mebooksfree.com mebooksfree.com mebooksfree.com mebooksfree.com mebooksfree.com mebooksfree.com

mebooksfree.com mebooksfree.com mebooksfree.com mebooksfree.com mebooksfree.com mebooksfree.com

mebooksfree.com mebooksfree.com mebooksfree.com mebooksfree.com mebooksfree.com mebooksfree.com

mebooksfree.com mebooksfree.com mebooksfree.com mebooksfree.com mebooksfree.com mebooksfree.com

mebooksfree.com mebooksfree.com mebooksfree.com mebooksfree.com mebooksfree.com mebooksfree.com

mebooksfree.com mebooksfree.com mebooksfree.com mebooksfree.com mebooksfree.com mebooksfree.com

mebooksfree.com mebooksfree.com mebooksfree.com mebooksfree.com mebooksfree.com mebooksfree.com mebooksfree.com mebooksfree.com mebooksfree.com mebooksfree.com mebooksfree.com mebooksfree.com

Review of mebooksfree.com mebooksfree.com mebooksfree.com mebooksfree.com mebooksfree.com mebooksfree.com

mebooksfree.com mebooksfree.com mebooksfree.com mebooksfree.com mebooksfree.com mebooksfree.com

mebooksfree.com mebooksfree.com mebooksfree.com mebooksfree.com mebooksfree.com mebooksfree.com

mebooksfree.com mebooksfree.com mebooksfree.com mebooksfree.com mebooksfree.com mebooksfree.com

mebooksfree.com mebooksfree.com mebooksfree.com mebooksfree.com mebooksfree.com mebooksfree.com

mebooksfree.com mebooksfree.com mebooksfree.com mebooksfree.com mebooksfree.com mebooksfree.com mebooksfree.com mebooksfree.com mebooksfree.com mebooksfree.com mebooksfree.com mebooksfree.com

mebooksfree.com mebooksfree.com mebooksfree.com mebooksfree.com mebooksfree.com mebooksfree.com

mebooksfree.com mebooksfree.com mebooksfree.com mebooksfree.com mebooksfree.com mebooksfree.com

mebooksfree.com mebooksfree.com mebooksfree.com mebooksfree.com mebooksfree.com mebooksfree.com

mebooksfree.com mebooksfree.com mebooksfree.com mebooksfree.com mebooksfree.com mebooksfree.com

mebooksfree.com mebooksfree.com mebooksfree.com mebooksfree.com mebooksfree.com mebooksfree.com

mebooksfree.com mebooksfree.com mebooksfree.com mebooksfree.com mebooksfree.com mebooksfree.com mebooksfree.com mebooksfree.com mebooksfree.com mebooksfree.com mebooksfree.com mebooksfree.com

Review of Psychiatry mebooksfree.com mebooksfree.com mebooksfree.com mebooksfree.com mebooksfree.com mebooksfree.com

Praveen Tripathi MBBS, MD Consultant, Psychiatry Kailash Hospital and Research Institute mebooksfree.com mebooksfree.com mebooksfree.comNoida, Uttar Pradesh,mebooksfree.com India mebooksfree.com mebooksfree.com

mebooksfree.com mebooksfree.com mebooksfree.com mebooksfree.com mebooksfree.com mebooksfree.com

Foreword Kailash Kedia MBBS, MD

mebooksfree.com mebooksfree.com mebooksfree.com mebooksfree.com mebooksfree.com mebooksfree.com

The Health Sciences Publisher New Delhi | | Panama | Philadelphia

mebooksfree.com mebooksfree.com mebooksfree.com mebooksfree.com mebooksfree.com mebooksfree.com

mebooksfree.com mebooksfree.com mebooksfree.com mebooksfree.com mebooksfree.com mebooksfree.com mebooksfree.com mebooksfree.com mebooksfree.com mebooksfree.com mebooksfree.com mebooksfree.com

mebooksfree.com mebooksfree.com mebooksfree.com mebooksfree.com mebooksfree.com mebooksfree.com Jaypee Brothers Medical Publishers (P) Ltd

Headquarters Jaypee Brothers Medical Publishers (P) Ltd 4838/24, Ansari Road, Daryaganj New Delhi 110 002, India Phone: +91-11-43574357 Fax: +91-11-43574314 Email: [email protected]

Overseas Offices J.P. Medical Ltd Jaypee-Highlights Medical Publishers Inc Jaypee Medical Inc 83 Victoria Street, London City of Knowledge, Bld. 235, 2nd Floor, Clayton 325 Chestnut Street SW1H 0HW (UK) Panama City, Panama Suite 412, Philadelphia, PA 19106, USA mebooksfree.com mebooksfree.comPhone: +44 20 3170 8910 mebooksfree.comPhone: +1 507-301-0496 mebooksfree.comPhone: +1 267-519-9789mebooksfree.com mebooksfree.com Fax: +44 (0)20 3008 6180 Fax: +1 507-301-0499 Email: [email protected] Email: [email protected] Email: [email protected]

Jaypee Brothers Medical Publishers (P) Ltd Jaypee Brothers Medical Publishers (P) Ltd 17/1-B Babar Road, Block-B, Shaymali Bhotahity, Kathmandu, Nepal Mohammadpur, Dhaka-1207 Phone +977-9741283608 Bangladesh Email: [email protected] Mobile: +08801912003485 Email: [email protected]

Website: www.jaypeebrothers.com Website: www.jaypeedigital.com

© 2016, Jaypee Brothers Medical Publishers mebooksfree.com mebooksfree.comThe views and opinions expressed mebooksfree.comin this book are solely those of the original contributor(s)/author(s)mebooksfree.com and do not necessarilymebooksfree.com represent those mebooksfree.com of editor(s) of the book. All rights reserved. No part of this publication may be reproduced, stored or transmitted in any form or by any means, electronic, mechanical, photocopying, recording or otherwise, without the prior permission in writing of the publishers. All brand names and product names used in this book are trade names, service marks, trademarks or registered trademarks of their respective owners. The publisher is not associated with any product or vendor mentioned in this book. Medical knowledge and practice change constantly. This book is designed to provide accurate, authoritative information about the subject matter in question. However, readers are advised to check the most current information available on procedures included and check information from the manufacturer of each product to be administered, to verify the recommended dose, formula, method and duration of administration, adverse effects and contraindications. It is the responsibility of the practitioner to take all appropriate safety precautions. Neither the publisher nor the author(s)/editor(s) assume any liability for any injury and/or damage to persons or property arising from or related to use of material in this book. This book is sold on the understanding that the publisher is not engaged in providing professional medical services. If such advice or services are required, the services of a competent medical professional should be sought. Every effort has been made where necessary to contact holders of copyright to obtain permission to reproduce copyright material. If any mebooksfree.com mebooksfree.comhave been inadvertently overlooked,mebooksfree.com the publisher will be pleased to make the necessarymebooksfree.com arrangements at the first opportunity.mebooksfree.com mebooksfree.com Inquiries for bulk sales may be solicited at: [email protected] Review of Psychiatry

First Edition: 2016 ISBN 978-93-85999-52-9 Printed at

mebooksfree.com mebooksfree.com mebooksfree.com mebooksfree.com mebooksfree.com mebooksfree.com

mebooksfree.com mebooksfree.com mebooksfree.com mebooksfree.com mebooksfree.com mebooksfree.com mebooksfree.com mebooksfree.com mebooksfree.com mebooksfree.com mebooksfree.com mebooksfree.com

mebooksfree.com mebooksfree.com mebooksfree.com mebooksfree.com mebooksfree.com mebooksfree.com

Dedicated to mebooksfree.com mebooksfree.com mebooksfree.comMy Parents mebooksfree.com mebooksfree.com mebooksfree.com

mebooksfree.com mebooksfree.com mebooksfree.com mebooksfree.com mebooksfree.com mebooksfree.com

mebooksfree.com mebooksfree.com mebooksfree.com mebooksfree.com mebooksfree.com mebooksfree.com

mebooksfree.com mebooksfree.com mebooksfree.com mebooksfree.com mebooksfree.com mebooksfree.com

mebooksfree.com mebooksfree.com mebooksfree.com mebooksfree.com mebooksfree.com mebooksfree.com mebooksfree.com mebooksfree.com mebooksfree.com mebooksfree.com mebooksfree.com mebooksfree.com

mebooksfree.com mebooksfree.com mebooksfree.com mebooksfree.com mebooksfree.com mebooksfree.com

mebooksfree.com mebooksfree.com mebooksfree.com mebooksfree.com mebooksfree.com mebooksfree.com

mebooksfree.com mebooksfree.com mebooksfree.com mebooksfree.com mebooksfree.com mebooksfree.com

mebooksfree.com mebooksfree.com mebooksfree.com mebooksfree.com mebooksfree.com mebooksfree.com

mebooksfree.com mebooksfree.com mebooksfree.com mebooksfree.com mebooksfree.com mebooksfree.com

mebooksfree.com mebooksfree.com mebooksfree.com mebooksfree.com mebooksfree.com mebooksfree.com mebooksfree.com mebooksfree.com mebooksfree.com mebooksfree.com mebooksfree.com mebooksfree.com

Foreword mebooksfree.com mebooksfree.com mebooksfree.com mebooksfree.com mebooksfree.com mebooksfree.com

Psychiatry is quite different from mainstream medical specialties and poses unique challenges when the novice medical graduate is attempting to understand these concepts. Psychiatry is also a fast evolving science and the recent introduction of DSM-5 has led to several diagnostic revisions. Most of the textbooks on psychiatry are fairly exhaustive and can be difficult to read for students preparing for entrance exams who are hard-pressed for time. Keeping these aspects in mind Dr Tripathi has made enthusiastic efforts to compile the exhaustive literature on mental health into a simple format that is highly readable and easy to understand. He has also included MCQs from past examinations for practice and to adapt to the exam questions. I recommend this book as a powerful and time efficient tool to prepare for psychiatry section of postgraduate entrance examinations. I wish all the readers good luck and congratulate Dr Tripathi for his efforts in writing this book. mebooksfree.com mebooksfree.com mebooksfree.com mebooksfree.com mebooksfree.com mebooksfree.com Kailash Kedia MBBS, MD Staff Specialist Princess Alexandra Hospital Woolloongabba, Queensland-4102 Associate Lecturer University of Queensland, Australia

mebooksfree.com mebooksfree.com mebooksfree.com mebooksfree.com mebooksfree.com mebooksfree.com

mebooksfree.com mebooksfree.com mebooksfree.com mebooksfree.com mebooksfree.com mebooksfree.com

mebooksfree.com mebooksfree.com mebooksfree.com mebooksfree.com mebooksfree.com mebooksfree.com

mebooksfree.com mebooksfree.com mebooksfree.com mebooksfree.com mebooksfree.com mebooksfree.com mebooksfree.com mebooksfree.com mebooksfree.com mebooksfree.com mebooksfree.com mebooksfree.com

mebooksfree.com mebooksfree.com mebooksfree.com mebooksfree.com mebooksfree.com mebooksfree.com

mebooksfree.com mebooksfree.com mebooksfree.com mebooksfree.com mebooksfree.com mebooksfree.com

mebooksfree.com mebooksfree.com mebooksfree.com mebooksfree.com mebooksfree.com mebooksfree.com

mebooksfree.com mebooksfree.com mebooksfree.com mebooksfree.com mebooksfree.com mebooksfree.com

mebooksfree.com mebooksfree.com mebooksfree.com mebooksfree.com mebooksfree.com mebooksfree.com

mebooksfree.com mebooksfree.com mebooksfree.com mebooksfree.com mebooksfree.com mebooksfree.com mebooksfree.com mebooksfree.com mebooksfree.com mebooksfree.com mebooksfree.com mebooksfree.com

Preface mebooksfree.com mebooksfree.com mebooksfree.com mebooksfree.com mebooksfree.com mebooksfree.com

Psychiatry is a complex subject and students have minimal exposure to psychiatric disorders during their MBBS training. The terminology used in psychiatry is quite different from other medical specialties and makes the subject tough to understand. Most of the students resort to rote memorization and struggle with the conceptual aspects. In this book, an attempt has been made to explain the concepts in a simple language and without using the psychiatry jargons. A large number of examples have been included in the text to explain the concepts and help in learning. Another important aspect of this book is that it has been fully updated with DSM-5. In DSM-5, a large number of new diagnoses have been introduced and diagnostic criterions of many existing disorders have been changed. All these changes have been incorporated in the book. This book has been written keeping in mind the needs of students preparing for various postgraduate entrance examinations and MCI screening test. Nowadays, mastery over short subjects has become a key to get a good rank. mebooksfree.com mebooksfree.comIn most of the exams (includingmebooksfree.com AIIMS, PGI and NEET), at leastmebooksfree.com 5-6 questions are being asked frommebooksfree.com psychiatry. If mebooksfree.com students can spare 5-6 days for psychiatry, they would be easily be able to get those questions correct and that will make a real difference in the final ranks achieved. Finally, a word of advise for the students. If you can keep yourself motivated for the entire duration of preparation, cracking the entrance becomes a child’s play. You should remain in regular touch with your seniors and take both tips and inspiration from them. Appearing regularly for mock tests and discussion with peers is a good way of assessing your strengths and weaknesses, it also motivates you to work harder and get better results next time. Remember you need to win many small battles, before you can win a war. So buckle up, get ready to bring your best to the table, work so hard that you surprise even yourself and achieve what you rightly deserve. My best wishes and blessings are always with you. mebooksfree.com mebooksfree.com mebooksfree.com mebooksfree.com mebooksfree.com mebooksfree.com April, 2016 Praveen Tripathi MBBS, MD Consultant, Psychiatry Kailash Hospital and Research Institute Noida, Uttar Pradesh, India [email protected] www.facebook.com/drpraveentripathipsychiatrist

mebooksfree.com mebooksfree.com mebooksfree.com mebooksfree.com mebooksfree.com mebooksfree.com

mebooksfree.com mebooksfree.com mebooksfree.com mebooksfree.com mebooksfree.com mebooksfree.com

mebooksfree.com mebooksfree.com mebooksfree.com mebooksfree.com mebooksfree.com mebooksfree.com mebooksfree.com mebooksfree.com mebooksfree.com mebooksfree.com mebooksfree.com mebooksfree.com

mebooksfree.com mebooksfree.com mebooksfree.com mebooksfree.com mebooksfree.com mebooksfree.com

mebooksfree.com mebooksfree.com mebooksfree.com mebooksfree.com mebooksfree.com mebooksfree.com

mebooksfree.com mebooksfree.com mebooksfree.com mebooksfree.com mebooksfree.com mebooksfree.com

mebooksfree.com mebooksfree.com mebooksfree.com mebooksfree.com mebooksfree.com mebooksfree.com

mebooksfree.com mebooksfree.com mebooksfree.com mebooksfree.com mebooksfree.com mebooksfree.com

mebooksfree.com mebooksfree.com mebooksfree.com mebooksfree.com mebooksfree.com mebooksfree.com mebooksfree.com mebooksfree.com mebooksfree.com mebooksfree.com mebooksfree.com mebooksfree.com

Acknowledgments mebooksfree.com mebooksfree.com mebooksfree.com mebooksfree.com mebooksfree.com mebooksfree.com

Every endeavor, however big or small needs contribution from many. This book is no exception. A large number of people have contributed directly or indirectly in the completion of this book. At the outset, I would like to thank my parents who have backed all the decisions, I have ever taken in my life and have supported me even when they did not agree with me. I am thankful to my elder brother, Dr. Anurag Tripathi, who gave me a lot of suggestions while I was writing this book and pushed me to put more and better efforts. I want to convey special thanks to my wife, Dr Priyanka Goyal, for bearing with me for the long months during which this book was written and helping me with the content as well as editing of the book. Without her help, this book would not have seen the light of day. I am extremely thankful to Dr Apurv Mehra, who brought me into the field of teaching and is like a friend and teacher to me. I am also grateful to Dr Pritesh Singh, who taught me the art of writing a book and who has made mebooksfree.com mebooksfree.comimportant contributions in formattingmebooksfree.com the book. mebooksfree.com mebooksfree.com mebooksfree.com I would also like to thank Shri Jitendar P Vij (Group Chairman), Mr Ankit Vij (Group President), Ms Chetna Malhotra Vohra (Associate Director—Content Strategy), Ms Payal Bharti (Project Manager), Mr Arun Sharma (Typesetter), Ms Priyanka Shahi, Mr Pankaj K Singh (Proof Readers), and the production team of Jaypee Brothers Medical Publishers (P) Ltd, New Delhi, India. Finally, I would like to thank my patients and my students. Both of them have taught me a lot and continue to be my favorite teachers.

Praveen Tripathi mebooksfree.com mebooksfree.com mebooksfree.com mebooksfree.com mebooksfree.com mebooksfree.com

mebooksfree.com mebooksfree.com mebooksfree.com mebooksfree.com mebooksfree.com mebooksfree.com

mebooksfree.com mebooksfree.com mebooksfree.com mebooksfree.com mebooksfree.com mebooksfree.com

mebooksfree.com mebooksfree.com mebooksfree.com mebooksfree.com mebooksfree.com mebooksfree.com mebooksfree.com mebooksfree.com mebooksfree.com mebooksfree.com mebooksfree.com mebooksfree.com

mebooksfree.com mebooksfree.com mebooksfree.com mebooksfree.com mebooksfree.com mebooksfree.com

mebooksfree.com mebooksfree.com mebooksfree.com mebooksfree.com mebooksfree.com mebooksfree.com

mebooksfree.com mebooksfree.com mebooksfree.com mebooksfree.com mebooksfree.com mebooksfree.com

mebooksfree.com mebooksfree.com mebooksfree.com mebooksfree.com mebooksfree.com mebooksfree.com

mebooksfree.com mebooksfree.com mebooksfree.com mebooksfree.com mebooksfree.com mebooksfree.com

mebooksfree.com mebooksfree.com mebooksfree.com mebooksfree.com mebooksfree.com mebooksfree.com mebooksfree.com mebooksfree.com mebooksfree.com mebooksfree.com mebooksfree.com mebooksfree.com

Contents mebooksfree.com mebooksfree.com mebooksfree.com mebooksfree.com mebooksfree.com mebooksfree.com

1. Basics 1

2. Schizophrenia Spectrum and Other Psychotic Disorders 10

3. Mood Disorders 27

4. Neurotic, Stress Related and Somatoform Disorders 43

5. Substance Related and Addictive Disorders 63 mebooksfree.com mebooksfree.com mebooksfree.com mebooksfree.com mebooksfree.com mebooksfree.com 6. Organic Mental Disorders 78

7. Personality Disorders 89

8. Eating Disorders 95

9. Sleep Disorders 98

10. Sexual Disorders 103 mebooksfree.com mebooksfree.com 11. Child Psychiatry mebooksfree.com mebooksfree.com mebooksfree.com107 mebooksfree.com

12. 115

13. Miscellaneous 122

mebooksfree.com mebooksfree.com mebooksfree.com mebooksfree.com mebooksfree.com mebooksfree.com

mebooksfree.com mebooksfree.com mebooksfree.com mebooksfree.com mebooksfree.com mebooksfree.com

mebooksfree.com mebooksfree.com mebooksfree.com mebooksfree.com mebooksfree.com mebooksfree.com mebooksfree.com mebooksfree.com mebooksfree.com mebooksfree.com mebooksfree.com mebooksfree.com

mebooksfree.com mebooksfree.com mebooksfree.com mebooksfree.com mebooksfree.com mebooksfree.com

Chapter 1 Basics

Psychiatry is the branch of medicine which deals with b. Depressed mood: Excessive sadness of mood, morbid psychological processes. To establish diagnosis which is usually seen in depression. mebooksfree.com mebooksfree.comof a psychiatric disorder bothmebooksfree.com history and clinical exami­ • mebooksfree.comFluctuations: It refers to the changes inmebooksfree.com mood/affect. mebooksfree.com nation are required. The clinical examination in psychia­ The common disturbances of fluctuations are as try, wherein the clinician records the psychiatric signs follows: and symptoms, is known as Mental Status Examination a. Labile mood: Excessive variations in mood with­ (MSE)Q. out any apparent reason. It is also known as emotional labilityQ. For example, a man starts Mental Status Examination crying and then starts laughing without any apparent reason. It is usually seen in mania. In mental status examination, following areas of mental b. Affective flattening: Absence of changes in mood functioning are assessed: irres­pective of the situation. In this condition, A. General appearance and behavior: The appearance of patient doesn’t experience any emotions hence the patient is described along with any gross abnor­ his affect remains the same. For example, a malities (such as abnormalities of dressing etc). schizophrenic patient would not look happy mebooksfree.com mebooksfree.com B. Speech: Various aspects ofmebooksfree.com speech such as rate, tone, mebooksfree.com mebooksfree.com mebooksfree.com during festivals and did not appear sad when volume, spontaneity of speech are described. C. Mood and affect: The terms “affect” and “mood” are his mother died. His mood remained the same both used to describe the emotions or emotional irrespective of the situation. state. “Affect”Q is the cross sectional emotional state • Appropriateness and congruency: Appropriateness whereas “mood” is the sustained or longitudinal of affect is described in relation to the social situa­ emotional state. For example, if an individual who tion. For example, in a funeral, the expected emo­ was extremely sad for last one month, gets extremely tional state is sadness. Hence, being sad in a funeral and unusually happy for a moment; it can be said is an appropriate affect. If a man starts laughing that his affect is happy (euphoric), whereas his mood and looks extremely happy in a funeral, it would is depressed. The term affect and mood are at times be diagnosed as inappropriate affect. Congruency used interchangeably. Affect and mood are further of mood is described in relation to the thought con­ mebooksfree.com mebooksfree.comdescribed under the followingmebooksfree.com three subheads: mebooksfree.comtent of the person. Congruency describesmebooksfree.com whether mebooksfree.com • Quality: It refers to the predominant affective (or the emotional state of person is in sync with his mood) state. There can be various disturbances in thought/speech or not. For example, if a man is the quality of mood, common ones include: thinking about or talking about the events which a. Euphoric mood (elevation of mood): Euphoria led to his mother’s death, he is expected to be sad. refers to a state of excessive happiness, without Hence, appearing sad while talking about mother’s any reason. It is usually seen in mania or hypo­ death is a congruent affect. If a person, looksvery mania. happy and smiles while describing his mothers

mebooksfree.com mebooksfree.com mebooksfree.com mebooksfree.com mebooksfree.com mebooksfree.com

mebooksfree.com mebooksfree.com mebooksfree.com mebooksfree.com mebooksfree.com mebooksfree.com mebooksfree.com mebooksfree.com mebooksfree.com mebooksfree.com mebooksfree.com mebooksfree.com

2 Review of Psychiatry

death, it would be considered as incongruent voices are coming from the wall or from outside mood. It must be stressed that while “appropri­ the house. (PseudohallucinationsQ are experi­ ateness” of affect is described after comparing the enced in the inner subjective space, or originat­ current affect with the expected affect in the given ing from within the mind. For example, a patient mebooksfree.com mebooksfree.comsocial situation, the congruencemebooksfree.com is described after mebooksfree.comwith auditory pseudohallucinationsmebooksfree.com will report mebooksfree.com comparing the current affect with the expected that the voices are originating within his mind affect in the context of the patients thoughts. and not from outside). Few other important disturbances of emotions d. Hallucinations are not under the willful con- include: trolQ of the patient. It means that the patient can a. Alexithymia: It refers to the inability to understand neither start the hallucinations nor can he stop emotions of others and inability to express emo­ them. tions of self. Although alexithymia is closely related Hallucinations can occur in any modality. The to affective flattening,alexithymia Q is “lack of most common hallucinations in psychiatric disorders words to describe emotions” rather than absence are auditory hallucinationsQ. The most common of emotions. hallucinations in organic psychiatric disorders b. Anhedonia: It refers to the loss of capacity to expe­ (such as delirium) are visual hallucinationsQ. In mebooksfree.com mebooksfree.comrience pleasure. The patientmebooksfree.com is unable to enjoy any­ mebooksfree.compatients with temporal lobe epilepsymebooksfree.comQ all kinds of mebooksfree.com thing in the life. hallucinations can be present including olfactory and tactile hallucinations. Tactile hallucinations are also a Neuroanatomical substrate of emotions: Limbic systemQ (which includes hippocampus, amygdala, hypothalamus, typical feature of cocaine intoxication. cingulate gyrus and related thalamic and cortical areas) Few specific hallucinations: Q is the neural substrate for the emotional experiences. The a. Hypnagogic hallucinations : These hallucina­ tions occur while falling asleep or while going regulation of emotions is a function of frontal lobeQ. to sleep. Since hypnagogic has the word “go” in D. Perception: Perception is the receiving of information it, hence its easy to remember that they occur using one of the sensory modalities (i.e. auditory, vis­ while “going” to sleep. Hypnagogic hallucina­ ual, tactile, olfactory and gustatory). Two most impor­ tions are seen in narcolepsy. tant disturbances of perception are: b. Hypnopompic hallucinationsQ: These halluci­ • IllusionsQ: Illusion is false perception of a real nations occur while getting up from the sleep. object. For example, a man mistakes a rope for mebooksfree.com mebooksfree.com mebooksfree.com mebooksfree.comc. Reflex hallucinations (Synesthesiamebooksfree.comQ): In reflex mebooksfree.com snake in night. hallucinations, stimulus in one sensory moda­ • Hallucinations: Hallucination is a false perception lity produces hallucinations in another sensory in the absence of any object or stimulus. For exam­ modality. For example, a patient reports that ple, a patient of delirium reported seeing snakes on whenever he sees a white bulb (stimulus in the ground of his room, when in reality there was visual modality), he starts hearing voices of god nothing there. Hallucinations have the following (hallucination in auditory modality). Reflex hal­ properties and all these properties must be present lucinations are a feature of cannabis and LSDQ to diagnose a perception as hallucination. (and other hallucinogens) intoxication. a. Hallucinations occur in the absence of any sen­ d. Functional hallucination: Here, stimulus in one sory or perceptual stimulus. sensory moda­lity, produces hallucinations in the b. Hallucinations are as vivid (clear or detailed) same sensory modality. For example, a patient mebooksfree.com mebooksfree.comas true perceptions.mebooksfree.com It means that the person mebooksfree.comreported that whenever he heardmebooksfree.com the sound of mebooksfree.com who experiences hallucinations­ is able to give a a ticking clock (stimulus in auditory modality), detailed description of what he is experiencing. he would also start hearing voices of god (hal­ c. Hallucinations are experienced in outer objec- lucinations in auditory modality).­ tive spaceQ. It means that patients experiences E. Thought (Cognition): The terms “thought” and that the source of hallucinations­ is in the outer “cognition”Q are at times used interchangeably, how­ world. For example, a patient who is having ever in a stricter sense cognition is the mental process auditory hallucinations will report that the of acquiring knowledge which includes thoughts but

mebooksfree.com mebooksfree.com mebooksfree.com mebooksfree.com mebooksfree.com mebooksfree.com

mebooksfree.com mebooksfree.com mebooksfree.com mebooksfree.com mebooksfree.com mebooksfree.com mebooksfree.com mebooksfree.com mebooksfree.com mebooksfree.com mebooksfree.com mebooksfree.com

Basics 3

also experiences and sensations. The thought distur­ d. CircumstantialityQ: It is a pattern of speech bances are primary in many psychiatric disorders like which progresses with inclusion of lots of schizophrenia. Thought and its disturbances can be unnecessary details and goes round and round described under the following subheads. before reaching the final goal. For example, a mebooksfree.com mebooksfree.com• Stream (Flow of thought)mebooksfree.com: It refers to the speed with mebooksfree.commedical student was asked aboutmebooksfree.com his preferred mebooksfree.com which thoughts follow each other. The disturbances branch in postgraduation and he replied by say­ of stream includes: ing “Sir, in the first year i was very interested in a. Flight of ideasQ: Here, the thoughts follow each physiology, however in the second year i started other very rapidly, and connection between dif­ liking pathology. In the third year, i started liking ferent thoughts appears to be due to chance fac­ ophthalmology however in the final year i rea­ tors or rhyming. It is usually seen in mania. For lized that i have a lot of liking for orthopedics example, a manic patient when asked about his too and i liked putting casts and working with hometown said “I live in Delhi…my cat has a big POP. I also think that after MBBS one should belly…..i like to eat Jelly…..lilly lilly lilly”. Some get married as soon as possible and that noone authors describe “flight of ideas” as an abnor­ should have more than two kids…Well..you see mality of form of thought. i like pediatrics as a subject and want to do my mebooksfree.com mebooksfree.comb. Inhibition of thinkingmebooksfree.com: Here thoughts come in mebooksfree.compostgraduation in the pediatrics”. mebooksfree.comIn this exam­ mebooksfree.com mind very slowly and thought progresses with ple the thought process progressed with inclu­ a slow rate. sion of lots of irrelevant details however in the • Form of thought: The form refers to the “organi­ end, the goal was reached as student said that zation” of thought or the “association” between he wants to become a pediatrician. the consecutive thoughts. Normally, the thoughts e. TangentialityQ: In tangentiality, the answer are well organized and there is a connection is related to the question in some distant way between various components of a single thought and the goal of thought is never reached. For and between the consecutive thoughts. In formal example, a patient was asked about his favorite thought disorders, there are disturbance in the bolly­wood actor and he replied “Well, you see organization, associations and connections of the the hindi movies are mostly hero centric and thoughts. The important formal thought disorders usually deal with the relationship issues whereas include: the hollywood movies have lots of action and mebooksfree.com mebooksfree.coma. Derailment: In derailment,mebooksfree.com the association bet­ mebooksfree.com mebooksfree.com mebooksfree.com science fiction. I think the Hindi Film Industry ween two successive thoughts is disturbed. For is growing rapidly and its a good medium for example, a patient said Jawahar Lal Nehru was entertainment of masses”. In this example, the the first prime minister of India and he was a congress leader. Sachin Tendulkar scored 100 patients answer was distantly related to ques­ international hundreds”. In this example, there tion, however the exact answer was never given. Q is no link between the first thought about Nehru f. Neologism: A neologism is coining of a new and second thought about Tendulkar. word, whose derivation cannot be understood. b. Loosening of associationQ: Here, the connection For example, a patient would use the word is lost bet­ween components of a single thought. “tintintapa” for a pen. Neologism is highly sug­ For example, a patient says “I thought that it will gestive of schizophrenia. rain today, Modi is the current prime minister g. Word approximations (metonyms): Here, old mebooksfree.com mebooksfree.comof India”. In this examplemebooksfree.com the phrase before the mebooksfree.comwords are used in a new or unconventionalmebooksfree.com way. mebooksfree.com comma is totally disconnected from the phrase The meaning will be easily evident, though the after the comma and hence this represents loos­ word in itself might appear strange. For exam­ ening of association. ple, a patient would us the world “time vessel” c. Incoherence: It is the total lack of organization so for watch, and use the word “handshoes” for that the thought is incomprehensible and does gloves. not make any sense. For example, a patient says h. Perseveration: It is repetition of the same res­ “India me churchgate pulses cricket computer”. ponse, beyond the point of relevance. For

mebooksfree.com mebooksfree.com mebooksfree.com mebooksfree.com mebooksfree.com mebooksfree.com

mebooksfree.com mebooksfree.com mebooksfree.com mebooksfree.com mebooksfree.com mebooksfree.com mebooksfree.com mebooksfree.com mebooksfree.com mebooksfree.com mebooksfree.com mebooksfree.com

4 Review of Psychiatry

example, a patient was asked the following may claim that they have committed unpardon­ questions. Q: What is your name. Ans. Mahesh able sins. It is usually seen in severe depression. kumar….Q: Where do you live. Ans: Mahesh Bizarre Vs Nonbizarre Delusions Kumar…..Q: How many children do you have… Bizarre delusions: The term bizarre is used for mebooksfree.com mebooksfree.comA: Mahesh Kumar. mebooksfree.com mebooksfree.comdelusions which are scientificallymebooksfree.com impossible mebooksfree.com It must be noted that the perseveration is in and culturally implausible (ununderstandable). response to a question and is not spontaneous. For example, if a patient says that aliens have • Content of thought: It refers to what person is actu­ stolen his heart, it would be an example of ally thinking about. Delusion is a disorder of con­ bizarre delusion. tent of thought. It is defined as a false, unshakeable Nonbizarre delusions: These are delusions which belief that cannot be explained on the basis of per­ are false but are possible, i.e. they can happen. sons social and cultural background. The following For example, if a patient develops a delusion are the types of delusion: that his family members wants to take away his a. Delusion of persecution: It is the most common property, it would be an example of nonbizarre type of delusion.The patient believes that some­ delusion, since it is not impossible for a family one wants to harm him. For example, a patient member to take away property of another family mebooksfree.com mebooksfree.comclaimed that Indianmebooksfree.com police along with CBI is mebooksfree.commember. mebooksfree.com mebooksfree.com hatching a conspiracy to kill him. • Possession of thought: Normally one experiences that b. Delusion of reference: The patient believes that their thoughts belong to themselves and no one else can influence their thinking process, also there is a events happening around him are somehow sense of control over one’s thought. In disturbances related to him. For example, a patient claimed of possession of thought either the patients experi­ that the tube light of his apartment was flicker­ ences that others are tampering with their thoughts ing as there was a camera fitted inside through or that they have lost control over their thoughts. The which his movements are being recorded. disorders of possession include the following: c. Delusion of grandeur or grandiosity: The patient a. ObsessionsQ: Here, a thought comes repeatedly belie­ves that he has some exceptional identity or into the mind of patient against his will. The power. For example, a patient claimed that he is patient recognizes the thought as his own, how­ the reincarnation of Lord Hanuman and that he ever is distressed by the repetitive and intrusive mebooksfree.com mebooksfree.comcan carry the mountainsmebooksfree.com on his shoulders. mebooksfree.comnature of the thought. The patientmebooksfree.com feels that he mebooksfree.com Q d. Delusion of love (erotomania , fantasy lover syn- has lost control over his thoughts. drome): Patient may have false belief that some­ b. Thought alienation: Here, the patient feels that one is in love with them. It is also known as de their thoughts are under control of an outside Clerambault syndrome. For example, a rickshaw agency or that others are interfering with their puller claimed that Katrina Kaif is in love with thought process. Thought alienation pheno­ him though he admitted that he has never met menon is of following types: her. – Thought insertion: Patient feels that some e. Nihilistic delusion (delusion of negation, Cotard’s external agency is inserting foreign thoughts syndromeQ): Here, the patient may deny exis­ into their mind. tence of their body, their mind, or the world in – Thought withdrawal: Patient experiences general. They may claim that everybody is dead, that his thoughts are being withdrawn from mebooksfree.com mebooksfree.comthe world has stopped,mebooksfree.com etc. The basic theme of mebooksfree.comtheir mind by an external agency.mebooksfree.com mebooksfree.com delusion is the “end of existence”. – Thought broadcast: Patient experiences that f. Delusion of infidelity (delusion of jealousy): The thoughts are escaping from their minds and patient has a false belief that his partner/spouse other people are able to access them. is having an affair. It is also known as morbid F. Higher mental functions: In this component of MSE, jealousy or Othello syndromeQ. various higher mental functions like attention, con­ g. Delusion of guilt: Here, the patient may develop centration, memory, judgement, abstract thinking and a delusion that they are bad or evil person and insight are assessed.

mebooksfree.com mebooksfree.com mebooksfree.com mebooksfree.com mebooksfree.com mebooksfree.com

mebooksfree.com mebooksfree.com mebooksfree.com mebooksfree.com mebooksfree.com mebooksfree.com mebooksfree.com mebooksfree.com mebooksfree.com mebooksfree.com mebooksfree.com mebooksfree.com

Basics 5 CLASSIFICATION are known to affect brain parenchyma) For example, delirium, dementia. At present, there are two major classificatory systems in B. Functional (Nonorganic) mental disorders: These dis­ psychiatry. orders do not have any demonstrable disturbance mebooksfree.com mebooksfree.com 1. ICD-10 (International classificationmebooksfree.com of diseases, 10th of mebooksfree.combrain parenchyma. For example, schizophrenia,mebooksfree.com mebooksfree.com edition): It is published by WHO and provides classi­ mania, etc. fication for all medical disorders (including psychia­ This classification is at best arbitrary, since with the tric disorders). The psychiatric disorders have been advent of science its possible to demonstrate brain Q classified in thechapter-V (F) of ICD-10. parenchyma disturbances even in so called “func­ 2. DSM-5 (Diagnostic and statistical manual of mental tional” mental disorders. disorders): It is published by American Psychiatric Psychoses vs neuroses: The functional disorders can be Association. The fifth edition of DSM was published further classified into psychotic disorders (psychoses) in 2013. and neurotic disorders (neuroses). Psychiatric disorders have been classified in multiple A. Psychoses: Psychotic disorders are characterized by ways. The most important classifications includes organic lack of awareness of illness (also known as lack of vs functional psychiatric disorders and psychosis vs neu­ insight)Q and impaired reality testing (i.e. the patients mebooksfree.com mebooksfree.comrosis. mebooksfree.com losesmebooksfree.com contact with reality and start livingmebooksfree.com in a fantasy mebooksfree.com Organic vs Functional (Nonorganic) mental disorders: world created by their ill minds). For example, schizo­ This was the first major classification of psychiatric/men­ phrenia, bipolar disorder. Delusions and hallucina­ tal disorders. tions are the prototype psychotic symptoms. A. Organic mental disorders: These disorders are caused B. Neuroses: Neurotic disorders are characterized by aware­ by demons­trable disturbances of brain (primary ness of the illness (insight is present) and reality contact brain disturbances or systemic disturbances which is also intact. For example, anxiety disorders, depression.

QUESTIONS AND ANSWERS mebooksfree.com mebooksfree.com mebooksfree.com mebooksfree.com mebooksfree.com mebooksfree.com QUESTIONS D. Inability to recognize and describe feelings E. Inappropriate mood 1. Which of the following are sections of Mental State Examination? (DNB NEET 2014-15) 4. A person who laughs at one minute and cries the A. Mood and affect B. Speech and language next minute without any clear stimulus is said to C. Cognition D. All of the above have: (AIIMS Nov 2005) A. Incongruent affect Affect and Mood B. Euphoria C. Labile affect 2. A 25-year-old woman complaints of intense D. Split personality depressed mood for last 6 months. She also reports 5. Emotion is controlled by: (PGI 1997) inability to enjoy previously pleasurable activities. A. Limbic system B. Frontal lobe This symptom is known as: (AIIMS Nov 2005) mebooksfree.com mebooksfree.com mebooksfree.com mebooksfree.comC. Temporal lobe D. Occipitalmebooksfree.com lobe mebooksfree.com A. Anhedonia B. Avolition C. Apathy D. Amotivation Perception 3. Alexithymia is: (Kerala 2000, DNB 2004) 6. Phantom limb is an example of disorder of: A. A feeling of intense rapture (DNB NEET 2104-15) B. Pathological sadness A. Thought B. Perception C. Affective flattening C. Cognition D. None of the above

mebooksfree.com mebooksfree.com mebooksfree.com mebooksfree.com mebooksfree.com mebooksfree.com

mebooksfree.com mebooksfree.com mebooksfree.com mebooksfree.com mebooksfree.com mebooksfree.com mebooksfree.com mebooksfree.com mebooksfree.com mebooksfree.com mebooksfree.com mebooksfree.com

6 Review of Psychiatry

7. A patient wanting to scratch for itching in his A. At the beginning of the sleep amputated limb is an example of: B. At the end of sleep, while getting up (DNB NEET 2014-15) C. After head trauma A. Illusion D. After convulsions B. Pseudohallucination mebooksfree.com mebooksfree.com mebooksfree.com 15. mebooksfree.comHallucinations which occur at the “start”mebooksfree.com of sleep mebooksfree.com C. Phantom limb hallucination are known as: (JIPMER 2002, DNB 2005) D. Autoscopic hallucination A. Hypnagogic hallucinations 8. A patient sees a rope and gets afraid that it is a B. Hypnopompic hallucinations snake. This sign is known as: C. Jactatio capitis nocturna (DNB NEET 2014-15, PGI 2002) D. Extracampine hallucinations A. Illusion B. Hallucination C. Delusion D. Depersonalization 16. Hallucinations are seen in all except: E. Derealization (MP 1999, DNB 2001) A. Schizophrenia 9. A 8-year-old child after a tonsillectomy sees a bear B. Seizures due to intracerebral space occupying in her room. She screams in fright. A nurse who lesions rushes on switching the light, finds a rug wrapped C. Lysergic acid diethyl amide intoxication (LSD mebooksfree.com mebooksfree.comon an armchair. Whatmebooksfree.com child experiences is best mebooksfree.com mebooksfree.com mebooksfree.com intoxication) described as? (DNB 2006, Kerala 1997) D. Anxiety A. Illusion B. Hallucination C. Delusion D. Depersonalization 17. Olfactory hallucinations are seen in: (PGI May 2011) 10. Which statement is not true about hallucinations? A. Schizophrenia (AIIMS 2009) B. Alzheimer’s disease A. It is as vivid as a real perception C. Mesial temporal sclerosis B. It occurs in inner subjective space D. Body dysmorphic disorder C. It is independent of will of observer E. Temporal lobe epilepsy D. It occurs in the absence of any perceptual stimulus 18. Visual hallucinations are seen in: (PGI Jun 2009) A. Hebephrenic schizophrenia 11. All of the following are features of hallucinations, mebooksfree.com mebooksfree.com mebooksfree.com mebooksfree.comB. Residual schizophrenia mebooksfree.com mebooksfree.com except: (AI 2003) C. Simple schizophrenia A. It is independent of will of observer D. Delirium B. Sensory organs are not involved E. Temporal lobe epilepsy C. It is as vivid as a real perception D. It occurs in the absence of any perceptual 19. Reflex hallucinations is a morbid variety of: stimulus (AIIMS May 2009, 2011) A. Kinesthesia B. Paresthesia 12. Formed visual hallucinations are seen in lesions C. Hyperesthesia D. Synesthesia of: (PGI 2006, 2000) A. Frontal lobe B. Temporal lobe Thought C. Occipital lobe D. Parietal lobe 20. The term “cognition” is used to imply about: 13. The following is suggestive of an organic cause of (AI 1997, Jharkhand 2003, DNB 1998) mebooksfree.com mebooksfree.combehavioral symptoms:mebooksfree.com (AI 2002) mebooksfree.comA. Affect B. Perceptionmebooksfree.com mebooksfree.com A. Formal thought disorder C. Thought D. Speech B. Auditory hallucinations 21. True about thought is all except: (PGI Feb 2007) C. Delusion of guilt A. Perseveration is out of context repetition D. Prominent visual hallucinations B. Circumstantiality is over inclusion of irrelevant 14. When is hypnopompic phenomenon experienced? details while eventually getting back to the origi­ (Bihar 2006, DNB 2002) nal point

mebooksfree.com mebooksfree.com mebooksfree.com mebooksfree.com mebooksfree.com mebooksfree.com

mebooksfree.com mebooksfree.com mebooksfree.com mebooksfree.com mebooksfree.com mebooksfree.com mebooksfree.com mebooksfree.com mebooksfree.com mebooksfree.com mebooksfree.com mebooksfree.com

Basics 7

C. Verbigeration is senseless repetition 29. A false belief which is unexplained by reality and D. Vorbeireden is skirting around the end point but is shared by a number of people is: never reaching it (AIIMS 2003, 2004 Jipmer 1998) E. Loosening of association is logically connected A. Illusion B. Delusion mebooksfree.com mebooksfree.comthoughts with loss ofmebooksfree.com goal. mebooksfree.comC. Obsession D. Superstitionmebooksfree.com mebooksfree.com 22. Perseveration is: (AI 2005) 30. The primary delusions are disorder of: (AI 1999) A. Persistent and inappropriate repetition of the A. Flow of thought same thoughts B. Form of thought B. Feeling of distress in a patient with schizophrenia C. Content of thought C. Characteristic of schizophrenia D. Possession of thought D. Characteristic of obsessive compulsive disorder 31. Delusions are not likely to be seen in: (AI 2012) 23. In schizophrenia, characteristic feature is: A. Dementia B. Depression (PGI 1997) C. Schizophrenia D. Conversion disorder A. Formal thought disorder 32. Delusions can be seen in all of the following except: B. Delusion (SGPGI 2002, DNB 2001) mebooksfree.com mebooksfree.comC. Hallucination mebooksfree.com mebooksfree.com mebooksfree.com mebooksfree.com A. OCD B. Depression D. Apathy C. Mania D. Schizophrenia 24. Loosening of association is an example of: 33. Delusion of persecution can be seen in: (AI 2006) (PGI Jun 2009) A. Formal thought disorder A. Schizophrenia B. Schneider’s first rank symptoms B. Delusional disorder C. Perseveration C. Manic episode D. Concrete thinking D. Melancholic depression 25. Not a disorder of form of thought is: 34. Delusion of grandiosity can be seen in: (AIIMS May 2012) PGI Nov 2010, May 2011) A. Tangentiality mebooksfree.com mebooksfree.com mebooksfree.com mebooksfree.comA. Hypomania mebooksfree.com mebooksfree.com B. Derailment B. Paranoid schizophrenia C. Thought block C. Schizoaffective disorder D. Loosening of association D. Kleptomania/Pyromania 26. Which of the following is/are thought disorder? E. Cyclothymia (DNB NEET 2014-15) 35. Nihilistic ideas are seen in: (PGI Dec 2008) A. Circumstantiality B. Tangentiality A. Simple schizophrenia C. Prolixity D. All of the above B. Paranoid schizophrenia 27. Schizophrenia and depression both have the fol- C. Cotard’s syndrome lowing features except: (PGI 2002) D. Depression A. Formal thought disorder E. Body dysmorphic disorder B. Social withdrawal 36. A 25-year-old university student had a fight with mebooksfree.com mebooksfree.comC. Poor personal caremebooksfree.com mebooksfree.comthe neighbouring boy. On the next daymebooksfree.com while out, mebooksfree.com D. Decreased interest in sex he started feeling that two men in police uniform E. Suicidal tendency were observing his movements. When he reached 28. Delusion is a disorder of: home in the evening he was frightened and told (DNB NEET 2014-15, AIIMS Nov 2006, AI 2007) his family members that police was after him and A. Perception B. Thought would arrest him. Despite reassurances by family C. Insight D. Affect members, he remained afraid that he is about to

mebooksfree.com mebooksfree.com mebooksfree.com mebooksfree.com mebooksfree.com mebooksfree.com

mebooksfree.com mebooksfree.com mebooksfree.com mebooksfree.com mebooksfree.com mebooksfree.com mebooksfree.com mebooksfree.com mebooksfree.com mebooksfree.com mebooksfree.com mebooksfree.com

8 Review of Psychiatry

be arrested. The history is suggestive of which 6. B. Perception. In phantom limb, the patient feels psychiatric sign/symptom: (AIIMS Nov 2003) sensations in the amputated limb. Hence, its a A. Delusion of persecution disorder of perception. B. Delusion of reference 7. C. Phantom limb hallucination. Since, patient expe­ mebooksfree.com mebooksfree.comC. Somatic passivity mebooksfree.com mebooksfree.comriences sensation in the absence ofmebooksfree.com any stimulus, mebooksfree.com D. Thought insertion it is a hallucination. In autoscopic hallucination, 37. A man had a fight with his neighbor. The next day patient sees himself in the mirror and feels that he started feeling that police is following him and “he” is the “image” i.e. what he is seeing is not his brain is being controlled by radio waves by only an image but him. his neighbor. The history is suggestive of which 8. A. Illusion. psychiatric sign/symptom: (AIIMS 1999) 9. A. Illusion. A. Thought insertion Illusion is false perception of a real object. B. Somatic passivity 10. B. It occurs in inner subjective space. Hallucinations C. Delusion of persecution occur in outer and objective space; pseudohal­ D. Obsession lucinations occur in inner and subjective space. 11. None > B. mebooksfree.com mebooksfree.com 38. Healthy thinking includesmebooksfree.com all of the following mebooksfree.com All the statements are correct. However,mebooksfree.com if one has mebooksfree.com except: (AIIMS 2011) to chose, the best answer would be B (sensory A. Continuity B. Constancy organs are not involved) as rest three options C. Organization D. Clarity form the criterion of hallucinations. 12. B. Temporal lobe. The lesions of temporal lobe can Insight cause all types of hallucinations and formed 39. The awareness regarding the disease in mental visual hallucinations (elaborate visual hallucina­ status examination is known as: tions) should raise a strong doubt of an organic (AIIMS Nov 2012, May 2013) cause, specifically a temporal lobe pathology. A. Insight B. Orientation 13. D. Prominent visual hallucinations. The presence C. Judgment D. Rapport of prominent visual hallucinations is a strong pointer towards an organic cause (i.e. a distur­ 40. Impaired insight is found in: (PGI 1997) mebooksfree.com mebooksfree.comA. Acute psychosis mebooksfree.com mebooksfree.combance of brain parenchyma such mebooksfree.comas tumors). mebooksfree.com B. Schizophrenia 14. B. At the end of sleep. While getting up. C. Anxiety disorder 15. A. Hypnagogic hallucinations. These occur while D. Obsessive compulsive disorder “going” to sleep. Jactatio capitis nocturna, or rhythmic movement disorder is a neurological 41. If a person is asked, “what will he do if he sees a disorder characterized by involuntary move­ house on fire”? Then what is being tested in that ments, usually of head and neck, before and person? (DNB NEET 2014-15) during the sleep. A. Social Judgment B. Test Judgment 16. D. Anxiety. C. Response Judgment D. None 17. A, B, C, E. Olfactory hallucinations can be seen in temporal ANSWERS lobe epi-lepsy, medial temporal sclerosis (which 1. D. All of the above is a common cause of epilepsy). Though rare, mebooksfree.com mebooksfree.com 2. A. Anhedonia. Anhedoniamebooksfree.com is seen in both depres­ mebooksfree.comolfactory hallucinations can alsomebooksfree.com be present in mebooksfree.com sion as well as schizophrenia. schizophrenia and Alzheimer’s disease. 3. D. Inability to recognize and describe feelings. 18. A, D, E. 4. C. Labile affect. Visual hallucinations are the most common type 5. B. Frontal lobe. The neuroanatomical substrate for of hallucinations in delirium. Temporal lobe gene­ration of emotions is limbic system however epilepsy can present with all types of hallucina­ the regulation/control of emotions is a function tions including visual hallucinations. In hebe­ of frontal lobe. phrenic schizophrenia, the primary symptom is

mebooksfree.com mebooksfree.com mebooksfree.com mebooksfree.com mebooksfree.com mebooksfree.com

mebooksfree.com mebooksfree.com mebooksfree.com mebooksfree.com mebooksfree.com mebooksfree.com mebooksfree.com mebooksfree.com mebooksfree.com mebooksfree.com mebooksfree.com mebooksfree.com

Basics 9

disorganized behavior and formal though dis­ 31. D. Conversion disorder. Conversion disorder is a orders however hallucinations can also be seen. neurotic disorder (described in later chapters). 19. D. Synesthesia. Delusion is not a feature of conversion disorder. 20. C. Thought. 32. None > A. mebooksfree.com mebooksfree.com 21. E. Loosening of associationmebooksfree.com is logically connected mebooksfree.com Delusion can be seen in schizophrenia,mebooksfree.com mania, mebooksfree.com thoughts with loss of goal. In loosening of asso­ depression as well as OCD. However the best ciation, the connec-tions between the thought is answer here would be OCD, as delusions are lost. The rest of the statements are true. Verbige­ rarely seen in OCD. ration is a senseless repetition of one or several 33. A, B, C, D. sentences or phrases. For example, a patient Delusions can be seen in all these disorders. Mel­ continued to repeat the following sentences for ancholic depression is usually seen in elderlies. hours “Life is great. The lord is great. Summer 34. B, C. will come soon” Its an example of verbigeration. Delusion of grandiosity can be seen in paranoid Vorbeireden or vorbeigehen is seen in Ganser’s schizophrenia­ and schizoaffective disorders. syndrome (described in later chapters) and is Delusion of grandiosity can be seen in mania but another name for approximate answers in which not in hypomania. 35. B, C, D. mebooksfree.com mebooksfree.compatient reaches closemebooksfree.com to the right answer, but mebooksfree.com mebooksfree.com mebooksfree.com Nihilistic delusions can be seen in paranoid never gives the right answer. schizophrenia, Cotard’s syndrome and depres­ 22. A. Persistent and inappropriate repetition of the sion. same thoughts. 36. A. Delusion of persecution. 23. A. Formal thought disorders are characteristic 37. C. Delusion of persecution. Here, in the question the abnormalities in schizophrenia. In schizophre­ history for delusion of persecution (i.e police is nia, the abnormalities of affect, perception, motor following) is clear. The second half where patient system as well as thought are present, however feels that his mind is being controlled by radio the characteristic abnormality in schizophrenia waves is suggestive of possible though alienation is that of thought, and more specifically the form phenomenon but we have not been provided of thought (known as formal thought disorder). with any further details. mebooksfree.com mebooksfree.com 24. A. Formal thought disorder.mebooksfree.com 38. mebooksfree.comD. Clarity. Healthy thinking has threemebooksfree.com characte­ mebooksfree.com 25. C. Thought block. ristics (1) Continuity (2) Organization and (3) 26. D. All of the above. Prolixity is a milder form of “flight Constancy. of ideas”. As mentioned in the text, flight of ideas 39. A. Insight can be considered as both a disorder of stream 40. A, B. of thought and form of thought. Only first two options are psychotic illnesses in 27. A. Formal thought disorder is seen only in schizo­ which insight is impaired. phrenia and not in depression. Rest all options 41. B. Test Judgment. In mental status examination, the can be present in either of the illnesses. judgment of the patient is also described. Patient 28. B. Thought. Delusion is a disorder of content of is given hypothetical scenarios such as “you see thought. that a house is on fire” or “you find a letter lying 29. D. Superstition. There are many beliefs which are on the road” and is asked “what will you do”. This mebooksfree.com mebooksfree.comfalse and are sharedmebooksfree.com by whole communities mebooksfree.comis called “test judgment” as patient’smebooksfree.com judgment is mebooksfree.com e.g. black magic, witches etc. These beliefs are being tested in a hypothetical scenario. There are considered as superstitions. In comparison, other forms of judgment like “social judgment” delusions are not shared by members of the which describes whether a person is able to inter­ same sociocultural background. For example, if a act socially in an appropriate manner. Finally, in villager starts claiming that he is lord hanuman, “personal judgment”, patient is asked about his no one in his village will share his belief. future plans and it is assessed whether he has a 30. C. Content of thought. logical plan for his future or not.

mebooksfree.com mebooksfree.com mebooksfree.com mebooksfree.com mebooksfree.com mebooksfree.com

mebooksfree.com mebooksfree.com mebooksfree.com mebooksfree.com mebooksfree.com mebooksfree.com mebooksfree.com mebooksfree.com mebooksfree.com mebooksfree.com mebooksfree.com mebooksfree.com

mebooksfree.com mebooksfree.com mebooksfree.com mebooksfree.com mebooksfree.com mebooksfree.com

Chapter Schizophrenia Spectrum and 2 Other Psychotic Disorders

Schizophrenia is the prototype of psychotic disorders. It is C. Affect disturbances: Disturbances of emotions such as one of the most common serious mental disorders. inappropriate affect. mebooksfree.com mebooksfree.com mebooksfree.com D. Associationmebooksfree.com disturbances: Disturbances mebooksfree.comof association mebooksfree.com HISTORY of thoughts such as formal thought disorders. Emil Kraepelin Kurt Schneider Kraepelin classified psychiatric illnesses into two clini­ Schneider described a group of symptoms, popularly Q cal types: Dementia Praecox and Manic Depressive known as Schneiderian First Rank Symptoms (SFRS)Q Q Illness . The basis of this classification is the course of which were frequently seen in patients of schizophrenia illness and the cognitive decline. and were characteristic of the illness. It must be however Dementia Praecox is characterized by a chronic and remembered that these symptoms can also be present in deteriorating course along with gradual decline of cog- other illnesses and hence are not specific or pathogno­ nitive functions (i.e. gradual decline of memory, atten­ monic of schizophrenia. There are11 Schneiderian First mebooksfree.com mebooksfree.comtion and goal directed behavior).mebooksfree.com The term “dementia” Rank Smebooksfree.comymptoms. mebooksfree.com mebooksfree.com was used to indicate gradual decline in cognitive func­ A. Three thought phenomenon: These three together are tions and the term “praecox’ was added since the onset known as thought alienation phenomenon in which of illness was in young age (praecox means early onset). patient feels as if some one is tampering with his mind In contrast Manic Depressive illness is characterized and thoughts. The thought alienation includes the fol­ by distinctQ episodes of illness alternating with period lowing: of normal functioning. Also, there is no cognitive • Thought insertion (patient reports that someone is decline. putting thoughts in his mind) • Thought withdrawal (patient experiences that Eugen Bleuler thoughts are being taken out of his mind) • Thought broadcast (patient experiences that Q Bleuler coined the term “Schizophrenia” , which thoughts are leaving his mind and that others are mebooksfree.com mebooksfree.comreplaced dementia praecox inmebooksfree.com scientific literature. Bleuler mebooksfree.comable to access his thoughts, e.g. patientmebooksfree.com would say mebooksfree.com proposed four symptoms which he called as fundamental that “everybody understands my thoughts, though (or primary) symptoms of schizophrenia. These symp­ I never say anything”. toms are also known as 4 A’s of BleulerQ. They include: B. Three made phenomenon: Here the patient experi­ A. Autistic thinking and behavior (Autism): Excessive ences that his emotions, actions and drives are being fantasy thinking which is irrational and withdrawn influenced by others. It includes the following: behavior. • Made volition: The patient experiences that his B. Ambivalence: Marked inability to take a decision. actions are being controlled by an external agency

mebooksfree.com mebooksfree.com mebooksfree.com mebooksfree.com mebooksfree.com mebooksfree.com

mebooksfree.com mebooksfree.com mebooksfree.com mebooksfree.com mebooksfree.com mebooksfree.com mebooksfree.com mebooksfree.com mebooksfree.com mebooksfree.com mebooksfree.com mebooksfree.com

Schizophrenia Spectrum and Other Psychotic Disorders 11

and not by himself. For example, a patient would said “I will have dinner at a restaurant tonight”. repeatedly put his hand in the fan, and on asking The German word Gedankenlautwerden“ ” or the the reason reported, “I don’t want to do it myself french word “echo de pensees” is occasionally but I am being controlled by aliens who can mani­ used to describe these audible thoughts. mebooksfree.com mebooksfree.compulate my actions, I ammebooksfree.com a robot for them and they D. Somaticmebooksfree.com passivity: In somatic passivity, mebooksfree.compatient expe­ mebooksfree.com have my remote control”. riences tactile or visceral hallucinations which he • Made affect: The patient experiences that someone believes are being imposed by some external agent. is changing his affect (emotions). For example, a For example, a patient reported that he feels intense patient reported “at times I start laughing loudly burning sensation inside his right knee and claimed and at times I cry. The neighbours control my emo­ that it is because of UV rays sent by FBI agents from tions, they can change it whenever they want to. I New York”. feel helpless”. E. Delusional perception: In Delusional perception, a • Made impulses: The patient experiences that some­ delusion is attached to a normal perception. For exam­ one is putting certain “drives” in his mind. For ple, a patient of schizop­hrenia looked at the ceiling fan example, a patient suddenly threw his coffee mug and immediately understood that the “all the people onto a nurse. On asking about it he reported “a sud­ in the city consider him a homosexual”. In this exam­ mebooksfree.com mebooksfree.comden impulse came overmebooksfree.com me, this impulse was sent plemebooksfree.com there was a normal perception in themebooksfree.com first step (i.e. mebooksfree.com by CBI officers who wanted me to throw the mug. the patient saw a ceiling fan) and in the second step a I tried resisting the impulse, but could not control delusion was attached to this normal perception (i.e. i t ”. the delusion that everybody in city considers patient C. Three auditory hallucinations: a homosexual). Delusional perception is a type of • Voices arguing or discussing: The patient reports “primary delusion”Q. Primary delusions are those hearing of two or more voices which argue or delusions which arise directly as a result of morbid discuss about the patient. The patient is usually psychological processes whereas secondary delusions referred to in third person (hence also called third develop secondarily to some other psychopathologi­ person auditory hallucinationsQ). For example, cal phenomenon. For example, a patient who had the first voice would say “he is a strange man, he continuous auditory hallucinations of a voice which doesn’t have any good qualities”. The second voice said “you will be killed”, started believing that “some­ would respond “yes, also look how fat he has mebooksfree.com mebooksfree.com mebooksfree.com bodymebooksfree.com wants to harm me”. Now, this “delusionmebooksfree.com of perse­ mebooksfree.com become”. In this example the patient is hearing two cution” which developed is a secondary delusion as it voices and the voices are using the word “he” to developed secondarily to the auditory hallucinations. refer to the patient, hence patient is being referred to in third person. • Voices commenting on patient’s action: Here, the EPIDEMIOLOGY patient hears voices which give a running com­ The lifetime prevalence of schizophrenia is 1% whereas mentary on the patient’s activities. For example, the point prevalence is 0.5–1%. Theincidence rate is a patient who was working in the kitchen heard 0.15–0.25 per thousand. the following voice “she has peeled the potato and A. Prevalence in specific population: Schizophrenia has now she is about to switch on the gas. Now, she high heritability­ Q. The prevalence in general popula­ has started to wash the potatoes”. The voice usually tion is 1% however in relatives of patients, the rate mebooksfree.com mebooksfree.comrefers to the patient in mebooksfree.comthird person, hence this can is higher.mebooksfree.com The following table mentionsmebooksfree.com the rates for mebooksfree.com again be an example of third person auditory hal­ specific population groups. lucinations. The usual age of onset of schizophrenia isadoles - • Audible thoughts: Here the patients hears a voice, cenceQ and young adulthood. When the onset occurs which would say aloud whatever patient would after age of 45 years, the disorder is called as late-onset think. For example, a patient had a thought that “I schizophreniaQ. will have dinner at a restaurant tonight”. Immediately It is equally prevalent in men and women, however he heard a voice of a middle aged women who the onset is earlier in men.

mebooksfree.com mebooksfree.com mebooksfree.com mebooksfree.com mebooksfree.com mebooksfree.com

mebooksfree.com mebooksfree.com mebooksfree.com mebooksfree.com mebooksfree.com mebooksfree.com mebooksfree.com mebooksfree.com mebooksfree.com mebooksfree.com mebooksfree.com mebooksfree.com

12 Review of Psychiatry

Table 1: Prevalence of Schizophrenia in specific populations. C. Neuropathological factors: The neuropathology of • General: 1% schizophrenia is still not clear. Abnormalities have been found in various structures, such as: • Non twin sibling of a schizophrenia patient: 8% • Cerebral ventricles: Reduction in cortical gray matter mebooksfree.com mebooksfree.com• Dizygotic twin of a schizophrenicmebooksfree.com patient: 12% mebooksfree.comvolume and enlargement of lateral andmebooksfree.com third ven­ mebooksfree.com • Monozygotic twin of a schizophrenic patient: 47% tricles has been consistently observed. • Child with one parent with schizophrenia: 12% • Limbic system: Abnormalities in limbic system • Child with both parents with schizophrenia: 40% components such as hippocampus (smaller in size and functionally abnormal), amygdala (smaller size) and parahippocampal gyrus (smaller size) Schizophrenia is more prevalent in lower socioeconomic have been observed. status. It was earlier believed that different body types • Prefrontal cortex: Anatomical abnormalities have were related to different personalities and also had dif­ been found. ferent vulnerability to some disorders. Three types of body • Thalamus: Neuronal loss especially in medial dor­ types were described: asthenic (thin and weak), athletic sal nucleus of thalamus. Q (muscular) and pyknic (short and fat). The asthenic • Basal ganglia and cerebellum: Abnormalities have mebooksfree.com mebooksfree.comand to a lesser extent athletic mebooksfree.compersons were believed to be mebooksfree.combeen reported without any conclusivemebooksfree.com proof. mebooksfree.com predisposed for development of schizophrenia whereas the pyknic were believed to be predisposed to manic SYMPTOMS depressive illness (bipolar disorder). The symptoms of schizophrenia can be divided into vari­ ETIOLOGY AND PATHOGENESIS ous symptom complexes, described as follows: A. Positive symptoms A. Genetic factors: (or psychotic symp­ H In DSM-4, the presence of • Schizophrenia has a genetic contribution as toms): The two bizarre delusions was considered enough to satisfy the Criterion A for reflected by higher monozygotic concordance rate positive symptoms schizophrenia, however in DSM-5, the than dizygotic concordance rate. Several genes include delusions concept of bizarre delusions has been and . removed and it no longer carries any appear to make a contribution to schizophrenia hallucinations special diagnostic signi­ficance. and nine linkage sites have been identified: 1q, 5q, They respondwell mebooksfree.com mebooksfree.com6p, 6q, 8p, 10p, 13q, 15qmebooksfree.com and 22q. to mebooksfree.commedications and the presence of positivemebooksfree.com symp­ mebooksfree.com Q • Several candidate genes contributing to schizo­ toms is a good prognostic factor in schizophrenia. phrenia have been identified, and they include • Delusions: The most common delusion in schizo­ a-7 nicotinic receptor, DISC 1 (Disrupted in schizo­ phrenia is delusion of persecution. A category of phrenia), COMT (catechol-o-methyl transferase), delusion that holds special significance in schizo­ phrenia is the so called . NRG 1 (Neuregulin 1), GRM-3 (Glutamate recep­ “bizarre delusions” Bizarre delusions are those that are considered tor metabotropic), RGS-4 (Regulator of G protein physically impossible and culturally implausible signalling) and DAOA (or G-72) (D-Amino acid (or ununderstandable). For example, “a patient oxidase activator). claimed that he has been sent by aliens from mars B. Biochemical factors: and his purpose is to evaporate all the water from • Dopamine hypothesis: This hypothesis proposes earth and make it dry”. This patient is having a Q mebooksfree.com mebooksfree.comthat excess of dopaminergicmebooksfree.com activity is responsi­ mebooksfree.combizarre delusion as his belief is bothmebooksfree.com impossible mebooksfree.com ble for schizophrenia. and ununderstandable. • Serotonin: Currently, along with dopamine, an • Hallucinations: The most common hallucinations excess of serotonin is also considered to be respon­ in schizophrenia are auditory hallucinationsQ. sible for symptoms of schizophrenia. Visual hallucinations are the second most com­ • Other neurotransmitters like GABA, glutamate, mon, however the presence of visual hallucination norepinephrine, acetylcholine, nicotine have also should always raise the suspicion of an organic been implicated in pathogenesis of schizophrenia. mental disorder.

mebooksfree.com mebooksfree.com mebooksfree.com mebooksfree.com mebooksfree.com mebooksfree.com

mebooksfree.com mebooksfree.com mebooksfree.com mebooksfree.com mebooksfree.com mebooksfree.com mebooksfree.com mebooksfree.com mebooksfree.com mebooksfree.com mebooksfree.com mebooksfree.com

Schizophrenia Spectrum and Other Psychotic Disorders 13

• The positive symptoms of schizophrenia are due • Stupor: Extreme hypoactivity or immobilityQ and to dopamine excess in mesolimbic tract (neural minimal responsiveness to stimuli. pathway from ventral segmental area to nucleus • Excitement: Extreme hyperactivity which is usually accumbens)Q. non goal directed (i.e. the patient is very active but mebooksfree.com mebooksfree.com B. Negative symptoms: Negativemebooksfree.com symptoms represent mebooksfree.comdoesn’t do any meaningful work). mebooksfree.com mebooksfree.com “loss of normal functions” in patients with schizo­ • Posturing/catalepsy: Spontaneous maintenance of phrenia. These symptomsrespond poorly to medica- posture for long periods of time. tions and their presence is a bad prognostic factorQ • Waxy flexibility: When examiner makes a passive in schizophrenia. Following are the negative symp­ movement on patient, there is a feeling of plastic toms: resistance which resembles bending of a soft wax • Avolition: Loss of will or drive to indulge in goal candle. directed activities (such as grooming and hygiene, • Automatic obedience: Excessive cooperation with education and occupational activities). examiner’s commands despite unpleasant conse­ • Apathy: Loss of concern for an idea or task or quences. For example, a patient kept on protruding results. For example, a student who had deve­ his tongue in response to examiner’s commands, loped schizophrenia failed in exams. However he despite the fact that his tongue would be pricked appeared unconcerned with his results. mebooksfree.com mebooksfree.com mebooksfree.com mebooksfree.comby a pin everytime he protruded it. mebooksfree.com mebooksfree.com • Anhedonia: Loss of ability to derive pleasure from • Echolalia: Mimicking of examiner’s speech. activities or relationships. • Echopraxia: Mimicking of examiner’s movements. • Asociality: Indifference to social relationships and • Negativism: Patient refuses to accept examiner’s decrease in the drive to socialize. instructions or any attempts to move him. • Affective flattening (or blunting): Inability of patient • GrimacingQ: Maintenance of odd facial expres- to under- stand emotions of others and inability to sions. express own emotions. • Stereotypy: Spontaneous repetition of odd, pur- • Alogia: Decrease in verbal communication. poseless movements. For example, making strange The negative symptoms are due to decreased movements of fingers repeatedlyQ. dopamine activity in mesocortical pathway (neu­ • Gegenhalten: Resistance to passive movement, ral pathway from ventral segmental area to prefron­ which is directly proportional to the strength of mebooksfree.com mebooksfree.comtal cortex). mebooksfree.com mebooksfree.com mebooksfree.com mebooksfree.com C. Disorganization symptoms: This symptom complex force applied. includes the following symptoms: • Mannerisms: Spontaneous repetition of odd, pur- • Formal thought disorder: These are the distur­ poseful movements. For example, repeatedly bances in the form of thought characterized by loss saluting the passerby. of organization of thought. • Perseveration: It is an induced movement which is • Disorganized behavior: It is the odd and inap- senselessly repeated. For example, A patient takes propriate behavior which may break the social his tongue out and in, when asked however then norms. For example, a hospitalized schizophrenic keeps on repeating the out and in movement , even patient would masturbate in front of the nursing when he is no longer asked. It must be noted that staff, another patient of schizophrenia would wear perseveration occurs in response to an instruction, sweaters and coats in hot summer season. whereas stereotypy and mannerisms are spontane­ mebooksfree.com mebooksfree.com• Inappropriate affect: Affectmebooksfree.com which is not in sync mebooksfree.comous. Perseveration is also a sign of mebooksfree.combrain damage mebooksfree.com with the social situation. (organic brain disorders)Q. D. Motor symptoms (catatonic symptoms): The term “cat- • Ambitendency: Inability to decide the desired motor atonia” was given by Karl KahlbaumQ who described movement. For example, when offered a hand for these motor symptoms for the first time. These symp­ handshake, patient may repeatedly bring his hand toms are sometimes described along with disorgani­ forward and backward as he is not able to decide zation symptoms. For more clarity, they have been whether he wants to shake the hand or not. It is described separately here. These include: ambivalence in motor movementsQ.

mebooksfree.com mebooksfree.com mebooksfree.com mebooksfree.com mebooksfree.com mebooksfree.com

mebooksfree.com mebooksfree.com mebooksfree.com mebooksfree.com mebooksfree.com mebooksfree.com mebooksfree.com mebooksfree.com mebooksfree.com mebooksfree.com mebooksfree.com mebooksfree.com

14 Review of Psychiatry DIAGNOSIS E. Residual schizophrenia: Residual schizophrenia is characterized by progression from an early stage (with According to DSM-5, two or more of the following symp­ prominent delusions and hallucinations) to a later toms should be present for a duration of 1 month period stage where the delusions and hallucinations have mebooksfree.com mebooksfree.comand at least one of these mustmebooksfree.com be either (1), (2) or (3) becomemebooksfree.com minimal and mostly negative symptomsmebooksfree.com are mebooksfree.com 1. Delusions present. 2. Hallucinations F. Simple schizophre­ H 3. Disorganized speech (or formal thought disorder) DSM-5 Update: The DSM-4 nia: There are des­cribed multiple subtypes of schizo- 4. Disorganized or catatonic behavior prominent negative phrenia (like paranoid, catatonic, dis- 5. Negative symptoms. organized, catatonic, undifferentiated, symptoms without residual). The DSM-5 has eliminated The total duration of illness should be at least any history of posi­ all of them and does not describe any subtypes. 6 months, and the 6 months period must include at least tive symptoms like one month of above mentioned symptoms. delusion and hallucinations. It has the worst prog- The ICD-10 also H nosis. uses similar criterion DSM-5 Update: In DSM-4, only one G. Post schizophrenic depression: A depressive episode for diagnosis of schizo­ of the above symptoms was required if which deve­lops after the resolution of schizophrenic phrenia however the the delusions were bizarre or halluci- mebooksfree.com mebooksfree.com nationsmebooksfree.com were one of schneiderian first symptoms.mebooksfree.com This disorder is associatedmebooksfree.com with an mebooksfree.com total duration of symp­ rank symptoms (either voices discuss- increased risk of suicide. toms should be more ing about the patient or voices giving a running commentary). However in than one month unlike DSM-5 this special attribution to bizarre Other Classifications DSM-5 which requires delusions and schneiderian auditory hallucinations has been removed. a total duration more Apart from ICD-10 and DSM-5, various other classifica­ than six months. tions have been proposed. A. TJ Crow divided schizophrenia into two subtypes, Types namely Type I and Type II schizophrenia: • Type I: Mostly According to ICD-10, the following are the types of positive symp- H schizophrenia: Substances which can cause schi­ toms with nor- zophrenia like symptoms: Ampheta­ A. Paranoid schizophrenia: This type is dominated by mines, cocaine, phencyclidine and mal ventricles, other hallucinogens, cannabis. mebooksfree.com mebooksfree.comhallucinations and delusions.mebooksfree.com This is themost com- mebooksfree.comgood response mebooksfree.com mebooksfree.com mon type of schizophrenia. It has a late onset and to medications and better prognosis. Q a good prognosis . The personality is usually pre- • Type II: Mostly negative symptoms with dilated served (the person is able to maintain daily activities ventricles, poor response to medications and and social interaction is normal). poor prognosis. B. Catatonic schizophrenia: This type is dominated by B. Pfopf schizophrenia: Schizophrenia in a patient with catatonic (motor) symptoms. It has the best progno- mental retardation. sis of all types. The first line treatment for catatonic C. Van Gogh syndrome: Self mutilation (injuring self) schizophrenia includes intravenous lorazepam and occurring in schizophrenia has also been called Van electroconvulsive therapy. Gogh syndrome. C. Hebephrenic (disorganized) schizophrenia: This type is dominated by prominent disorganization symp­ TREATMENT mebooksfree.com mebooksfree.comtoms and negative symptoms.mebooksfree.com It has an early onset mebooksfree.com mebooksfree.com mebooksfree.com and bad prognosis. There is severe deterioration of Antipsychotics (also known as neuroleptics) are the main­ personality (patient is not able to maintain hygiene, stay of treatment for psychotic disorders like schizophre­ social interaction is inappropriate, odd behaviors are nia, schizoaffective disorders, delusional disorders and present). others. Antipsychotics have been divided into two classes: D. Undifferentiated schizophrenia: The schizophrenia not (1) Typical antipsychotics and (2) Atypical antipsychotics conforming to any of the above subtypes or exhibiting 1. Typical antipsychotics or first generation antipsycho­ features of more than one of them. tics or dopamine receptor antagonists (DRAs): These

mebooksfree.com mebooksfree.com mebooksfree.com mebooksfree.com mebooksfree.com mebooksfree.com

mebooksfree.com mebooksfree.com mebooksfree.com mebooksfree.com mebooksfree.com mebooksfree.com mebooksfree.com mebooksfree.com mebooksfree.com mebooksfree.com mebooksfree.com mebooksfree.com

Schizophrenia Spectrum and Other Psychotic Disorders 15

drugs mainly act through dopamine, D2 receptor prophylactic use of oral anticholinergics is suggested antagonism. They were the first antipsychotics that while prescribing typical antipsychotics. were used in the clinical practice. They are effec- • Acute akathisia: It is the commonest side effect tive against positive symptoms but have mini- of antipsychotics and is characterized by an inner mebooksfree.com mebooksfree.commal effecton negative symptomsmebooksfree.com. The therapeutic mebooksfree.comsense of restlessness along with objectivemebooksfree.com, observ- mebooksfree.com effect of improvement in psychotic symptoms is able movements such as fidgetingQ of legs, pacing mediated by D2 receptor antagonism in mesolim­ around, inability to sit or stand in one place for a bic tract. The typical antipsychotics can further be long time. The treatment options includeb block- classified according to their chemical groups, as ersQ such as propranolol (drug of choice), anticho- described below: linergics and benzodiazepines. The antipsychotic • Phenothiazines: Chlorpromazine, Thioridazine, Tri­ can also be changed to a second generation or low fluoperazine, Prochlorperazine, Triflupromazine, potency first generation antipsychotics, which have Fluphenazine, Perphenazine­ lesser incidence of akathisia. • Thioxanthenes: Thiothixene, flupenthixol • Drug induced parkinsonism: It is characterized • Butyrophenones: Haloperidol, droperidol, penfluri­ by the triad of rigidity, bradykinesia and resting dol tremors. The treatment options include use of mebooksfree.com mebooksfree.com• Miscellaneous: Pimozide,mebooksfree.com loxapine, molindone. mebooksfree.comanticholinergics or change of antipsychoticsmebooksfree.com to mebooksfree.com The typical antipsychotics can further be classified second generation or low potency first generation as low potency (like chlorpromazine, thioridazine) antipsychotics. The dose reduction can also be tried. and high potency (like haloperidol and fluphenazine). Often, use of prophylactic anticholinergics prevents Apart from differing in potency, the low potency and high the development of drug induced parkinsonism. potency antipsychotics also differ in their side effects • Tardive dyskinesia: The term “tardive” refers to profile. The common side effects of typical antipsychotics features which develop after prolonged exposure. are as follows: Tardive dyskinesia develops after long-term treat­ A. Movement disorders: The antipsychotics can cause vari­ ment with antipsychotics and can present as invol­ ous movement disorders, which collectively are often untary movements of the tongue (e.g. twisting, referred as extrapyramidal symptoms (or extrapy- protrusion), jaw (e.g. chewing), lips (e.g. smacking, ramidal side effects). These side effects are caused by puckering), trunk or extremities. Patient may also mebooksfree.com mebooksfree.comblockade of dopamine receptorsmebooksfree.com in nigrostriatal tract mebooksfree.comhave rapid, jerky movements (choreiformmebooksfree.com move­ mebooksfree.com (neural pathway from substantia nigra to striatum). ments) or slow, sinusoid movements (athetoid The movement disorders aremore commonly seen movements). The management usually includes with typical antipsychotics in comparison to atypical shifting to a second generation medication. antipsychotics and amongst typical antipsychotics, • Neuroleptic malignant syndrome: It is a fatal side high potency typical antipsychotics are more likely effect of antipsychotic use. It is characterized by to cause this side effect. The movement disorders can muscle rigidity, elevated temperature (greater be of the following types: than 38°C), and increased CPK (creatine phos- • Acute dystonia: It is the earliest side effectQ of phokinase) levels. The other symptoms include antipsycho­tics and can be seen within minutes diaphoresis, tremors, confusion, autonomic distur­ of receiving an injectable antipsychotic (also with bances, liver enzyme elevation and leukocytosis. oral antipsychotic). It is characterized by sudden The pathophysiology involves D2 antagonism at mebooksfree.com mebooksfree.comcontraction of a musclemebooksfree.com group and can result in mebooksfree.comvarious levels. The D2 receptors blockademebooksfree.com in corpus mebooksfree.com symptoms like torticollisQ, trismus (contraction striatum causes muscle contraction and rigidity­ of jaw muscles),Q deviation of eye balls that initiates heat generation, whereas blockade (oculogyric crisis due to contraction of extraocular of dopamine receptors in hypothalamus interferes muscles), laryngospasm, etc. The management with heat regulation. The autonomic disturbances includes immediate administration of parenteral are caused by dopamine blockade of spinal neu­ anticholinergicsQ like benztropine, promethazine rons. The increased CPK indicates muscle injury. or diphenhydramineQ. To prevent acute dystonia, The early recognition of symptoms and prompt

mebooksfree.com mebooksfree.com mebooksfree.com mebooksfree.com mebooksfree.com mebooksfree.com

mebooksfree.com mebooksfree.com mebooksfree.com mebooksfree.com mebooksfree.com mebooksfree.com mebooksfree.com mebooksfree.com mebooksfree.com mebooksfree.com mebooksfree.com mebooksfree.com

16 Review of Psychiatry

withdrawal of antipsychotics is of paramount earlier, however the incidence is lesser in comparison impor­tance, otherwise the continuing muscle dam­ to the typical antipsychotics. age can cause myoglobinuria and renal failure. B. Endocrine side effects: The incidence of hyperpro­ The treatment includes skeletal muscle relax­ lactinemia is also lesser with atypical antipsychotics mebooksfree.com mebooksfree.comants like dantroleneQmebooksfree.com, dopamine agonists such (exceptmebooksfree.com risperidone and amisulpride whichmebooksfree.com have a mebooksfree.com as amantadine and bromocriptine are also useful. higher incidence). Supportive measures including adequate hydration C. Weight gain and increased risk of dyslipidemia, dia­ are also important in the management. When drug betes and cardiovascular disease is more commonly treatment with antipsychotics is restarted, second seen with atypical antipsychotics in comparison to generation antipsychotics should be used. typical antipsychotics. B. Endocrine side effects: The blockage of dopamine D. Other side effects include sedation, QTc prolongation receptors in tuberoinfundibular tract results in and seizures. hyperprolactinemia (remember dopamine inhibits (especially with ziprasidone) prolactin secretion and hence dopamine blockade causes hyperprolactinemia) and can cause galactor­ Clozapine rhea, menstrual disturbances in females and impo­ It was the first atypical antipsychotic to be synthesized. mebooksfree.com mebooksfree.comtence in males. mebooksfree.comClozapinemebooksfree.com is the drug of choice in treatmentmebooksfree.com resistance mebooksfree.com C. Sedation, orthostatic hypotension and anticholinergic schizophrenia. Clozapine is a unique drug as unlike side effects are usually see with low potency typical other antipsychotics, it has a relatively low affinity for D2 antipsychotics. receptors. This low affinity for D2 receptor explains lack 2. Atypical antipsychotics or second generation anti­ of extrapyramidal side effects on clozapine. Clozapine has psychotics or serotonin dopamine antagonists: These a strong affinity for D4 receptors and also acts as an drugs act through anta- gonism of 5HT 2 receptors as antagonist at 5 HT2A, D1, D3 and a (alpha) adrenergic well of D2 receptors. These drugs have a higher ratio receptors. The lack of extrapyramidal symptoms, makes of 5 HT2 to D2 blockade, in contrast the typical anti­ clozapine a preferred antipsychotic in patients who are psychotics primarily act on D2 receptors. Due to lesser intolerant to other antipsychotics­ because of extrapyrami­ D2 blockade, atypical antipsychotics have lesser risk of dal side effects including tardive dyskinesia. causing extrapyramidal side effects as well as hyper­ prolactinemia. Atypical antipsychotics are effective in Side effects: The common side effects of clozapine include mebooksfree.com mebooksfree.comtreatment of both positivemebooksfree.com and negative symptoms. sedation,mebooksfree.com syncope, hypotension, tachycardia,mebooksfree.com nausea and mebooksfree.com The following drugs are classified as atypical anti­ vomiting. Other side effects include weight gain (clozapine psychotics: causes highest weight gain amongst all antipsychotics), • Clozapine constipation, anticholinergic side effects. A particularly • Olanzapine problematic side effect issialorrhea or hypersalivation. • Risperidone Clozapine can also cause life threatening side effects • Paliperidone which include agranulocytosis, seizures and myocar- • Iloperidone ditis. In view of possibility of agranulocytosis, during the • Quetiapine first six months of clozapine treatment, WBC and neu­ • Ziprasidone trophil counts should be measured every week. Also, if • Aripiprazole during the therapy, WBC counts fall below 3000/mm3 • Sertindole or neutrophil counts fall below 1500/mm3, the cloza­ mebooksfree.com mebooksfree.com• Zotepine mebooksfree.compine therapymebooksfree.com should be stopped. The agranulocytosismebooksfree.com and mebooksfree.com • Lurasidone myocarditis are dose independent side effects of cloza­ • Asenapine pine whereas seizures are dose dependent­ Q (seen only • Amisulpride at higher dosages). The side effect profile of atypical antipsychotics is as The only contraindication to clozapine use is aWBC follows: count of less than 3500/dL at the time of starting cloza­ A. Movement disorders: Atypical antipsychotics can pine, a history of agranulocytosis during clozapine treat­ cause all kind of extrapyramidal side effects described ment or use of other drug that is known to suppress the

mebooksfree.com mebooksfree.com mebooksfree.com mebooksfree.com mebooksfree.com mebooksfree.com

mebooksfree.com mebooksfree.com mebooksfree.com mebooksfree.com mebooksfree.com mebooksfree.com mebooksfree.com mebooksfree.com mebooksfree.com mebooksfree.com mebooksfree.com mebooksfree.com

Schizophrenia Spectrum and Other Psychotic Disorders 17

bone marrow (e.g clozapine and carbamazepine cannot 3. Simple, disorganized, undifferentiated subtype be given together as both are bone marrow suppressants. 4. Male sex 5. Prominent negative symptoms Specific Points about Antipsychotics 6. Absence of affective symptoms mebooksfree.com mebooksfree.com mebooksfree.com 7. Familymebooksfree.com history of schizophrenia. mebooksfree.com mebooksfree.com A. Long acting injectable antipsychotics (Depot anti­ psychotics): In patients who have poor complianceQ with medications (i.e who refuse to take medications) OTHER PSYCHOTIC DISORDERS long acting injectable antipsychotics can be used. A. Acute psychotic disorders: There are disorders which The patients typically receives theintramuscular have symptoms (e.g. delusions, hallucinations and injections of antipsychotics once a month or once a disorganisation symptoms) similar to schizophrenia, fortnight. Long acting injectable preparations are however do not meet the duration criterion. These available for following antipsychotics: disorders have been classified separately in DSM-5 • Flupenthixol and ICD-10. These disorders frequently are preceded • Fluphenazine by a stressor (stressful life event), have an acute onset • Haloperidol and often resolve completely. These disorders may mebooksfree.com mebooksfree.com• Pipotiazine mebooksfree.com alsomebooksfree.com be precipitated by feverQ. mebooksfree.com mebooksfree.com • Zuclopenthixol In ICD-10, if the symptoms (delusions, hallucina­ • Risperidone tions, disorganization) are present for less than one • Olanzapine month, a diagnosis of acute and transient psychotic • Paliperidone disorder is made. • Aripiprazole In DSM-5, if symptoms (delusions, hallucinations, B. Thioridazine can cause irreversibleretinal pigmenta- disorganisation) are present for less than one month, Q tion . Thiori­dazine can also cause cardiac arrhyth- a diagnosis of brief psychotic disorder is made; and Q mias (prolongation of QT interval). It is also the if symptoms last between 1-6 months, a diagnosis of drug with least extrapyramidal side effects amongst schizophreniform disorder is made. typical antipsychotics, overall clozapine is the anti­ Treatment: Antipsychotics and benzodiazepines are psychotic with least extrapyramidal side effect. used for the treatment of acute psychotic disorders. C. Chlorpromazine can cause corneal and lenticular B. Schizoaffective disorder: Schizoaffective disorder has mebooksfree.com mebooksfree.comQ mebooksfree.com mebooksfree.com mebooksfree.com mebooksfree.com deposits . features of both schizophrenia and mood disorders D. Penfluridol is the Q. longest acting antipsychotic concurrently. Depen­ding on whether manic episode or E. Ziprasidone is known to cause cardiac arrhythmias depressive episode is present along with schizophrenia (prolonga- tion of QT interval). symptoms, there are two subtypes: F. Aripiprazole is a partial agonist at D2 receptors (all • Schizoaffective disorder (Bipolar type or manic other antipsychotics are D2 antagonists). type): With manic symptoms • Schizoaffective disorder (Depressive type): With PROGNOSIS depressive symptoms. Good prognostic factors: Treatment: It involves combination of mood stabilis­ 1. Acute or abrupt onset ers, antipsychotics and antidepressants depending 2. Late onset (age > 35 years)Q on the presentation. In schizoaffective (manic type mebooksfree.com mebooksfree.com 3. Catatonic subtype and paranoidmebooksfree.com subtype episodes)mebooksfree.com a combination of antipsychoticsmebooksfree.com and mood mebooksfree.com 4. Female sex stabiliser is commonly used. In schizoaffective 5. Prominent positive symptoms (depressive type episodes) a combination of antipsy­ 6. Presence of affective symptoms (such as depressionQ) chotics, and antidepressants is often used. 7. Family historyQ of mood disorder. C. Delusional disorder: These disorders are characterized Bad prognostic factor: by development­ of either a single delusion or a set 1. Insidious onset of related delusions, which are usually persistent and 2. Early onset (age <20 years) sometimes are life long. Other psychotic symptoms

mebooksfree.com mebooksfree.com mebooksfree.com mebooksfree.com mebooksfree.com mebooksfree.com

mebooksfree.com mebooksfree.com mebooksfree.com mebooksfree.com mebooksfree.com mebooksfree.com mebooksfree.com mebooksfree.com mebooksfree.com mebooksfree.com mebooksfree.com mebooksfree.com

18 Review of Psychiatry

like hallucinations, disorganization, negative symp­ –– Fregoli syndrome: Patient believes that familiar toms are usually absent. If hallucinations occur they persons are taking the guise of strangers. For are for a very short duration, presence of frequent hal­ example, a patient saw a beggar, and claimed lucinations goes against the diagnosis of delusional that his brother is following him in the guise of mebooksfree.com mebooksfree.comdisorder. The following mebooksfree.comare the types of delusional mebooksfree.comthe beggar. mebooksfree.com mebooksfree.com disorder: –– Syndrome of inter metamorphosis: Patient • Persecutory type: Delusion of persecution. believes that people can undergo changes in • Jealous type: Delusion of infidelity. physical and psychological identity and become • Erotomanic type: Delusion of love. a different person altogether. • Somatic type: Patient may have delusion that he is –– Syndrome of subjective doubles: Patient believes that he has many doubles who are living life of infested by parasites (delusional parasitosis), that their own. he has misshaped body parts (delusion of dysmor­ D. Shared psychotic disorders (or induced delusional phophobia) or that his body has a foul odor (delu- disorder): This disorder is characterized by spread of sion of halitosis). delusions from one person to another. The individual • Grandiose type: Delusion of grandiosity. who has the delu­sion (the primary case) is typically • Unspecified type: In patients where the above men­ the influential member of close relationship with a mebooksfree.com mebooksfree.comtioned categories are notmebooksfree.com applicable. Delusion of moremebooksfree.com suggestible person (the secondarymebooksfree.com case) who mebooksfree.com misidentification is an example of unspecified type. also develops the delusion. When two people are Delusion of misidentification can be of many types involved, the term “folie a deux” is used. Occasionally like: more than two individuals are involved (known as –– Capgras syndrome: Patient believes that a folie a trois, folie a quatre, etc). familiar person has been replaced by an impos­ The patients of delusional disorder are usually able tor. For example, a patient believed that his to function normally in domains which are unaf- wife has been H fected by the delusion. For example, a patient with replaced by a DSM-5 update: The DSM-4 requi­ delusion of infidelity may incessantly doubt his wife red that the delusions should be non stranger who bizarre, however DSM-5 has removed and fight with her, however he may be perfectly nor­ looks exactly this condition from the diagnosis of mal at work place. like his wife. delusional disorders. Treatment: Antipsychotics are the drug of choice. mebooksfree.com mebooksfree.com mebooksfree.com mebooksfree.com mebooksfree.com mebooksfree.com QUESTIONS AND ANSWERS

QUESTIONS 4. Bleuler’s symptoms of schizophrenia include all except: (PGI Dec 2005) History A. Autism 1. The term “Dementia precox” was coined by: B. Automatism (AI 2008) C. Affect disturbance A. Freud B. Bleuler D. Loosening of association C. Kraepelin D. Schneider E. Ambivalence mebooksfree.com mebooksfree.com mebooksfree.com mebooksfree.com mebooksfree.com mebooksfree.com 2. The term schizophrenia was coined by: 5. Schneiderian First Rank Symptoms are found in: (DNB NEET 2014-15) (PGI Nov 2011) A. Eugen Bleuler B. Emil Kraepelin A. Schizophrenia C. Hecker D. Kurt Schneider B. Organic mental disorders 3. The term “catatonia” was coined by: C. Schizoaffective disorder A. Kahlbaum B. Freud D. Mood disorder C. Maxwell D. Adler E. Delusional disorder

mebooksfree.com mebooksfree.com mebooksfree.com mebooksfree.com mebooksfree.com mebooksfree.com

mebooksfree.com mebooksfree.com mebooksfree.com mebooksfree.com mebooksfree.com mebooksfree.com mebooksfree.com mebooksfree.com mebooksfree.com mebooksfree.com mebooksfree.com mebooksfree.com

Schizophrenia Spectrum and Other Psychotic Disorders 19 Epidemiology 13. Schizophrenia is characterized by all of the fol- lowing symptoms except: (AI 1993) 6. Schizophrenia is associated with which of the A. Delusion of reference B. Delusion of control following personalities: (AIIMS 1997) C. Waxy flexibility D. Altered sensorium A. Athletic B. Pyknic mebooksfree.com mebooksfree.com mebooksfree.com 14. mebooksfree.comThe characteristic clinical manifestationmebooksfree.com of schizo- mebooksfree.com C. Asthenic D. All of the above phrenia are: (PGI 1998) 7. True about late onset schizophrenia: A. Confusion (AIIMS Nov 2010) B. Anxiety A. Onset is after 45 years C. Auditory hallucinations B. Onset is between 25-30 years D. Visual hallucinations C. Prognosis is poor 15. Which of the following hallucinations is pathog- D. Olfactory hallucinations are common nomonic of schizophrenia?(AIIMS 2K, Delhi 2003) 8. Maximum heritability is seen in which of the fol- A. Auditory hallucinations commanding the patient lowing illness: B. Auditory hallucinations giving running com­ mentary (DNB 2005, MP 2004, WB 2003, UP 2001) C. Auditory hallucinations criticising the patient A. Depression B. Mania mebooksfree.com mebooksfree.com mebooksfree.com mebooksfree.comD. Auditory hallucinations talking tomebooksfree.com the patient mebooksfree.com C. Schizophrenia D. Panic disorder 16. All of the following are characteristic symptoms Etiology and Pathogenesis of schizophrenia except: (AIIMS Nov 2007, MCI Screening) 9. Neurotransmitter related to the pathology of A. Third person hallucinations schizophrenia is: (PGI 1997) B. Inappropriate emotions A. Acetylcholine B. Dopamine C. Sustained mood changes C. Serotonin D. Norepinephrine D. Formal thought disorder 10. Blood sample of a 45 years old male shows 17. Hallucinations in schizophrenia are characterized increased levels of homovanillic acid (HVA). This by all of the following except: patient is most likely suffering from: A. Hallucinations commanding and controlling mebooksfree.com mebooksfree.com mebooksfree.com(AIIMS Nov 2008) mebooksfree.comaction of the person mebooksfree.com mebooksfree.com A. Dementia B. Schizophrenia B. Hallucinations of voices, singing songs C. Depression D. Parkinson’s disease C. Hallucinations are almost always continuous D. Hallucinations commenting on action of the 11. Schizophrenia is caused by overactivity in which person of the fol- lowing dopaminergic systems? 18. Which of the following sign is not a part of catato- (DNB 2007) nia? (AIIMS May 2015) A. Nigrostriatal pathway A. Akathisia B. Ambivalence B. Tuberoinfundibular pathway C. Ambitendency D. Akinesia C. Mesolimbic/Mesocortical pathway D. None of the above 19. All of the following are features of catatonia except: (DNB NEET 2014-15) A. Automatic obedience B. Cataplexy mebooksfree.com mebooksfree.comSymptoms and Diagnosismebooksfree.com mebooksfree.comC. Catalepsy D. Negativismmebooksfree.com mebooksfree.com 12. Schizophrenia is characterized by all of the following 20. The following are features of catatonic schizophre- symptoms except: (AIIMS 1998, 2000) nia, except: (MP 2000) A. Delusion A. Mutism B. Auditory hallucination B. Echolalia C. Elation C. Waxy flexibility D. Catatonia D. Deep tendon reflexes are increased

mebooksfree.com mebooksfree.com mebooksfree.com mebooksfree.com mebooksfree.com mebooksfree.com

mebooksfree.com mebooksfree.com mebooksfree.com mebooksfree.com mebooksfree.com mebooksfree.com mebooksfree.com mebooksfree.com mebooksfree.com mebooksfree.com mebooksfree.com mebooksfree.com

20 Review of Psychiatry

21. In catatonic schizophrenia, which of the following B. Affective symptoms sign is not found: (PGI Dec 2008) C. Emotional blunting A. Waxy flexibility D. Insidious onset B. Automatic obedience 29. All of the following are associated with better mebooksfree.com mebooksfree.comC. Somatic passivity mebooksfree.com mebooksfree.com mebooksfree.com mebooksfree.com prognosis in schizophrenia except: D. Gegenhalten (AI 2006, MCI Screening) E. Hallucinations A. Late onset B. Married 22. True about schizophrenia: (PGI 2003) C. Negative symptoms D. Acute onset A. Thought broadcasting B. Third person hallucinations 30. Prognosis of schizophrenia is less favorable in the C. Violent behavior following clinical scenario: (MCI Screening) D. Elated mood A. Occurring in women E. Good self care B. Anxiety is prominent C. Emotional blunting is present 23. All of the following are true about paranoid schizo- D. In presence of rapid onset of psychosis phrenia except: (MP 1997) A. Most common type of schizophrenia 31. Type two schizophrenia is characterized by all of mebooksfree.com mebooksfree.comB. Onset in 3rd/4th decademebooksfree.com mebooksfree.comthe fol- lowing features except: (AIIMSmebooksfree.com Nov 2008) mebooksfree.com C. Delusion of grandeur is a symptom A. Negative symptoms D. Rapid deterioration of personality B. Poor response to treatment C. Disorganised behavior 24. Defect of conation is typically seen in: D. CT scan abnormalities (PGI 1997, AIIMS 1996, UP 2006) A. Simple schizophrenia 32. Van Gogh syndrome is seen in: (PGI 2003) B. Hebephrenic schizophrenia A. Mania B. Depression C. Catatonic schizophrenia C. Schizophrenia D. OCD D. Paranoid schizophrenia 33. Which of the following is the most common cause 25. Waxy flexibility is a characteristic sign of: of premature death in schizophrenia? (AI 2011) (Orissa 2004, Jharkhand 2006) A. Homicide mebooksfree.com mebooksfree.comA. Excitatory catatoniamebooksfree.com mebooksfree.comB. Suicide mebooksfree.com mebooksfree.com B. Stuporous catatonia C. Toxicity of antipsychotic drugs C. Obsessive compulsive disorder D. Hospital acquired infections D. All of the above 34. Expressed emotionality is related to which of the 26. Early onset and bad prognosis is seen in: following illnesses: (MH 2010) (AIIMS 1991) A. Depression A. Catatonic schizophrenia B. Schizophrenia B. Hebephrenic schizophrenia C. Mania C. Paranoid schizophrenia D. Somatoform disorder D. Undifferentiated schizophrenia 27. Schizophrenia with late onset and best prognosis: Clinical Vignettes mebooksfree.com mebooksfree.com mebooksfree.com(DNB NEET 2014-15) 35. mebooksfree.comA patient of Schizophrenia was startedmebooksfree.com on neuro- mebooksfree.com A. Simple schizophrenia leptics, his psychotic symptoms began to improve B. Hebephrenic schizophrenia however he developed sadness, would talks less to C. Catatonic schizophrenia others , would mostly remain on bed. This presen- D. Paranoid schizophrenia tation could be caused by all of following except: 28. Good prognosis in schizophrenia is indicated by: (AIIMS 2000) (PGI 1998) A. Parkinsonism A. Soft neurological signs B. Major depression

mebooksfree.com mebooksfree.com mebooksfree.com mebooksfree.com mebooksfree.com mebooksfree.com

mebooksfree.com mebooksfree.com mebooksfree.com mebooksfree.com mebooksfree.com mebooksfree.com mebooksfree.com mebooksfree.com mebooksfree.com mebooksfree.com mebooksfree.com mebooksfree.com

Schizophrenia Spectrum and Other Psychotic Disorders 21

C. Negative symptoms are still persisting talking to himself and sometimes laughing loudly. D. He is reacting to external stimuli The likely diagnosis is: (AIIMS May 2002) A. Schizophrenia B. Conversion disorder 36. Kallu, a 24-year-old occasional alcoholic was C. Major depression D. Delusional disorder mebooksfree.com mebooksfree.combrought to psychiatrymebooksfree.com OPD with a history of mebooksfree.com mebooksfree.com mebooksfree.com behavioral changes. According to family members, 41. A 16-year-old boy does not attend school because he has become suspicious that people are trying to of the fear of being harmed by school mates. He conspire against him, though his father states that thinks that his classmates laugh at him and talk there is no reason for his fears. Kallu also reports about him. He is even scared of going out to the of hearing voices that comment on his actions. market. He is most likely suffering from: What is the most probable diagnosis:(AIIMS 2000) (AI 2004) A. Delirium tremens A. Anxiety disorder B. Alcohol induced psychosis B. Manic depressive psychosis (bipolar disorder) C. Schizophrenia C. Adjustment reaction D. Delusional disorder D. Schizophrenia 37. A 70-year-old male, Babulal was brought to the Treatment mebooksfree.com mebooksfree.comhospital with the historymebooksfree.com of third person auditory mebooksfree.com mebooksfree.com mebooksfree.com hallucinations. He has no history of similar prob- 42. Depot preparations are available for: lems previously. What is the most likely diagno- (PGI Nov 2010) sis? (AIIMS 2001) A. Haloperidol B. Risperidone A. Dementia B. Delusional disorder C. Olanzapine D. Imipramine C. Schizophrenia D. Acute psychosis E. Fluphenazine 38. A 60-year-old man is brought to a psychiatrist with 43. A 23-year-old boy with schizophrenia is well- a 10-year history, that he suspects his neighbors maintained on risperidone for the last 2 months. and he feels that whenever he passes by they He has no family history of the disease. For how sneeze and plan against him behind his back. He long will you continue treatment in this patient? feels that his wife has been replaced by a double (AIIMS Nov 2015) and calls police for help. He is quite well-groomed, A. 5 years B. 6 months mebooksfree.com mebooksfree.comalert, occasionally consumesmebooksfree.com alcohol, likely diag- mebooksfree.comC. 2 years D. 12 monthsmebooksfree.com mebooksfree.com nosis is: (AIIMS May 2002) A. Paranoid personality disorder 44. A person with violent behavior and agitation was B. Paranoid schizophrenia diagnosed to have schizophrenia and was started C. Alcohol withdrawal on haloperidol. Following this he developed rigidity D. Conversion disorder and inability to move his eyes. Which of the follow- ing drugs should be added to his treatment intrave- 39. Lallo, a 40-year-old male has recently started writ- nously for this condition? (AIIMS May 2015) ing books. But the matter in his book could not be A. Promethazine B. Haloperidol understood by anybody since it contained words C. Risperidone D. Diazepam which were never there in any dictionary and the theme was very disjoint. Nowadays he has become 45. Antipsychotic drug with least incidence of extrapy- very shy and self absorbed. When he addresses ramidal side effects is: (DNB NEET 2014-15) mebooksfree.com mebooksfree.compeople he speaks aboutmebooksfree.com meta philosophical ideas. mebooksfree.comA. Pimozide B. Thioridazinemebooksfree.com mebooksfree.com What is the likely diagnosis? (AIIMS 2000) C. Clozapine D. Chlorpromazine A. Mania B. Schizophrenia 46. Not true about clozapine is: (AI-2012) C. A genius writer D. Delusional disorder A. Should be discontinued, if WBC counts <3000/ 40. A patient is brought with 6 months history of odd mm3 behavior. There is history of a family member hav- B. Blood levels should be maintained <350 ng/mL ing disappeared some years back. He seems to be to avoid agranulocytosis

mebooksfree.com mebooksfree.com mebooksfree.com mebooksfree.com mebooksfree.com mebooksfree.com

mebooksfree.com mebooksfree.com mebooksfree.com mebooksfree.com mebooksfree.com mebooksfree.com mebooksfree.com mebooksfree.com mebooksfree.com mebooksfree.com mebooksfree.com mebooksfree.com

22 Review of Psychiatry

C. Should not be used along with carbamazepine C. Addition of beta-blocker D. The action is more on D4 receptors than D2 D. Adding another antipsychotic drug receptors 52. Akathisia is treated by all except: (AI 1994) mebooksfree.com mebooksfree.com 47. A patient of schizophreniamebooksfree.com on chlorpromazine mebooksfree.comA. Trihexyphenidyl B. Diazepammebooksfree.com mebooksfree.com (CPZ) develops auditory hallucination again. The C. Haloperidol D. Promethazine next drug to be given is: (AI 2000) 53. A psychotic patient on antipsychotic drugs deve­ A. Haloperidol B. Clozapine lops torticollis­ within 4 days of starting therapy. C. Sulpiride D. Tianeptin What is the appropriate medication that should 48. A patient with acute psychosis, who is on halo­ be added in the treatment regimen? peridol 20 mg/day for last 2 days, has an episode (DNB NEET 2014-15) characterized by tongue protrusion, oculo- A. Central anticholinergic gyric crisis, stiffness and abnormal posture of B. Peripheral anticholinergic limbs and trunk without loss of consciousness C. Beta-blocker for last 20 minutes before presenting to casualty. D. Dantrolene This improved within a few minutes after 54. A patient who is taking antipsychotics for 3 weeks, administration of diphenhydramine HCl. The mebooksfree.com mebooksfree.com mebooksfree.com mebooksfree.compresents with high grade fever, raisedmebooksfree.com CPK and mebooksfree.com most likely diagnosis is: (AIIMS 2011, May 2006) myoglobinuria. What is the most probable diag­ A. Acute dystonia nosis? (DNB NEET 2014-15) B. Akathisia A. Neuroleptic malignant syndrome C. Tardive dyskinesia B. Tardive dyskinesia D. Neuroleptic malignant syndrome C. Acute dystonia 49. 16-year-old boy who was started on an antipsy- D. Akathisia chotic drug, presents with sudden onset of torti- 55. A 31-year-old male, with mood disorder, on 30 mg collis. What is the most probable diagnosis? of haloperidol and 100 mg of lithium, is brought (DNB NEET 2014-15) to the hospital emergency room with history of A. Acute muscular dystonia acute onset of fever, excessive sweating, confusion, mebooksfree.com mebooksfree.comB. Akathisia mebooksfree.com mebooksfree.comrigidity of limbs and decreased communicationmebooksfree.com mebooksfree.com C. Neuroleptic malignant syndrome for a day. Examination reveals tachycardia and D. Tardive dyskinesia labile blood pressure and investigations reveal 50. An elderly woman suffering from schizophrenia is on increased CPK enzyme levels and leucocytosis. antipsychotic medication. She developed pur- He is likely to have developed: (AIIMS May 2004) poseless involuntary facial and limb movements, A. Lithium toxicity constant chewing and puffing of cheeks. Which of B. Tardive dyskinesia the following drugs is least likely to be involved in C. Neuroleptic malignant syndrome this side effect? (AIIMS Nov 2003) D. Hypertensive encephalopathy A. Haloperidol B. Clozapine 56. A patient was on treatment with trifluoperazine C. Fluphenazine D. Loxapine for some time. He presents with complaint of 51. A 19-year-old boy suffering from chronic schizo- hyperthermia, lethargy and sweating. Needed (AIIMS 2000) mebooksfree.com mebooksfree.comphrenia is put on haloperidolmebooksfree.com at the dose of 20 mg/ mebooksfree.cominvestigations are: mebooksfree.com mebooksfree.com A. CT scan brain and hemogram day. A week after the initiation of medication the B. Hemogram, electrolyte level and creatinine patient shows restlessness, fidgetiness, irritability C. ECG, chest X-ray and hemogram and cannot sit still at one place. The most appro- D. Hemogram, CPK and renal function test priate treatment strategy is: (AIIMS May 2004) A. Increase in the dose of haloperidol 57. Which of the following is a symptom of neuroleptic B. Addition of anticholinergic drug malignant syndrome? (DNB NEET 2014-15)

mebooksfree.com mebooksfree.com mebooksfree.com mebooksfree.com mebooksfree.com mebooksfree.com

mebooksfree.com mebooksfree.com mebooksfree.com mebooksfree.com mebooksfree.com mebooksfree.com mebooksfree.com mebooksfree.com mebooksfree.com mebooksfree.com mebooksfree.com mebooksfree.com

Schizophrenia Spectrum and Other Psychotic Disorders 23

A. Hypotension 65. Alcoholic paranoia is associated with: (AI 2010) B. Hypothermia A. Fixed delusions B. Hallucinations C. Increased magnesium level in blood C. Drowsiness D. Impulsivity D. Catatonia and stupor mebooksfree.com mebooksfree.com mebooksfree.com 66. mebooksfree.comDelusion of doubles is seen in: mebooksfree.com(AIIMS 1999) mebooksfree.com 58. What is produced by the supersensitivity of dopa- A. Schizoaffective disorder mine receptors? (DNB NEET 2014-15) B. Capgras syndrome A. Dyskinesia B. Hyperphagia C. Reactive psychosis D. Paranoid schizophrenia C. Hyperpathia D. Hypomania 67. Characteristic symptom in induced psychotic 59. Drug of choice for treatment of neuroleptic disorder is: (AIIMS 1992) malignant syndrome is: (DNB NEET 2014-15) A. Insomnia A. Dantrolene B. Profound mood disturbance B. Beta-blockers C. Accepting delusions of other person C. Central anticholinergics D. Suicidal ideation D. None of the above 68. A person aged 35 years is having firm belief about mebooksfree.com mebooksfree.com 60. A young patient of schizophreniamebooksfree.com is intolerant to mebooksfree.cominfidelity involving the spouse. Hemebooksfree.com never allows mebooksfree.com antipsychotic medications. Which drug is most her to go out of home alone. He often locks his preferred? house, while going to the office. In spite of all this, A. Clozapine B. Olanzapine he is persistently suspicious about the character C. Risperidone D. Haloperidol of his wife. The probable diagnosis is: (AIIMS 1999) 61. Antipsychotic drug causing retinal pigmentation A. Schizophrenia disorder is: (DNB NEET 2014-15) B. Delusional parasitosis A. Thioridazine B. Clozapine C. De Clerambault’s syndrome C. Chlorpromazine D. None of the above D. Othello syndrome 62. In comparison to haloperidol, clozapine causes 69. Basanti, 27-year-female thinks that her nose is more: (PGI May 2015) ugly. Her idea is fixed and is not shared by anyone mebooksfree.com mebooksfree.comA. Weight gain mebooksfree.com mebooksfree.comelse. Whenever she goes out of the home,mebooksfree.com she hides mebooksfree.com B. Agranulocytosis her face? She visits a surgeon for plastic surgery. The appropriate next step would be: (AI 2001) C. Sedation A. Investigate and then operate D. Severe extrapyramidal symptoms B. Reassure the patient E. Less epileptogenic potential C. Immediate operation 63. Cognitive remediation is used for: (AIIMS 2013) D. Refer to psychiatrist A. Cognitive restructuring 70. A 41-year-old woman working as an executive in B. Memory improvement a company is convinced that the management C. Correcting cognitive distortion has denied her promotion by preparing false D. Improving study habits reports about her competence and have forged her signature on sensitive documents so as to convict mebooksfree.com mebooksfree.comOther Psychotic Disordersmebooksfree.com mebooksfree.comher. She files a complaint in the policemebooksfree.com station and mebooksfree.com requests for security. Despite all this she attends 64. What is the content of most common type of per- to her work and manages the household. What is sistent delusional disorder? (DNB NEET 2014-15) the most likely diagnosis? (AI 2004) A. Delusion of persecution A. Paranoid schizophrenia B. Somatic delusion B. Late onset psychosis C. Delusion of jealousy C. Persistent delusional disorder D. Delusion of grandeur D. Obsessive compulsive disorder

mebooksfree.com mebooksfree.com mebooksfree.com mebooksfree.com mebooksfree.com mebooksfree.com

mebooksfree.com mebooksfree.com mebooksfree.com mebooksfree.com mebooksfree.com mebooksfree.com mebooksfree.com mebooksfree.com mebooksfree.com mebooksfree.com mebooksfree.com mebooksfree.com

24 Review of Psychiatry

71. A 30-year-old unmarried woman from a low 5. A, B, C, D, E. socioeconomic status family believes that a rich Although, SFRS were described in relation to boy staying in her neighborhood is in deep love schizophre-nia, however they are not specific with her. The boy clearly denies his love towards to schizophrenia. They can be found in all the mebooksfree.com mebooksfree.comthis lady. Still the ladymebooksfree.com insists that his is mebooksfree.comdisorders mentioned in this question.mebooksfree.com However, mebooksfree.com a secret affirmation of his love towards her. She if it was not a PGI question, and only option had makes desperate attempts to meet the boy despite to be chosen, it would be schizophrenia. resistance from her family. She also develops sad- 6. C. ness at times when her effort to meet the boy does 7. A not materialize. She is able to maintain her daily 8. C. Schizophrenia has more heritability than bipolar routine. She however, remains preoccupied with disorder (mania) which in turn has higher heri­ tability in comparison to depression. the thoughts of this boy. She is likely to be suffering 9. A, B, C, D. from: (AI 2004) All have been implicated but most important are A. Delusional disorder B. Depression dopamine and serotonin. C. Mania D. Schizophrenia 10. B. The HVA is a metabolite of dopamine and mebooksfree.com mebooksfree.com 72. A 20-year-old boy complaintsmebooksfree.com of hearing of voices and mebooksfree.comdopamine is usually increased in schizophrenia.mebooksfree.com mebooksfree.com aggressive behavior for last 2 days. He had fever A large number of studies have found that levels before the onset of these symptoms. The family of HVA are increased in schizophrenia. members report that he has been muttering to self 11. C. and gesticulating as if he is talking to someone. 12. C. There is no history of any past psychiatric illness. 13. D. Please remember altered sensorium (or clouding The likely diagnosis is: (AIIMS Nov 2010) of consciousness) is a sign of delirium. This is a A. Dementia B. Acute psychosis frequently repeated question. C. Delirium D. Delusional disorder 14. C. Auditory hallucinations are the most common type of hallucinations in schizophrenia and the 73. A 30-year-old man has become suspicious that his third person auditory hallucinations are quite wife is having an affair with his boss. He thinks his characteristic for schizophrenia. friend is also involved from abroad and is provid- 15. B. Actually, the correct answer is none. No single mebooksfree.com mebooksfree.coming technology support.mebooksfree.com He also thinks that people mebooksfree.comsymptom or sign is pathognomonicmebooksfree.com of schizo­ mebooksfree.com talk ill about him. His friends tried to convince him phrenia. However, earlier, the Schneider’s first but failed to do so. The patient otherwise is normal, rank symptoms were considered to be patho­ doesn’t have any thought disorder or any other gnomonic. Hence the best answer here is B. inappropriate behavior. The most likely diagnosis 16. C. Formal thought disorder, third person hallucina­ is: (AI 2010) tions and inappropriate emotions (inappropriate A. Paranoid personality disorder affect) are characteristic of schizophrenia. B. Persistent delusional disorder 17. C. Hallucinations in schizophrenia are usually not C. Schizophrenia continuous. D. Obsessive compulsive disorder 18. A. Akathisia is a side effect of antipsychotics. Ambi­ valence might be confusing here, but please ANSWERS remember ambiten­dency is nothing but ambi­ valence of motor movements. Akinesia, which mebooksfree.com mebooksfree.com 1. C. mebooksfree.com mebooksfree.comis lack of voluntary movements ismebooksfree.com another term mebooksfree.com 2. A. for stupor. 3. A. 19. B. Cataplexy is a feature of narcolepsy. 4. B. Automatisms are usually a feature of epilepsy. 20. D. They are apparently meaningful behaviors, for 21. C, E. which patient doesn’t have any memory later The other three options are classical catatonic on. It is not seen in schizophrenia. signs. While in catatonic schizophrenia, hallu­

mebooksfree.com mebooksfree.com mebooksfree.com mebooksfree.com mebooksfree.com mebooksfree.com

mebooksfree.com mebooksfree.com mebooksfree.com mebooksfree.com mebooksfree.com mebooksfree.com mebooksfree.com mebooksfree.com mebooksfree.com mebooksfree.com mebooksfree.com mebooksfree.com

Schizophrenia Spectrum and Other Psychotic Disorders 25

cinations and delusions can be seen, however 38. B. Kindly note that this patient also has Capgras they are not prominent. syndrome (feeling that his wife has been 22. A, B, C. replaced by a double) which is usually seen in Schizophrenic patients are much more likely to patients with schizophrenia. Also the history is mebooksfree.com mebooksfree.comengage in violent actsmebooksfree.com in comparison to those mebooksfree.comsuggestive of delusion of persecutionmebooksfree.com (neighbors mebooksfree.com without schizophrenia. are planning against him). Also, note that he is 23. D. quite well groomed suggesting that personality 24. C. is preserved as is seen in patients with paranoid 25. B. Stuporous catatonia has stupor as a prominent schizophrenia. symptom. Waxy flexibility is seen in stuporous 39. B. The history is suggestive of neologisms (words catatonia more commonly. which are not present in any dictionary) and 26. B. formal thought disorders (theme is very disjoint). 27. D. The best prognosis is of catatonic schizophrenia. Further, there are negative symptoms (shy and However in this question, the better answer is self absorbed). All point towards the diagnosis paranoid schizophrenia, as it is the one which of schizophrenia. has both late onset and good prognosis. 40. A There is history of disorganised behavior (odd mebooksfree.com mebooksfree.com 28. B. Presence of affectivemebooksfree.com symptoms (manic or mebooksfree.combehavior), hallucinations (talkingmebooksfree.com to self and mebooksfree.com depressive) is a good prognostic sign. laughing loudly is most likely a result of patient 29. C. hearing some voices and communicating with 30. C. Emotional blunting is quite similar to affective flat­ the voices), the history of disappeared family tening and hence is a negative prognostic sign. member is again suggesting that some family 31. C. member may have had a mental illness because 32. C. of which either he got lost or committed suicide. 33. B. Suicide is the most common cause of premature All factors combined, the likely diagnosis is death. Around 5-10% of patients with schizophre­ schizophrenia. nia commit suicide. 41. D. The history is suggestive of delusion of persecu­ 34. B. “Expressed emotions” is a term which is used tion (fear that schoolmates may harm him) and to describe certain attitudes of family members delusion of reference (belief that classmates of patients with schizophrenia, which have an laugh at him and talk about him). mebooksfree.com mebooksfree.comimpact on the illnessmebooksfree.com itself. These attitudes 42. mebooksfree.comA, B, C, D, E. mebooksfree.com mebooksfree.com includes over involvement, hostility, passing There is a depot preparation available for imipra­ critical comments, etc. mine, which is an antidepressant. 35. D. Kindly remember that the negative symptoms 43. C. The history is suggesting that the patient had of schizophrenia have a similar presentation first episode of schizophrenia (i.e he developed as depression. Speaking less, staying on bed schizophrenia for the first time and no history mostly can be due to either negative symptoms or of any relapse has been provided) and is now depression. Further, the use of antipsychotics can maintaining well for last two months. It is gene­ cause drug induced parkinsonism which again rally recommended that after first episode, the looks quite similar to negative symptoms. treatment with antipsychotics should be conti­ 36. C. There is history of delusions and auditory hal­ nued for at least two years. If there are more than lucinations, (running commentary type). Hence, one episodes (i.e. there is history of relapses) the mebooksfree.com mebooksfree.comthe diagnosis is mostmebooksfree.com likely schizophrenia. Occa­ mebooksfree.comtreatment should continue for at leastmebooksfree.com 5 years. In mebooksfree.com sional alcohol use is unlikely to cause psychosis. patients with multiple relapses, indefinite treat­ 37. C. Third person auditory hallucination is suggestive ment is given. of schizophrenia. Although its an incomplete 44. A. The symptoms are suggestive of acute dystonia question, and information about duration of (inability to move eyes is most likely due to oculo­ symptoms, any memory disturbances would gyric crisis) and drug induced parkinsonism have helped in making a more definitive diag­ (development of rigidity). For both, an anticho­ nosis. linergic needs to be added.

mebooksfree.com mebooksfree.com mebooksfree.com mebooksfree.com mebooksfree.com mebooksfree.com

mebooksfree.com mebooksfree.com mebooksfree.com mebooksfree.com mebooksfree.com mebooksfree.com mebooksfree.com mebooksfree.com mebooksfree.com mebooksfree.com mebooksfree.com mebooksfree.com

26 Review of Psychiatry

45. C. (primary case) induces the delusion in another 46. B. Agranulocytosis is an idiosyncratic reaction and person (secondary case). is not related to blood levels. 68. D. Here, there is only one delusion, i.e. delusion of 47. B. Clozapine. This question intends to give history infidelity, also known as Othello syndrome. mebooksfree.com mebooksfree.comfor treatment resistancemebooksfree.com schizophrenia. However, 69. mebooksfree.comD. This appears to be a case of delusionalmebooksfree.com disorder, mebooksfree.com treatment resis­tance schizophrenia is defined as somatic type. Its important to differentiate it lack of response to two different antipsychotics from body dysmorphic disorder. Here, the ques­ (belonging to different chemical classes) whereas in this question only one antipsychotic has been tion says that the idea is fixed (fixed means that used. This might be due to wrong recall of question. the belief persists despite evidences to contrary 48. A. and despite reassurances by others) and is not 49. A. shared by anyone else and patient is further 50. B. The history here is suggestive of tardive dyskine­ hiding her face when visiting outside (i.e. acting sia. Cloza­pine is the antipsychotic with minimum on her belief). In body dysmorphic disorder, the incidence of tardive dyskinesia. belief is not fixed and may be at least temporarily­ 51. C. The history here is suggestive of akathisia. changed by reassurances of others. In body mebooksfree.com mebooksfree.com 52. C. mebooksfree.com mebooksfree.comdysmorphic disorder, the problemmebooksfree.com is more of a mebooksfree.com 53. A. pre­occupation with the thought that a body part 54. A. is deformed, this preoccupation is however not 55. C. fixed (which means person can be reassured at 56. D. Here, we need to rule out the neuroleptic malignant syndrome and also check the renal least for some time). functions (as NMS can result in renal failure 70. C. In this question, there is a single delusion that secondary to myoglobinuria). management is against her (delusion of perse­ 57. D. The symptoms of NMS are quite similar to cata- cution) and her actions are according to that tonia (increased rigidity, stupor). Infact, cata­ delusion. Please remember that in delusional tonia is an important differential diagnosis in disorders, the areas of functioning which does patients with NMS. not involve the delusion, remain unaffected. In 58. A. It is believed that long-term blockade of D2 recep­ this patient also, the history that she is able to do mebooksfree.com mebooksfree.comtors by antipsychoticsmebooksfree.com causes super sensitivity of mebooksfree.comher work and manage householdmebooksfree.com is suggesting mebooksfree.com the receptors which results in tardive dyskinesia. that she is able to manage the areas of her life 59. A. which are not affected by the delusion. In ques­ 60. A. In patients who are intolerant to the extrapy­ ramidal side effects, clozapine is the preferred tions of delusional disorder, this history is very antipsychotic. important and should be looked for. 61. A. 71. A. 62. A,B,C. 72. B. The duration of symptoms is less than one month. 63. B. Cognitive remediation is a therapy usually used Also please remember that in a large number of in schizophrenia for improvement of cognitive cases acute psychosis is preceded by fever, hence functions such as attention, concentration, don’t get confused. In this case if the history also memory, planning and execution. mentioned disturbances of consciousness or his­ 64. A. tory of disorientation, the likely diagnosis would mebooksfree.com mebooksfree.com 65. A. Alcoholic paranoiamebooksfree.com usually presents with mebooksfree.combe delirium. mebooksfree.com mebooksfree.com delusion of infidelity (also known as morbid 73. B. Here again, there is a central delusion that wife is jealousy). having an affair, and the rest of history is exten­ 66. B. Please remember delusion of doubles is also sion of that delusion (i.e. friend is providing known as Capgras syndrome and is usually seen support and people are talking ill). The question in patients with schizophrenia. has mentioned the lack of any thought disorder 67. C. In Induced psychotic disorder or shared psy­ and inappropriate behavior to provide evidence chotic disorder, one person who has the delusion against the diagnosis of schizophrenia. mebooksfree.com mebooksfree.com mebooksfree.com mebooksfree.com mebooksfree.com mebooksfree.com

mebooksfree.com mebooksfree.com mebooksfree.com mebooksfree.com mebooksfree.com mebooksfree.com mebooksfree.com mebooksfree.com mebooksfree.com mebooksfree.com mebooksfree.com mebooksfree.com

mebooksfree.com mebooksfree.com mebooksfree.com mebooksfree.com mebooksfree.com mebooksfree.com

Chapter 3 Mood Disorders

Mood disorders are so called as their main feature is Symptoms abnormality of mood. They are also sometimes referred The symptoms of depression can be remembered using mebooksfree.com mebooksfree.comto as affective disorders. Moodmebooksfree.com disorder include: mebooksfree.com mebooksfree.com mebooksfree.com the pneumonic, SIGECAPS. These symptoms must last for A. Major depressive disorder (or Unipolar depression or more than two weeksQ, for the diagnosis of depression. Depression): Patients have only depressive episodes. A. Sleep disturbances: Usually insomnia is seen however B. Bipolar disorder: Patients have both manic and hypersomnia can also be a symptom. The two char­ depressive episodes or may present with manic epi­ acteristic sleep disturbances in depression are “early sodes only. morning awakeningQ” and “reduced latency of REM C. Hypomania: It is an episode similar to mania, how­ sleep”. ever is less severe, and does not meet the criterion for B. Interest (loss): Patient looses interest in the activities mania. which used to interest him earlier (anhedonia). D. Cyclothymia: It is a less severe form of bipolar disorder. C. Guilt: Patient may have excessive guilt feelings and E. Dysthymia: Less severe and chronic form of major may blame himself for trivial matters. depression. mebooksfree.com mebooksfree.com mebooksfree.com D. Energymebooksfree.com (lack): Patient may have decreasedmebooksfree.com energy mebooksfree.com DEPRESSION levels and easy fatigability. E. Cognition/Concentration: Patient may have negative Various terms such as major depressive disorder, unipolar cognitions (negative thoughts) and may have poor depres­sion and depression have been used for the same concentration. illness. It is characterized by major depressive episodes F. Appetite: Usually the appetite and weight are lost, in (also known as depres­sive episodes) in the absence of any some patients weight gain may be seen. manic, mixed or hypomanic episodes. G. Psychomotor agitation or retardation: The term ‘psy­ It is one of the commonest psychiatric disorder (life­ chomotor’ refers to the changes in motor activity time prevalence is 17%). It is twice as prevalent in women secondary to psychological causes. It may be increased as in men and the mean age of onset is around 40 years (e.g. in restless patients) or may be decreased (e.g. a (so its most commonly seen in middle aged females). patient who keeps on lying on the bed and rarely gets mebooksfree.com mebooksfree.comIt is more commonly seen mebooksfree.comin divorced and separated up).mebooksfree.com mebooksfree.com mebooksfree.com persons. H. Suicidal thoughts and sadness of mood (depressed Depression is also responsible for maximum DALYs mood). (disability adjusted life years) amongst all the psychia­ Out of the above symptoms, SWAG (suicidality, weight tric disorders. It is also the most commonQ cause of loss, anhedonia and guilt feelings) are quite suggestive of suicide. depression.

mebooksfree.com mebooksfree.com mebooksfree.com mebooksfree.com mebooksfree.com mebooksfree.com

mebooksfree.com mebooksfree.com mebooksfree.com mebooksfree.com mebooksfree.com mebooksfree.com mebooksfree.com mebooksfree.com mebooksfree.com mebooksfree.com mebooksfree.com mebooksfree.com

28 Review of Psychiatry Physical Signs retardation and feelings of guilt and higher suicide risk. The symptoms of endogenous depression were quite Q A. Veraguth fold : Otto veraguth described a triangular similar to today’s psychotic and melancholic depres­ shape fold in the nasal corner of upper eyelid, called sion. The exogenous depression (reactive depression) mebooksfree.com mebooksfree.comveraguth fold in patients withmebooksfree.com depression. was believedmebooksfree.com to occur in response to amebooksfree.com negative life mebooksfree.com B. Omega sign: It is the omega shaped fold (like the event and the symptoms which were described included Greek letter omega, Ω ) in the forehead above the root initial insomnia (difficulty in falling asleep), absence of of the nose, seen in patients with depression. psychotic symptoms and multiple somatic complaints Apart from the above mentioned symptoms, the and lower suicide risk. patient may have few other special features which must be mentioned along with the diagnosis, such as psychotic Etiology features, atypical features, melancholic features and cata­ A. Biological factors: tonic features. These have been described below: • Neurotransmitters disturbances: Decreased lev- A. With psychotic features: Patients with severe depres­ els of serotonin­ and norepinephrineQ are most sion may develop psychotic symptoms (delusions and important factors implicated in the pathophysio­ mebooksfree.com mebooksfree.comhallucinations). These psychoticmebooksfree.com symptoms could be mebooksfree.comlogy of depression. Dopamine has alsomebooksfree.com been found mebooksfree.com mood congruent (i.e. content of delusion/halluci­ to be decreased in a subset of patients. nation is consistent with the depressed mood, e.g. a • Hormonal disturbances: Elevated HPA activity severely depressed patient developed a delusion that (hypothalamic-pituitary-adrenal axis activity) has the world is about to end, nihilistic delusion) or mood been documented. Also, hypothyroidismQ is a incongruent (i.e. content of delusion is inconsistent common cause of depression. with the depressed mood, e.g. a severely depressed • Neuroanatomical considerations: Decreased acti- patient developed the delusion that he is the richest vity in dorso­lateral prefrontal cortexQ and man on earth). It must be remembered that psychotic increased activity in amygdala (and other limbic symptoms are present only in severe depression and tissue) has been found in depression. sometimes the term “ Q” is used psychotic depression B. Genetic factors: Gene mapping studies have found for depression with psychotic features. evidence of linkage to locus for cAMP response ele­ B. With atypical features: These patients present with ment binding protein (CREB 1) on chromosome 2. mebooksfree.com mebooksfree.comreverse biological symptomsmebooksfree.com such as hypersomnia, mebooksfree.com mebooksfree.com mebooksfree.com Serotonin transporter gene has also shown linkage. overeating and weight gain. C. Psychological theories: C. With melancholic features: Depression with melan­ • Cognitive theory: It was proposed by Aaron BeckQ. cholic features (or involutional melancholiaQ) is According to this theory negative thoughts have a usually seen in old age. It is characterized by severe central role in development of depression. He pro­ anhedonia, profound guilt feelings, early morning posed that there are three central thoughts/ideas in awakening and weight loss, agitation and high sui­ depression, the so called cognitive triad of depres- cide risk. sionQ. These include: D. With catatonic features: Patient with depression may 1. Negative view of self (ideas of worthlessnessQ) develop catatonic symptoms such as stuporQ, negati­ 2. Negative views about environment—A tendency vism, etc. to experience world as hostile (ideas of help- mebooksfree.com mebooksfree.com mebooksfree.com mebooksfree.comlessnessQ) and mebooksfree.com mebooksfree.com Endogenous vs Exogenous (Reactive) Depression 3. negative view about future (ideas of hopeless- In older classi­ficatory system, two subtypes of depres­ ness). sion were described. Endogenous depression which • Learned helplessness: According to this theory, due occurred in the absence of any precipitating negative to repeated adverse events, patient starts believ­ life event, and was considered to be caused by biologi­ ing that he has no control over events happening cal factors. The symptoms described were early morn­ around him and loses the motivation to act which ing awakening, psychotic symptoms, psychomotor results in depression.

mebooksfree.com mebooksfree.com mebooksfree.com mebooksfree.com mebooksfree.com mebooksfree.com

mebooksfree.com mebooksfree.com mebooksfree.com mebooksfree.com mebooksfree.com mebooksfree.com mebooksfree.com mebooksfree.com mebooksfree.com mebooksfree.com mebooksfree.com mebooksfree.com

Mood Disorders 29 Treatment channels. The blockage of sodium channels in brain can cause seizures. 1. Pharmacotherapy: The use of specific pharmaco­ • Sedation due to blockage of H1 histaminic therapy doubles the chances that a depressed patient receptors. mebooksfree.com mebooksfree.comwill recover in 1 month.mebooksfree.com All the available anti­ mebooksfree.com• Important properties of individualmebooksfree.com drugs: mebooksfree.com depressants take up to 3–4 weeks to exert signi­ficant a. AmoxapineQ has D2 blocking action and therapeutic effects. The available antidepressants hence can cause extrapyramidal side effects do not differ in the overall efficacy, speed of like antipsychotics. response or long-term effectiveness and the choice b. ImipramineQ is used in the treatment of noc­ of antidepressants is mostly determined by the side turnal enuresis (however the drug of choice Q effect profile of the drugs. Antidepressant treatment is desmopressin; the treatment of choice is should be maintained for at least 6 months or equal behavioral methods like night alarms). to the duration of a previous episode, whichever is c. Clomipramine is the first line therapy in greater. Prophylactic­ treatment with antidepressants OCD, however due to better side effect pro­ is effective in reducing the number and severity of file, SSRIs are preferred over clomipramine. episodes. It should be given to patients who have B. Selective serotonin reuptake inhibitors (SSRIs): mebooksfree.com mebooksfree.comhad three or more prior depressivemebooksfree.com episodes or who mebooksfree.comThese are the most commonly prescribedmebooksfree.com antide­ mebooksfree.com have chronic major depressive disorder (> 2 years pressants. They act by blocking the reuptake of duration is chronic depression). The following classes serotonin and do not have problematic side effects of medications can be used. seen with TCAs. The SSRIs include fluoxetine, flu­ A. Tricyclic and tetracyclic antidepressants (TCAs): voxamine, citalopram, escitalopram, sertraline, These were the first class of antidepressants that paroxetine and vilazodone. The SSRIs are the first were widely used in clinical practice. They act by line drugs for depression, obsessive compulsive blocking the transporters of serotonin and norep- disorder, post-traumatic stress disorder, panic dis­ inephrine and hence increase the levels of these order, generalized anxiety disorder and phobias. neurotransmitters in synapses. Secondary effects The side effects of the SSRIs include nausea (most of TCAs include antagonism of muscarinic, his­ common) followed by anxiety and diarrhoea. Other taminic H1, a 1 and a 2 adrener­gic receptors and side effects include delayed ejaculation (hence blockage of cardiac sodium channels. These secon­ SSRIs are used in the treatment of premature eja­ mebooksfree.com mebooksfree.comdary effects are responsiblemebooksfree.com for the unfavorable side mebooksfree.comculation), decreased , anorgasmia,mebooksfree.com sedation, mebooksfree.com effect profile of these drugs. delayed platelet aggregation, sweating and weight The class TCAs include the following drugs: Imi­ gain. pramine, desipramine, trimipramine, amitriptyline, Serotonin syndromeQ: Concurrent administration nortriptyline, protri­ptyline, amoxapine, doxepin, of an SSRI with MAO inhibitor, L-tryptophan or maprotiline and clomipramine. The TCAs differ in lithium can raise plasma serotonin concentration, their affinity for transporters, with clomipramine producing serotonin syndrome. The symptoms are being the most serotonin selective and desipra­ diarrhea, restlessness, hyperreflexia, myoclonus, mine the most norepinephrine selective of TCAs. seizures, may result in death. It is treated using The side effects of TCAs include the following: cyproheptadine and supportive care. • Anticholinergic side effectsQ like constipation, Vortioxetine: A recently introduced antidepres­ urinary retentionQ, blurred vision, dry mouth, sant works as an inhibitor of serotonin reuptake, mebooksfree.com mebooksfree.comdecreased sweating mebooksfree.comand delirium. Due to signi­ mebooksfree.combut also has other actions like agonismmebooksfree.com at 5-HT1A mebooksfree.com ficant anticholinergic side effects TCAs should receptor, partial agonism at 5-HT1B receptor and be avoided in glaucomaQ and prostate hypertro­ antagonism at 5-HT3. 5-HT1D and 5-HT7 receptors. phy. C. SNRIs (Serotonin Norepinephrine Reup­take Inhibi­ • Side effects due to blockade ofa (alpha) recep­ tors): These drugs produce blockade of neuronal tors like postural hypotension. serotonin and norepinephrine uptake transporters • Severe side effects like cardiacarrhythmias , and hence are also refer­red as dual reuptake inhibi­ hypotension due to blocking of cardiac sodium tors. They include venlafaxine,desvenlafaxine,

mebooksfree.com mebooksfree.com mebooksfree.com mebooksfree.com mebooksfree.com mebooksfree.com

mebooksfree.com mebooksfree.com mebooksfree.com mebooksfree.com mebooksfree.com mebooksfree.com mebooksfree.com mebooksfree.com mebooksfree.com mebooksfree.com mebooksfree.com mebooksfree.com

30 Review of Psychiatry

duloxetine, milnacipran, levomilnacipran. The side of reuptake of both norepinephrine and dopa- effect profile is quite similar to SSRIs. In addition, mineQ. The advantage of bupropion is a good SNRIs can cause hypertension at higher dosages. side effect profile with low risk of sexual side D. Monoamine oxidase inhibitors: These drugs act by effects, weight gain or sedation. The common mebooksfree.com mebooksfree.cominhibiting the metabolismmebooksfree.com of monoamines. There mebooksfree.comside effects are insomnia, tremors,mebooksfree.com restlessness mebooksfree.com are two isoforms of the enzymes (MAO), MAO-A and nausea. A particular worrisome side effect (involved in metabolism of serotonin, norepi­ is seizures (usually seen at higher dosages). nephrine and dopamine) and MAO-B (preferential Bupropion is also used for smoking cessation. metabolism of dopamine). The nonselective MAO • Tianeptine and amineptine: These antidepres­ inhibitors which includes tranylcypromine, phen­ sants work by enhancingQ the reuptake of sero­ elzine and isocarboxazid inhibits both the isoforms tonin (serotonin reuptake enhancer). irreversibly. These drugs are rarely used now as • Antipsychotics: If patient has depression with they can cause hypertensive crisis. psychotic symptoms, a combination of antide­ Cheese reaction: Cheese, red wine and beer con­ pressants and antipsychotics is used. tains tyramine (which is an indirectly acting sym­ 2. Psychotherapy: It is the treatment using psychological pathomimetic). Normally, when these items are techniques. The following psychotherapeutic tech­ mebooksfree.com mebooksfree.comconsumed, the MAO-Amebooksfree.com present in the gastrointes­ niquesmebooksfree.com are effective in depression: mebooksfree.com mebooksfree.com tinal tract degrades the tyramine. However when A. Cognitive behavioral therapy: This therapy aims MAO inhibitors are used, the tyramine escapes at correcting cognitive distortions (faulty ways of degradation and gets absorbed resulting in dan­ thinking) and faulty behaviors. It is the most effec- gerous elevation of blood pressure, causing hyper­ tiveQ psychotherapeutic technique in depression. tensive crisis (also called cheese reaction). Hence B. Interpersonal therapy: In interpersonal therapy, the these food items are restricted in a patient who is focus is on management of patient’s current inter­ on MAO inhibitors. Phentolamine is the drug of personal problems (e.g. relationship problems). choice for cheese reaction. C. Other less commonly used therapeutic techniques E. Atypical antidepressants: There are many other include behavior therapy, family therapy and psy­ antidepressants which have novel mechanisms of choanalytically oriented therapy. actions. These include: 3. Other somatic treatments: • Trazodone and nefazodone: These drugs are A. Electroconvulsive therapy (ECTs): The indications mebooksfree.com mebooksfree.comclassified as SARImebooksfree.com (serotonin antagonist and mebooksfree.comfor ECT in depression includes: mebooksfree.com mebooksfree.com reuptake inhibitors). The mechanism of action • Severe depression with suicide riskQ (If the is weak inhibition of serotonin reuptake and patient is suicidal, ECT is the preferred treat- strong antagonism at 5 HT2A and 5 HT2C recep­ ment modality)Q. tors. Trazodone can cause priapismQ as a side • Severe depression with stuporQ. effect. • Other indications include depression with psy­ • Mirtazapine: Mirtazapine belongs to a class chotic symptoms, refractoriness to other treat­ called NSSA (nor adrenergic and specific sero­ ment modalities. tonergic antidepressant). The mechanism of B. Transcranial magnetic stimulationQ: It is a newer action is antagonism of central presynaptic a-2 moda­lity which uses magnetic energy to stimulate (alpha-2) receptors which results in increased nerve cells. It is nonconvulsive, requires no anes­ firing of norepinephrine and serotonin neu­ thesia, has a safe side effect profile and is not asso­ mebooksfree.com mebooksfree.comrons. The other importantmebooksfree.com action is antago­ mebooksfree.comciated with cognitive side effects. Itsmebooksfree.com use is yet not mebooksfree.com nism of postsynaptic serotonin 5 HT2 and widespread. 5 HT3 receptors. Mirtazapine causes sedation C. Vagal nerve stimulationQ: This modality involves and weight gain but doesn’t have problematic stimulation of vagal nerve using an electrode. sexual side effects. D. Deep brain stimulationQ: This modality involves • Bupropion: Bupropion belongs to a class called implantation of leads into specific brain areas and NDRI (norepinephrine dopamine reuptake has been used in patients with chronic and intrac­ inhibitors). The mechanism of action is inhibition table depression.

mebooksfree.com mebooksfree.com mebooksfree.com mebooksfree.com mebooksfree.com mebooksfree.com

mebooksfree.com mebooksfree.com mebooksfree.com mebooksfree.com mebooksfree.com mebooksfree.com mebooksfree.com mebooksfree.com mebooksfree.com mebooksfree.com mebooksfree.com mebooksfree.com

Mood Disorders 31

E. Sleep deprivation: Sleep deprivation can produce B. Increased self esteem or grandiosity (e.g. patient significant bene­fits however these are transient believes himself to be the richest, most powerful, most and are typically reversed by next night of sleep. goodlooking person on the earth, etc.) Research is ongoing to produce sustained benefits. C. Decreased need for sleepQ (e.g. patient feels rested mebooksfree.com mebooksfree.comF. Phototherapy: It has mebooksfree.combeen primarily used for aftermebooksfree.com 2 hours of sleep) mebooksfree.com mebooksfree.com seasonal affective disorders (mood disorder D. Over-talkativeness with seasonal patternQ). In this dis­order patients E. Flight of ideasQ typically develop depressive symptoms during F. DistractibilityQ (not able to concentrate on task in winter seasons which are associated with decreased hand) day time. The phototherapy involves exposure to G. Increase in goal directed activities (overactivity, hyper­ bright light in range of 1500–10,000 lux or more. sexuality, overfamiliarity) or psychomotor agitation Usually a combination of pharmacotherapy and psy­ H. Excessive involvement in activities that have high chotherapy is used in management of depressed patients, potential for painful consequences (e.g. unrestrained in cases of suicide risk, ECT is the preferred treatment. buying sprees, sexual indiscretions or foolish business investments). Q BIPOLAR DISORDER These symptoms should last for atleast7 days and mebooksfree.com mebooksfree.com mebooksfree.commust mebooksfree.comcause marked impairment in socialmebooksfree.com and occupa­ mebooksfree.com Bipolar disorder is characterized by episodes of both tional functioning. mania and depression. Even if a patient has only manic Psychotic symptoms: Apart from the above mentioned Q episodes , he would still be diagnosed with bipolar dis­ symptoms, patient may also develop psychotic symp­ order, as in all likelihood he would develop a depressive toms (delusions and hallucinations). These may be episode in future. Bipolar disorder is equally prevalent mood congruent (e.g delusion of grandiosity) or mood among men and women. Manic episodes are more com­ incongruent (e.g. delusion of persecution). In the mon in men, and depressive episodes are more common presence of psychotic symptoms, the diagnosis made is in women. The average age of onset of bipolar disorder is manic episode with psychotic symptoms. 30 years. It is more commonly seen in divorced and sin­ Hypomania: The symptoms of hypomania are similar gle persons. Bipolar disorder has multiple subtypes which to mania however they are not severe enough to cause have been illustrated in the following Table 1. marked impairment in social and occupational function­ mebooksfree.com mebooksfree.com mebooksfree.coming. Also,mebooksfree.com the duration criterion for hypomaniamebooksfree.com is 4 days. mebooksfree.com Symptoms Mixed episodes: Mixed episodes have both manic and The symptoms of manic episode are as follows: depressive symptoms lasting for at least 7 days. A. Elevated mood (undue happiness) or irritable mood Etiology

Table 1: Types of bipolar disorders. • Neurotransmitters: Increased levels of dopamine has been implicated in pathophysiology of manic episode. Bipolar 1/2 Schizobipolar disorder (schizoaffective disorder) The changes in depression have been already discussed. • Genetic factors: The chromosomes 18qQ and 22q have Bipolar I Mania with depression (or mania alone) the strongest evidence of linkage to bipolar disorder. Bipolar I 1/2 Depression with protracted hypomania Chromosome 21q has also been linked. Bipolar II Depression with discrete hypomanic episodes mebooksfree.com mebooksfree.comBipolar II 1/2 Depression superimposedmebooksfree.com on cyclothymia Treatmentmebooksfree.com mebooksfree.com mebooksfree.com Bipolar III Depression plus induced hypomania (e.g. The treatment in bipolar disorder depends on the phase. hypomania occurring solely in association with Patient requires treatment during acute illness (acute antidepressants or other somatic treatment manic or mixed or depressive episodes) and also need Bipolar III 1/2 Bipolar disorder associated with substance use prophylaxis to prevent further episodes (maintenance Bipolar IV Depression superimposed on hyperthymic treatment). The following classes of drugs are usually temperament used in bipolar disorder:

mebooksfree.com mebooksfree.com mebooksfree.com mebooksfree.com mebooksfree.com mebooksfree.com

mebooksfree.com mebooksfree.com mebooksfree.com mebooksfree.com mebooksfree.com mebooksfree.com mebooksfree.com mebooksfree.com mebooksfree.com mebooksfree.com mebooksfree.com mebooksfree.com

32 Review of Psychiatry

A. Mood stabilizers: Commonly used mood stabiliz­ C. Maintenance ing drugs include lithium, valproate, carbamazepine • Usually given after two or more acute episodes in oxcarbazepine and, lamotrigine. Many atypical anti­ bipolar I illness or after a single manic episode if psychotics also have mood stabilizing properties: it was associated with significant risk. mebooksfree.com mebooksfree.com• Lithium is considered mebooksfree.comthe prototypical mood sta- • mebooksfree.comLithium and valproate have the best mebooksfree.comevidence. mebooksfree.com bilizerQ. However it takes around 1-2 weeks to start • Treatment should be continued for at least two acting. It is usually supplemented by other mood years. stabilizers, antipsychotics or benzodiazepi­nes in early phase of treatment. Lithium • Valproate: Valproate has surpassed lithium in use for acute mania due to better tolerability. Lithium is used for treatment of acute episodes (both • Lamotrigine: It is mostly used in treatment of acute mania and depression) as well as prophylaxis in bipolar disorder. Lithium is a monovalent cation and gets rapidly depressive episode of bipolar disorder (bipolar and completely absorbed after oral administration. The depressionQ). B. Antipsychotics: Usually atypical antipsychotics are plasma half-life is initially 1.3 days and gets increased used due to better tolerance and side effect profile. to 2.4 days after continued administration for more than mebooksfree.com mebooksfree.com C. Benzodiazepines: High potencymebooksfree.com benzodiazepines such one year,mebooksfree.com lithium is not metabolized in the mebooksfree.combody and gets mebooksfree.com as lorazepam and clonazepam are frequently used in excreted unchanged through the kidney. acute mania due to their calming effect. D. Antidepressants: Antidepressants are never used Indications alone in bipolar disorder. When used alone in bipolar A. Acute manic episode: Lithium is an effective treatment depression they can cause switch (patient may go into for acute mania however since its onset of action is mania), hence they are always used along with mood delayed (1-3 weeks), an antipsychotic, benzodiaz­ stabilizers. epine or valproate is usually added for initial period. Lithium is also effective for prophylaxis against future Treatment Guidelines manic episodes. A. Acute manic or mixed episode: B. Bipolar depression: Lithium is effective for treatment • For severe mania or mixed episode, initiate lithium of bipolar depression and prophylaxis of same, how­ mebooksfree.com mebooksfree.comin combination with anmebooksfree.com antipsychotic or valproate evermebooksfree.com the antimanic efficacy of lithium mebooksfree.comis more than its mebooksfree.com in combination with an antipsychotic. antidepressive efficacy. • For less ill patients, monotherapy with lithium, val­ C. Maintenance treatment: Maintenance treatment with proate or an atypical antipsychotic such as olan­ lithium decreases the frequency, severity and dura­ zapine may be sufficient. tion of manic and depressive episodes in patients with • Short-term treatment with benzodiazepines is often bipolar disorders. used. D. Lithium is also use in patients with schizoaffective • For mixed episodes, valproate is preferred over lithium. disorders as well as an adjuvant to antidepressants in • If patient has psychotic symptoms, antipsychotics major depressive dis­order. must be added to the treatment regimen. E. Other indications in which lithium has been used B. Acute depression (bipolar depression): but is not the first line treatment include obsessive • Initiate lithium or lamotrigine. compulsive disorder, aggression, headache (cluster, mebooksfree.com mebooksfree.com• In severely ill patients, mebooksfree.cominitiate treatment with both migrainemebooksfree.comQ), gout, epilepsy, movement disorders,mebooksfree.com neu- mebooksfree.com lithium and an antidepressant. tropeniaQ, ulcerative colitis. • Quetiapine alone and combination of olanzapine Lithium has a narrow therapeutic index and therapeu­ and fluoxetine are other treatment options. tic drug monitoring is required. The effective serum con­ • Antidepressant mono therapy should never be given. centration for treat ment of acute mania is 1.0–1.5 mEq/ • Electroconvulsive therapy for patients with high dLQ. The serum concentration required for maintenance suicide risk. treatment is 0.6–1.2 mEq/dLQ.

mebooksfree.com mebooksfree.com mebooksfree.com mebooksfree.com mebooksfree.com mebooksfree.com

mebooksfree.com mebooksfree.com mebooksfree.com mebooksfree.com mebooksfree.com mebooksfree.com mebooksfree.com mebooksfree.com mebooksfree.com mebooksfree.com mebooksfree.com mebooksfree.com

Mood Disorders 33

Side Effects B. Dysthymia: It is the H presence of mild DSM-5 Update: In DSM-5 a new A. Neurological side effects: Lithium can cause postural depressive symp­ diagnosis of “premenstrual dysphoric tremors (usually treated with beta blockers like pro- disorder” has been added. This dis- toms (not enough order is characterized by symptoms pranololQ), lack of spontaneity and memory distur­ mebooksfree.com mebooksfree.com mebooksfree.com to mebooksfree.comdiagnose a major such as mood swings,mebooksfree.com irritability, leth- mebooksfree.com bances, rarely it can cause raised intracranial tension argy, physical symptoms like breast and peripheral neuropathy. depressive episode) tenderness, which start before the for a period of more onset of menses and improve after B. Endocrine: Hypothyroidism, rarely hyperthyroidism, onset of menses. hyperpara- thyroidism. than two years. C. Renal: Most common is polyuria at time progress­ Chronic depression: If the depression continues for ing to diabetes insipidus which is treated with use of more than 2 years, it is known as chronic depression. thiazide diuretics or potassium sparing diuretics (like C. Cyclothymia: It is a milder form of bipolar disorder, amiloride, spironolactone or triamterene) . Rarely in which manic symptoms and depressive symptoms nephrotic syndrome, renal tubular acidosis or inter­ occur, but they are never severe enough to make a stitial fibrosis can be seen. diagnosis of mania/hypomania or depression. The D. Others include acne, psoriasis, nausea, vomiting, diar­ symptoms should last for atleast 2 years. rhoea, weight gain, benign T wave changes. Rapid cycling: If a patient of bipolar disorder has four mebooksfree.com mebooksfree.comLithium toxicity: The risk factorsmebooksfree.com of lithium toxicity include or mebooksfree.commore than four episodes of mania/hypomania/mebooksfree.com mebooksfree.com renal impairment, dehydrationQ and low sodium dietQ. depression in one calendar year. Usually the sign of toxicity starts to appear at levels above 1.5 mEq/dL. The early signs include GI symptoms like SUICIDE abdominal pain, vomiting and neurological symptoms like Q coarse tremors, ataxia and dysarthria. The later signs and The psychiatric illnesses associated with highest risk of symptoms include impairment of consciousness, muscular suicide are depressive disorder, schizophrenia, alco- Q fasciculations, increased deep tendon reflexes and con­ hol dependence and other substance dependence and vulsions. There might be circulatory failure and death. The personality disorders (especially borderline personality management involves stopping lithium, correcting dehy­ disorder and antisocial personality disorder. Low CSF dration, use of polyethy lene glycol (and not activated levels of 5-hydro­xyindoleacetic acid (5 HIAA)Q, which charcoal) to remove unabsorbed lithium from GI tract. is a metabolite of serotonin, are associated with higher mebooksfree.com mebooksfree.comIn severe cases, hemodialysismebooksfree.com may be required. mebooksfree.com mebooksfree.com mebooksfree.com suicide risk. OTHER MOOD DISORDERS The following are the risk factors for suicide: 1. Male sexQ A. Recurrent depressive H 2. Age > 45 years disorder: If there DSM-5 Update: In DSM-5, the are more than one diagnosis of dysthymia has been 3. Divorced, widowed removed. A new diagnostic category 4. Unemployed depressive episodes, of “persistent depressive disorder” diagnosis of recur­ has been made, which includes both 5. Chronic illness chronic depression and previous dys- 6. Family history of suicide rent depressive dis­ thymic disorder. order is made. 7. Poor social support. mebooksfree.com mebooksfree.com mebooksfree.com mebooksfree.com mebooksfree.com mebooksfree.com

mebooksfree.com mebooksfree.com mebooksfree.com mebooksfree.com mebooksfree.com mebooksfree.com

mebooksfree.com mebooksfree.com mebooksfree.com mebooksfree.com mebooksfree.com mebooksfree.com mebooksfree.com mebooksfree.com mebooksfree.com mebooksfree.com mebooksfree.com mebooksfree.com

34 Review of Psychiatry

QUESTIONS AND ANSWERS mebooksfree.com mebooksfree.comQUESTIONS mebooksfree.comSymptomsmebooksfree.com and Diagnosis mebooksfree.com mebooksfree.com Depression 9. Persistent feeling of guilt is seen in: (DNB NEET 2014-15) Epidemiology and Etiology A. Obsessive compulsive disorder 1. Most commonly depression is seen in: B. Mania (AI 1996, 1998) C. Depression A. Middle aged men B. Middle aged female D. Schizophrenia C. Young girl D. Children 10. DSM IV, duration criterion for diagnosis of depres- 2. Neurotransmitters involved in depression are: sion is: (DNB NEET 2014-15) (AI 1995) A. 1 week B. 2 weeks A. GABA and dopamine C. 3 weeks D. 4 weeks B. Serotonin and norepinephrine 11. Which of the following symptoms must be present mebooksfree.com mebooksfree.comC. Serotonin and dopaminemebooksfree.com mebooksfree.com mebooksfree.com mebooksfree.com D. Norepinephrine and GABA for the diagnosis of major depressive disorder: (MH 2010, 2007) 3. Which of the following is not a part of cognitive triad A. Loss of interest or pleasure of beck? (AIIMS Nov 2015) B. Recurrent suicidal tendencies A. Hopelessness B. Worthlessness C. Insomnia C. Helplessness D. Guilt D. Indecisiveness 4. All of the following about “Aaron becks cognitive 12. Disruption or disorganization of biological rhythm is theory of depression” is true, except: observed in: (DNB Dec 2010) A. Schizophrenia B. Anxiety A. Negative thought of past B. Negative thought of future C. Depression D. Mania mebooksfree.com mebooksfree.comC. Negative thought ofmebooksfree.com environment 13. mebooksfree.com“Nihilistic delusions” are seen in: mebooksfree.com(PGI 2000) mebooksfree.com D. Negative about self A. Endogenous depression 5. True about major depressive disorder: B. Double depression (PGI Nov 2011) C. Depression in involutional stage A. Abnormally diminished activity in prefrontal D. Cyclothymia cortex E. Dysthymia B. Lesion of corticospinal tract 14. True about psychotic feature in depression: C. Monoaminergic system disturbances (PGI Dec 2004) D. Genetic predisposition is present A. Found in severe depression 6. Depression is seen in: B. Found in moderate depression A. Hyperthyroidism B. Hypogylcemia C. Mood incongruent psychotic feature C. Adrenal disorder D. Pheochromocytoma D. Cyclothymia mebooksfree.com mebooksfree.com 7. Depression is a featuremebooksfree.com of which of the following mebooksfree.comE. Dysthymia mebooksfree.com mebooksfree.com condition: 15. Intense nihilism, somatization and agitation in A. Hypopituitarism B. Hyperthyroidism old age are the hallmark symptoms of: C. Hypothyroidism D. Hypoglycemia A. Involutional melancholia 8. Depression is not caused by: B. Atypical depression A. Clonazepam B. Levodopa C. Somatized depression C. Metronidazole D. Corticosteroid D. Depressive stupor

mebooksfree.com mebooksfree.com mebooksfree.com mebooksfree.com mebooksfree.com mebooksfree.com

mebooksfree.com mebooksfree.com mebooksfree.com mebooksfree.com mebooksfree.com mebooksfree.com mebooksfree.com mebooksfree.com mebooksfree.com mebooksfree.com mebooksfree.com mebooksfree.com

Mood Disorders 35

16. True about major depressive disorder:(PGI 2003) A. Leave him as normal adolescent problem A. Commonly seen in female B. Rule out depression B. Recovery is complete after treatment C. Rule out migraine C. Associated with hypothyroidism D. Rule out an oppositional defiant disorder mebooksfree.com mebooksfree.comD. Family history of majormebooksfree.com depression mebooksfree.com mebooksfree.com mebooksfree.com 22. A 40-years-old female patient presents with his- 17. Dysthymia is: (DNB NEET 2014-15) tory of depressed mood, loss of appetite, insomnia A. Chronic mild depression and lack of interest in surroundings for past one B. Chronic severe depression year. These symptoms followed soon after a busi- C. Bipolar disorder ness loss one year back. Which of the following D. Personality disorder statements is true regarding the management of 18. Most common type of postpuerperal psychosis is: this patient: (PGI 1999) A. No treatment is necessary as it is due to business A. Depression B. Anxiety loss C. Mania D. Suicide B. SSRI is the most efficacious of the available drugs C. Antidepressant treatment is based on the side mebooksfree.com mebooksfree.comClinical Vignettes mebooksfree.com mebooksfree.comeffect profile of the drugs mebooksfree.com mebooksfree.com 19. A 41-year-old woman presented with a history of D. Combination therapy of two antidepressant aches and pains all over the body and generalized should be given weakness for four years. She cannot sleep because 23. A patient presents with depressed mood, loss of of the illness and has lost her appetite as well. She sleep, loss of hope, feeling of worthlessness and has lack of interest in work and doesn’t like to diminished concentration for last 1 month. meet friends and relatives. She denies feelings of Which of the following is the drug of choice in this sadness. Her most likely diagnosis is: patient? (DNB NEET 2014-15) (AIIMS Nov 2002) A. SSRIs A. Somatoform pain disorder B. Atypical antidepressants B. Major depression C. Lithium C. Somatization disorder D. Tricyclic antidepressants mebooksfree.com mebooksfree.comD. Dissociative disordermebooksfree.com mebooksfree.com mebooksfree.com mebooksfree.com 24. A woman has mild depressive symptoms after few 20. A 60-year-old male is brought by his wife. He thinks days of delivery which disappeared after 2 week that he had committed sins throughout his life. He in postpartum period. The most likely cause is: is very much depressed and has considered com- (PGI May 2015) mitting suicide but has not taken any such steps. A. Postpartum blue B. Mania He is also taking sessions with a spiritual guru. He C. Postpartum depression D. Mild depression does not get convinced when his wife tells him that E. Postpartum psychosis he has led a pious life. How will you treat him: 25. A patient presents to the emergency department A. Antipsychotic plus antidepressant with self harm and indicates suicidal intent. Which B. Antidepressant with cognitive behavioral therapy of the following conditions does not warrant an C. Guidance and recounselling with guru plus immediate specialist assessment: (AI 2010) mebooksfree.com mebooksfree.comantidepressant mebooksfree.com mebooksfree.comA. Formal thought disorder mebooksfree.com mebooksfree.com D. Antidepressant alone B. Acute alcohol intoxication 21. An 18-year-old student complaints of lack of inter- C. Chronic severe physical illness D. Social isolation est in studies for last 6 months. He has frequent quarrels with his parents and has frequent head- 26. A 50-year-old male presents with a three year aches. The most appropriate clinical approach history of irritability, low mood, lack of interest would be: (AI 2005) in surroundings and general dissatisfaction with

mebooksfree.com mebooksfree.com mebooksfree.com mebooksfree.com mebooksfree.com mebooksfree.com

mebooksfree.com mebooksfree.com mebooksfree.com mebooksfree.com mebooksfree.com mebooksfree.com mebooksfree.com mebooksfree.com mebooksfree.com mebooksfree.com mebooksfree.com mebooksfree.com

36 Review of Psychiatry

everything. There is no significant disruption in 33. Repetitive transcranial magnetic stimulation sleep and appetite. He is likely to be suffering (rTMS) is approved by USFDA for the treatment from: of: (AI 2012) A. Major depression A. Resistant schizophrenia mebooksfree.com mebooksfree.comB. No psychiatric disordermebooksfree.com mebooksfree.comB. Obsessive compulsive disorder mebooksfree.com mebooksfree.com C. Dysthymia C. Acute psychosis D. Chronic fatigue syndrome D. Depression 27. A patient is depressed for past 3 years, does not 34. A young female on antidepressants presents to the go out of his house much and is cut off from the emergency with altered sensorium and hypoten- society. But with normal sleep and normal weight. sion. ECG reveals wide QRS complexes and right Most probable diagnosis is? axis deviation. What is the next best step? (DNB December 2011) (AIIMS Nov 2015) A. Major depression A. Sodium bicarbonate B. Hemodialysis B. Dysthymia C. Fomepizole D. Flumazenil C. Chronic fatigue syndrome 35. Rathi, 26-years-old female has been diagnosed D. No psychiatric illness mebooksfree.com mebooksfree.com mebooksfree.com mebooksfree.comto be suffering from depression. Nowmebooksfree.com for the past mebooksfree.com 2 days she has suicidal tendency, thought and Treatment ideas. The best treatment is: (AIIMS 2001) 28. Which is not a serotonin norepinephrine reuptake A. Amitriptyline inhibitor: (PGI Nov 2009) B. Selegiline A. Paroxetine B. Mirtazapine C. Haloperidol + chlorpromazine C. Escitalopram D. Venlafaxine D. ECT 29. SSRIs should be carefully used in the young for the 36. A patient comes in stuporous condition. Patient’s management of depression due to increase in: parents give history of patient being continu- (DNB NEET 2014-15) ally sad and suicidal attempts and not eating and A. Nihilism ideation B. Guilt ideation sleeping for most of the time. The treatment is: C. Suicidal ideation D. Envious ideation (AIIMS 2000) mebooksfree.com mebooksfree.com mebooksfree.com mebooksfree.comA. ECT B. Antidepressantmebooksfree.com mebooksfree.com 30. Features of serotonin syndrome associated with C. Antipsychotic D. Sedative SSRIs, and MOAIs are all except: (DNB NEET 2014-15) 37. A patient on antidepressant therapy developed A. Tremors B. Agitation sudden hypertension on consuming cheese. The C. Cardiovascular collapse D. Hypothermia antidepressant is possibly: (PGI 1999) A. Amitriptyline B. Tranylcypromine 31. The clinical effects of the antidepressant drugs is C. Fluoxetine D. Sertraline mainly based on: (DNB NEET 2014-15) A. Change in neurotransmitter receptor sensitivity 38. Tricyclic antidepressants have all of the following B. Decreased levels of neurotransmitters actions except: (PGI 1999) C. Change in efficacy of neurotransmitters A. Anticholinergic action D. None of the above B. Anti MAO (monoamine acid oxidase) action C. Blocks 5 HT (serotonin) or NE (norepinephrine) mebooksfree.com mebooksfree.com 32. Mechanism of action ofmebooksfree.com bupropion is: mebooksfree.com mebooksfree.com mebooksfree.com reuptake (DNB NEET 2014-15) D. Causes sedation A. Increased levels of GABA B. Increased levels of norepinephrine in the synap­ 39. A patient on treatment for psychiatric disorder tic cleft takes overdose of a drug, develops bradycardia, C. Increased levels of dopamine in the synaptic cleft hypotension, decreased sweating and salivation. D. Both B and C The likely drug is: (AIIMS 1999)

mebooksfree.com mebooksfree.com mebooksfree.com mebooksfree.com mebooksfree.com mebooksfree.com

mebooksfree.com mebooksfree.com mebooksfree.com mebooksfree.com mebooksfree.com mebooksfree.com mebooksfree.com mebooksfree.com mebooksfree.com mebooksfree.com mebooksfree.com mebooksfree.com

Mood Disorders 37

A. Amitriptyline B. Lithium 48. Phototherapy is used to treat which of the follow- C. Selegiline D. Amphetamine ing psychiatric condition: (DNB NEET 2014-15) A. Depression 40. Tricyclic antidepressant are contraindicated in: B. Mental retardation (DNB 1997, AI 1991) mebooksfree.com mebooksfree.com mebooksfree.com mebooksfree.comC. Schizophrenia mebooksfree.com mebooksfree.com A. Glucoma B. Brain tumor D. Obsessive compulsive disorder C. Bronchial asthma D. Hypertension 41. Following drugs have abuse liability except: Suicide (DNB 2003) 49. Increased suicidal tendency is associated with: A. Buprenorphine (DNB NEET 2014-15) B. Alprazolam A. Increased noradrenaline C. Fluoxetine B. Decreased serotonin D. Dextropropoxyphene C. Decreased dopamine 42. Tianeptine acts by: (AIIMS 1998) D. Increased GABA A. MAO inhibitor 50. Risk factor for suicide in depression are all except: B. Serotonin uptake inhibitor mebooksfree.com mebooksfree.com mebooksfree.com mebooksfree.com (DNBmebooksfree.com June 2011) mebooksfree.com C. Serotonin uptake enhancer A. Female D. 5-HT agonist B. Male > 45 years 43. What is/are the side effects of SSRI: C. Child with conduct disorder A. Insomnia B. Sedation D. Family history C. Nausea D. Seizure precipitation 51. Suicidal tendencies are most common in: E. Weight gain (PGI 2000) 44. Not true regarding serotonin syndrome is: A. Involutional depression A. It is predictable and not idiosyncratic B. Reactive depression B. SSRIs and MAOIs cause it C. Psychotic depression C. IV dantrolene is the treatment of choice D. Childhood depression D. Hypertension, hyperthermia and hyperreflexia mebooksfree.com mebooksfree.com mebooksfree.com 52. mebooksfree.comEmile Durkheim is linked with workmebooksfree.com on which of mebooksfree.com are the signs the following conditions in psychiatry? 45. Stimulation of which of the following nerve cause (DNB NEET 2014-15) elevation of mood: (AIIMS Nov 2009) A. Suicide A. Olfactory nerve B. Optic nerve B. Obsessive compulsive disorder C. Trigeminal nerve D. Vagus nerve C. Anxiety disorder D. Schizophrenia 46. Following are the somatic therapies used in depression, except: (DNB NEET 2014-15) 53. Incidence of suicide in India is: (PGI June 2005) A. Electroconvulsive therapy A. 8-10/100 population B. Deep brain stimulation B. 8-10/1000 population C. Transcranial magnetic stimulation C. 8-10/10000 population mebooksfree.com mebooksfree.comD. Ultrasound brain stemmebooksfree.com stimulation mebooksfree.comD. 8-10/100000 population mebooksfree.com mebooksfree.com 47. The evidence-based psychological therapy of 54. Suicidal tendencies are seen in: (PGI 2002) choice for depression is: (AIIMS May 2014) A. Depression A. Group discussion therapy B. Post-traumatic stress disorder B. Counselling C. Schizophrenia C. Cognitive behavioral therapy D. Substance abuse D. Psychological psychotherapy E. Anxiety

mebooksfree.com mebooksfree.com mebooksfree.com mebooksfree.com mebooksfree.com mebooksfree.com

mebooksfree.com mebooksfree.com mebooksfree.com mebooksfree.com mebooksfree.com mebooksfree.com mebooksfree.com mebooksfree.com mebooksfree.com mebooksfree.com mebooksfree.com mebooksfree.com

38 Review of Psychiatry Bipolar Disorder 62. Mania is characterized by: (PGI 1999) A. Paranoid delusions B. Loss of orientation Classification C. High self esteem D. Loss of insight mebooksfree.com mebooksfree.com 55. Chromosome associatedmebooksfree.com with bipolar disorder: 63. mebooksfree.comWhich of the following is not a symptommebooksfree.com of mania: mebooksfree.com (PGI Dec 2005) (DNB NEET 2014-15) A. Chromosome16 B. Chromosome 13 A. Distractibility B. Elated mood C. Chromosome 18 D. Chromosome 11 C. Delusion of grandeur D. Increased sleep E. Chromosome 23 56. Bipolar II disorder includes: Clinical Vignettes (DNB NEET 2014-15, AIIMS 2011) A. Cyclothymic disorder 64. A 20-year-old man has presented with increased B. Dysthymia alcohol consumption and sexual indulgence, C. Single manic episode irritability, lack of sleep, and not feeling fatigued D. Major depression and hypomania even on prolonged periods of activity. All these changes have been present for 3 weeks. The most 57. All of the following are included in diagnosis of likely diagnosis is: (AI 2003) mebooksfree.com mebooksfree.combipolar disorder exceptmebooksfree.com: (AI, 2007) mebooksfree.comA. Alcohol dependence mebooksfree.com mebooksfree.com A. Mania alone B. Schizophrenia B. Depression alone C. Mania C. Mania and depression D. Impulsive control disorder D. Mania and anxiety 65. A 67-year-old lady is brought in by her 6 children 58. Which of the following is/are included in bipolar saying that she has gone senile. Six months after disorders: (PGI Nov 2010) her husband’s death she has become more reli- A. Hypomania B. Cyclothymia gious, spiritual and gives lots of money in dona- C. Paranoid disorder D. Hyperthymia tion. She is occupied in too many acti­vities and E. Kleptomania sleeps less. She now believes that she has a goal 59. The period of normalcy is seen between two psy- to change the society. She does not like being chosis. The diagnosis is: (AI 1999) brought to the hospital and is argumentative on mebooksfree.com mebooksfree.comA. Schizophrenia mebooksfree.com mebooksfree.combeing questioned on her doings. Themebooksfree.com diagnosis is: mebooksfree.com B. Manic depressive psyhosis (AI 2002) C. Alcoholism A. Depression D. Depression B. Schizophrenia C. Mania Symptoms and Diagnosis D. Impulse control disorder 66. A 42-year-old male with a past history of a manic 60. According to the ICD-10 revision, for establishing episode presents with an illness of 1 month dura- a diagnosis of mania, the symptoms should persist tion characterized by depressed mood, anhedonia for at least: (AIIMS May 2014, DNB 2010) and profound psychomotor retardation. The most A. 1 week B. 2 weeks appropriate management strategy is prescribing C. 3 weeks D. 4 weeks a combination of: (AIIMS 2004) mebooksfree.com mebooksfree.com 61. The clinical features mebooksfree.comof mania include: mebooksfree.comA. Antipsychotics and antidepressantsmebooksfree.com mebooksfree.com (PGI 2006, 2002) B. Antidepressants and mood stabilizers A. Anhedonia C. Antipsychotics and mood stabilizers B. Elated mood D. Antidepressants and benzodiazepines C. Avolition D. Delusion of grandiosity Treatment E. Distractibility 67. Drug of choice in acute mania is: (DNB NEET 2015)

mebooksfree.com mebooksfree.com mebooksfree.com mebooksfree.com mebooksfree.com mebooksfree.com

mebooksfree.com mebooksfree.com mebooksfree.com mebooksfree.com mebooksfree.com mebooksfree.com mebooksfree.com mebooksfree.com mebooksfree.com mebooksfree.com mebooksfree.com mebooksfree.com

Mood Disorders 39

A. Lithium B. Chlorpromazine C. Treatment of first depressive episode C. Valproic acid D. Risperidone D. Prevention of recurrence in bipolar mood dis­ order 68. Which of the following drugs is/are used in treat- mebooksfree.com mebooksfree.comment of acute mania?mebooksfree.com (DNB NEET 2014-15) 76. mebooksfree.comA male patient with bipolar disordermebooksfree.com is controlled mebooksfree.com A. Lithium B. Oxcarbazepine on medications. Symptoms of mania start to C. Risperidone D. All the above appear whenever he himself tapers down the drugs. What type of treatment can improve com- 69. Treatment of bipolar disorder includes: (PGI 2011) A. Antidepressant drugs pliance in this patient? (AIIMS Nov 2015) B. Aversion therapy A. Psychoeducation C. ECT B. CBT D. Lithium carbonate C. Supportive psychotherapy D. Insight oriented psychotherapy 70. Drug of choice for rapid cycling manic depressive psychosis (bipolar disorder) is:(DNB 2004, AI 1999) 77. All of the following are mood stabilizing drug in A. Lithium B. Carbamazepine bipolar disorder except: (PGI MAY 2013) C. Sodium valproate D. Haloperidol A. Lithium B. Valproate mebooksfree.com mebooksfree.com mebooksfree.com mebooksfree.comC. Carbamazepine D. Clonazepammebooksfree.com mebooksfree.com 71. Prophylactic maintenance serum levels of lithium E. Lamotrigine is: (AI 1994, DNB 1997) A. 0.2–0.8 mEq/L B. 0.7–1.2 mEq/L 78. A 30-years-old pregnant woman comes to your C. 1.2–2.0 mEq/L D. 2.0–2.5 mEq/L clinic with decreased sleep, increased appetite and hyperactivity for last 2 weeks. A diagnosis of 72. True about Lithium toxicity: (PGI 2012) mania is made. Further probing reveals four epi- A. Causes ebstein anomaly sodes of major depression in the past two years. B. Decreases neutrophil count What drug will you prescribe to this patient? C. Decreases eosinophil count (AIIMS Nov 2015) D. Optimum concentration is 0.2–0.6 mEq/L A. Haloperidol B. Lithium E. Decreases sodium excretion C. Promethazine D. Clonazepam 73. True about Lithium treatment in mania: mebooksfree.com mebooksfree.comA. Commonest side effectmebooksfree.com is tremor ANSWERSmebooksfree.com mebooksfree.com mebooksfree.com B. Toxic level is <1.5 mg/ dL serum level C. Amyloid is DOC for Li induced diabetes insipidus 1. B. D. Lithium is 90% protein bound 2. B. E. Tremor is treated with propranolol 3. D. 4. A. 74. A patient is brought to the casualty in the state of 5. A, C and D. altered sensorium. He was on lithium treatment 6. A, C. for affective disorder and has suffered through Depression is the most common psychiatric ill­ an attack of epileptic fits. On examination he has ness associated with both hypothyroidism and worsening tremors, increased DTR’s and inconti- hyperthyroidism. Also, in adrenal disorders nence of urine. He has also undergone an episode like Cushing’s syndrome and Addison’s disease, mebooksfree.com mebooksfree.comof severe gastroenteritismebooksfree.com 2 days ago. The serum mebooksfree.comdepression is commonly associated.mebooksfree.com mebooksfree.com lithium was found to be 1.95 mEq/L. The probable 7. A, B, C. In hypopituitarism, depression is commonly cause for his present state is: (AIIMS 2001) seen. A. Lithium toxicity B. Dehydration 8. C. Few common medications which cause C. Manic episode D. Depressive stupor depression include antihypertensives (reserpine, 75. Best use of lithium is in: (DNB NEET 2014-15) methyldopa, beta blockers), steroids (cortico- A. Treatment of schizophrenia steroids, oral contraceptive pills), barbiturates B. Treatment of recurrent depression and benzodiazepines (like clonazepam).

mebooksfree.com mebooksfree.com mebooksfree.com mebooksfree.com mebooksfree.com mebooksfree.com

mebooksfree.com mebooksfree.com mebooksfree.com mebooksfree.com mebooksfree.com mebooksfree.com mebooksfree.com mebooksfree.com mebooksfree.com mebooksfree.com mebooksfree.com mebooksfree.com

40 Review of Psychiatry

9. C. 22. C. The diagnosis in this case is depression. The 10. B. depression can be precipitated by various 11. A. According to DSM-IV and DSM-5, to diagnose stressors, and irrespective of what precipitated depression at least one of the following two it, it should always be treated. Further, all the mebooksfree.com mebooksfree.comsymptoms should bemebooksfree.com present (1) depressed mood mebooksfree.comavailable antidepressants have similarmebooksfree.com efficacy mebooksfree.com (2) loss of interest or pleasure. and the choice of antidepressants is usually 12. C. There are characteristic disturbances of sleep dictated by the side effect profile of the drug. (early morning insomnia and reduced latency SSRIs are usually used as the first antidepressants of REM sleep) in depression. because of their favorable side effect profile. 13. A,B,C. 23. A. Psychotic symptoms are seen more commonly 24. A. The postpartum blue includes transient symp­ in both endogenous and melancholic depres­ toms such as irritability, tearfulness, sadness sion (depression in involutional stage). It can of mood, decreased sleep and appetite. These also be seen in double depression, though less symptoms usually get resolved by 10th day commonly (double depression is depression postpartum. superimposed over dysthymia). 25. B. The suicidal intent in a person with formal mebooksfree.com mebooksfree.com 14. A,C. mebooksfree.com mebooksfree.comthought disorder (most likely amebooksfree.com patient with mebooksfree.com The presence of psychotic symptoms itself makes schizo­phrenia), chronic severe physical illness a depression severe, hence psychotic symptoms and social isolation, should be taken very serious­ are seen only in severe depression. The psychotic ly and immediate measures taken must include symptoms can be either mood congruent or assessment by a specialist. If patient has mood incongruent. expressed suicidal intent in an inebriated state, 15. A. it must still be ensured that he doesn’t harm 16. A, C and D. himself however a specialist assessment can be The recovery is often incomplete in patients with deferred till he is sober. depression. 26. C. Long standing and less severe depressive symp­ 17. A. toms along with normal sleep and appetite indi­ 18. A. The most common type of postpuerperal psycho­ cates dysthymia. sis is depression. 27. A. In this case, the better answer would be mebooksfree.com mebooksfree.com 19. B. In this case, patient hasmebooksfree.com significant somatic symp­ mebooksfree.com mebooksfree.com mebooksfree.com depression. Though the question, mentions toms such as aches and pain and generalized that sleep and appetite is normal, however weakness. In a large number of patients, depres­ sion presents mostly with somatic complaints the fact that he is cut off from society indi­ and patient may deny psychological symptoms cates severe symptomatology and hence such as sadness of mood. Further, in this patient depression would be a better diagnosis than there are sleep and appetite abnormalities along dysthymia here. Usually in patients with dys­ with loss of interest which clinches the diagnosis thymia, the overall functioning is minimally of depression. affected and that’s an important clue for the 20. A. This patient has depression with psychotic symp­ diagnosis. toms. The patient belief that he committed sins 28. A,B,C. in his life, and the fact that despite his wife assur­ 29. C. The use of SSRIs can increase suicidal ideations. mebooksfree.com mebooksfree.comances he continues mebooksfree.comto hold the belief is suggestive mebooksfree.comThis side effect is more common mebooksfree.comin children and mebooksfree.com of delusion. Hence, this patient should be treated adolescents and hence these medications should with antidepressants and antipsychotics. be used cautiously in that age group. 21. B. In children and adolescents, depression 30. D. In serotonin syndrome, hyperthermia is a feature. frequently presents with irritability, lack of 31. A. The recent research has shown that its not the interest and changes in behavior such as increase in neurotransmitters levels in synapse withdrawn behavior or quarrelsome behavior. which causes antidepressant effect. Rather, sec­ Its important to rule out depression first. ondary to increased neurotransmitter levels, the

mebooksfree.com mebooksfree.com mebooksfree.com mebooksfree.com mebooksfree.com mebooksfree.com

mebooksfree.com mebooksfree.com mebooksfree.com mebooksfree.com mebooksfree.com mebooksfree.com mebooksfree.com mebooksfree.com mebooksfree.com mebooksfree.com mebooksfree.com mebooksfree.com

Mood Disorders 41

receptor sensitivity changes over a course of time 49. B. Decreased levels of 5 HIAA (which is a metabo­ and that is responsible for antidepressant effect. lite of serotonin) are related to increased risk of 32. D. suicide. 33. D. 50. A. Males have higher suicide risk than females. Please mebooksfree.com mebooksfree.com 34. A. This patient was mebooksfree.commost likely on tricyclic antide­ mebooksfree.comremember that females make mebooksfree.commore suicide mebooksfree.com pressants and it appears to be a case of tricyclic attempts than males, however males complete antidepressant over-dosage as the patient is suicide more commonly than females. This dif­ experiencing arrhythmias, hypotension and has ference is mostly due to method used, males also developed altered sensorium. The mainstay tend to use more lethal methods such as gun of treatment in TCA induced cardiotoxicity is and hence are more likely to complete suicide. intravenous sodium bicarbonate. It is used if the 51. A, C. QRS interval is prolonged (usually more than 100 Endogenous depression, depression with psy­ milliseconds) and can reverse the toxic effects of chotic symptoms (psychotic depression) and TCAs. Because of large volume of distribution involutional depression (depression with mel­ and high protein binding of TCAs, hemodialysis ancholic features) are associated with higher is not effective. Further flumazenil and fomepi­ suicide risk. mebooksfree.com mebooksfree.comzole have no role. mebooksfree.com 52. mebooksfree.comA. Emile Durkheim studied extensivelymebooksfree.com the social mebooksfree.com 35. D. In depression with suicide risk, ECT is the treat­ factors associated with suicide. ment of choice. 53. D. The data for incidence of suicide is released 36. A. This patient most likely has depression with by government every year. According to NCRB stupor. ECT is again the treatment of choice. (National Crime Record Bureau), in 2014, the 37. B. This is history of cheese reaction on MAO inhibi­ suicide rate in India was 10.6/lac of population. tors. 54. A,C,D. 38. B. 55. C. 39. A. The symptoms are suggestive of tricyclic antide­ 56. D. pressants overdose (anticholinergic side effects). 57. B. Even a single episode of mania is sufficient to make 40. A. Due to anticholinergic action, TCAs should be a diagnosis of bipolar disorder. avoided in glaucoma. 58. A,B. mebooksfree.com mebooksfree.com 41. C. SSRIs do not havemebooksfree.com any abuse liability. Both 59. mebooksfree.comB. Manic depressive psychosis was themebooksfree.com older name mebooksfree.com opioids (buprenorphine, dextropropoxyphene) for bipolar disorder. In bipolar disorder, in bet­ and benzodiazepines (alprazolam) have abuse ween the episodes, patient is usually normal. liability. 60. A. 42. C. 61. B,D,E. 43. A,B,C,E. 62. C,D. SSRIs can cause both sedation as well as insom­ Insight is absent in mania and usually high self nia. In long-term they can cause weight gain. esteem is also a clinical feature. Nausea, diarrhoea, anxiety and sweating are 63. D. The sleep is usually decreased in mania some common side effects. 64. C. Please remember that in manic stages, the sub­ 44. C. Dantrolene is not the treatment of choice, though stance intake also frequently increases. it is at times used to control the hyperthermia. 65. C. Kindly don’t get confused with the fact that mebooksfree.com mebooksfree.com 45. D. Vagal nerve stimulationmebooksfree.com can be used for treatment mebooksfree.comthe symptoms are following husband’smebooksfree.com death. mebooksfree.com of depression. Even negative life events can precipitate manic 46. D. ECT, deep brain stimulation as well as transcra­ episode. This patient has increased religio­ nial magnetic stimulation can be used for treat­ sity, overspending (giving excessive donation), ment of depression. increased activity levels, decreased sleep, new 47. C. interests and goals (of changing society) and lack 48. A. Depression associated with a seasonal pattern of insight (doesn’t want to come to hospital). All can be treated with phototherapy. these symptoms are suggestive of mania.

mebooksfree.com mebooksfree.com mebooksfree.com mebooksfree.com mebooksfree.com mebooksfree.com

mebooksfree.com mebooksfree.com mebooksfree.com mebooksfree.com mebooksfree.com mebooksfree.com mebooksfree.com mebooksfree.com mebooksfree.com mebooksfree.com mebooksfree.com mebooksfree.com

42 Review of Psychiatry

66. B. The patient had a manic episode in past and cur­ 75. D. rently he is in severe depression (as suggested 76. A. Psychoeducation is a form of psychological by profound psychomotor retardation). The intervention in which patient as well as family complete diagnosis would be bipolar disorder members are educated about various aspects of mebooksfree.com mebooksfree.com(currently severe depressivemebooksfree.com episode). Hence, mebooksfree.comdisease and its treatment. It involvesmebooksfree.com discussion mebooksfree.com this patient should receive both mood stabilizers about the symptoms, the need for medications and antidepressants. as well as maintenance of a regular life style. Psy­ 67. A. This question doesn’t make much sense. There is choeducation decreases the chances of relapses no drug of choice in acute mania. The drug is cho­ in bipolar disorder. sen depending on the symptoms. If symptoms 77. D. are severe and we need immediate improve­ 78. A. This is an interesting question and slightly con­ ment, a combination of mood stabilizers­ with troversial too. We need to consider various fact. antipsychotic would be preferred. If symptoms First of all, since this patient had 4 episodes are less severe, either lithium, valproate or an (mania + depression) in last 5 years, prior to antipsychotic can be used. Here, the answer is the current manic episode, ideally she should given as lithium as it is the prototypical drug already be on a prophylaxis. However we have mebooksfree.com mebooksfree.comused in mania andmebooksfree.com can be considered as a gold mebooksfree.comnot been provided with any informationmebooksfree.com about mebooksfree.com standard. It is true that lithium takes 1-3 weeks the prophylaxis. Now, the question is asking for its onset of action, but that doesn’t mean about the management of acute manic episode. that it can’t be used in acute mania. In fact, all According to APA (American Psychiatric Associa­ the guidelines start the treatment algorithm for tion) practice guidelines, in a patient with severe acute mania with lithium only. mania a combination of mood stabilizers (like 68. D. lithium or valproate) and antipsychotics should 69. A,C,D. be used, whereas in mild to moderate mania, Lithium and antidepressant are obviously used in monotherapy with lithium, valproate or an an­ bipolar disorders. In cases of severe mania which tipsychotic (such as haloperidol) can be used. is not responding to medications, or in bipolar This is for all patients and not specifically for depression with high suicide risk, ECTs can also pregnancy. Further, APA guidelines mention that mebooksfree.com mebooksfree.combe used. mebooksfree.com mebooksfree.com“in pregnancy, antipsychotics maymebooksfree.com represent an mebooksfree.com 70. C. alternative to lithium for treating the symptoms 71. B. of mania. In addition, there is no evidence of 72. E > A. teratogenicity with exposure to haloperidol, Dehydration as well as low sodium levels pre­ perphenazine, thiothixene and trifluopera­ dispose to lithium toxicity. Ebstein anomaly is a zine”. It must also be remembered that lithium teratogenic effect of lithium and as such is not a takes around 1-3 weeks to start acting whereas sign of lithium toxicity. Lithium causes neutro­ antipsychotics have a much faster onset, and in philia and eosinophilia. a patient who is in acute manic episode for last 73. A, C, E. two weeks, we need faster response. Maudsley 74. A. The gastroenteritis causes dehydration and prescribing guidelines, which is another well may result in lithium toxicity (the body handles accepted prescribing guideline says “In acute lithium similarly to sodium. In presence of mania in pregnancy, use an antipsychotic, if mebooksfree.com mebooksfree.comdehydration, sodiummebooksfree.com absorption is increased and mebooksfree.comineffective, consider ECT”. Hence, mebooksfree.comin view of these mebooksfree.com lithium absorption is also increased in kidneys). reference for acute management of mania in The lithium toxicity may present with tremors, pregnancy, haloperidol would be a better answer increased reflexes and seizure. than lithium.

mebooksfree.com mebooksfree.com mebooksfree.com mebooksfree.com mebooksfree.com mebooksfree.com

mebooksfree.com mebooksfree.com mebooksfree.com mebooksfree.com mebooksfree.com mebooksfree.com mebooksfree.com mebooksfree.com mebooksfree.com mebooksfree.com mebooksfree.com mebooksfree.com

mebooksfree.com mebooksfree.com mebooksfree.com mebooksfree.com mebooksfree.com mebooksfree.com

Chapter Neurotic, Stress Related and 4 Somatoform Disorders

ANXIETY In panic disorder, the patients have recurrent panic attacks which are not restricted to any particular situa- mebooksfree.com mebooksfree.comAnxiety is a common experience.mebooksfree.com It is an alerting signal tion ormebooksfree.com setting. The patient is usually freemebooksfree.com from anxiety mebooksfree.com and helps a person to take measures to deal with a threat. symptoms in between the attack however anticipatory It must be differentiated from fear. Fear is the response anxiety (fear that next panic attack can occur anytime) one would have if he sees a snake. The fear is a response is common. The mean age of presentation is around 25 to a known, external and definite threat. Anxiety is the years and females are two to three times more commonly response one would have before exams. It is the response affected than men. Panic disorder presents with a number to an unknown, internal and vague threat. of comorbid conditions, most commonly agoraphobia. The neurotransmitters which have been implicated in Manifestations of Anxiety: panic disorders include norepinephrine, serotonin and • Feeling of nervousness GABAQ. Recently cholecystokininQ has also been found • Sweating, tachycardia, restlessness, tremors, mydriasis as a mediating neurotransmitter in panic disorder. • Diarrhea, urinary frequency Q mebooksfree.com mebooksfree.com• Cold clammy skin , hyperreflexia.mebooksfree.comDifferentialmebooksfree.com Diagnosis mebooksfree.com mebooksfree.com ANXIETY DISORDERS Due to predominance of somatic symptoms, panic dis- order must be differentiated from common physical dis- Anxiety disorders are a group of related disorders which orders such as myocardial infarctionQ, angina, mitral include: valve prolapse asthma, pulmonary embolism, pheo- • Panic disorder chromocytoma, carcinoid syndrome, hypoglycemia, • Agoraphobia hyperthyroidism. • Specific phobia • Social anxiety disorder Treatment • Generalized anxiety disorder. Usually a combination of pharmacotherapy and psycho- Panic Disorder therapy is used. mebooksfree.com mebooksfree.com mebooksfree.com A. Pharmacotherapymebooksfree.com: The drugs mostlymebooksfree.com used include mebooksfree.com Panic attack is an acute attack of intense anxiety accom- benzodiazepinesQ and SSRIs. Frequently, both ben- panied by “feeling of impending doom”. The symptoms zodiazepines and SSRIs are started concurrently, fol- during panic attack usually involve sudden onset of pal- lowed by slow tapering of benzodiazepines. Other pitations, chest pain, choking sensations, dizziness and medications which are used include venlafaxine, bus- feeling of unreality (depersonalization or derealization). pirone and clomipramine. Along with these physical symptoms there is also a fear B. Psychotherapy: Cognitive behavioral therapyQ is quite of dying, losing control or going mad. effective in management of panic disorder. Other less

mebooksfree.com mebooksfree.com mebooksfree.com mebooksfree.com mebooksfree.com mebooksfree.com

mebooksfree.com mebooksfree.com mebooksfree.com mebooksfree.com mebooksfree.com mebooksfree.com mebooksfree.com mebooksfree.com mebooksfree.com mebooksfree.com mebooksfree.com mebooksfree.com

44 Review of Psychiatry

commonly used therapies include Relaxation tech- 2. Natural environment type (storms, water, height, etc.) niques and psychodynamic psychotherapy. 3. Blood-injection-injury type (needles, invasive medical procedures) Agoraphobia 4. Situational type (cars, elevators, planes) mebooksfree.com mebooksfree.com mebooksfree.com 5. Others.mebooksfree.com mebooksfree.com mebooksfree.com It is the fear of places from where escape might be diffi- cultQ. This basic fear can manifest in various forms such as: Treatment • Fear of being in open spacesQ A. Pharmacotherapy: The pharmacotherapy is at best • Fear of crowded placesQ used as an adjunct to psychotherapy and includes • Fear of enclosed placesQ benzodiazepines, beta blockers and, SSRIs. • Fear of travelling aloneQ B. Psychotherapy: Behavior therapy is the most effective • Fear of using public transportations. treatmentQ for phobias. A variety of behavioral tech- Agoraphobia and panic disorder usually coexist. niques, all of which involve exposure to phobic stimu- Agoraphobia is the most disabling phobia and patient lus, have been used, which are described as follows: may become home bound. • Systematic desensitization: In this method, the patient is exposed to a series of anxiety provoking Treatment mebooksfree.com mebooksfree.com mebooksfree.com mebooksfree.comstimuli, starting with the least anxietymebooksfree.com provoking mebooksfree.com A. Pharmacotherapy: The pharmacotherapy usually stimulus. After the exposure, relaxation techniques includes benzodiazepines and SSRIs. Other medica- (usually progressive muscle relaxation) are used to tions which are used include venlafaxine, buspirone induce relaxation. As the patient masters the tech- and clomipramine. nique of relaxation in the presence of an anxiety B. Psychotherapy: Cognitive behavioral therapy is fre- provoking stimuli, he moves up to the next stimu- Q quently used. Behavioral therapy (using techniques lus. This technique hasbest evidence in treatment such as systematic desensi­tization, exposure and of phobias. response prevention, floodingQ) is also effective. Less • Therapeutic graded exposure or in vivo exposure (or exposure and response prevention): It is similar to commonly used are relaxation techniques and psy- systematic desensitization except that no relaxa- chodynamic psychotherapy. tion techniques are used. The patient learns to get mebooksfree.com mebooksfree.com mebooksfree.com mebooksfree.comhabituated to anxiety. mebooksfree.com mebooksfree.com Specific Phobias • Flooding (Implosion): Here, the patient is exposed A specific phobia is a strong, persistent and irrational fear to phobic stimulus in its most severe form. The of an object or a situation. The DSM-5 includes distinctive patient experiences intense anxiety which gradu- types of phobias: ally decreases. Q 1. Animal type (spiders, insects, dogs) • Modeling (Participant modeling ): Here, therapist him­self makes the contact with phobic stimulus and demonstrates this to the patient. Patient learns Table 1: Common phobias. by imitation, primarily by observation. Apart from Acrophobia Fear of heights behavioral therapy, other less commonly used psy- Ailurophobia Fear of cats chotherapeutic techniques include Psychodynamic Hydrophobia Fear of water psychotherapy (Insight oriented psychotherapy), hypnosis, supportive therapy and family therapy. mebooksfree.com mebooksfree.comClaustrophobia mebooksfree.comFear of closed spaces mebooksfree.com mebooksfree.com mebooksfree.com Cynophobia Fear of dogs Social Anxiety Disorder (Social Phobia) Mysophobia Fear of dirt and germs Pyrophobia Fear of fire It involves the fear of social situations, including situa- tions that involve contact with strangers. Patients with Xenophobia Fear of strangers this disorder are afraid of embarrassing themselves in a Zoophobia Fear of animals social situation. The treatment is usually similar to speci­ Thanatophobia Fear of death fic phobias.

mebooksfree.com mebooksfree.com mebooksfree.com mebooksfree.com mebooksfree.com mebooksfree.com

mebooksfree.com mebooksfree.com mebooksfree.com mebooksfree.com mebooksfree.com mebooksfree.com mebooksfree.com mebooksfree.com mebooksfree.com mebooksfree.com mebooksfree.com mebooksfree.com

Neurotic, Stress Related and Somatoform Disorders 45 Generalized Anxiety Disorder Etiology This disorder is characterized by excessive anxiety which Serotonin dysregulationQ is considered to be involved is gene­ralized and persistent and is not restricted to any in the etiopathogenesis­ of OCD. Less evidence exists for mebooksfree.com mebooksfree.comparticular situation (also calledmebooksfree.com “freely floating” anxiety) dysregulationmebooksfree.com of noradrenergic system in OCD.mebooksfree.com mebooksfree.com and excessive worries. The physical symptoms associ- The neuroanatomical model of OCD emphasizes ated with anxiety are also present. The treatment includes the role of cortico-striatal-thalamic-cortical circuitry pharmacotherapy (SSRIs, benzodiazepines, buspirone (CSTC). This circuit starts with prefrontal cortex and pro- and venlafaxine) and psychotherapy (cognitive behavio- jects to striatum which further projects to thalamus and ral, insight oriented psychotherapy and supportive psy- then back to prefrontal cortex. Dysfunction in this circuit chotherapy). is considered to be responsible for the symptoms of OCD.

OBSESSIVE COMPULSIVE AND Symptoms RELATED DISORDERS OCD has four major symptom patterns. A. Contamination: Most commonly patients present with mebooksfree.com mebooksfree.comObsessive-Compulsivemebooksfree.com Disorder (OCD) obsessionmebooksfree.com of contamination followedmebooksfree.com by washing mebooksfree.com behavior and avoidance of situations which provoke The essential feature of this disorder includes recurrent obsessive thoughts. For example, a patient repeatedly obsessional thoughts and compulsive acts. gets thought that his hands are dirty, which causes Obsessions are defined by the following properties: anxiety, he understands that this thought is senseless A. Recurrent and intrusive thoughts, images or impulses which cause marked anxiety or distress and tries to stop this thought (obsessional thought) B. The person recognizes that the obsessional thoughts, however is forced to repeatedly wash his hands (com- images or impulses are a product of their own mindQ pulsive behavior) which decreases this thought for (and not imposed by others such as is in thought some time. He further avoids using public toilet as insertion) these thoughts get increased in a dirty environment (avoidance). C. The person recognizes that the thoughts, images or B. Pathological doubt: Second most common pattern impulses are irrational and senselessQ and experi- is the obsession of doubt which is usually followed mebooksfree.com mebooksfree.comences the obsessions andmebooksfree.com compulsions as ego dys- mebooksfree.com mebooksfree.com mebooksfree.com by compulsion of checking. For example, a patient tonic (i.e. unwanted and unacceptable) (in contrast would repeatedly doubt if he had locked the door a patient with a delusion, believes in the delusion properly (obsession) and would repeatedly check the and doesn’t find it senseless or irrelevant) lock (compulsion). D. The person attempts to suppress or resist such C. Intrusive thoughts: Here, patient gets intrusive obses- thoughts, images or impulses or tries to neutralize sional thoughts without an observable compulsion, them, with some other thoughts or actions. though mental compulsions are commonly present. Compulsions are defined by following properties: The thoughts are usually with sexual and aggres- A. Repetitive behaviors (such as hand washing, check- sive contentQ. For example, a patient repeatedly ing) or mental acts (such as counting, praying) that gets the thought about having sex with god, this the person performs in response to an obsession. thought causes intense anxiety and patient under- mebooksfree.com mebooksfree.com B. The repetitive behaviors mebooksfree.comand mental acts are done to standmebooksfree.com that this thought is senseless andmebooksfree.com tries to stop mebooksfree.com reduce the distress and anxiety caused by obsessions. the thought but is not able to do so (obsessional The symptoms of obsessions and compulsions should thought), to decrease the anxiety patient starts to be present for at least two weeks for the diagnosis of OCD chant prayers in his mind which decreases the anxi- The lifetime prevalence of OCD is around 2-3%. ety temporarily (mental compulsions). Depression is the most common comorbidity in OCD and D. Symmetry: The patient has aneed for symmetry or both must be treated together. precision. This can result incompulsion of slowness.

mebooksfree.com mebooksfree.com mebooksfree.com mebooksfree.com mebooksfree.com mebooksfree.com

mebooksfree.com mebooksfree.com mebooksfree.com mebooksfree.com mebooksfree.com mebooksfree.com mebooksfree.com mebooksfree.com mebooksfree.com mebooksfree.com mebooksfree.com mebooksfree.com

46 Review of Psychiatry For example, a patient would take hours while arrang- TRAUMA AND STRESSOR RELATED ing pens on the table. He would ensure that all the DISORDERS pens are aligned exactly parallel to each other and are at exact same distance to each other. Post-traumatic Stress Disorder (PTSD) mebooksfree.com mebooksfree.com mebooksfree.comand mebooksfree.comAcute Stress Disorder (ASDmebooksfree.com) mebooksfree.com Course and Prognosis These disorders follow significant traumatic events in Around 50% of patients with OCD have a sudden onset of which there is a serious injury or threat of serious injury symptoms. The course is usually chronic. Around 20–30% to self or others and a feeling of helplessness and horror of patients have significant improvement in their symp- during the event. The traumatic events causing PTSD and toms, around 40–50% have moderate improvement and ASD are sufficiently overwhelming to affect anyone (such as war, earthquake, floods, rape, serious accidents). The remaining 20–40% have no improvement or further dete- clinical symptoms are usually seen in the following three rioration. domains: • Intrusion symptoms: These are characterized byflash - Treatment backsQ (individual may feel as if trauma is reoccur- mebooksfree.com mebooksfree.comA combinationQ of pharmacotherapymebooksfree.com and psychotherapy ring)mebooksfree.com and nightmares (dreams about themebooksfree.com trauma). mebooksfree.com is the preferred approach. • Avoidance: The patient avoids all those stimuli which can remind him of the trauma. A. Pharmacotherapy: The standard approach is to start • Arousal symptoms: These include hypervigilance, treatment with an SSRIQ. Clomipramine­ is also consi­ exaggerated startle response, insomnia, poor concen- dered the first line treatment however due to its adverse tration. side effect profile, it is rarely used as a first drug. If In addition, symptoms such as emotional numbingQ, treatment with SSRIs or clomipramine is unsuccess- emotional detachmentQ and anhedoniaQ can also be ful, augmentation with antipsychotics (like haloperi- present. The onset of symptoms may be delayed, if symp- dolQ, quetiapine, risperidone and olanzapine) is used. toms appear 6 monthsQ after the trauma, it is diagnosed Other drugs which have been used include venlafax- as PTSD with delayed onsetQ. ine, lithium, valproate and carbamazepineQ. For a diagnosis of post-traumatic stress disorder, the mebooksfree.com mebooksfree.com B. Psychotherapy: Cognitivemebooksfree.com behavioral therapy rely- above mebooksfree.commentioned symptoms should be presentmebooksfree.com for more mebooksfree.com ing primarily on behavioral technique of exposure than one month, if the duration of symptoms is less than Q and response prevention (ERP) has the best evi- one month, a diagnosis of acute stress disorder is made. dence amongst all the psychotherapeutic techniques. The area of brain involved in the pathogenesis of Exposure and response prevention involves exposure PTSD are hippocampus and amygdalaQ. of patient to a stimulus which is known to produce obsessional thoughts (exposure) followed by asking Treatment the patient to not indulge in the compulsive behavior Q (response prevention). Selective serotonin reuptake inhibitors (SSRIs) are the first line pharmacological treatment in PTSD. Psycho­ Other types of behavioral therapy such as desen- therapeutic interventions include cognitive behavioral sitization, thought stopping, flooding, and aversive therapyQ (treatment of choice), psychodynamic­ psycho- conditioning have also been used. therapy and eye movement desensitization and repro- Psychodynamic psychotherapy, family therapy can mebooksfree.com mebooksfree.com mebooksfree.comcessingmebooksfree.com (EMDR). mebooksfree.com mebooksfree.com also be used. C. Other treatment modalities: In extreme cases that are Adjustment Disorders treatment resistant electroconvulsive therapy and psy- chosurgery can be considered. The psychosurgical These disorders are characterized by emotional responses techniques usually include cingulotomy and capsulo­ to stressful events like financial problems, medical ill- tomy (also known as sub caudate tractotomy). ness, relationship problems or death of a loved one. The

mebooksfree.com mebooksfree.com mebooksfree.com mebooksfree.com mebooksfree.com mebooksfree.com

mebooksfree.com mebooksfree.com mebooksfree.com mebooksfree.com mebooksfree.com mebooksfree.com mebooksfree.com mebooksfree.com mebooksfree.com mebooksfree.com mebooksfree.com mebooksfree.com

Neurotic, Stress Related and Somatoform Disorders 47

symptom complex that deve­lops usually involve anxiety not able to perform a task, however if he suddenly deve­ and depressive symptoms. The symptoms of adjustment lops para­lysis, now his guilt will decrease, as it is under- disorders include depressed mood, anxiety, worry, a feel- stood that paralyzed patient can’t work. So, this patients ing of inability to cope and some degree of disturbance in psyche is unconsciously producing symptoms of paralysis mebooksfree.com mebooksfree.comindividuals daily functioning.mebooksfree.com It is at times difficult to dif- to reducemebooksfree.com the unpleasant guilt feelings. mebooksfree.com mebooksfree.com ferentiate adjustment disorder from depression (depres- Secondary gain: It refers to external psychological­ moti- sion can also follow a negative life event). If the symptoms vation. For example, this patient who developed sudden are severe and a diagnosis of depression can be made, paralysis is now not expected to work outside or make the diagnosis of depression will always get precedence money for the family and he is relieved of his duties. over the diagnosis of adjustment disorder. Also, one needs to differentiate adjustment disorder from uncom- Tertiary gain: It refers to the gain that a third person plicated bereavement/grief reactions (in uncomplicated derives because of patients symptoms. For example, the bereavement, the symptoms and dysfunctions which wife of this paralyzed patient starts to get lots of money develop after death of a loved one are within expected from her parents as they feel sympathetic towards her. limits, whereas in adjustment disorder the symptoms and dysfunction are beyond the expectable reaction to Types mebooksfree.com mebooksfree.comthe stressor). Other differentialmebooksfree.com diagnosis of adjustment mebooksfree.com mebooksfree.com mebooksfree.com A. Dissociative amnesia: Here, the main feature is loss of disorder includes depression, PTSD and brief psychotic memory. The amnesia is usually for traumatic events disorders. These diagnoses should be given precedence of personal significance­ Q (such as accidents or unex- if their diagnostic criterion are met, irrespective of the pected bereavements). For example, a rape survivor is presence of stressors. not able to recall any thing about her rape. Treatment B. Dissociative fugue: It is characterized by a sudden, unexpected travelQ away from home or work place, Psychotherapy is the treatment of choice. Supportive psy- with inability to recall some or all of one’s past. The chotherapy is commonly used. The medications are used basic self care is maintainedQ during the travel and as an adjuvant to psychotherapy and include antidepres- patients behavior during this time may appear com- sants and anti­anxiety drugs. pletely normal to independent observers. Alongside mebooksfree.com mebooksfree.com mebooksfree.com whenmebooksfree.com asked, the patient may be confusedmebooksfree.com about his mebooksfree.com DISSOCIATIVE DISORDERS personal identity or may even assume a new identity (CONVERSION DISORDERS) (e.g. a doctor may claim that he is in H These disorders were previously classified as “hysteria” In DSM-5, dissociative fugue is not fact a cab driver a separate diagnosis. Instead it has however that term is no longer used. Dissociative and give a different been made a specifier (special kind disorders are characterized by disturbances in one or of) of dissociative amnesia. name when asked). more of mental functions such as memory, identity, C. Dissociative stupor: Here, the patient is in stupor perception, consciousness and motor behavior. These which is caused by psychological factors. symptoms are produced by the “psyche” (mind) to deal D. Dissociative trance and possession disorder: It is char- with the unconscious conflicts that are producing anxiety. acterized by loss of sense of identity and full aware- These symptoms are produced unconsciously and help ness of the surroundings. The patient behaves as the patient to get attention. The symptoms appear if taken over by another personality such as a god- mebooksfree.com mebooksfree.comsuddenly and are caused by mebooksfree.compsychological trauma (such mebooksfree.com mebooksfree.com mebooksfree.com dess or a spirit. For example, a middle aged women as stressful events or disturbed relationship). Quite often, claimed that she has been possessed by a goddess and the genesis of dissociative disorders is explained in terms demanded that everybody should pray in front of her. of primary, secondary and tertiary gains. All these gains E. Dissociative disorders of movement and sensation: function unconsciously. Here the patient presents with symptoms that suggest Primary gain: It refers to internal psychological motiva- deficit in motor or sensory functions, however there is tion. For example, a person might be feeling guilty as he is no evidence of any physical disorder. The symptoms

mebooksfree.com mebooksfree.com mebooksfree.com mebooksfree.com mebooksfree.com mebooksfree.com

mebooksfree.com mebooksfree.com mebooksfree.com mebooksfree.com mebooksfree.com mebooksfree.com mebooksfree.com mebooksfree.com mebooksfree.com mebooksfree.com mebooksfree.com mebooksfree.com

48 Review of Psychiatry

are instead caused H indicating that the question was understood. For by psychological The DSM-4 had the diagnosis example, when asked the color of sky, patient may factors.­ Depending of “depersonalization disorder”, in answer it red. Although, the answer is not correct but DSM-5 the name has been changed on the symptoms, to Depersonalization/Derealization it is obvious that patient understood that the question mebooksfree.com mebooksfree.compatient may be dia­ dismebooksfree.com­order. wasmebooksfree.com about color. Other symptoms includemebooksfree.com clouding of mebooksfree.com gnosed with disso- consciousnessQ, auditory and visual hallucinationsQ ciative motor disorder (e.g. paralysis, ataxia), dissocia- and other dissociative symptoms. Ganser’s syndrome tive convulsions (e.g. pseudoseizures) and dissociative is frequently seen in prisoners, however is not con- anaesthesia and sensory loss (e.g. sensory losses, fined only to themQ and can be seen in other popula- visual disturbance). The symptoms often do not con- tions also. firmQ with anatomical and physiological principles (e.g. sensory loss which doesn’t confirm to any nerve Treatment lesion). Usually psychological modalities are used in the treat- The DSM-5 uses the diagnosis ofconversion disor- ment of dissociative disorders. It is important that patient Q (functional neurological symptom disorder) spe- der is not encouraged to assume a “sick-role” and it must be cifically for this cate­gory and classifies it along with the emphasized that the patient is normal. The secondary somatoform disorders. In ICD-10, the term conversion mebooksfree.com mebooksfree.com mebooksfree.comand tertiarymebooksfree.com gains should not be allowed mebooksfree.comotherwise the mebooksfree.com disorder is synonymous with dissociative disorder. In symptoms tend to become persistent. The treatment moda­ the questions given at the end of this chapter, the lities include behavioral therapy, abreactionQ (in abreac- diagnosis of “conversion disorder” refers to the DSM-5 tion, attempt is made to bring the unconscious memories diagnosis.­ La belle indifferenceQ is a phrase used to and emotions, into conscious awareness using hypnosis, describe the feeling of indifference which patients of medications and other techniques) and psychoanalysis. conversion disorders have towards their symptoms. For example, if a person suddenly has a sensory The use of drugs is limited. Benzodiazepines, thiopen- loss, say loss of vision, he would be expected to get tone and amytal have been used for abreaction. extremely concerned about it, however the patient of conversion disorder looks completely unconcerned SOMATOFORM DISORDERS and this unconcern/indifference towards their symp- The patients with somato­ Q H mebooksfree.com mebooksfree.comtoms is called “la belle indifferencemebooksfree.com” . form disordersmebooksfree.com typically The DSM-5 has replacedmebooksfree.com the dia­ mebooksfree.com F. Depersonalization/derealization disorder: In dep- present with physical gnosis of somatoform disorders with “somatic symptom and related dis­ ersonalization patient has a feeling of unreality of symptoms which cannot orders”. self. He feels “as if” he has changed. The patients fre- be explained by any quently report that they feel as if they have detached known medical condition. These patients persistently from their body and are watching themselves like in re­quest for investigations despite repeated negative a movie. The depersonalization is often accompanied findings and reassurances by doctors. The symptoms by derealization, which is a feeling of unreality of the are significantly distres­sing to the patient and cause external world, as if the world is unreal. impairment in social and occupational functioning. G. Dissociative identity disorder (multiple personality dis- There are various types of somatoform disorders: order): Here, two or more distinct personalities exist A. Somatization disorder: The main feature is presence within an individual, with only one of them being evi- mebooksfree.com mebooksfree.comdent at any particular time.mebooksfree.com The different personali- of mebooksfree.commultiple physical symptoms for whichmebooksfree.com no medical mebooksfree.com ties are known as “alters” and the personalities are cause can be ascer- H tained. According In DSM-5, the diagnosis of soma- unaware of each others existence. tization disorder has been removed H. Other dissociative disorders: This category includes to DSM-IV, for a and these patients are given the dia­gnosis of “somatic symptom disor- Q Ganser’s syndrome. The characteristic symptom is diagnosis of somati- der”. The criterion of somatic symp- approximate answersQ (vorbeigehen). The approxi- zation disorder, the toms disorder include the presence of somatic symptoms and also exces- mate answer are the answers which are not cor- following symptoms sive thoughts, feelings and behaviors rect, but bear an obvious relation to the question, should be present: related to these somatic symptoms.

mebooksfree.com mebooksfree.com mebooksfree.com mebooksfree.com mebooksfree.com mebooksfree.com

mebooksfree.com mebooksfree.com mebooksfree.com mebooksfree.com mebooksfree.com mebooksfree.com mebooksfree.com mebooksfree.com mebooksfree.com mebooksfree.com mebooksfree.com mebooksfree.com

Neurotic, Stress Related and Somatoform Disorders 49

• Four pain symptoms (pain at four different sites) The location of the H • Two gastrointestinal symptoms (such as nausea, imagined defect is In DSM-5, the diagnosis of body usually hair, nose dysmorphic disorder has been reor- vomiting, belching) dered. It is now considered as a type • One sexual symptoms (such as erectile or ejacula- and skin. of OCD and related disorders and has mebooksfree.com mebooksfree.com mebooksfree.com DSM-IVmebooksfree.com consid- been removed frommebooksfree.com the category of mebooksfree.com tory dysfunction) somatoform disorders (somatic symp- • One pseudoneurological symptom (such as weak- ers body dysmor- toms and related disorders). ness, visual disturbance, etc). phic disorder as a The patient usually refuses to accept the advice or separate entity and a type of somatoform­ disorder. reassurance of the doctors that there is no physical How­ever ICD-10 clubs this dia­gnosis under the cause of the symptoms. The onset and progression of broader diagnosis of hypochondriasis. symptoms usually bears a close relationship to unplea­ E. Somatoform autonomic dysfunction: This type is characterized by specific and unexplained autonomic sant life events and psychological stressors. symptoms such as palpitations, tremors, sweating, Treatment usually involves psychotherapy. The belching, etc. patient should be made aware that the physical F. Persistent somatoform pain disorder: Here, the symptoms are expression of underlying emotions main complaint is persistent and unexplained pain. and should be helped to cope with the symptoms and In DSM-5, this category has been subsumed under the mebooksfree.com mebooksfree.comunderlying emotions. mebooksfree.com mebooksfree.com mebooksfree.com mebooksfree.com new diagnosis of “somatic symptom disorder”. B. Undifferentiated somatoform disorder: This diag- G. Pseudocyesis: It is a type of somatoform disorder in nosis is used when which the patient has a false belief that she is preg- multiple physical H Q. There are associated objective signs of preg- symptoms are pre- In DSM-5, the diagnosis of undif- nant ferentiated somatoform disorder has nancy like abdominal enlargement (although umbili- sent but full picture been removed and these patients of somatization dis- would also get the broader diagnosis cus does not become everted), reduced menstrual of “somatic symptom disorder”. order is not fulfilled. flow or amenorrhea, subjective sense of fetal move- C. Hypochondriasis: This disorder is characterized by ments, breast engorgement and labour pains at the a preoccupation with the fear of having, or the idea expected date of delivery. Some endocrine changes that one has one or may also be present. H more serious phys- In DSM-5, the diagnosis of hypo- ical illnessesQ. The chondriasis has been replaced by “ill- OTHER NEUROTIC DISORDERS mebooksfree.com mebooksfree.com nessmebooksfree.com anxiety disorder”. mebooksfree.com mebooksfree.com mebooksfree.com preoccupation per- A. Neurasthenia: This disorder is characterized by com- sists despite normal investigationQ results as well as plaints of increased mental and physical fatigue after doctors reassurances. mild efforts. The patient is often concerned about It is important to differentiate hypochondriasis lowered physical and mental efficiency. Associated from somatization disorder. The emphasis in hypo- symptoms include muscular aches and pain, sleep chondriasis is on the diagnosis whereas the emphasis disturbances, irritability, dyspepsia, headache and in somatization disorder is on the symptoms. inability to relax. The ICD-10 includes the diagnosis Also, hypochondriasis must be differentiated from of fatigue syndromes under the category of neuras- delusional disorder (somatic type). In patients with hypochondriasis the belief is not as fixed as it is in thenia. delusional disorder. The patient with hypochondriasis B. Chronic fatigue syndrome (myalgic encephalo- mebooksfree.com mebooksfree.commay doubt his belief atleastmebooksfree.com for short-term, after a nor- myelitis)mebooksfree.com: This syndrome is frequentlymebooksfree.com diagnosed in mebooksfree.com mal investigation or medical reassurance. In contrast western countries. The symptoms are severe, debili- in delusional disorder, the belief is fixed and totally tating fatigue, malaise, headaches, pharyngitis, low unshakeable. grade fever, cognitive complaints, gastrointestinal D. Body dysmorphic disorder: It is characterized by symptoms and tender lymph nodes. The diagnosis the preoccupation with an imagined defectQ in body of chronic fatigue syndrome is not covered by either appearance. In case a slight physical anomaly is pre- ICD-10 or DSM-5, however the symptoms have some sent, patient’s concern for the same is exaggerated. resemblance to neurasthenia.

mebooksfree.com mebooksfree.com mebooksfree.com mebooksfree.com mebooksfree.com mebooksfree.com

mebooksfree.com mebooksfree.com mebooksfree.com mebooksfree.com mebooksfree.com mebooksfree.com mebooksfree.com mebooksfree.com mebooksfree.com mebooksfree.com mebooksfree.com mebooksfree.com

50 Review of Psychiatry

C. Culture bound syndrome: These are limited to a Stage 3, the stage of exhaustion: If the stress continues, particular culture and are not seen world wide. It is the resis­tance of body gradually decreases and finally col- believed that local cultural beliefs and patterns of lapses. behavior have strong influence on the presentation of Almost all the organ systems may be involved in psy- mebooksfree.com mebooksfree.comthese syndromes. Few commonmebooksfree.com culture bound syn- chosomaticmebooksfree.com disorders. The important onesmebooksfree.com include: mebooksfree.com dromes are: A. Gastrointestinal system: A large number of GI disor- • Dhat syndromeQ: It is prevalent in Indian subcon- ders such as peptic ulcers, Crohn’s disease, ulcerative tinent. The patient has a belief that he is passing colitis are affected by psychological causes. Irritable semen in urine and this is resulting in physical and bowel syndrome, which is characterized by symptoms such as abdominal pain, cramps, alteration of bowel mental weakness. habits (diarrhea or constipation) is a well known • KoroQ: The patient has a fear that his penis will example of psychosomatic disorder. retract into the abdomen and would result in death B. Respiratory system: Asthma, COPD and hyperventila- • Latah: This is characterized by automatic obedi- tion syndrome are known to have psychological com- ence, echolalia and echopraxia. ponent. Hyper­ventilation syndrome is characterized Q D. Factitious disorder (munchausen syndrome ): Facti­ by rapid and deep breathing for several minutes and tious disorder (also known as hospital addiction) is accompanying symptoms of suffocation, giddiness, mebooksfree.com mebooksfree.coma disorder in which patientsmebooksfree.com produces fake symp- mebooksfree.com mebooksfree.com mebooksfree.com paraesthesia and syncope due to falling PCO2 levels toms with the sole aim of obtaining medical attention in blood. (hence called professional patientsQ). Unlike malin- C. Cardiovascular system: Cardiovascular disorders such gering, in which the motive is usually financial gains as hypertension, coronary artery diseases, cardiac or avoidance of duty, the patients with factitious dis- arrhythmias are known to be affected by psychological orders have no recognizable motives apart from wish causes. Of particular interest is the association of so to get medical attention. These patients distort the called type A personality with coronary artery dis- history and make stories (pseudologia fantastica) to easeQ. The type A personality is characterized by eas- convince the doctors. The patients are often from the ily aroused anger, impatience, aggression, competitive medical and related fieldsQ and have basic under- striving and hostility. Type A pattern is associated with standing of symptoms/signs of various disorders. a nearly two fold risk of MI and CAD related mortality. In comparison type B-personality is characterized by mebooksfree.com mebooksfree.comPSYCHOLOGICAL FACTORSmebooksfree.com AFFECTING calmness,mebooksfree.com relaxed attitude, low competitivenessmebooksfree.com and mebooksfree.com lesser chances of coronary artery diseases. OTHER MEDICAL CONDITIONS D. Musculoskeletal system: Disorders like rheumatoid The concept of psychosomatic disorders (physical disor- arthritis, systemic lupus erythematosus are known to ders caused by or aggravated by psychological factors) have psychological components. Of particular note has been known for a long time. It is clear that stress is fibromyalgia, a disease characterized by pain and can result in many somatic symptoms. Stress is described stiffness of soft tissues such as muscle and ligaments. as any circumstance, that disturbs or is likely to disturb, The patient often reports of local areas of tenderness, the normal physiological or psychological functioning of also known as “trigger points”. There might be associ- ated symptoms such as anxiety, fatigue and inability an individual. to sleep. Hans Selye described a model of stress that is known E. Other disorders such as endocrinological disorders, as gene­ral adaptation syndromeQ. According to this skin disorders, headaches also have psychological mebooksfree.com mebooksfree.commodel, body reacts to stress inmebooksfree.com three stages. mebooksfree.com mebooksfree.com mebooksfree.com contributions. Stage 1, the alarm reaction: This is the immediate response Treatment: Patients with all forms of somatoform disor- characterized by fight or flight response. ders usually resist psychiatric treatment. The treatment Stage 2, the stage of resistance: This is also known as is usually focused on helping the patient understand the stage of adaptation. Here, the body adapts to the stress. effect of psychological factors in the genesis of symptoms For example, if the stress is starvation, body reduces the while acknowledging that the symptoms are real and dis- energy consumption and decreases physical activity. tressing to the patient. Psychotherapeutic techniques like

mebooksfree.com mebooksfree.com mebooksfree.com mebooksfree.com mebooksfree.com mebooksfree.com

mebooksfree.com mebooksfree.com mebooksfree.com mebooksfree.com mebooksfree.com mebooksfree.com mebooksfree.com mebooksfree.com mebooksfree.com mebooksfree.com mebooksfree.com mebooksfree.com

Neurotic, Stress Related and Somatoform Disorders 51 group psychotherapy, insight oriented psychotherapy, Grief, Bereavement and Mourning behavior therapy, cognitive therapy and hypnosis may be useful. Relaxation techniques and stress management Although these terms have been used interchangeably, training may also be required. they have specific meanings.Bereavement means the mebooksfree.com mebooksfree.com mebooksfree.comstate ofmebooksfree.com being deprived of someone due tomebooksfree.com death. Grief mebooksfree.com DEATH AND DYING reaction is the psychological feeling precipitated by the death of a loved one. Mourning is the process through When an individual is informed about his impending which grief is resolved. Mourning involves societal prac- death, he usually goes through a series of responses. These tices like funerals, burial and memorial services. stages of death and dying, were proposed by Elizabeth Kubler-RossQ. Complicated Bereavement (Complicated Grief Stage 1: Denial and shock—This is characterized by refusal Reactions) to accept the diagnosis and a reaction of shock. Complicated bereavement includes prolonged grief Stage 2: Anger—In this stage patients become irritable and reactions­ (chronic grief) or extraordinarily intense grief angry at family members, friends , doctors and even God. reactions (hypertrophic grief) or delayed grief reactions mebooksfree.com mebooksfree.comStage 3: Bargaining—In this mebooksfree.comstage patient try to bargain (delayedmebooksfree.com grief). Traumatic Bereavement mebooksfree.comrefers to grief mebooksfree.com with family members and even God. For example, they that is both chronic and hypertrophic. may pledge to god that they will regularly go to temples if god cures them. Bereavement and Depression Stage 4: Depression—The patient now start showing symp- Grief is a complex experience in which both positive emo- toms of depression such as sadness of mood, withdrawal tions (happy memories of the deceased) and negative and suicidal thoughts emotions (sadness) coexist and alternate. In depression, Stage 5: Acceptance—Finally patient accepts that death the negative emotions predominate and do not change. is inevitable and their feelings may change to neutral or Also the symptoms in depression are severe and cause even happiness. significant dysfunction. mebooksfree.com mebooksfree.com mebooksfree.com mebooksfree.com mebooksfree.com mebooksfree.com QUESTIONS AND ANSWERS

QUESTIONS C. Pattern of psychological response to stress D. Pattern of autonomic nervous system (ANS) and General physio­logical response when we are aroused by 1. All of the following are seen in anxiety except: a stressful situation (Kerala 1996) 4. Which of the following is the most common psy- A. Decreased sweating B. Hyperventilation chiatric disorder? (DNB NEET 2014-15) C. Cold extremities D. Palpitations A. Anxiety disorder E. Pupillary dilatation B. Schizophrenia mebooksfree.com mebooksfree.com 2. General adaptation syndromemebooksfree.com (GAS) is seen in: mebooksfree.comC. Depression mebooksfree.com mebooksfree.com (AI 2012) D. Mania A. Panic attacks B. Depression C. Anxiety D. Stressful situations Generalized Anxiety Disorders 3. General adaptation syndrome relates to:(AI 2012) 5. All are seen in generalized anxiety disorder A. How we achieve homeostasis except: (DNB June 2009) B. How well we adapt to new situations A. Muscle tension

mebooksfree.com mebooksfree.com mebooksfree.com mebooksfree.com mebooksfree.com mebooksfree.com

mebooksfree.com mebooksfree.com mebooksfree.com mebooksfree.com mebooksfree.com mebooksfree.com mebooksfree.com mebooksfree.com mebooksfree.com mebooksfree.com mebooksfree.com mebooksfree.com

52 Review of Psychiatry

B. Irritability C. Short-term benzodiazepine plus SSRI plus CBT C. Fear of impending doom D. Long-term benzodiazepine plus venlafaxine D. Restlessness Phobic Anxiety Disorders mebooksfree.com mebooksfree.com 6. Drug of choice for generalizedmebooksfree.com anxiety disorder is: mebooksfree.com mebooksfree.com mebooksfree.com (DNB NEET 2014-15) 12. Agoraphobia is: (SGPGI 2003) A. Alprazolam B. Buspirone A. Fear of getting caught in places from where C. Venlafaxine D. Beta-blockers escape would be difficult 7. A 25-year-old lady presented with sadness, palpi- B. Fear of heights tation, loss of appetite and insomnia. There is no C. Fear of animals complaint of hopelessness, suicidal thoughts and D. Fear of closed spaces there is no past history of any precipitating event. 13. True about social phobia is: (UP 2001) She is remarkably well in other areas of life. She A. Fear of closed spaces is doing her office job normally and her social life B. Irrational fear of situation is also normal. What is the probable diagnosis in C. Irrational fear of activities (AI 2010) this case? D. Irrational fear of specified objects mebooksfree.com mebooksfree.comA. Generalized anxietymebooksfree.com disorder mebooksfree.com mebooksfree.com mebooksfree.com B. Mixed anxiety depression 14. A middle aged person reported to psychiatric OPD C. Adjustment disorder with the complaints of fear of leaving home, fear D. Mild depressive episode of travelling alone and fear of being in a crowd. He develops marked anxiety with palpitations Panic Disorder and sweating if he is in these situations. He often avoids public transport to go to his place of work. 8. A 30-year-old lady presents with sudden onset The most likely diagnosis is: (AIIMS May 2006) breathlessness, anxiety, palpitation and feeling A. Generalized anxiety disorder of impending doom. Physical examination does B. Schizophrenia not reveal any abnormality. What is the probable C. Personality disorder diagnosis in this case? (AIIMS Nov 2010) D. Agoraphobia A. Panic attack B. Anxiety disorder mebooksfree.com mebooksfree.comC. Conversion disordermebooksfree.com D. Acute psychosis 15. mebooksfree.comA medical student could not delivermebooksfree.com semi- mebooksfree.com nar, fearing his seniors despite knowing that 9. Panic attack is associated with a disturbance in all they are supportive. He further has difficulty of the following neurotransmitters except: (AIIMS Nov 2011) speaking in front of others and also avoids A. Serotonin going to the parties. The most likely diagnosis is: B. GABA (AIIMS 1999, MH 2008) C. Glutamate A. Agoraphobia B. Claustrophobia D. Dopamine, CCK, pentagastrin C. Social phobia D. Acrophobia 10. Differential diagnosis of panic disorder are: 16. A fifty-year-old male feels uncomfortable in using (PGI June 2004) lift, being in crowded places and traveling. The A. Pheochromocytoma most appropriate line of treatment is: B. Myocardial infarction (AIIMS Nov 2005) mebooksfree.com mebooksfree.comC. Mitral valve prolapsemebooksfree.com mebooksfree.comA. Counseling mebooksfree.com mebooksfree.com D. Depression B. Relaxation therapy E. Carcinoid syndrome C. Exposure and response prevention D. Covert sensitization 11. Which of the following is the most appropriate treatment for panic disorder? (AIIMS 2009) 17. Treatment of choice in phobic disorder is : A. Buspirone plus benzodiazepines (AIIMS 1994), (Assam 1999) B. Benzodiazepines plus supportive therapy A. Psychotherapy

mebooksfree.com mebooksfree.com mebooksfree.com mebooksfree.com mebooksfree.com mebooksfree.com

mebooksfree.com mebooksfree.com mebooksfree.com mebooksfree.com mebooksfree.com mebooksfree.com mebooksfree.com mebooksfree.com mebooksfree.com mebooksfree.com mebooksfree.com mebooksfree.com

Neurotic, Stress Related and Somatoform Disorders 53

B. Benzodiazepines 24. True about obsessive compulsive disorders are C. Behavior therapy all except: (DNB NEET 2014-15) D. 5-HT reuptake inhibitor A. Obsessions are ego-alien B. Patient tries to resist against obsessions and 18. Agoraphobia is treated with: (PGI Dec 2007) mebooksfree.com mebooksfree.com mebooksfree.com mebooksfree.comcompulsions mebooksfree.com mebooksfree.com A. Systematic desensitization C. Obsessions are egosyntonic B. Psychodynamic therapy D. Insight is present C. Exposure therapy D. Relaxation therapy 25. Most common major symptom in obsessive com- E. Behavior therapy pulsive disorder is: (DNB NEET 2014-15) A. Compulsive washing of hand Obsessive Compulsive Disorder B. Obsessive thoughts of contamination C. Compulsive checking 19. Feeling of uncertainty and excessive sense of D. Obsessive precision responsibility is seen in: (AIIMS May 2015) A. Generalized anxiety disorder 26. Transmitters mainly involved in OCD is: B. OCD (AIIMS 1995) mebooksfree.com mebooksfree.comC. Phobia mebooksfree.com mebooksfree.comA. GABA B. Norepinephrinemebooksfree.com mebooksfree.com D. Personality disorder C. Dopamine D. Serotonin 20. Abnormal thought possession is found in: (AI 1994) 27. A 15-year-old boy feels that the dirt has hung onto A. Organic brain syndrome him whenever he passes through the dirty street. B. Hysteria This repetitive thought causes much distress C. Obsessive compulsive disorder and anxiety. He knows that there is actually no D. Neurasthenia such thing after he has cleaned once but he is not 21. True statements about obsession: (PGI 2003) satisfied and is compelled to think so. This has A. It is a repetitive thought or image led to social withdrawal. He spends much of his B. Patient believes that the images or thoughts are time thinking about the dirt and contamination. imposed by others This has affected his stu­dies also. The most likely mebooksfree.com mebooksfree.comC. Content of obsessionmebooksfree.com are about sex or God mebooksfree.comdiagnosis is: mebooksfree.com(AI 2003) mebooksfree.com D. Patient gets disturbed when unable to remove A. Obsessive compulsive disorder the ideas or thoughts B. Conduct disorder C. Agoraphobia 22. Which of the following statements differentiates D. Adjustment disorder obsessional idea from delusion: (DNB NEET 2014-15, AIIMS Nov 2005) 28. An obsessive compulsive neurosis patient is likely A. Obsessional idea is not a conventional belief to develop: (AIIMS 1993) B. Obsessional idea is held in spite of evidence to A. Hallucination B. Depression the contrary C. Delusion D. Schizophrenia C. Obsessional idea is regarded as senseless by 29. Drug of choice for OCD is: (DNB June 2009) patient A. Clomipramine B. Fluoxetine mebooksfree.com mebooksfree.comD. Obsessional idea ismebooksfree.com held on inadequate ground mebooksfree.comC. Carbamazepine D. Chlorpromazinemebooksfree.com mebooksfree.com 23. Following are the major symptoms of obsessive 30. In obsessive-compulsive disorder, which is not compulsive disorder: (DNB NEET 2014-15) given: (DNB 2002, Jharkhand 2006) A. Doubts of contamination A. Clomipramine B. Pathological doubts B. Haloperidol C. Intrusive thoughts C. Sertraline D. All of the above D. Carbamazepine

mebooksfree.com mebooksfree.com mebooksfree.com mebooksfree.com mebooksfree.com mebooksfree.com

mebooksfree.com mebooksfree.com mebooksfree.com mebooksfree.com mebooksfree.com mebooksfree.com mebooksfree.com mebooksfree.com mebooksfree.com mebooksfree.com mebooksfree.com mebooksfree.com

54 Review of Psychiatry 31. All drugs are used for treatment of OCD except: Post-traumatic Stress Disorder (ONB 2009, PCI 1999), (JIPMEr 2002) (MAHE 2003) 38. Most common disorder(s) after trauma is: A. Carbamazepine B. Lithium (PGI May 2015) C. Fluoxetine D. Diazepam mebooksfree.com mebooksfree.com mebooksfree.com mebooksfree.comA. Major depression B. Maniamebooksfree.com mebooksfree.com 32. Drug used for long-term treatment of OCD C. Schizophrenia D. PTSD includes: (PGI May 2013) E. Acute stress reaction A. Clomipramine B. Fluoxetine 39. Which of the following is not a clinical feature of C. Fluvoxamine D. Citalopram post-traumatic stress disorder (PTSD)? (AI 2008) E. Trifluperidol A. Flashbacks B. Hyperarousal 33. Treatment of obsessive-compulsive disorder C. Hallucinations D. Emotional numbing includes: (PGI Dec 2008) 40. Post-traumatic stress-disorder is associated with A. Exposure and response prevention all except: (PGI 2000) B. Flooding A. Flashback C. Psychoanalytic therapy B. Severe traumatic injury D. Supportive psychotherapy involving family C. Re-experiencing of stressful event mebooksfree.com mebooksfree.commembers mebooksfree.com mebooksfree.comD. Anhedonia mebooksfree.com mebooksfree.com E. Systematic desensitization E. It does not develop after 6 months of stress 34. Treatment of choice for OCD is: 41. False statement about post-traumatic stress dis- (DNB 2004, MP 2006) order: (DNB NEET 2014-15) A. Behavior therapy A. Symptoms develop immediately after the event B. Drug therapy B. Symptoms include insomnia, poor concentration C. Psychosurgery C. It is the response to an exceptionally stressful or D. Combination of behavior and drug therapy catastrophic stimuli D. Anxiolytics are given only, if anxiety develops 35. A 35-year-female has been diagnosed with obsessive compulsive disorder and she washes 42. True for PTSD are all except: (PGI 2001) her hands many times a day. Which would be the A. Patients have past history of psychiatric illness mebooksfree.com mebooksfree.combest CBT technique formebooksfree.com her treatment? (AI 2012) mebooksfree.comB. Women are more predisposed mebooksfree.com mebooksfree.com A. Thought stopping B. Response prevention C. Occur in intellectuals C. Relaxation D. Exposure D. Feeling of numbness E. Feeling of detachment 36. Exposure and response prevention technique is/ are used in: (PGI May 2015) 43. Post-traumatic stress disorder (PTSD) is differ- A. Schizophrenia B. OCD entiated from other disorders by presence of: (AIIMS May 2012) C. Phobia D. Mania A. Nightmares about events E. Depression B. Autonomic arousal and anxiety 37. A woman comes to psychiatrist with history of C. Recall of events and avoidance of similar experi- spending a lot of time in washing her hands. She ences D. Depression mebooksfree.com mebooksfree.comis distressed about it butmebooksfree.com says that she is not able to mebooksfree.com mebooksfree.com mebooksfree.com stop washing. This has started to affect her social 44. All are true for PTSD except: (PGI 2002) life as well. What is the best mode of treatment for A. Hippocampus and amygdala are the brain areas her? (AIIMS May 2015) involved in PTSD A. Cognitive behavioral therapy B. Anhedonia B. Exposure and response prevention C. Depression and guilt C. Systematic desensitization D. Insomnia and poor concentration D. Pharmacological agents E. Anxiolytics are the treatment of choice

mebooksfree.com mebooksfree.com mebooksfree.com mebooksfree.com mebooksfree.com mebooksfree.com

mebooksfree.com mebooksfree.com mebooksfree.com mebooksfree.com mebooksfree.com mebooksfree.com mebooksfree.com mebooksfree.com mebooksfree.com mebooksfree.com mebooksfree.com mebooksfree.com

Neurotic, Stress Related and Somatoform Disorders 55

45. Three years back a woman suffered during an This however, provoked anxiety and sadness of earthquake and she was successfully saved. After mood when she would remain preoccupied with recovery she has nightmares about the episode thoughts about him. She should be treated with: and she also gets up in the night and feels terrified. (AIIMS May 2003) mebooksfree.com mebooksfree.comThe most probable diagnosismebooksfree.com is: mebooksfree.comA. Clomipramine mebooksfree.com mebooksfree.com (AIIMS May 2002) B. Alprazolam A. Major depression C. Electroconvulsive therapy B. Post-traumatic stress disorder D. Haloperidol. C. Mania D. Schizophrenia 51. Ms. B. a 27-year-old nurse had extracurricular interests in trekking and painting. She broke up 46. Which of the following is the most effective treatment relationship with her boyfriend. Two months later modality for post-traumatic stress disorder she lost interest in her hobbies and was convinced (PTSD)? (AIIMS Nov 2014) that she would not be able to work again. She A. Cognitive behavioral therapy thought life was not worth living and consumed B. Eye movement desensitization and reprocessing 60 tablets of phenobarbitone to end her life. She C. Hypnosis is most likely suffering from: (AI 2004) mebooksfree.com mebooksfree.comD. Rational and emotivemebooksfree.com therapy mebooksfree.com mebooksfree.com mebooksfree.com A. Adjustment disorder 47. SSRl are first line treatment for: (PGI 2010) B. Acute stress disorder A. OCD B. Panic disorder C. Depressive disorder C. Social phobia D. Post-traumatic stress D. Post-traumatic stress disorder disorder E. Adjustment disorder 52. Two months after knowing that his son was suffering from leukemia, a 45-year-old father presents with Grief and Adjustment Disorder sleep deprivation, lethargy, headache, and low mood. He interacts reasonably well with others, 48. A man coming from mountain whose wife died 6 but has absented himself from work. The most months prior says that his wife appeared to him probable diagnosis is: (AI 2008) and asked him to join her. The diagnosis is: A. Depression mebooksfree.com mebooksfree.com mebooksfree.com(AIIMS 2000) mebooksfree.comB. Psychogenic headache mebooksfree.com mebooksfree.com A. Normal grief C. Adjustment disorder B. Grief psychosis D. Somatization disorder C. Bereavement reaction D. Supernatural phenomenon Somatoform and Factitious Disorders 49. Which of the following is not a part of Kubler- 53. Which of the following is/are the form/subtype of Ross’s stages of impeding death? (DNB Dec 2010) somatoform disorder(s)? (PGI May 2012) A. Depression B. Bargain A. Post-traumatic stress disorder C. Aggression D. Anger B. Depersonalization 50. An elderly house wife lost her husband who died C. Somatic passivity suddenly of myocardial infarction couple of years D. Conversion disorder mebooksfree.com mebooksfree.comago. They had been stayingmebooksfree.com alone for almost a mebooksfree.comE. Hypochondriasis mebooksfree.com mebooksfree.com decade with infrequent visits from her son and grandchildren. About a week after the death she 54. Which of the following is not a specific somatoform heard his voice clearly talking to her as he would disorder?­ (AIIMS Nov 2011) in a routine manner from the next room. She A. Somatization disorder went to check but saw nothing. Subsequently she B. Chronic fatigue syndrome often heard his voice conversing with her and she C. Irritable bowel syndrome would also discuss her daily matters with him. D. Fibromyalgia

mebooksfree.com mebooksfree.com mebooksfree.com mebooksfree.com mebooksfree.com mebooksfree.com

mebooksfree.com mebooksfree.com mebooksfree.com mebooksfree.com mebooksfree.com mebooksfree.com mebooksfree.com mebooksfree.com mebooksfree.com mebooksfree.com mebooksfree.com mebooksfree.com

56 Review of Psychiatry

55. A 45-year-old male presents with history of head- A. Carcinoma lung ache, and vague body pains, off and on diarrhea B. Hypochondriacal disorder and constipation, impotence and tingling and C. Delusional disorder paresthesia in glove stocking pattern. The most D. Malingering mebooksfree.com mebooksfree.comprobable diagnosis is:mebooksfree.com mebooksfree.com mebooksfree.com mebooksfree.com 60. A young 20-year-old girl presents with complaints (AI 2K, JIPMER 2002, DNB 2004) of pain in legs, intermittent vomiting, and head- A. Hypochondriasis ache since 2 months. Her physical examination B. Somatization disorder was normal. What is the most possible diagnosis? C. Conversion disorder (AIIMS Nov 2009) D. Factitious disorder A. Generalized anxiety disorder 56. A 41-year-old married female presented with B. Conversion disorder headache for the last 6 months. She had several C. Somatoform pain disorder consultations. All her investigations were found to D. Somatization disorder be within normal limits. She still insists that there 61. A 40-year-old male is admitted with complaints of is something wrong in her head and seeks another abdo­minal pain and headache. General physical consultation. The most likely diagnosis is: mebooksfree.com mebooksfree.com mebooksfree.com mebooksfree.comexamination revealed six scars on mebooksfree.comthe abdomen mebooksfree.com (AI 2003) A. Phobia from previous surgeries. He seems to maintain B. Psychogenic headache a sick role and seeks attention from the nurses. C. Hypochondriasis He demands multiple diagnostic tests including D. Depression a liver biopsy. The treating team failed to diag- nose any major physical illness in the patient. (AI 1994) 57. Hypochondriasis is: His mental status examination did not reveal any A. Normal preoccupation with abnormal body major psychopathology. One of the treating staff function recognized him to have appeared in several other B. Abnormal preoccupation with abnormal body hospitals with abdominal pain and some other function vague complaints. He is most likely suffering from: C. Normal preoccupation with normal body func- (AIIMS Nov 2003) mebooksfree.com mebooksfree.comtion mebooksfree.com mebooksfree.com mebooksfree.com mebooksfree.com A. Schizophrenia B. Malingering D. Abnormal preoccupation with normal body C. Somatization disorder D. Factitious disorder function 62. A 30-year-old lady presented to physician with 58. A man with intermittent hiccups feels that he is complaints of hematuria. On evaluation RBCs about to die because he is suffering from gastric were found in urine but no cause was found. On cancer. All his radiological investigations prove further enquiry it was found that she has gone to the contrary. Which is the most probable diagno- many doctors with the same complaints and would sis? (DNB NEET 2014-15) demand in patient care. She would prick her finger A. Somatization disorder B. Hypochondriasis and mix blood in urine sample. Her diagnosis is: C. Conversion disorder D. Delusional disorder (Karnataka 2011) 59. A 35-year-old male, with premorbid anxious traits A. Malingering mebooksfree.com mebooksfree.comand heavy smoker believesmebooksfree.com that he has been suffer- mebooksfree.comB. Factitious illness mebooksfree.com mebooksfree.com ing from ‘lung carcinoma’ for a year. No significant C. Dissociative disorder clinical finding is detected on examination and D. Hypochondriasis relevant investigations. In the process, he has 63. Munchausen’s syndrome by proxy involves: spent a huge amount of money, time and energy (MH 2011) in getting himself unduly investigated. He is most A. Drug abuse likely suffering from: (AIIMS Nov 2004) B. Toxin mediated neuropsychiatric disorder

mebooksfree.com mebooksfree.com mebooksfree.com mebooksfree.com mebooksfree.com mebooksfree.com

mebooksfree.com mebooksfree.com mebooksfree.com mebooksfree.com mebooksfree.com mebooksfree.com mebooksfree.com mebooksfree.com mebooksfree.com mebooksfree.com mebooksfree.com mebooksfree.com

Neurotic, Stress Related and Somatoform Disorders 57

C. Illness caused by care giver 71. Which of the following can differentiate hysterical D. All of the above fits from epileptic fits? (DPG 2009, Calcutta 2002) A. Occur in sleep B. Injuries to person 64. Maintaining sick role by any means is a characteristic C. Incontinence D. Occur when people mebooksfree.com mebooksfree.comfeature of: mebooksfree.com mebooksfree.com mebooksfree.com mebooksfree.com (JIPMER 2002, Mahe 2004, Rohtak 2002, DNB 2003) are watching A. Hypochondriasis 72. The most common form of dissociative hysteria B. Somatization disorder is: (MH 2000) C. Conversion disorder A. Fugue B. Amnesia D. Factitious disorder C. Multiple personality D. Somnambulism Dissociative Disorders 73. Psychogenic amnesia is characterized by: (AIIMS 1997) (Conversion Disorders) A. Anterograde amnesia 65. La belle indifference is seen in: B. Retrograde amnesia (DNB NEET 2014-15, AIIMS 1998) C. Both with confabulation A. Conversion disorder B. Schizophrenia D. Patchy impairment of personal memories mebooksfree.com mebooksfree.comC. Mania mebooksfree.comD. Depression mebooksfree.com mebooksfree.com mebooksfree.com 74. A person missing from home is found wander- 66. In conversion disorder, all of the following state- ing purposefully. He is well-groomed and denies ments are true except: (DNB NEET 2014-15) remembering how he reached at the new place. A. Autonomic nervous system is involved Most likely diagnosis is: (AI 2001) B. There is primary and secondary gain A. Dementia B. Dissociative amnesia C. La belle indifference is a feature C. Dissociative fugue D. Schizophrenia D. Patient does not intentionally produce symptoms 75. An 18-year-old boy came to psychiatry OPD with 67. All are true about conversion disorder except: a complaint of feeling as, if he is changed from (DNB June 2011) inside. He reports feeling strange as, if he is dif- A. Presence of secondary gain ferent from his normal self. He was very tense and B. Onset in late age anxious yet could not point out the precise change mebooksfree.com mebooksfree.comC. Patient does not consciouslymebooksfree.com produce symptom mebooksfree.comin him. This phenomena is best calledmebooksfree.com as: mebooksfree.com D. Relation with stress (AI 2005) 68. Which of the following is a conversion disorder? A. Delusional mood (DNB June 2009) B. Depersonalization A. Hysterical fits B. Derealization C. Autochthonous delusion C. Depersonalization D. Amnesia D. Overvalued idea 69. In conversion disorders, all are found except: 76. Regarding Ganser’s syndrome, which of the fol- (DNB NEET 2014-15) lowing statement is true: (AIIMS 1998) A. Jealousy B. Paralysis A. Repeated lying C. Anesthesia D. Abnormal gait B. Approximate answers C. Unconscious episodes 70. Following are included in dissociative disorder: D. Malingering mebooksfree.com mebooksfree.com mebooksfree.com(PGI June 2007, 2003) mebooksfree.com mebooksfree.com mebooksfree.com A. Multiple personality disorder 77. All are true about Ganser’s syndrome except: B. Fugue (JIPMER/UP 2K, PGI 1999, DNB 1998) C. Hypochondriasis A. Approximate answer D. Somatization disorder B. Apparent clouding of consciousness E. Obsession C. Only found in prisoners F. Borderline personality D. Hallucinations

mebooksfree.com mebooksfree.com mebooksfree.com mebooksfree.com mebooksfree.com mebooksfree.com

mebooksfree.com mebooksfree.com mebooksfree.com mebooksfree.com mebooksfree.com mebooksfree.com mebooksfree.com mebooksfree.com mebooksfree.com mebooksfree.com mebooksfree.com mebooksfree.com

58 Review of Psychiatry

78. Ganser syndrome is a type of: (DNB NEET 2014-15) expected that there would be at least some distur- A. Dementia B. Malingering bance in professional and social life. Further her C. Dissociative disorder D. Personality disorder symptoms are not enough to make a diagnosis of depression. In view of the above, the diagnosis 79. All is true about pseudocyesis except: mebooksfree.com mebooksfree.com mebooksfree.com mebooksfree.comof mild depression cannot be made.mebooksfree.com This patient mebooksfree.com (DNB NEET 2014-15) doesn’t have any history of precipitating event A. Abdominal enlargement hence the diagnosis of adjustment disorder can B. Patient is pregnant be easily ruled out. Few guides are giving the C. Labor pains at expected date of delivery answer as generalized anxiety disorder which D. Amenorrhea does not make any sense as the only anxiety 80. The difference between malingering and hysteria symptom mentioned here is palpitation. The is: (AI 1994, DNB 2006) core feature of generalized anxiety disorder i.e A. Hypnosis “generalized­ and persistent anxiety” is not there. B. Malingering has poor prognosis Hence, we are left with mixed anxiety depression. C. Hysteria is more common in females The diagnosis of mixed anxiety depression is D. Conscious motive in malingering made when there are “symptoms of both anxiety mebooksfree.com mebooksfree.com mebooksfree.com mebooksfree.comand depression, but neither set ofmebooksfree.com symptoms is mebooksfree.com 81. Differential diagnosis of premenstrual tension severe enough to make an independent anxiety or includes all of the following except: depressive disorder diagnosis”. This description (AIIMS Nov 2002) suits best to the clinical scenario provided here. A. Psychiatric depressive disorder 8. A. B. Panic disorder 9. C. Serotonin, norepinephrine and GABA are the C. Generalized anxiety disorder major neuro­transmitters involved. Cholecys- D. Chronic fatigue syndrome tokinin and pentagastrin (which acts on CCK receptors) are known to cause panic attacks. ANSWERS 10. A, B, C, E. 1. A. 11. C. 2. D. 12. A. mebooksfree.com mebooksfree.com 3. D. mebooksfree.com 13. mebooksfree.comB. Social phobia is defined as irrationalmebooksfree.com fear of mebooksfree.com 4. A. As a group anxiety disorders are the most com- social situations. Though it can be said that mon psychiatric disorders. it also includes certain activities, however 5. C. Fear of impending doom is typically seen in panic please remember that it’s the context (situa- attacks. tion) that is central to social phobia and not the 6. A. Benzodiazepines are the drug of choice for gen- activity. For example, many patients with social eralized anxiety disorder. However, it must be phobia have difficulty eating in a restaurant. remembered that benzodiazepines can cause However, they have no problem doing the same dependence. The other drugs which can be used activity (i.e eating) when alone. It’s the situation include SSRIs, buspirone and venla-faxine. (i.e. the restaurant) that produces anxiety. 7. B. In this patient the best answer would be 14. D. “mixed anxiety depression”. This patient has 15. C. Here the fear is performing in public and there is mebooksfree.com mebooksfree.comsome depressive symptomsmebooksfree.com (sadness, loss of mebooksfree.comalso avoidance of social situations mebooksfree.com(i.e. avoidance mebooksfree.com appetite and insomnia), however the ques- of parties). tion explicitly mentions that there is no hope- 16. C. The diagnosis here is agoraphobia as this gen- lessness, no suicidal thoughts and that her tleman is uncomfortable with closed places job and social life is normal. The question (lift), crowded places and also travelling alone. goes on to add that “she is doing remarkable The best treatment option here is exposure and well in other areas of life”. Please remember response prevention. that even in a patient with mild depression, it is 17. C.

mebooksfree.com mebooksfree.com mebooksfree.com mebooksfree.com mebooksfree.com mebooksfree.com

mebooksfree.com mebooksfree.com mebooksfree.com mebooksfree.com mebooksfree.com mebooksfree.com mebooksfree.com mebooksfree.com mebooksfree.com mebooksfree.com mebooksfree.com mebooksfree.com

Neurotic, Stress Related and Somatoform Disorders 59

18. A, B, C, D, E. dence is for haloperidol, risperidone, quetiapine 19. B. The cognitive theory of OCD says that the typi- and olanzapine. In comparison, carbamazepine cal abnormalities in OCD include, “excessive is rarely used in OCD and has very weak evidence or inflated sense of responsibility”, “feeling of in comparison to haloperidol. Hence, the best mebooksfree.com mebooksfree.comuncertainty” and “overestimationmebooksfree.com of threat”. Few mebooksfree.comanswer here would be carbamazepine.mebooksfree.com mebooksfree.com books are giving the answer as “generalized anxi- 31. D. Again, we have to choose between carbamaz- ety disorder” which is incorrect. epine and diazepam. Now, diazepam is a benzo- 20. C. Obsessions are disturbances of thought posses- diazepine and can improve anxiety temporarily sion. however it doesn’t act at core symptoms of OCD. 21. A, C, D. Whereas, carbamazepine, though has minimal Obsessions are repetitive thoughts, images evidence, but it has been found to act on core or impulses. Often the content of thoughts is symptoms of OCD. about sex or god and patient tries to stop these 32. A, B, C, D. anxiety provoking thoughts unsuccessfully. SSRIs and clomipramine are first line agents. Please remember, that a patient with obsession Trifluperidol doesn’t have any evidence in man- identifies the repetitive thought as his “own agement of OCD. mebooksfree.com mebooksfree.comthought” and not mebooksfree.comsomething that is imposed 33. mebooksfree.comA, B, C, D and E. mebooksfree.com mebooksfree.com 34. D. A combination of pharmacotherapy and psycho- by others. Also remember, if the patient indeed therapy has the best evidence in the management believes that the thought has been imposed by of OCD. others, it would then be diagnosed as “thought 35. B. The technique is actually exposure and response insertion” and not an obsession. prevention. In OCD, the primary aim is to stop 22. C. Obsessions are considered senseless by the the compulsions; hence response prevention is patient whereas patient has full belief in the the better answer here. delusions. For example, a patient who gets 36. B, C. obsessive thoughts that “his hands are unclean” 37. B. According to american psychiatric association understands that his thought is not true and guidelines “The first line treatments for OCD gets bothered by this repetitive thought whereas are cognitive behavioral therapy that relies on a patient with “delusion of infidelity” actually behavioral technique of exposure and response mebooksfree.com mebooksfree.combelieves that his wifemebooksfree.com is cheating on him and mebooksfree.comprevention and serotonin reuptakemebooksfree.com inhibitors”. mebooksfree.com continues to believe so irrespective of what Now, this question is just mentioning pharma- others say. cological agents without specifying anything 23. D. about which agent. Also few studies have found, 24. C. Obsessions are ego dystonic and not ego syn- that exposure and response prevention has more tonic. lasting effect than pharmacological agents. Said 25. B. that, the choice of treatment between ERP and 26. D. If we have to choose one, it would be serotonin. pharmacological agents depends on patients 27. A. characteristics, which have not been provided, 28. B. Most of the patients with OCD, develop secon­ hence its tough to choose. However in this case, dary depression. exposure and response prevention appears to be 29. B. Both SSRIs and clomipramine are considered the best answer. mebooksfree.com mebooksfree.comfirst line treatment,mebooksfree.com however due to better side 38. mebooksfree.comA, D, E. mebooksfree.com mebooksfree.com effect profile, SSRIs are preferred. 39. C. 30. D. Many guides are giving the answer as haloperidol 40. E. PTSD may have a delayed onset, i.e after 6 months but that is not the right answer here. The Ameri- of trauma. can Psychiatric Association guidelines clearly 41. A. state that, if patients do not respond to SSRIs and 42. C. The patients who have a past history of psychiatric clomipramine, one of the treatment strategy is illness are more predisposed and so are women. augmentation with antipsychotics. The best evi- There is no such correlation with intellect.

mebooksfree.com mebooksfree.com mebooksfree.com mebooksfree.com mebooksfree.com mebooksfree.com

mebooksfree.com mebooksfree.com mebooksfree.com mebooksfree.com mebooksfree.com mebooksfree.com mebooksfree.com mebooksfree.com mebooksfree.com mebooksfree.com mebooksfree.com mebooksfree.com

60 Review of Psychiatry

43. C. Recall of events and avoidance is quite typical remember patient can develop all kind of psychi- of PTSD. Nightmares, autonomic arousal and atric disorders like depression, anxiety, PTSD in depressive symptoms can be seen in other dis- association with grief and if the symptoms are orders also. severe enough, they should receive separate mebooksfree.com mebooksfree.com 44. E. The treatment of choicemebooksfree.com is cognitive behavioral mebooksfree.comdiagnosis. This patient has psychoticmebooksfree.com symptoms mebooksfree.com therapy. All other statements are correct. (i.e. hallucinations) and should be treated with 45. B. There is history of a traumatic event followed an antipsychotic, haloperidol. The treatment by intrusion symptoms (nightmares). The most depends on symptoms, in case of occasional likely diagnosis is post-traumatic stress disorder. anxiety, alprazolam could have been used. 46. A. In case of significant depressive symptoms 47. A, B, C, D, E. antidepressant could have been used, but 48. A. The answer here is debatable. First of all, lets since the psychotic symptoms are prominent, review some facts. There is no clear cut duration we must use an antipsychotic. in which grief should get resolved. The most 51. C. This question has been answered wrongly by accepted duration for grief is 6–12 months. most of the guides. Please remember few basic However, every single textbook says that grief things about adjustment disorder and depres- mebooksfree.com mebooksfree.comusually continues beyondmebooksfree.com that period. Second, mebooksfree.comsion. Adjustment disorder is alwaysmebooksfree.com seen after mebooksfree.com brief hallucinations can be a part of normal grief, a stressful event, which is usually a negative life however continuous­ hallucinations are not seen. event. The symptoms of adjustment disorder In this case the history is that the man reported are quite similar to depression and include that wife asked him to join her. The question has depressed mood, anxiety, worry, a feeling of not mentioned if it was an auditory perception inability to cope and some degree of disturbance (i.e. he heard voice of wife) or visual percep- in individuals daily functioning. Now, a negative tion (i.e. he saw his wife) , what was the state life event can also precipitate the depressive of consciousness (whether he was awake or episode. So, the presence of a stressor cannot sleeping). In view of above its difficult to even be used to differentiate between adjustment call this phenomenon as a hallucination. Even disorder and depression. If a patient has the if we accept it as a hallucination, it appears to be symptoms severe enough to qualify the diagnosis mebooksfree.com mebooksfree.coma single episode. Theremebooksfree.com is no history of any other mebooksfree.comof depression, depression would alwaysmebooksfree.com be diag- mebooksfree.com associated symptoms. Hence, the better answer nosed ahead of adjustment disorder, irrespective here would be normal grief. Also, please remem- of whether there was a stressor or not. In this ber that grief and bereavement are often used case patient has severe symptoms such as loss interchangeably, however strictly speaking, of interest, ideas of hopelessness (patient is con- bereavement is a state of loss, whereas grief is vinced that she won’t be able to work again) and the emotional and behavioral response to loss. most importantly suicide attempt, all of which The question is talking about the behavioral and are highly suggestive of depression. Hence, the emotional response here. All in all, its a poorly diagnosis would be depressive disorder. framed and incomplete question. 52. C. Here, the diagnosis is adjustment disorder. The 49. C. symptoms are not severe enough to qualify for 50. D. In this case, death happened “couple of years the diagnosis of depression and there is a clear mebooksfree.com mebooksfree.comago”. The first timemebooksfree.com she had auditory hallucina- mebooksfree.comhistory of a stressor (diagnosismebooksfree.com of leukemia mebooksfree.com tions was after a week of his death and since in son). then it has been happening. Now, in grief “brief 53. D, E. hallucinations” can occur however here the According to DSM-IV, the following are the hallucinations are often and patient is even dis- somatoform disorders (1) somatization disorder cussing the daily matters with the “voice”. This (2) conversion disorder (3) hypochondriasis (4) clearly shows presence of psychotic symptoms body dysmorphic disorder (5) pain disorder which should be diagnosed separately. Please (6) undifferentiated somatoform disorder (7)

mebooksfree.com mebooksfree.com mebooksfree.com mebooksfree.com mebooksfree.com mebooksfree.com

mebooksfree.com mebooksfree.com mebooksfree.com mebooksfree.com mebooksfree.com mebooksfree.com mebooksfree.com mebooksfree.com mebooksfree.com mebooksfree.com mebooksfree.com mebooksfree.com

Neurotic, Stress Related and Somatoform Disorders 61

somatoform disorder, not otherwise specified. tious disorders frequently fake symptoms to get Please remember that in DSM-5, the somatoform the “sick role”. disorders are now referred as somatic symptom 65. A. La belle indifference is a phrase used to describe and related disorders. the feeling of indifference which patients of con- mebooksfree.com mebooksfree.com 54. A. The classificationmebooksfree.com of somatoform disorders mebooksfree.comversion disorders have towards theirmebooksfree.com symptoms. mebooksfree.com mentioned in the preceding answer is fre- 66. A. Sensory and motor system are involved and not quently not used by nonpsychiatrist practition- the autonomic nervous system. ers. These practitioners use other diagnoses, 67. B. The onset of conversion disorder is usually seen in which are frequently referred to as functional late childhood to early adulthood and is rare after somatic syndromes. These include chronic fatigue 35 years of age. syndrome, fibromyalgia and irritable bowel 68. A. The term “hysterical fits” is no longer used in syndrome. Somatization disorder is not a part modern terminology. The current classificatory of functional somatic syndromes. system will use the diagnosis of conversion 55. B. This patient has pain symptoms, gastrointestinal disorder with seizure The DSM diagnosis of symptoms, sexual symptoms and pseudoneuro- conversion disorder can present with either motor logical symptoms. symptoms, sensory symptoms or convulsions. mebooksfree.com mebooksfree.com 56. C. The patient had multiplemebooksfree.com normal investigations mebooksfree.comPlease remember that in ICD-10mebooksfree.com , conversion mebooksfree.com but conti­nues to believe that there is something disorder is another name for dissociative disorders. So, if we follow ICD-10, all four options are true. wrong in her head and continues to seek mul- But usually, in exams the term conversion disorder tiple consultations. The most likely diagnosis is refers to the DSM diagnosis and not the ICD. hypochondriasis. 69. A. Jealousy is not a neurological sign, the rest three 57. D. are. 58. B. 70. A, B. 59. B. 71. D. Hysterical fits or dissociative convulsions/sei- 60. D. The best answer here is somatization disorder. zures or conversion disorders with convulsions/ The patient has pain symptoms and gastroin- seizures do not occur in sleep, are not associated testinal symptoms. Going by strict definition with any injuries, are not associated with any of DSM-IV, there should be 4 pain symptoms, 2 incontinence and there is no postseizure amne- mebooksfree.com mebooksfree.comGI symptoms, 1 sexualmebooksfree.com symptom and 1 pseudo- mebooksfree.com mebooksfree.com mebooksfree.com sia or confusion. They usually occur when others neurological symptoms. However the ICD-10, are watching. simply says that there should be “multiple and 72. B. Dissociative amnesia is the most common type variable physical symptoms for which no ade­ of dissociative disorder. quate explanation has been found”. The other 73. D. In psychogenic amnesia (or dissociative amne- plausible option is somatoform pain disorder sia), usually memory is lost for events which have however it is characterized by only pain symp- some personal significance, whereas memories toms whereas in this patient intermittent vomit- for neutral events (e.g. national events) is intact. ing is also present. Hence, the memory loss is patchy and mostly for 61. D. The history of multiple scars from previous personal memories. surgeries, seeking attention from nurses, main- 74. C. mebooksfree.com mebooksfree.comtenance of sick role,mebooksfree.com demands for multiple 75. mebooksfree.comB. mebooksfree.com mebooksfree.com dia­gnostic tests and identification by a staff all 76. B. suggest a factitious disorder. 77. C. Though Ganser syndrome is usually seen in 62. B. prisoners but it is not exclusive to them. 63. C. 78. C. 64. D. Sick role means that the patient wants others to 79. B. Patient is not pregnant is pseudocyesis. Though accept him as “sick” and treat him accordingly she falsely believes that and there are also associ- by giving attention and care. Patients with facti- ated changes suggestive of pregnancy.

mebooksfree.com mebooksfree.com mebooksfree.com mebooksfree.com mebooksfree.com mebooksfree.com

mebooksfree.com mebooksfree.com mebooksfree.com mebooksfree.com mebooksfree.com mebooksfree.com mebooksfree.com mebooksfree.com mebooksfree.com mebooksfree.com mebooksfree.com mebooksfree.com

62 Review of Psychiatry

80. D. The symptoms is malingering are produced con- their resolution after the onset of menses or sciously for some conscious motive (e.g. mon- within few days of onset of menses. These symp- etary gain). In hysteria (dissociative disorders) toms are not present during the other period of the symptoms are produces unconsciously and menstrual cycles. If the depressive and anxiety mebooksfree.com mebooksfree.comthe motive is also unconsciousmebooksfree.com (e.g. attention or mebooksfree.comsymptoms are present throughoutmebooksfree.com the cycle the mebooksfree.com love from others). differential diagnose is depression, anxiety dis- 81. D. Premenstrual tension or Premenstrual syndrome orders like generalized anxiety disorder, panic is characterized by depressive and anxiety symp- disorder. Chronic fatigue syndrome is not a dif- toms one week before the onset of menses, and ferential here.

mebooksfree.com mebooksfree.com mebooksfree.com mebooksfree.com mebooksfree.com mebooksfree.com

mebooksfree.com mebooksfree.com mebooksfree.com mebooksfree.com mebooksfree.com mebooksfree.com

mebooksfree.com mebooksfree.com mebooksfree.com mebooksfree.com mebooksfree.com mebooksfree.com

mebooksfree.com mebooksfree.com mebooksfree.com mebooksfree.com mebooksfree.com mebooksfree.com

mebooksfree.com mebooksfree.com mebooksfree.com mebooksfree.com mebooksfree.com mebooksfree.com mebooksfree.com mebooksfree.com mebooksfree.com mebooksfree.com mebooksfree.com mebooksfree.com

mebooksfree.com mebooksfree.com mebooksfree.com mebooksfree.com mebooksfree.com mebooksfree.com

Chapter Substance Related 5 and Addictive Disorders

The substance relateddisorders encompass 10 separate B. Harmful use: It is a state where substance use is caus- classes of drugs which includes alcohol, caffeine, canna- ing harm but still criterion of dependence are not met. mebooksfree.com mebooksfree.combis, hallucinogens, inhalants,mebooksfree.com opioids, sedatives and hyp- Accordingmebooksfree.com to ICD-10, the harmful use ismebooksfree.com defined as a mebooksfree.com notics, stimulants, tobacco and other substances. pattern of substance use which is causing damage to physical health (e.g. hepatitis due to alcohol use) or Terminology mental health (e.g. episode of depression secondary to heavy alcohol consumption). A. Dependence: It is defined as a pattern in which the C. Abuse: The DSM-IV, does not use the concept of use of a substance or a class of substances takes on a “harmful use”. It instead uses the concept of “abuse” much higher priority for a given individual than other which is defined as a pattern of substance use that behaviors that once had a greater value. It encom- leads to one or more of the following (1) failure to ful- passes behavioral dependence (substance seeking fil obligations at work, school or home (2) substance behaviors), physical dependence (physiological use in situations in which it is physically hazardous effects of multiple episodes of substance use) and (such as while driving) (3) legal problems and (4) mebooksfree.com mebooksfree.compsychological dependencemebooksfree.com (continuous or intermit- mebooksfree.com mebooksfree.com mebooksfree.com social or interpersonal problems. tent craving). D. Intoxication: A transient condition that develops According to ICD-10, the presence of three or more following administration of a substance, in which of the following in past one year is required for diag- various mental functions such as consciousness, nosis of dependence on a substance: thinking, perception or behavior are altered. • Strong desire or sense of compulsion to take a sub- E. Withdrawal: Specific symptoms that occur after stop- stance (craving) ping or reducing the amount of substance that has • Difficulty in controlling substance taking behavior been used regularly over a prolonged period. in terms of its onset, termination or levels of use • Withdrawal symptoms (typical physiological Etiology H symptoms that develop when substance use is DSM-5 Update: In DSM-5, the cate­ gories of “dependence” and “abuse” reduced or stopped) The development of have been removed and clubbed mebooksfree.com mebooksfree.com• Tolerance (increased dosesmebooksfree.com of substance is required substancemebooksfree.com use disor- under a single diagnosticmebooksfree.com category of mebooksfree.com to achieve the effects originally produced by lower ders is best explained “substance use disorders”. doses) by a biopsychosocial H • Progressive neglect of alternative pleasures or model. It means that DSM-5 Update: Pathological gam- interests because of substance use there is an interaction bling has been included along with substance related disorders under • Persistence with substance use despite clear evi- of biological factors, the diagnostic entity of “gambling dence of harmful consequences. psychological factors disorder”.

mebooksfree.com mebooksfree.com mebooksfree.com mebooksfree.com mebooksfree.com mebooksfree.com

mebooksfree.com mebooksfree.com mebooksfree.com mebooksfree.com mebooksfree.com mebooksfree.com mebooksfree.com mebooksfree.com mebooksfree.com mebooksfree.com mebooksfree.com mebooksfree.com

64 Review of Psychiatry

and social factors which results in development of sub- Table 1: Absolute alcohol concentration in various preparations. stance use disorders (dependence, harmful use or abuse). Concentration of alcohol by The drugs act on particular receptors and brain path- Preparation volume (% ABV) ways and these receptors and pathways have been found Spirits (whiskey, rum, gin, 40 mebooksfree.com mebooksfree.comto play a central role in developmentmebooksfree.com of substance use vodka,mebooksfree.com brandy, etc.) mebooksfree.com mebooksfree.com disorders. Of parti­cular importance are the dopaminergic Arrack 33 neutrons in the ventral tegmental area which project to Fortified wines 14–20 cortical and limbic regions, especially the nucleus accum- Wines 5–13 bens. This pathway is involved in the sensation of reward Beer (strong) 8–11 (or pleasure) and is believed to be the major mediator Beer (standard) 3–4 of effects of substances. This pathway is also known as “brain reward pathway”. The major neurotransmitters involved in development depending on whether the alcohol was ingested on of substance used disorders include opioids, catechola- an empty stomach (absorption is faster) or with food mines (particularly dopamine) and g-aminobutyric acid (absorption is slower). (GABA). mebooksfree.com mebooksfree.com mebooksfree.comMellanbymebooksfree.com effectQ: Studies have shown thatmebooksfree.com intoxicating mebooksfree.com The evidence from studies of twin, adoptees and sib­ effects of alcohol are greater at a given blood alcohol level lings has also suggested the role of genetic factors in when BAC (blood alcohol concentration) is increasing than development of substance abuse. for the same BAC when the blood alcohol level is falling. Apart from biological factors, learning and condi- tioning is also known to contribute to development Reverse tolerance: This refers to the phenomenon where of the substance use disorder. The use of substance the intoxi­cating effects of alcohol are seen progressively Q can result in an intense sense of euphoria, it also with lower dosages . A patient may report that he ­frequently alleviates the negative emotions (such as sad- gets intoxicated with much smaller amounts of alcohol ness, anxiety). This results in reinforcement of substance now in comparison to the past. It is believed to be taking behavior. Other factors like peer pressure, social secondary to decreasing levels of alcohol metabolizing acceptance, easy availability and the personality type enzymes secondary to progressive liver dysfunction. A mebooksfree.com mebooksfree.comof the individual also contributemebooksfree.com to the development of similarmebooksfree.com concept of “sensitization” is seenmebooksfree.com in cocaine, mebooksfree.com ­substance use disorders. amphetamines, opioids and cannabis where in aug­ mented stimulant response is observed with repeated, ALCOHOL intermittent exposure to a specific drug. It is believed to be due to changes in the brain reward pathways. Ethyl alcohol is the active ingredient of alcoholic drinks. Metabolism: About 90% of absorbed alcohol is The concentration of ethyl alcohol (ethanol) varies across metabolized through oxidation in the liver, the remaining the preparations. The standard drink or a unit of alcohol 10% is excreted unchanged by the kidneys and the corresponds to 10 mL of absolute alcohol or 7.8 gram of lungs. The alcohol in alveolar air is in equilibrium with absolute alcohol (specific gravity of alcohol = 0.78). alcohol in blood passing through pulmonary capillaries, One standard drink = 1 peg (30 mL) of spirits = 1 glass hence determining the alcohol levels in breath by breath (125 mL) of wine = 1 glass (60 mL) of fortified wine = 1/2 analyzer gives a good estimate of blood alcohol levels. mebooksfree.com mebooksfree.compacket of arrack = 1/2 bottle ofmebooksfree.com standard beer = 1/4 bottle Themebooksfree.com rate of oxidation by the liver is constantmebooksfree.com and is mebooksfree.com of strong beer. around 7–10 gram an hour (which equals to amount of Arrack is the country made liquor. Fortified wines are alcohol in one standard drink). Alcohol is converted by prepared by adding brandy to wine. activity of enzyme alcohol dehydrogenase into acetalde- Absorption: About 10% of alcohol is absorbed from sto­ hyde, which is further oxidized by aldehyde dehydroge- mach and remainder from small intestineQ. Peak blood nase into acetate. Acetate is converted to carbon dioxide alcohol concentration is reached in 30–90 minutes, and water.

mebooksfree.com mebooksfree.com mebooksfree.com mebooksfree.com mebooksfree.com mebooksfree.com

mebooksfree.com mebooksfree.com mebooksfree.com mebooksfree.com mebooksfree.com mebooksfree.com mebooksfree.com mebooksfree.com mebooksfree.com mebooksfree.com mebooksfree.com mebooksfree.com

Substance Related and Addictive Disorders 65 Acute Intoxication include disturbances of consciousness, disorientation to time, place and person, hallucinations (most commonly Alcohol is a depressant of the central nervous system. The visual) coarse tremors and autonomic hyperactivity. excitement that follows alcohol use is due to decrease in mebooksfree.com mebooksfree.comconscious self control. The symptomsmebooksfree.com and signs of alcohol Alcoholmebooksfree.com Induced Disorders mebooksfree.com mebooksfree.com intoxication depends on the blood alcohol concentration. Following symptoms develop: The useof alcohol may be associated with development Blood levels Symptoms of various mental disorders. Usually alcohol induced dis- 20-30 mg/dL: Slowness of motor performance and orders, resolve within one month of cessation of alcohol decreased thinking ability. 30 mg/dLQ is intake. If the symptoms of mental disorder persist beyond the legal limit for driving in India that, the possibility of an independent mental disorder 30-80 mg/dL: Worsening of motor performance and should be entertained. The following disorders have been further decrease in thinking ability described: 80-200 mg/dL: Incoordination, judgment errors, mood 1. Alcohol induced psychotic disorders 2. Alcohol induced bipolar disorders lability 3. Alcohol induced depressive disorders 200-300 mg/dL: Nystagmus, slurring of speech, alcoholic 4. Alcohol induced anxiety disorders, alcohol induced mebooksfree.com mebooksfree.comblackoutsQ mebooksfree.com mebooksfree.com mebooksfree.com mebooksfree.com sleep disorder >300 mg/dL: Impaired vital signs and possible death 5. Alcohol induced sexual dysfunction Q Alcoholic blackout: It refers to anterograde amnesia 6. Alcohol induced neurocognitive disorders. seen during intoxication. The person is unable to recall Alcohol induced neurocognitive disorders: Long-term the events that happened when his blood alcohol levels alcohol use can cause amnestic disorders characterized were between 200-300 mg/dL. by disturbances in short-term memory. The classic names Alcohol Withdrawal for alcohol induced amnestic disorders are Wernicke’s encephalopathy and Korsakoffs syndrome. It refers to the symptoms which develop after cessation A. Wernicke’s encephalopathy: It is the acute neurolo­ of alcohol intake. In most patients the following sequence gical complication characterized by the following is seen, though all symptoms do not necessarily occur in symptoms (pneumonic GOA): mebooksfree.com mebooksfree.comall patients. mebooksfree.com G: mebooksfree.comGlobal confusionQ mebooksfree.com mebooksfree.com Q After 6-8 hours: The classic and most common sign of O: Ophthalmoplegia, usually 6th nerve palsy (sec- alcohol withdrawal is tremulousness (coarse tremors)Q. ond most common is 3rd nerve palsy) causing, Other symptoms include gastrointestinal symptoms (like horizontal nystagmus and gaze palsy) Q nausea and vomiting), sympathetic autonomic hyper­ A: Ataxia activity including arousal, anxiety, sweating, hypertension, Although Wernicke’s encephalopathy can be com- Q mydriasis and tachycardia. pletely reversed with treatment, often residual ataxia and horizontal nystagmus remain despite treatment. Q After 12-24 hours: Alcoholic hallucinosis . It refers to Wernicke’s encephalopathy may clear spontaneously in hallucinations in the absence of any disturbances of cons­ days to weeks or progress to Korsakoff’s syndrome. ciousness. Usually auditory hallucinations are present. B. Korsakoff’s syndrome: It is the chronic neurological After 24-48 hours: Alcohol withdrawal seizures. The complication of long-term alcohol use. It is charac- mebooksfree.com mebooksfree.comseizures are usually generalizedmebooksfree.com and tonic-clonic. Usually terizedmebooksfree.com by impaired recent memory, mebooksfree.comanterograde mebooksfree.com patients have more than one seizures in a span of 3-6 amnesiaQ (inability to form new memory), retro- hours, hence often the term cluster seizures is used for grade amnesiaQ (inability to recall old memories) alcohol withdrawal seizures. and confabulationsQ (making of false stories to fill After 48-72 hours: Delirium tremens. Alcohol withdrawal memory gaps, which is unintentional). The antero- delirium is a medical emergency and if untreated the grade amnesia is much more prominent than the mortality rate is around 20%. The symptoms and signs retrograde amnesia.

mebooksfree.com mebooksfree.com mebooksfree.com mebooksfree.com mebooksfree.com mebooksfree.com

mebooksfree.com mebooksfree.com mebooksfree.com mebooksfree.com mebooksfree.com mebooksfree.com mebooksfree.com mebooksfree.com mebooksfree.com mebooksfree.com mebooksfree.com mebooksfree.com

66 Review of Psychiatry

The pathophysiology for both Wernicke’s syndrome Blood alcohol concentration is usually measured and Korsakoff’s syndrome isthiamine deficiencyQ. The using breath analyzers. It can also be estimated by neuropathological lesions are usually symmetrical and using Widmark formula, if the amount of alcohol involve mammillary bodiesQ. Other sites of lesion include consumed and body weight is known. mebooksfree.com mebooksfree.comthalamus, hypothalamus, midbrain,mebooksfree.com pons, medulla, fornix • mebooksfree.comCarbohydrate deficit transferrin (CDT)mebooksfree.com: The most mebooksfree.com and cerebellum. sensitive and specific laboratory test for the identi­ The treatment of Wernicke’s encephalopathy is high fication of heavy drinking is elevated blood levels dose of parenteral thiamine. Treatment of Korsakoff of carbohydrate deficit transferrin. syndrome is oral thiamine for 3–12 months. Only around • Gamma-glutamyl transferase (GGT): Elevated levels 20% of patients with Korsakoff syndrome recover. of GGT are again suggestive of heavy drinking. The C. Marchiafava bignami disease: It is a rare neurologi- levels of both CDT and GGT return towards normal cal complication of long-term alcohol use. It is charac- within days to weeks of stopping drinking. terized by epilepsy, ataxia, dysarthria, hallucinations • Mean corpuscular volume: MCV is frequently ele­ and intellectual deterioration. The pathophysiology is vated in individuals who indulge in heavy drinking. demyelination of corpus callosum, optic tracts and • Other test include elevated levels of ALT (alanine cerebellar peduncles. mebooksfree.com mebooksfree.com mebooksfree.com mebooksfree.comamino­transferase) and alkaline phosphatase,mebooksfree.com which mebooksfree.com Evaluation indicate liver injury secondary to heavy drinking. A. Screening test: One of the most commonly used Treatment screening test is CAGE questionnaireQ, which The treatment of alcohol dependence is done in the includes the following four questions: following phases. • Have you ever felt that you should Cut down on A. Detoxification: It is the first phase of treatment which your drinking? involves management of withdrawal symptoms. • Have people Annoyed you by criticizing your drink- The usual duration of detoxification is 7-14 days. ing? BenzodiazepinesQ are the drugs of choice (particu- • Have you ever felt bad or Guilty about your drink- larly chlordiazepoxideQ) for all the withdrawal symp- ing? toms ranging from common ones like tremors and mebooksfree.com mebooksfree.com• Have you ever had a drinkmebooksfree.com first thing in the morning nauseamebooksfree.com to severe withdrawal symptomsmebooksfree.com like alcohol mebooksfree.com to steady your nerves or to get rid of hangover (Eye withdrawal seizures and delirium tremens. In addi- opener)? tion vitamins (particularly thiamine) must be given as A positive response on two or more than two of the patients usually are deficient in vitamins. above questions, is suggestive of alcohol use disorder. Carbamazepine can also be used in place of benzo­ Another commonly used screening test is AUDIT diazepines however other anticonvulsants do not have (alcohol use disorders identification test). Others tests any role. The antipsychotics can be used in patients such as SADQ (severity of alcohol dependence ques- with delirium tremens and alcoholic hallucinosis. tionnaire) are used to determine the severity of depen­ B. Maintenance of abstinence: After the completion of dence. detoxification, the next phase involves long-term treatment to maintain the abstinence. It involves both B. Diagnostic markers: Apart from the screening tests, pharmacological and nonpharmacological treatment. the blood test may also help in the identification of • Pharmacological treatment: The drugs used are of heavy drinkers who are susceptible to development mebooksfree.com mebooksfree.com mebooksfree.com mebooksfree.comtwo types: mebooksfree.com mebooksfree.com of alcohol use disorders. a. Deterrent agents: The most commonly used • Blood alcohol concentration: It can be used to judge deterrent agent is disulfiramQ. It is an irre- tolerance to alcohol. For example, if a person has versible inhibitor­ of aldehyde dehydrogenase, high blood alcohol concentration without showing the enzyme which metabolites acetaldehyde. any signs of intoxication, it indicates the pre­sence Acetaldehyde is the first breakdown product of of tole­rance and high chances of presence of alco- alcohol. If a patient who is on disulfiram, con- hol use disorders. sumes alcohol, it results in accumulation of toxic mebooksfree.com mebooksfree.com mebooksfree.com mebooksfree.com mebooksfree.com mebooksfree.com

mebooksfree.com mebooksfree.com mebooksfree.com mebooksfree.com mebooksfree.com mebooksfree.com mebooksfree.com mebooksfree.com mebooksfree.com mebooksfree.com mebooksfree.com mebooksfree.com

Substance Related and Addictive Disorders 67

levels of acetaldehyde and causes a number progress to subcutaneous administration, once he is not of unplea­sant signs and symptoms, termed as able to find any patent vein. The subcutaneous route is disulfiram ethanol reaction (DER). known as “skin popping”. Other deterrent agents include citrated cal- mebooksfree.com mebooksfree.comcium carbimide andmebooksfree.com metronidazole. Intoxicationmebooksfree.com mebooksfree.com mebooksfree.com b. Anticraving agents: These agent reduce craving, which is an important reason for relapse. The Opioids when taken (especially intravenously) produce a anticraving agents include naltrexoneQ, acam- feeling of intense euphoria. The other symptoms include prosateQ, topiramate, serotonergic agents like a feeling of warmth, heaviness of extremities and facial fluoxetine and baclofen. flushing. This initial euphoria is followed by a period of • Nonpharmacological treatment: These are psycho- sedation (known as “nodding off”). social treatment methods and include: Opioids overdose can be lethal due to respiratory a. Cognitive behavioral therapies: A large number depression. The symptoms of overdose include coma, of therapies have been found to have efficacy in slow respiration, hypothermia, hypotensionQ, bradycar- maintaining abstinence. These include motiva- dia, pin point pupils, cyanosis. tional enhancement therapy, relapse preven- mebooksfree.com mebooksfree.comtion model and cognitivemebooksfree.com therapy. Withdrawalmebooksfree.com Symptoms mebooksfree.com mebooksfree.com b. Alcoholic anonymous: It is a self help group, The sudden stopping of opioids after prolonged use or which follows 12 steps to quit alcohol use. The intake of opioid antagonists like naltrexone can produce members include patients who have recovered withdrawal symptoms. The short-term use of opioids from alcoholism, current alcohol users and also decreases the activity of noradre­nergic neurons and the volunteers. long-term use results in compensatory hyperactivity. c. Family therapy When opioids are suddenly stopped, there are symptoms d. Group therapy of rebound noradrenergic hyperactivity. This hypothe­ sis also explains the mechanism of action of clonidine OPIOIDS (alpha-2 adrenergic receptor agonist, which decreases The termopiates is used to describe the psychoactive norepinephrine release) in management of opioid with- alkaloids (like morphine and codeine) which are present drawal. mebooksfree.com mebooksfree.comin opium (derived from papavermebooksfree.com somniferum, the poppy Themebooksfree.com withdrawal symptoms usually appearmebooksfree.com around mebooksfree.com plant). The termopioids is a broader term which also 6–8 hoursQ after the last dose, peak during the second includes synthetic compounds like heroin and metha- or third day and subside during the next 7–10 days. The done, which share the action and effects of opiates. withdrawal from opioids produces a flu-like syndromeQ Heroin (diacetyl morphine) is the most commonlyQ with the following symptoms. abused opioid. Since, it is more lipid soluble than mor- 1. LacrimationQ, rhinorrheaQ, sweating, diarrheaQ phine, it crosses blood brain barrier faster and has a more 2. Yawning and piloerectionQ rapid onset of action. Heroin was initially used as a treat- 3. Pupillary dilationQ ment for morphine addiction, however, it was realized 4. Muscle cramps and generalized bodyache that dependence forming potential of heroin is higher 5. InsomniaQ, anxiety, hypertension and tachycardia than morphine. The street names of heroin includes 6. Nausea, vomiting and anorexia. “smack” and “brown sugar” amongst others. The street mebooksfree.com mebooksfree.comforms are often impure and mebooksfree.comhave adulterants like starch Treatmentmebooksfree.com mebooksfree.com mebooksfree.com (fructose and sucrose), quinine, chalk powder, paraceta- mol and talcum powder, etc. A. Detoxification: In this stage, the main focus is on Opioids can be taken orally, snorted intranasally the manage­ment of withdrawal symptoms. The (also called chasing the dragon), and injected intrave- medications used are usually long acting opioids like nously or subcutaneously. The intravenous users tend methadoneQ or buprenorphine. Both medications, to gradually shift from peripheral veins to larger veins in view of their agonist action at opioid receptors, (a phenomenon called mainliningQ). The user may suppress the withdrawal symptoms. Other opioids

mebooksfree.com mebooksfree.com mebooksfree.com mebooksfree.com mebooksfree.com mebooksfree.com

mebooksfree.com mebooksfree.com mebooksfree.com mebooksfree.com mebooksfree.com mebooksfree.com mebooksfree.com mebooksfree.com mebooksfree.com mebooksfree.com mebooksfree.com mebooksfree.com

68 Review of Psychiatry like dextropropoxyphene can also be used. Usually CANNABIS detoxification medicines are required for 2-3 weeks. Cannabis is derived from the hemp plant, cannabis Another method is use of clonidineQ for detoxification. sativa. The plant has several varieties named after the However, clonidine provides considerably less reduction mebooksfree.com mebooksfree.com mebooksfree.comregionsmebooksfree.com where it is found (e.g. cannabis sativamebooksfree.com indica in mebooksfree.com in symptoms in comparison to buprenorphine or metha­ India, cannabis sativa americana in USA). Cannabis is the done. Clonidine is thus mostly used as an adjunct to most commonly used illegal drugQ in the world and in methadone or buprenorphine during detoxification. India. The street names include joints, marijuana, grass, Accelerated detoxification: In this method, initially low pot, weed, etc. doses of naltrexone is given to patient. Naltrexone being The active ingredient, which is responsible for the an opioid antagonist, produces severe withdrawal symp- psychoactive effects of cannabis isd -9 tetrahydrocan- toms. After that, clonidine is used to control the symp- nabinol (THC)Q. The various preparations of cannabis toms. This method reduces the detoxification period to includes. 4-5 days. Table 2: THC concentration in various cannabis preparations. B. Maintenance treatment: It follows the detoxification Cannabis preparation THC content (%) and the aim is to prevent the relapse. There are two Bhang (derived from dried leaves) 1 mebooksfree.com mebooksfree.comdifferent pharmacologicalmebooksfree.com approaches for mainte- Ganja mebooksfree.com(derived from inflorescence) 1–2 mebooksfree.com mebooksfree.com nance phase. Hashish/Charas (derived from resinous 8–14 • Opioid substitution therapy: In this method, the exudates) illicit, parenterally administered and short acting Hash oil (lipid soluble plant extract) 15–40 opioids (like heroin) are replaced by medically safe, The cannabis can be ingested orally or is more com- orally taken and long acting opioids. The long act- monly smoked. It is unsuitable for intravenous use because ing opioids such as methadone, buprenorphine are of poor solubility in water and risk of anaphylaxis due to mostly used. Levo alpha acetylmethadol was also undissolved particulate matter. used in past, however it has since been stopped as it is known to cause torsades de pointes. Intoxication These orally used opioids are given at govern- It is characterized by euphoria, subjective sense of ment approved centres. Though the patient conti­ slowing­ of time, sense of floating in air, reddening of nues to remain dependent,­ however he is protected mebooksfree.com mebooksfree.com mebooksfree.comconjunctivamebooksfree.comQ (due to dilatation of conjunctivalmebooksfree.com blood mebooksfree.com from medical consequence of parenteral opioids vessels), increased appetite and dryness of mouth. (like HBV, HIV infection) and does not need to Other symptoms ­include depersonalization, derealiza- indulge in criminal­ activities to fund the illicit opi- tion, synesthesiaQ (cross over of sensory perceptions. For oid use. example,­ patient may report that he is “seeing” music and Q • Opioid antagonist treatment: Naltrexone can be “hearing” ­ lights). given to the patient after detoxification is complete. Sometimes, after consumption of cannabis, the per- The rationale is that naltrexone will block the opi- son might feel restless, fearful, extremely anxious (similar oid receptors and any opioid use would fail to pro- to panic attack) and may feel that he is about to go crazy. duce the euphoric response and hence would not This unpleasant experience is known as “bad trip”Q. be repeated. • Nonpharmacological approaches like cognitive Withdrawal Symptoms mebooksfree.com mebooksfree.combehavioral therapy, narcoticmebooksfree.com anonymous (12 step It was mebooksfree.comearlier believed that cannabis doesn’tmebooksfree.com cause physi- mebooksfree.com self help groups), family therapy and group therapy cal dependence and produces no withdrawal symptoms, are also useful. however recent studies have shown that there are mild C. Overdose treatment: The opioids are lethal in over- withdrawal symptoms within 1-2 weeks of cessation and dose. The drug of choice for treatment of opioid over- include insomnia, anxiety, decreased appetite, irritability, ­ dose is i.v. naloxoneQ (short acting opioid antagonist). etc.

mebooksfree.com mebooksfree.com mebooksfree.com mebooksfree.com mebooksfree.com mebooksfree.com

mebooksfree.com mebooksfree.com mebooksfree.com mebooksfree.com mebooksfree.com mebooksfree.com mebooksfree.com mebooksfree.com mebooksfree.com mebooksfree.com mebooksfree.com mebooksfree.com

Substance Related and Addictive Disorders 69 Cannabis Related Disorders Withdrawal Symptoms 1. Cannabis induced psychotic disorder: It is also some- Hallucinogens do not cause any physical dependence, times referred to as “hemp insanity”. The patient has hence tolerance and withdrawal symptoms are not seen. mebooksfree.com mebooksfree.compsychotic symptoms suchmebooksfree.com as delusions and hallucina- Themebooksfree.com use of hallucinogens like LSD canmebooksfree.com be associated mebooksfree.com tions. with flashback phenomenonQ which refers to recurrence 2. Cannabis induced anxiety disorders. of LSD use experience in the absence of current LSD use. 3. Flash back phenomenonQ: It is characterized by a recurrence of cannabis use experience in the absence Treatment of current cannabis use. Mostly psychotherapeutic techniques are used to prevent 4. Running amokQ: It is described as development of rage following cannabis use, in which person may relapse. hurt or even kill others in an indiscriminate fashion. 5. Amotivational syndromeQ: It is characterized by an STIMULANTS unwillingness to persist in any task, whether at school or at work. The patient appears uninterested, lethargic Cocaine mebooksfree.com mebooksfree.comand apathetic. mebooksfree.comCocainemebooksfree.com is derived from the plant erythroxymebooksfree.comlum­ coca. mebooksfree.com Sigmund FreudQ had studied its pharmacological­ effects Treatment and is also believed to be addicted to cocaine for a long time. Coca cola used to contain cocaine till 1903 after As withdrawal symptoms are mild, no medications are which it ceased to be an ingre­dient. usually used. If required, benzodiazepines can be used Cocaine was initially used as a local anestheticQ and for short-term. still is used in eye, nose and throat surgeryQ. The local Long-term treatment usually involves the psycho­ anesthetic effect is mediated by blockade of fast, sodium therapeutic approach and patient may be offered cognitive channels. behavioral therapy, family therapy or group therapies. Cocaine acts primarily by blocking dopamine recep- torsQ (D1 and D2) and increasing dopamine concentra- HALLUCINOGENS tion in synaptic cleft. It is also an inhibitor of uptake of norepinephrine and hence has significant sympathomi- mebooksfree.com mebooksfree.comThis class includes a varietymebooksfree.com of drugs like LSD (Lysergic metic mebooksfree.comeffect. It causes marked vasoconstrictionmebooksfree.com of peri- mebooksfree.com acid diethy­lamide), mescaline, psilocybin, methylene­ pheral arteries, which results in hypertensionQ, further, dioxyamphetamine (MDMA, also called ecstasy), phen­ vasoconstriction of the epicardial coronary arteries, can cyclidine (angel dust) and ketamine. lead to ischemic myocardial injury. Cocaine use can also cause seizures. Cocaine (most common) and ampheta- Intoxication mines (second most common) are the substances mostly associated with seizures. The characteristic symptoms of LSD (and other halluci- Cocaine is usually inhaled (known as snorting). Due nogens) intoxication are depersonalization, derealization, to its vasoconstrictive properties nasal inhalation of Q synesthesia (also called as reflex hallucinations ­wherein cocaine causes nasal congestion and can even result in patient may report cross over of sensory perceptions), nasal septal perforationQ. Long-term use can also cause illusions and hallucinations, autonomic hyperactivity fea- jet black pigmentation of tongueQ. mebooksfree.com mebooksfree.comtures such as pupillary dilatation,mebooksfree.com tachycardia, sweating, Othermebooksfree.com methods of intake are smoking (knownmebooksfree.com as free- mebooksfree.com palpitations, tremors, etc. basingQ) and subcutaneous or intravenous injections. Similar to cannabis,at times, patient may become Freebasing involves mixing street cocaine (which usu- ­restless, fearful and may develop panic reaction (bad ally has procaine or sugar as adulterants) with freebase trip)Q. Usually patient can be calmed down by reassur- (chemically extracted pure cocaine). A particular potent ance. However in cases with extreme agitation, benzodia­ way is consumption of cocaine and heroin (called speed- zepines or antipsychotics may be required. ball) together.

mebooksfree.com mebooksfree.com mebooksfree.com mebooksfree.com mebooksfree.com mebooksfree.com

mebooksfree.com mebooksfree.com mebooksfree.com mebooksfree.com mebooksfree.com mebooksfree.com mebooksfree.com mebooksfree.com mebooksfree.com mebooksfree.com mebooksfree.com mebooksfree.com

70 Review of Psychiatry

Crack, is a freebase form of cocaine which is smoked. which includes smoking, chewing, applying, sucking It is extremely potent and even a single use can cause and gargling. Beedi smoking is the most common form intense craving. followed by cigarette smoking. The active ingredient of Intoxication: The intoxication is characterized by eupho- tobacco, which causes addiction is nicotine. The constitu- mebooksfree.com mebooksfree.comria, pupillary dilatation, tachycardia,mebooksfree.com hypertension and ents responsiblemebooksfree.com for cardiovascular disordersmebooksfree.com are nicotine mebooksfree.com sweating. Acute intoxication with moderate to high and carbon monoxide. dose of cocaine may be associated with paranoid idea- Nicotine has a stimulant action and improves the tions, auditory hallucinationsQ and visual illusions. The attention, learning, reaction time and problem solving patients also occasionally report of tactile hallucinations ability. (feeling of insects crawling under the skin), also known as The withdrawal symptoms can develop within two cocaine bugs. (also known as formication and magnan hours of smoking the last cigarette and peak in 24-48 pheno­menonQ). hours. These symptoms include craving for nicotine, irri- tability, anxiety, difficulty concentrating,bradycardia Q, Withdrawal symptoms: Cocaine causes strong psycho­ drowsiness and paradoxical trouble sleeping, increased logical dependence­ Q however physiological dependence appetite and weight gain. (tolerance and withdrawal symptoms) is mildQ in comparison. The withdrawal symptoms includes feeling mebooksfree.com mebooksfree.comlow, exhaustion, lethargy, fatigue,mebooksfree.com insatiable hunger. The Treatmentmebooksfree.com mebooksfree.com mebooksfree.com most severe withdrawal symptom is depression, which Pharmacotherapy can be associated with suicidal ideation. Cocaine induced psychotic disorder: It is most commonly 1. Nicotine replacement therapy: It is used to relieve seen with intravenous use and crack users. The hallmark the withdrawal symptoms by substituting nicotine in is paranoid delusions (delusion of persecution) and tobacco with nicotine in safer forms as they do not auditory hallucinationsQ. Visual and tactile hallucinations contain other harmful constituents present in tobacco. (cocaine bugs) can also be present. The disorder is quite The various preparations include nicotine gums, nico- similar to paranoid schizophreniaQ in its presentation. tine lozenges, nicotine patches, nicotine inhalers and nicotine spray). : The withdrawal symptoms are usually mild Treatment 2. Medications which can be used include bupropion and no specific pharmacological agents reduces the (first lineQ) and clonidine and nortriptyline (second intensity of withdrawal. Treatment mostly relies on mebooksfree.com mebooksfree.com mebooksfree.com line).mebooksfree.com Varenicline is a new medication whichmebooksfree.com has been mebooksfree.com psychotherapeutic interventions like cognitive behavioral approved for use in tobacco dependence. Varenicline therapy, group therapy, and support groups such as acts as an agonist at a 7 nicotinic acetylcholine recep- narcotic anonymous. tors and partial agonist on a4b2 receptors. Amphetamines Apart from medications behavioral therapy is also considered beneficial. The major amphetamines include dextroampheta- mine, methamphetamine. Methylphe­ni­date is also an OTHER DRUGS amphetamine like compound. Amphetamines are used to increase performance and induce a euphoric feeling. 1. Inhalants or volatile solvents: These include gaso- Long-term use can result in amphetamine induced psy- line (petrol), glues, thinners, industrial solvents. These chotic disorder, whose hallmark is presence of paranoid solvents are soaked in a cloth and than are sniffed mebooksfree.com mebooksfree.comdelusions (delusion of persecutionmebooksfree.comQ) and auditory hal- (vaporsmebooksfree.com are inhaled). It is more commonmebooksfree.com seen in mebooksfree.com lucinationsQ. children and adolescents. Long-term use may cause irreversible damage to livers and kidneys, peripheral TOBACCO neuropathy and brain damage. 2. Benzodiazepines and other sedative hypnotics: It is the most commonly used substance in India (caf- Benzodia­zepines can produces physical and psy- feine not considered) and is used in a variety of ways chological dependence. The withdrawal symptoms

mebooksfree.com mebooksfree.com mebooksfree.com mebooksfree.com mebooksfree.com mebooksfree.com

mebooksfree.com mebooksfree.com mebooksfree.com mebooksfree.com mebooksfree.com mebooksfree.com mebooksfree.com mebooksfree.com mebooksfree.com mebooksfree.com mebooksfree.com mebooksfree.com

Substance Related and Addictive Disorders 71

usually include anxiety, irritability, insomnia and in with feeling of improved efficiency, increased energy some cases seizures. The treatment usually involves levels and concentration. Excessive use can produce slow tapering and then stopping of benzodiazepines anxiety, restlessness, irritability. Caffeine can also along with supportive measures. produce physiological dependence and withdrawal mebooksfree.com mebooksfree.com 3. Caffeine: Caffeine is themebooksfree.com most widely used psycho­ symptomsmebooksfree.com include anxiety,irritability, midmebooksfree.com depressive mebooksfree.com active substance worldwide. Caffeine use is associated symptoms, nausea and vomiting.

QUESTIONS AND ANSWERS

QUESTIONS Alcohol Substance Use Disorders 6. Irresistible urge to drink alcohol is known as: (DNB June 2011) 1. Which of the following is not an important factor A. Kleptomania B. Pyromania mebooksfree.com mebooksfree.comin development of substancemebooksfree.com dependence? mebooksfree.comC. Dipsomania D. Trichotillomaniamebooksfree.com mebooksfree.com (AIIMS Nov 2009) A. Personality B. Family history 7. All of the following statements are true about C. Peer pressure D. Intelligence blackouts except: (AIIMS May 2014) A. The person appears confused to the onlookers 2. Not included in definition of substance abuse B. Remote memory is relatively intact during the syndrome: (PGI May 2011) blackout A. Withdrawal symptom C. It is a discrete episode of anterograde amnesia B. Use despite knowing that it can cause physical/ D. It is associated with alcohol intoxication. mental harm C. Tolerance to drug 8. A patient taking 120 mL alcohol everyday since last D. Recurrent substance abuse 12 years is brought to the hospital by his wife and is E. Use despite substance related legal problems diagnosed to have alcohol dependence syndrome. mebooksfree.com mebooksfree.com mebooksfree.com mebooksfree.comWhich of the following drug should mebooksfree.combe avoided in mebooksfree.com 3. All of the following are criteria for substance the management? (AIIMS Nov 2014) dependence­ except: (AI 2012) A. Phenytoin B. Disulfiram A. Repeated unsuccessful attempts to quit the sub- C. Naltrexone D. Acamprosate stance B. Recurrent substance related legal problems/use 9. All of the following are true about alcohol depen­ of illegal substances dence syndrome except: (DNB NEET 2014-15) C. Characteristic withdrawal symptoms; substance A. No tolerance taken to relieve withdrawal B. Withdrawal symptoms D. Substance taken in larger amount and for longer C. CAGE questionnaire than intended D. Physical dependence 4. Symptomatic treatment is not required in with- 10. First symptom to appear in alcohol withdrawal is: mebooksfree.com mebooksfree.comdrawal of: mebooksfree.com(AI 1998) mebooksfree.com (AIIMSmebooksfree.com May 2015) mebooksfree.com A. Cannabis B. Morphine A. Visual hallucinations C. Alcohol D. Cocaine B. Sleep disturbance C. Tremors 5. Drugs which cause both physical and psychologi- D. Delirium cal depen­dence are: (DNB NEET 2014-15) A. Opioids B. Alcohol 11. Most common symptom of alcohol withdrawal is: C. Nicotine D. All of the above (DNB NEET 2014-15, AI 2007)

mebooksfree.com mebooksfree.com mebooksfree.com mebooksfree.com mebooksfree.com mebooksfree.com

mebooksfree.com mebooksfree.com mebooksfree.com mebooksfree.com mebooksfree.com mebooksfree.com mebooksfree.com mebooksfree.com mebooksfree.com mebooksfree.com mebooksfree.com mebooksfree.com

72 Review of Psychiatry

A. Bodyache B. Tremor 20. Wernicke’s encephalopathy is due to deficiency of: C. Diarrhea D. Rhinorrhea (DNB NEET 2014-15) A. Folic acid B. Thiamine 12. Which of the following is characteristic of alcohol C. Ascorbic acid D. Pyridoxine mebooksfree.com mebooksfree.comwithdrawal? mebooksfree.com(AIIMS 1991) mebooksfree.com mebooksfree.com mebooksfree.com A. Hallucination B. Illusion 21. Which of the following is included in the classical C. Delusion D. Drowsiness triad of Wernicke’s encephalopathy? A. Peripheral neuropathy (DNB NEET 2014-15) 13. Widmark formula is used for: (AIIMS 1993) A. Opium B. Cannabis B. Autonomic dysfunction C. Alcohol D. Amphetamine C. Ataxia D. Abdominal pain 14. Male started drinking alcohol at age of 20 years, 22. Not affected in Wernicke’s disease: presently taking 3 quarters daily over 30 years, (DNB NEET 2014-15) complains that now he gets the kick in 1 quarter. A. Hypothalamus B. Thalamus Probable diagnosis is: (AIIMS Nov 2012) A. Withdrawal C. Hippocampus D. Mammillary bodies B. Mellanby phenomenon 23. An alcoholic patient comes to your office, he mebooksfree.com mebooksfree.comC. Reverse tolerance mebooksfree.com mebooksfree.comcan’t tell his name. There is gross incoordinationmebooksfree.com mebooksfree.com D. Cross tolerance in walking, and his eyes are deviated to one side. 15. Psychiatric complications of alcohol dependence What is the probable diagnosis? (Bihar 2006) are: (PGI 2001) A. Wernicke’s encephalopathy A. Anxiety B. Suicide B. Korsakoff’s psychosis C. Depression D. Schizophrenia C. Alcoholic hallucinosis E. Mania D. Delirium tremens 16. Not a feature of delirium tremens is: (AI 2011) 24. Feature(s) of Korsakoff psychosis: A. Confusion (clouding of consciousness) (PGI NOV 2014) B. Visual hallucinations A. Confabulation B. Retrograde amnesia C. Coarse tremors C. Ophthalmoplegia D. Delirium D. Oculomotor nerve palsy (ophthalmoplegia) mebooksfree.com mebooksfree.com mebooksfree.com 25. mebooksfree.comKorsakoff syndrome true is/are: mebooksfree.com mebooksfree.com 17. True about delirium tremens: (PGI June 2005) (DNB NEET 2014-15) A. Clouding of consciousness A. Can be seen in chronic alcoholics B. Coarse tremors B. Absence of intellectual decline C. Chronic delirious behavior C. Chronic amnestic syndrome D. Hallucination D. All of the above E. Autonomic dysfunction 26. All are relatively normal in Korsakoff’s psychosis 18. Wernicke’s encephalopathy involves which part except: (MAHE 2003, KA 2003; J & K 2000) of central nervous system: (PGI 2000) A. Mammillary body B. Thalamus A. Implicit memory B. Intelligence C. Frontal lobe D. Arcuate fasciculus C. Language D. Learning mebooksfree.com mebooksfree.com 19. A 45-year male with amebooksfree.com history of alcohol depen­ 27. mebooksfree.comTrue statement about Korsakoff’s mebooksfree.compsychosis is: mebooksfree.com dence presents with confusion, nystagmus and (Rohtak 2000; JIPMER 1999) (UP 1999; PGI 1997) ataxia. Examination reveals 6th cranial nerve A. Severe anterograde + Mild retrograde memory weakness. He is most likely to be suffering from: defect A. Korsakoff’s psychosis. (AI 2005) B. Mild anterograde + severe retrograde memory B. Wernicke encephalopathy. defect C. De Clerambault syndrome. C. Only anterograde memory defect D. Delirium tremens. D. Only retrograde memory defect

mebooksfree.com mebooksfree.com mebooksfree.com mebooksfree.com mebooksfree.com mebooksfree.com

mebooksfree.com mebooksfree.com mebooksfree.com mebooksfree.com mebooksfree.com mebooksfree.com mebooksfree.com mebooksfree.com mebooksfree.com mebooksfree.com mebooksfree.com mebooksfree.com

Substance Related and Addictive Disorders 73

28. In Korsakoff psychosis all are seenexcept : day he had GTCS followed by another episode of A. Loss of remote memory (JIPMER 1998) GTCS after few hours. Drug which should be given B. Loss of intellectual function but preservation of to control the symptoms: (AIIMS May 2013) memory A. Sodium valproate B. Phenytoin mebooksfree.com mebooksfree.comC. Lack of insight, unablemebooksfree.com to understand the dis- mebooksfree.comC. Diazepam D. Clonidinemebooksfree.com mebooksfree.com ability 33. In alcohol withdrawal drug of choice is: D. Reversible state (DNB NEET 2014-15, PGI June 2007, AIIMS 1990) 29. A 35-year-old male comes with h/o 10-years of A. Haloperidol B. Chlordiazepoxide alcoholism and past history of ataxia with bilateral C. Naltrexone D. Disulfiram rectus palsy. He was admitted and treated. What 34. Drugs used for treatment of delirium tremens is/ changes can be expected to be seen in such condi- are: (DNB NEET 2014-15, MCI screening) tion? (PGI June 2008) A. Diazepam B. Quetiapine A. Progression to Korasakoff’s psychosis C. Chlordiazepoxide D. Both A and C B. Residual ataxia in 50% of patients C. Extraocular palsy disappears in hours 35. All of the following agents are used in the treat- mebooksfree.com mebooksfree.comD. Immediate relief frommebooksfree.com symptoms mebooksfree.comment of alcohol dependence exceptmebooksfree.com: mebooksfree.com (DNB NEET 2014-15, AI 2011) 30. A 30-year-old male with history of alcohol abuse for A. Flumazenil B. Acamprosate 15 years is brought to the hospital emergency with C. Naltrexone D. Disulfiram ‘complaints of fearfulness, misrecognition, talk- ing to self, aggressive behavior, tremulousness and 36. In patients of substance-abuse, drugs used are: seeing snakes and reptiles that are not visible to (PGI 2002) others around him. There is history of last drinking A. Naltrexone B. Naloxone C. Clonidine D. Lithium alcohol two days prior to the onset of the present E. Disulfiram complaints. He is most likely suffering from: (AIIMS Nov 2003) 37. All are anticraving agent for alcohol except: A. Delirium tremens (AIIMS May 2009) B. Alcoholic hallucinosis A. Lorazepam B. Naltrexone mebooksfree.com mebooksfree.comC. Schizophrenia mebooksfree.com mebooksfree.comC. Topiramate D. Acamprosatemebooksfree.com mebooksfree.com D. Seizure disorder 38. Which of the following is not used in delirium? 31. A 40-year-old man presents to casualty with his- (PGI Dec 2005) tory of regular and heavy use of alcohol for ten A. Haloperidol B. Lithium years and morning drinking for one year. The last C. Diazepam D. Olanzapine alcohol intake was three days back. There is no E. Risperidone history of head injury or seizures. On examina- tion, there is no icterus, sign of hepatic encepha- Opioid lopathy or focal neurological sign. The patient had coarse tremors, visual hallucinations and 39. Which of the following is not an opioid peptide? had­disorientation to time. Which of the following (AIlMS May 2005) mebooksfree.com mebooksfree.comis the best medicine tomebooksfree.com be prescribed for such a mebooksfree.comA. Endorphins B. Epinephrinemebooksfree.com mebooksfree.com patient? (AI 2004) C. Leu-enkephalins D. Met-enkephalins A. Diazepam B. Haloperidol 40. All are seen in morphine poisoning except: C. Imipramine D. Naltrexone A. Cyanosis (AI 1997) 32. A chronic alcoholic patient stopped alcohol intake B. Pinpoint pupil for 2 days due to religious reasons, developed C. Hypertension symptoms of withdrawal on first day. On second D. Respiratory depression

mebooksfree.com mebooksfree.com mebooksfree.com mebooksfree.com mebooksfree.com mebooksfree.com

mebooksfree.com mebooksfree.com mebooksfree.com mebooksfree.com mebooksfree.com mebooksfree.com mebooksfree.com mebooksfree.com mebooksfree.com mebooksfree.com mebooksfree.com mebooksfree.com

74 Review of Psychiatry

41. Opioids can cause which of the following? C. Prevent relapse A. Physical dependence (DNB NEET 2014-15) D. Has addiction potential; used for detoxification B. Psychological dependence of opioid C. Both A and B mebooksfree.com mebooksfree.comD. None of the above mebooksfree.comCannabismebooksfree.com mebooksfree.com mebooksfree.com 42. Usual sign of morphine withdrawal are all except: 49. After use of some drug, a person develops episodes (PGI May 2013, 1999, 1993) of rage in which he runs about and indiscrimi- A. Dryness of secretion nately injures a person who is encountered in way. B. Constipation He is probably addict of: (AIIMS 1997) C. Miosis A. Alcohol B. Cannabis D. Lacrimation, diarrhea, rhinorrhea C. Opium D. Cocaine E. Generally occur after 6-8 hours of last use 50. Which of the following substances is associated 43. Withdrawal of which of the following causes yawn- with flashback phenomenon? (KA 1999) ing and piloerection ? (DNB NEET 2014-15) A. Cannabis A. Morphine B. Cannabis B. LSD mebooksfree.com mebooksfree.comC. Smoking mebooksfree.comD. Alcohol mebooksfree.comC. Psilocybin mebooksfree.com mebooksfree.com D. All of the above 44. A boy is having diarrhea, rhinorrhea, sweating and lacrimation. What is the most probable diagnosis? 51. Amotivational syndrome is seen in: A. Cocaine withdrawal (AIIMS Nov 2010) (DNB NEET 2014-15, MH 2010, TN 1999) B. Heroin withdrawal A. Cannabis B. Cocaine C. Alcohol withdrawal C. Amphetamine D. Heroin D. LSD withdrawal 52. Which of the following substance intoxication 45. Treatment of opioid dependence includes: causes conjunctival congestion, increased appe- (PGI May 2011) tite, dry mouth, tachycardia and synesthesia? A. Naloxone B. Naltrexone (MH 2009) C. Acamprosate D. Buprenorphine A. Cannabis B. Caffeine mebooksfree.com mebooksfree.comE. Topiramate mebooksfree.com mebooksfree.comC. Cocaine D. Codeinemebooksfree.com mebooksfree.com 46. Which drug is most commonly used worldwide in 53. Bad trip is seen with which of the following drugs: maintenance treatment for opioid dependence? (DNB NEET 2014-15) (AI 2011) A. Cocaine B. Cannabis A. Naltrexone B. Methadone C. LSD D. Heroin C. Imipramine D. Disulfiram Others 47. Which of the following is an alternative to methadone for maintenance treatment of opiate 54. Correct statement about cocaine abuse: dependence? (AIIMS May 2005) (PGI May 2011) A. Diazepam A. Block uptake of dopamine in CNS B. Chlordiazepoxide B. Strong physical dependence mebooksfree.com mebooksfree.comC. Buprenorphine mebooksfree.com mebooksfree.comC. Increased BP mebooksfree.com mebooksfree.com D. Dextropropoxyphene D. Severe tolerance E. Cause impairment of nerve conduction 48. Naltrexone is used in opioid addiction because: (AIIMS May 2010, 2007, 2006, AI 2007) 55. Paranoid delusions are associated with use of: A. To treat withdrawal symptoms (AI 2012) B. To treat overdose of opioids and prevent respira- A. Cocaine B. Heroine tory depression C. Cannabis D. GHB

mebooksfree.com mebooksfree.com mebooksfree.com mebooksfree.com mebooksfree.com mebooksfree.com

mebooksfree.com mebooksfree.com mebooksfree.com mebooksfree.com mebooksfree.com mebooksfree.com mebooksfree.com mebooksfree.com mebooksfree.com mebooksfree.com mebooksfree.com mebooksfree.com

Substance Related and Addictive Disorders 75

56. Jet black pigmentation of tongue with tactile hal- ANSWERS lucination and visual hallucinations is a feature 1. D. The personality, family history and peer pressure of which substance use: (RJ 1998) all play a role in development of dependence. A. Cocaine B. Cannabis There is no correlation between intelligence and mebooksfree.com mebooksfree.comC. Heroin mebooksfree.comD. LSD mebooksfree.com mebooksfree.com mebooksfree.com substance use. 57. Paranoid psychosis observed with cocaine abuse 2. A, C. can be explained by: (AI 2011, 2012) The DSM-IV, diagnosis of substance abuse A. Tolerance B. Intoxication includes the following four criterion (1) recurrent C. Reverse tolerance D. Withdrawal use resulting in failure to fulfil major obligations 58. Formication and delusion of persecution, both are at work, school or home. (2) recurrent use in together seen in: (AIIMS May 2011, 2009) situations in which it is physically hazardous A. LSD psychosis (such as while driving) (3) substance use causing B. Amphetamine psychosis legal problems and (4) substance use causing C. Cocaine psychosis social or interpersonal problems (e.g. fights D. Cannabis psychosis with spouse). Withdrawal and tolerance are a criterion for “substance dependence” but not mebooksfree.com mebooksfree.com 59. A 16-year-old boy sufferingmebooksfree.com from drug abuse pre- mebooksfree.com“substance abuse”. Please remembermebooksfree.com in DSM-5, mebooksfree.com sents with crossover of sensory perceptions, such both these diagnosis of “substance dependence” that, sounds can be seen and colors can be heard. and “substance abuse” have been removed and Which of the following is the most likely agents replaced by “substance use disorders”. responsible for drug abuse? (AI 2012) 3. B. Neither presence of legal problems related to A. Cocaine B. LSD substance use nor use of illegal substances, is a C. Marijuana D. PCP (phencyclidine) criterion for substance dependence. 60. Psychosis resulting due to chronic amphetamine 4. A. Since cannabis causes very mild withdrawal intake most commonly resembles: (Orissa 1999) symptoms hence, no symptomatic treatment A. Delirium is required. LSD and other hallucinogens also B. Mania do not cause any withdrawal symptoms or tole­ C. Paranoid schizophrenia rance. mebooksfree.com mebooksfree.comD. Dissociative disordermebooksfree.com 5. mebooksfree.comD. mebooksfree.com mebooksfree.com 6. C. Dipsomania is compulsive drinking or an irresis­ 61. Used for averting tobacco dependence is: tible urge to drink alcohol. (DPG 2008) 7. A. In alcoholic balackouts, which is an anterograde A. Buspirone B. Methadone amnesia, the person later doesn’t remember, C. Bupropion D. Buprenorphine however at that time he appears to be totally 62. Most common substance of abuse in India is: in control and his behavior appears purpose- (DNB NEET 2014-15, AIIMS May 2010, May 2007, ful to others. He doesn’t look confused to the AI 2007) onlookers. A. Tobacco B. Cannabis 8. B. Since this patient, has been taking alcohol C. Alcohol D. Opium every day, at the time of presentation, disulfiram should be avoided as it may precipitate a severe 63. Which is not a feature of nicotine withdrawal? disulfiram like reaction. Disulfiram should not be mebooksfree.com mebooksfree.com mebooksfree.com(DNB December 2011) mebooksfree.com mebooksfree.com mebooksfree.com used until person has abstained from alcohol for A. Depression B. Headache atleast 12 hours. Also, please remember that phe- C. Tachycardia D. Anxiety nytoin doesn’t have any role in the management 64. Which is not a feature of caffeine withdrawal? of alcohol dependence. However, this question (DNB December 2011) is specifically asking for the drug that should be A. Headache B. Hallucination avoided and hence disulfiram is the best answer. C. Depression D. Weight gain 9. A. Alcohol does produce tolerance.

mebooksfree.com mebooksfree.com mebooksfree.com mebooksfree.com mebooksfree.com mebooksfree.com

mebooksfree.com mebooksfree.com mebooksfree.com mebooksfree.com mebooksfree.com mebooksfree.com mebooksfree.com mebooksfree.com mebooksfree.com mebooksfree.com mebooksfree.com mebooksfree.com

76 Review of Psychiatry

10. C. Tremors usually appear 6-8 hours after last The diagnosis in this patient is Wernicke’s alcohol intake. encephalopathy. The patients when treated 11. B. Tremor is the most common withdrawal symp- adequately have the following course (1) Oph- tom (excluding the hangover). thalmoplegia starts to resolve within hours, mebooksfree.com mebooksfree.com 12. A. Alcoholic hallucinosismebooksfree.com is a characteristic with- mebooksfree.comthough horizontal nystagmus oftenmebooksfree.com persists (2) mebooksfree.com drawal symptom of alcohol. Delusion of infidelity Ataxia begins to improve within first week how- (morbid jealousy) is also seen in chronic alcoho­ ever around 50% of patient will be left with some lism but it is not related to withdrawal state. residual abnormalities. (3) Global confusion 13. C. begins to recover within 2-3 weeks and would 14. C. Reverse tolerance refers to the phenomenon usually clear completely in 1-2 months. Despite where the intoxicating effects of alcohol are seen treatment, patient can progress to Korsakoff progressively with lower dosages. syndrome. 15. A, B, C, E. 30. A. The onset of symptoms is after 2 days of last See the list of alcohol induced disorders in the intake. There is history of chronic alcohol use. text. There is history of disorientation (misrecogni- 16. D. Oculomotor nerve plays causing ophthalmople- tion), visual hallucination (seeing snakes and mebooksfree.com mebooksfree.comgia is a feature of Wernicke’smebooksfree.com encephalopathy and mebooksfree.comreptiles), hyperactivity. All these putmebooksfree.com together is mebooksfree.com not delirium tremens. suggestive of delirium tremens. 17. A, B, D, E. 31. A. The diagnosis is delirium tremens and the drug Delirium tremens is usually not a chronic condi- of choice is benzodiazepines like diazepam. tion. 32. C. The diagnosis is alcohol withdrawal seizures 18. A, B. and the drug of choice is benzodiazepines like Kindly see text. diazepam. 19. B. 33. B. Benzodiazepines are the drug of choice in alcohol 20. B. withdrawal. If the question asks you to chose a 21. C. specific benzodiazepine, the best choice would 22. C. Kindly see text. be chlordiazepoxide. 23. A. Here, there is history of ataxia (incoordination) 34. D. The best answer here is both diazepam and mebooksfree.com mebooksfree.comand ophthalmoplegia.mebooksfree.com The inability to tell name mebooksfree.comchlordiazepoxide as the benzodiazepinesmebooksfree.com are mebooksfree.com might be because of confusional state. The likely the drugs of choice. However, please remember diagnosis is Wernicke’s encephalopathy. antipsychotics can also be used if patient is hav- 24. A, B. ing excessive hallucinations or is excessively 25. D. Korsakoff syndrome is due to thiamine defi- agitated and these symptoms are not responding ciency. Apart from alcoholism, malnutrition can to benzodiazepines alone. also cause it. Also it presents with amnesia and 35. A. Flumazenil has no role. It is used in benzodiaz- confabulations. epine overdose. 26. D. In Korsakoff psychosis, there is prominent antero- 36. A, B, C, E. grade amnesia. Whenever there is anterograde Naltrexone is used in alcohol as well as opioid amnesia (i.e. new memories cannot be made), dependence. Naloxone is used in opioid over- learning would be severely affected. dose. Clonidine can be used in opioid withdrawal 27. A. and disulfiram in alcohol dependence. mebooksfree.com mebooksfree.com 28. A, B, D. mebooksfree.com 37. mebooksfree.comA. See text. mebooksfree.com mebooksfree.com There is some mistake in the language of question 38. B. As explained above, benzodiazepines and anti­ as only option C is correct and all other options psychotics can be used in delirium. are wrong statement. In Korsakoff, both remote 39. B. Epinephrine is not an opioid peptide. The endo­ memory and intellect remains preserved and the genous opioid peptides include b endorphins, patient doesn’t have insight into his symptoms. Met and Leu enkephalins and Dynorphins. 29. A, B, C. 40. C. Hypotension is a feature and not hypertension.

mebooksfree.com mebooksfree.com mebooksfree.com mebooksfree.com mebooksfree.com mebooksfree.com

mebooksfree.com mebooksfree.com mebooksfree.com mebooksfree.com mebooksfree.com mebooksfree.com mebooksfree.com mebooksfree.com mebooksfree.com mebooksfree.com mebooksfree.com mebooksfree.com

Substance Related and Addictive Disorders 77

41. C. and auditory hallucinations and it resembles 42. A, B, C. paranoid schizophrenia. 43. A. 61. C. 44. B. 62. C. This is a controversial question. Now, if the ques- mebooksfree.com mebooksfree.com 45. B, D. mebooksfree.com mebooksfree.comtion was simply, most commonly usedmebooksfree.com substance mebooksfree.com 46. B. Methadone is used as methadone maintenance in India, the ans­wer would have been tobacco treatment, in long-term treatment of opioid without any controversy. Since the question dependence. mentions most common “substance abuse”, the 47. C. Methadone, buprenorphine, levo alpha acetyl- controversy arises. According to DSM-IV, there methadol can be used for maintenance treatment can be two types of substance use disorders (1) of opiate depen­dence. substance dependence (2) substance abuse. 48. C. The only indication for naltrexone in opioid These two can be considered as two different dependence is relapse prevention in highly levels of addiction, substance abuse is a lower motivated patients. For opioid overdose nalox- level and substance dependence is higher level. one is used and not naltrexone. Now, tobacco can cause dependence but not 49. B. The description is suggestive of run amok which abuse i.e. DSM-IV provides criterion for tobacco mebooksfree.com mebooksfree.comis seen with cannabismebooksfree.com use. mebooksfree.comdependence but says that “substancemebooksfree.com abuse” is mebooksfree.com 50. D. Cannabis and hallucinogens can cause flash back not applicable for tobacco. Whereas for alco- phenomenon.­ hol which is the second most commonly used 51. A. substance in India, both “alcohol dependence” 52. A. and “alcohol abuse” has been described.If this 53. B and C. question is interpreted strictly in terms of DSM- 54. A, C and E. IV diagnoses, of substance abuse, the answer Cocaine causes strong psychological dependence becomes alcohol, as there is no diagnosis of however physiological dependence (tolerance “tobacco abuse”. However if the term abuse is and withdrawal symptoms) is mild in compari- used literally, the answer becomes tobacco. son. Cocaine blocks dopa­mine and norepineph- The book published by AIIMS, on substance rine uptake and hence causes hypertension. It use disorders, says that “alcohol is the most mebooksfree.com mebooksfree.comblocks nerve conductionmebooksfree.com and is also used as an mebooksfree.comfrequently used substance as seenmebooksfree.com in the NHS mebooksfree.com anesthetic agent. and DAMS’. Here “NHS” and “DAMS” refers to 55. A. Cocaine. name of surveys which were conducted by Indian 56. A. government. Further, a table titled “major drugs 57. B. The delusion of persecution and auditory hal- of abuse in India” again mentions alcohol on the lucinations can be seen in cocaine intoxication. top and doesn’t mention anything about tobacco. 58. C. Since, this question has been asked by AIIMS, its 59. B. The sign here is synesthesia (sounds can be seen likely that they will follow their own book. So, my and colors can be heard) which is in with LSD advise is mark alcohol as the answer. and cannabis intoxication. 63. C. Bradycardia is a symptom of nicotine withdrawal 60. C. The symptoms of amphetamine induced psy- and not tachycardia. chotic disorder include delusion of persecution 64. B. See text. mebooksfree.com mebooksfree.com mebooksfree.com mebooksfree.com mebooksfree.com mebooksfree.com

mebooksfree.com mebooksfree.com mebooksfree.com mebooksfree.com mebooksfree.com mebooksfree.com

mebooksfree.com mebooksfree.com mebooksfree.com mebooksfree.com mebooksfree.com mebooksfree.com mebooksfree.com mebooksfree.com mebooksfree.com mebooksfree.com mebooksfree.com mebooksfree.com

mebooksfree.com mebooksfree.com mebooksfree.com mebooksfree.com mebooksfree.com mebooksfree.com

Chapter 6 Organic Mental Disorders

Organic mental disorders are caused by either a demon- eyes closed. Various other terms such as “confusional strable cerebral disease, brain injury or other insults state”, “clouding of consciousness” and “altered sen- mebooksfree.com mebooksfree.comleading to cerebral dysfunctionmebooksfree.com. Following are the com­ sorium”mebooksfree.com are used to describe the disturbancesmebooksfree.com of con­ mebooksfree.com mon symptoms seen in organic mental disorders: sciousness in delirium. A. Cognitive impairment: The term “cognition” is used C. Hallucinations: These patients most commonly have to describe all the mental processes that are utilized visual hallucinationsQ although auditory, olfactory, to gain knowledge. These processes include memory, gustatory and tactile hallucinations can also be pre­ language, orientation, judgment, performing actions sent. (praxis) and problem solving. At times the term “cog­ D. Delusions: The delusions are usuallytransient Q. nition” is used to describe the thoughts. In organic Complex delusions are rareQ. mental disorders one or more of cognitive functions The organic mental disorders are classified in the fol­ are impaired. Frequently patient presents with diso- lowing groups: rientation (to time, place and person), impaired A. Delirium mebooksfree.com mebooksfree.comattention and concentration,mebooksfree.com disturbances in memory B. Dementiamebooksfree.com mebooksfree.com mebooksfree.com (especially recent memory resulting in anterograde C. Amnestic disorders amnesia), etc. As organic mental disorders commonly have disturbances of cognition, they are also known DELIRIUM as cognitive disorders. Q B. Disturbances of consciousness: The consciousness It is the most common organic mental disorder. It is characte­rized by an acute onsetQ of symptoms and a has different levels ranging from alertness to coma. fluctuating courseQ. It is most commonly seen in elderly Usually the term “alertness” is used when one is aware population. The patients who have been hospitalized of the internal and external stimuli and can respond for medical and surgical disorders frequently develop to them. The patients with organic mental disorders delirium. The patients withhip fracturesQ, open heart usually have disturbances of consciousness which surgeriesQ, severe burnsQ, pneumoniaQ, postoperative can be of varying severity. The term “somnolence or patientsQ and critically ill patients have high prevalence mebooksfree.com mebooksfree.comlethargy” is used when patientmebooksfree.com tends to drift off to of delirium.mebooksfree.com The history of a medical disordermebooksfree.com followed by mebooksfree.com sleep when not actively stimulated. The next level is sudden development of disturbances of consciousness, “obtundation” in which patient is difficult to arouse cognition and psychiatric symptoms such as hallucina­ and when aroused appears confused. The next level tions and delusions is strongly suggestive of delirium. The is “stupor or semicoma” in which patient is mute and other causes includes use of multiple medications (espe­ immobile. When stimulated persistently and vigor­ cially those with anticholinergic actions). Withdrawal of ously he may groan or mumble. Finally, in “coma” , psychoactive substances (such as alcohol and sedatives/ patient is totally unarousable and remain with their hypnotics) is another common cause. Delirium can

mebooksfree.com mebooksfree.com mebooksfree.com mebooksfree.com mebooksfree.com mebooksfree.com

mebooksfree.com mebooksfree.com mebooksfree.com mebooksfree.com mebooksfree.com mebooksfree.com mebooksfree.com mebooksfree.com mebooksfree.com mebooksfree.com mebooksfree.com mebooksfree.com

Organic Mental Disorders 79

develop in older patients wearing eye patches after cata­ Delirium versus dementia: The acute presentation and fluc­ ract surgery (due to sensory deprivation), also known as tuations of symptoms is suggestive of delirium. Dementia black-patch deliriumQ. develops slowly and usually the symptoms are stable over time. Further, a patient with delirium presents with distur­ mebooksfree.com mebooksfree.comSymptoms mebooksfree.combancesmebooksfree.com of consciousness whereas a patientmebooksfree.com with demen­ mebooksfree.com tia doesn’t have any consciousness disturbances. In some The clinical features of delirium are: cases, a patient of dementia may develop superimposed Q • Disturbances of consciousness (ranging from som­ delirium, a condition called as “beclouded dementia”. nolence to coma) • Impairment of attention Delirium versus schizophrenia: A patient of delirium • Disorientation to time, place and person may have pronounced hallucinations and delusion and • Memory disturbances (impairment of immediate may resemble schizophrenia. However, in delirium the hal­ and recent memory with relatively intact remote lucinations are not constant and delusions are transient memoryQ) and not systematized (not organized) whereas in schizo­ • Perceptual disturbances like illusions and hallucina­ phrenia the hallucination are more constant and delusions tions (most commonly visualQ) and transient delu­ are also better organized. Further, the patient of delirium sions has disturbances of attention and disturbed consciousness mebooksfree.com mebooksfree.com• Hyperactivity or hypoactivity,mebooksfree.com agitation which mebooksfree.comis not seen in patient with schizophrenia.mebooksfree.com mebooksfree.com • Autonomic disturbances • Disturbances of sleep wake cycle (insomnia or rever­ Treatment sal of sleep wake cycle) A. Treat the underlying cause. • Sundowning: It refers to diurnal variation of symp­ B. Antipsychotics can be used for management of delu­ toms with worsening of symptoms in the evening (i.e. sions, hallucinations and agitation seen in delirium. with downing of sun) C. Benzodiazepines are used for insomnia and are the • Floccillations (or carphologia): Aimless picking beha­ drugs of choice in alcohol withdrawal delirium (delir­ vior, where patient appears to be picking at his ium tremens). clothes/bed • Occupational delirium: Patient behaves as if he is still DEMENTIA on his job, despite being in hospital (e.g. a tailor may mebooksfree.com mebooksfree.com mebooksfree.comDementiamebooksfree.com is defined as a mebooksfree.com mebooksfree.com ask for clothes and scissors, while lying on the bed of H the hospital). progressive impairment DSM-5 Update: The DSM-4 diagno- sis of dementia and amnestic disor- The neurotransmitter involved in delirium is acetyl- of cognitive functions in der are sub-sumed under the newly choline and the neuroanatomical area involved is the the absence of any dis- named entity major neurocognitive disorders (NCD). reticular formation (kindly remember reticular ascend­ turbances of conscious- Q ing system is responsible for arousal in a person). ness . The prevalence of H dementia increases with DSM-5 Update: In DSM-5, a new age, with prevalence of diagnostic category of mild neuro- Diagnosis cognitive disorders (NCD) has been around 5% in the popu­ added, for the patients who present Q The diagnosis of delirium is made clinically , on the basis lation older than 65 with milder cognitive impairment of above mentioned symptoms. The sudden onset and (which is not sever enough of diag- years and prevalence of nosis of dementia or major neurocog- nitive disorder). fluctuations in symptoms are important pointers towards 20–40% in the popula­ the diagnosis. Bedside examinations such as mini mental mebooksfree.com mebooksfree.com mebooksfree.comtion oldermebooksfree.com than 85 years. The underlying causemebooksfree.com of dementia mebooksfree.com status examination (MMSE)Q and mental status exami­ can be permanent or reversible. nation (MSE) are used to provide a measure of cognitive impairment. Generalized slowingQ on EEG is a common finding Symptoms in patients with delirium, however delirium caused by The following are the symptoms of dementia: alcohol or sedative-hypnotic withdrawal has low voltage A. Cognitive impairment: The cognitive impairment is charac­ fast activity on EEG. terized by 4 A’s: amnesia, aphasia, apraxia and agnosia.

mebooksfree.com mebooksfree.com mebooksfree.com mebooksfree.com mebooksfree.com mebooksfree.com

mebooksfree.com mebooksfree.com mebooksfree.com mebooksfree.com mebooksfree.com mebooksfree.com mebooksfree.com mebooksfree.com mebooksfree.com mebooksfree.com mebooksfree.com mebooksfree.com

80 Review of Psychiatry

• Amnesia refers to the memory impairment. may result in an emotional outburst in a patient Initially the loss is of recent memory followed by of dementia. This is known as“catastrophic immediate memory and lastly the remote mem­ reaction”Q. ory. Another way of describing memory impair­ C. Focal neurological signs and symptoms: These are usu­ mebooksfree.com mebooksfree.comment is in terms of episodicmebooksfree.com (memory for events), allymebooksfree.com seen in vascular dementia (multi-infarctmebooksfree.com demen­ mebooksfree.com semantic memory (memory for facts such as rules, tia) and correspond to the site of vascular insults. words and language) and visuospatial deficits. In These include exaggerated tendon reflexes, extensor episodic memory, there is a gradient of loss with plantar response, gait abnormalities, etc. more recent events being lost before remote events. Semantic memory is preserved in the early course Types of disease and is gradually lost as the disease pro­ The dementia can be divided in to reversible and irrever­ gresses. Visuospatial skills deficits manifests with sible dementias. It is extremely important to do detailed symptoms of disorientation in strange environ­ work up of a patient of dementia as around 15% of cases ments and later, wandering and getting lost in even are reversible. The Q are: familiar environments. reversible causes of dementia • Aphasia refers to the disturbances of language A. Neurosurgical conditions (subdural hematoma, nor­ mebooksfree.com mebooksfree.comfunction. The initial disturbancemebooksfree.com is usually “word- malmebooksfree.com pressure hydrocephalus, intracranialmebooksfree.com tumors, mebooksfree.com finding difficulties” which gradually progresses to intracranial abscess). more severe abnormalities. B. Infectious causes (meningitis, encephalitis, neuro­ • Apraxia is inability to perform learned motor syphilis, lyme disease). functions. For example, patient may start having C. Metabolic causes (vitamin B12 or folate deficiency, difficulties in functions like buttoning the shirt or niacin deficiency, hypo and hyperthyroidism, hypo combing the hair. and hyperparathyroidism). • Agnosia is inability to interpret a sensory D. Others (drugs and toxins, alcohol abuse, autoimmune stimulus. One of the common disturbance is encephalitis). “prosopagnosia”Q which is inability to identify the Dementia can also be classified into cortical and sub­ face. At times patient may be unable to identify his cortical types depending on the area of brain which is own face , a condition known as “autoprosopag- affected first by the dementing process. mebooksfree.com mebooksfree.comnosia”. mebooksfree.com mebooksfree.com mebooksfree.com mebooksfree.com • Apart from the 4 A’s, disturbances in executive Cortical dementias: These disorders are characterized by functioning (i.e. planning, organizing, sequen­cing early involvement of cortical structures and hence early and abstracting) is another important cognitive appearance of cortical dysfunction. These disorders have impairment. early and severe presentation of the As: amnesia, apraxia, B . Behavioral and psychological symptoms: These may aphasia, agnosia and acalculia (impaired mathematical include: skills) indicating cortical involve­ment. Alzheimer’s • Personality changes: There might be a significant diseaseQ is the prototype of cortical dementia. Others change in the personality. Patient may become include Creutzfeldt-Jakob disease, Pick’s disease and introvert and seem to be unconcerned about other frontotemporal dementias. others or patients may become hostile. The per­ Subcortical dementia: These disorders are characterized­ sonality changes are mostly seen in patients with by early involvement of subcortical structures like basal mebooksfree.com mebooksfree.comfrontal and temporal lobemebooksfree.com involvement. mebooksfree.com mebooksfree.com mebooksfree.com • Hallucinations and delusions: Delusion mostly seen ganglia, brain stem nuclei and cerebellum. These dis­ is delusion of persecution and delusion of theft. orders are characterized by early presentation of motor • Depression, manic and anxiety symptoms. symptoms (abnormal movements like tics, chorea, dysar­ • Apathy, agitation, aggression, wandering and circa­ thria, etc), significant disturbances of executive functioning dian rhythm disturbances. and prominent behavioral and psychological symptoms • Catastrophic reaction: The subjective awareness of like apathy, depression, bradyphrenia (slowness of think­ intellectual deficits while in a stressful situation ing). The examples include Parkinson’s disease, Wilson’s

mebooksfree.com mebooksfree.com mebooksfree.com mebooksfree.com mebooksfree.com mebooksfree.com

mebooksfree.com mebooksfree.com mebooksfree.com mebooksfree.com mebooksfree.com mebooksfree.com mebooksfree.com mebooksfree.com mebooksfree.com mebooksfree.com mebooksfree.com mebooksfree.com

Organic Mental Disorders 81

disease, Huntington’s disease, multiple sclerosis, progres­ in tangles is in a highly phosphorylated form and has sive supra nuclear palsy, normal pressure hydrocephalus. abnormal functioning. Normally, tau protein binds Some dementias such as vascular dementia, dementia and stabilizes microtubules, which are essential for with lewy body have mixed presentation. axonal transport, however in Alzheimer’s this func­ mebooksfree.com mebooksfree.com mebooksfree.comtion ismebooksfree.com deranged. The neurofibrillary­ tanglesmebooksfree.com are widely mebooksfree.com Alzheimer’s Disease (Dementia of distributed in cortical structures and hippocampus, Alzheimer’s Type) but always spare cerebellumQ. Multiple studies have established that amount and distribution of NFTs It is the most commonQ cause of dementia. The preva­ correlates with the duration and severity of dementiaQ. lence of Alzheimers disease increases with age, the rates Both senile plaques and neurofibrillary tangles can are around 5% for all those aged 65 years and older, be present in elderlies without any dementia. However increasing to around 20-30% for all those aged above 85 in patients with dementia, these findings are extensive years. The Alzheimers disease can be divided into early and wide spread. The neuropathological diagnosis of onset (presenile), if the age of onset is 65 years or ear­ Alzheimer disease requires extensive presence of both lier; or late onset (senile), if the age of onset is after 65 senile plaques (extracellular deposits) and neurofibrillary years. At all ages, females outnumber males by a ratio of tangles (intracellular inclusions). mebooksfree.com mebooksfree.com2 or 3:1 except in early onset mebooksfree.comfamilial forms (inherited as Granulovacuolarmebooksfree.com degeneration (GVD)mebooksfree.comQ and Hirano mebooksfree.com Q autosomal dominant disorder) in which sex ratio is 1. The bodies (eosinophilic inclusions) are abnormalities seen onset is usually insidious and progression is gradual. The in the cytoplasm of hippocampal neurons in patients with insightQ (awareness of illness) is lost relatively early in the Alzheimer disease. Both of them are present in elderlies course of illness. In the initial phase symptoms include without dementia, however they are much more severe and widespread in Alzheimers disease. memory disturbances, gradually apraxia, agnosia, apha­ sia and acalculia develop and executive functions are lost. Amyloid cascade hypothesis: According to this hypo­ In the later stages neurological disabilities like tremors, thesis, mutation in APP gene near cleavage site favor the rigidity and spasticity may develop. cleavage by b and g secretase, resulting in the produc­ tion of Ab. The bA peptides form Ab oligomers which in Pathophysiology: The classical gross neuroanatomical turn induce tau phosphorylation, producing neurofibril­ finding in Alzheimers disease isdiffuse atrophy with flat- lary tangles. The tau protein in this highly phosphory­lated mebooksfree.com mebooksfree.comtened cortical sulci and enlargedmebooksfree.com cerebral ventricles. form ismebooksfree.com not able to stabilize microtubules,mebooksfree.com resulting in mebooksfree.com The classical microscopic findings areneuritic granulovascular degeneration of neurons, neuronal loss (senile) plaquesQ and neurofibrillary tanglesQ. Senile and synaptic loss. plaques, also referred to as amyloid plaques are com­ Neurochemistry: Alzheimer’s disease is predominantly a b posed of a particular protein A . This protein is derived disorder of cholinergic neuronsQ and loss of cholinergic from amyloid precursor protein (APP) by the action of neurons in nucleus basalis of meynert is a consistent find­ b- and g-secretase enzymes. The bA protein combines to ing. Apart from acetylcholine, norepinephrine and sero­ form fibrils. The senile plaques areextracellular deposits tonin have also been implicated in some cases. of Ab and are found in all cortical areas and also in striatum Genetics: Alzheimer’s disease has shown linkage to and cerebellum. The amyloid-b peptide not only deposits chromosome 1,14 and 21. A small number of cases of in the brain parenchyma in the form of amyloid plaques Alzheimer disease are early onset and familial and are but also in the vessel walls in the form of cerebral amy- inherited in autosomal dominant fashion. Mutations Q mebooksfree.com mebooksfree.comloid angiopathy (CAA) . mebooksfree.comin threemebooksfree.com genes, amyloid precursor proteinmebooksfree.comQ (chromo- mebooksfree.com The senile plaques can also be seen in elderlies who some 21), presenilin-1Q (chromosome 14) and presenilin-2Q do not have Alzheimer’s and their number increases with (chromo­some 1) have been found in most cases with age. Hence senile plaques are not specific for Alzheimer familial Alzheimer’s disease. The majority of cases disease. The amyloid plaques are not correlated with the are however sporadic and late onset. Apo E4 geneQ is severity of dementia. associated with the risk of development of Alzheimers The neurofibrillary tangles (NFTs) areintraneu - disease, however its testing is not recommended as it is ronal aggregates of tau protein. The tau protein present neither sensitive nor specific for Alzheimer’s disease.

mebooksfree.com mebooksfree.com mebooksfree.com mebooksfree.com mebooksfree.com mebooksfree.com

mebooksfree.com mebooksfree.com mebooksfree.com mebooksfree.com mebooksfree.com mebooksfree.com mebooksfree.com mebooksfree.com mebooksfree.com mebooksfree.com mebooksfree.com mebooksfree.com

82 Review of Psychiatry

The patients withDown’s syndromeQ have signifi­ subcortical type with more motor abnormalities and less cantly higher risk for development of Alzheimer’s disease. of amnesia, apraxia, aphasia and agnosia. The gene for APP (amyloid precursor protein) is located on chromosome 21. HIV Related Dementia mebooksfree.com mebooksfree.comRisk factors: Age is the mostmebooksfree.com important risk factors. The diagnosismebooksfree.com of HIV dementia (AIDS dementiamebooksfree.com complex) mebooksfree.com Other risk factors include head injury, hypertension, is made by lab evidence of systemic HIV infection, cogni­ insulin resistance, depression. Few studies have claimed tive deficits, presence of motor abnormalities or persona­ that smokingQ is a protective factor against Alzheimer’s lity changes. Personality changes are characterized by but this finding has been contradicted by other studies. apathy, emotional lability or disinhibition. High education levels and remaining physically and men­ tally active till late in life are protective factors against Head Trauma Related Dementia Alzheimer’s disease. Dementia can develop as a sequelae of head trauma. Dementia pugilistica (punch drunk syndrome) can Vascular Dementia or Multi-infarct Dementia develop in boxers after repeated head trauma. This is the second most common type of dementia. mebooksfree.com mebooksfree.comOccurrence of multiple cerebralmebooksfree.com infarctions (caused by Frontotemporalmebooksfree.com Dementia (FTD) mebooksfree.com mebooksfree.com occlusion of cerebral vessels by arteriosclerotic plaques Frontotemporal dementias are a group which have simi­ or thromboemboli) results in progressive deterioration of lar presentation but may be caused by a variety of neuro­ brain functions, finally resulting in dementia. There are pathological substrates. Pick’s diseaseQ is one pathological acute exacerbations which correspond to the new infarcts, variant of FTD, and is characterized by presence of pick’s and result is stepwise deterioration of symptoms (step- bodies. The frontotemporal dementia’s have anearlier ladder pattern). The general symptoms of dementia onsetQ, around 45-65 years and mainly present with beha­ are present. In addition patient has focal neurological vioral symptoms and change in personality with relative deficits which correspond to site of infarction. There is preservation of memory. Three distinctive forms of FTD usually history of previous stroke or transient ischemic have been described on the basis of clinical presentation. attacks. The patients usually have hypertension and A. Frontal variant FTD: The symptoms are primarily of other cardiovascular risk factors. The treatment involves loss of frontal lobe function. The classical feature is mebooksfree.com mebooksfree.commanagement of risk factors andmebooksfree.com cholinesterase inhibitors. stereotypedmebooksfree.com behavior, disinhibition andmebooksfree.com apathy. mebooksfree.com Binswanger’s diseaseQ: It is also known as subcortical B. Semantic dementia: The symptoms are primarily of arteriosclerotic encephalopathy, and is characterized by loss of temporal lobe functions and is characterized multiple small white matter infarctions and can produce by complaints of loss of memory for words. C. Progressive nonfluent aphasia: It presents with symptoms of subcortical dementia. speech dysfluency and word finding difficulties.

Lewy Body Disease (Dementia with Lewy Body) Pseudodementia The clinical signs and symptoms are similar to Alzheimer The depression in elderly patients may mimic symptoms disease. Apart these patients also have fluctuating levels of dementia and hence is known as pseudodementiaQ. of attention and alertness, recurrent visual hallucinations A depressed patient may get a low score on MMSE, as and parkinsonian features (tremors, rigidity and bradyki­ depressed individual lacks motivation to solve the ques­ nesia). Antipsychotic medications should be avoided as tions. Hence low score on MMSE should be carefully mebooksfree.com mebooksfree.comthese patients are extremelymebooksfree.com sensitive to antipsychotics interpreted,mebooksfree.com if depression is suspected. mebooksfree.com mebooksfree.com and can develop drug induced parkinsonism. Management of Dementia Huntington’s Disease, Parkinson’s Disease, The evaluation of cognitive functions is usually done using Wilson’s Disease and Multiple Sclerosis the screening test of mini mental status examination These predominantly motor diseases are associated with (MMSE)Q. A score of less than 24 (out of a maximum 30) is the development­ of dementia. The dementia seen is of suggestive of dementia. In accordance with the cholinergic

mebooksfree.com mebooksfree.com mebooksfree.com mebooksfree.com mebooksfree.com mebooksfree.com

mebooksfree.com mebooksfree.com mebooksfree.com mebooksfree.com mebooksfree.com mebooksfree.com mebooksfree.com mebooksfree.com mebooksfree.com mebooksfree.com mebooksfree.com mebooksfree.com

Organic Mental Disorders 83

hypothesis, cholinesterase inhibitors are widely used Amnestic syndrome is characterized by inability to form for treatment of cognitive deficits in Alzheimer’s disease. new memories (anterograde amnesia) and the inability Donepezil, rivastigmine, galantamine and tacrine are few to recall previously remembered knowledge (retrograde of the drugs belonging to this category. amnesia). Short-term and recent memory are usually mebooksfree.com mebooksfree.com Memantine, a NMDA receptormebooksfree.com antagonist has also impairedmebooksfree.com with preservation of remote andmebooksfree.com immediate mebooksfree.com been approved for the treatment. For behavioral and memory. Themajor causesQ of amnestic disorders are: psychological symptoms of dementia, symptomatic treat­ A. Thiamine deficiency (Korsakoff syndrome) ment is used and may include antidepressants, antipsy­ B. Hypoglycemia chotics and benzodiazepines. C. Primary brain conditions (head trauma, seizures, cere­ bral tumors, cerebrovascular disease, hypoxia, elec­ AMNESTIC DISORDERS troconvulsive therapy, multiple sclerosis) Amnestic disorder is a broad category that includes a vari­ D. Substance related disorders (alcohol, benzodiaz­ ety of conditions which present with amnestic syndrome. epines).

mebooksfree.com mebooksfree.com mebooksfree.comQUESTIONS AND ANSWERSmebooksfree.com mebooksfree.com mebooksfree.com

QUESTIONS 5. Disorientation occurs in: (AI 1993) A. Schizophrenia Organic Mental Disorders B. Organic brain syndrome 1. Which of the following behavioral problems C. Depression would suggest an organic brain lesion? D. Mania (SGPGI 2005, DNB 2006) 6. Which of the following suggest a psychotic rather A. Formal thought disorder than an organic disorder? (DNB June 2009) B. Auditory hallucinations A. Confusion C. Visual hallucinations B. Complex delusions mebooksfree.com mebooksfree.comD. Depression mebooksfree.com mebooksfree.comC. Impairment of consciousness mebooksfree.com mebooksfree.com D. Lack of insight 2. Organic mental disease is indicated by: (AIIMS 1991, DNB 1993) 7. Feature(s) suggestive of schizophrenia rather than A. Incoherence organic psychosis is/are: (PGI June 2009) B. Delusion A. Third person hallucination C. Flight of idea B. Split personality D. Perseveration of speech C. Visual hallucination D. Altered sensorium 3. Mini mental status examination is: E. Systematized delusion (DNB 2004, JIPMER 2002) A. Method to investigate common psychiatric 8. In India psychiatric disorder in people above 60- problem year of age is mostly due to: mebooksfree.com mebooksfree.comB. 30 point program tomebooksfree.com evaluate cognitive functions mebooksfree.com (DNB 2003,mebooksfree.com Calcutta 2K) mebooksfree.com C. To evaluate schizophrenia A. Depression B. Dementia D. Instrument to measure delirium C. Hysteria D. Schizophrenia (PGI June 2006, 2007) 4. Cognitive disorders are: Delirium A. Intellectualization B. Depersonalization C. Dementia D. Delirium 9. Most important feature of delirium is: E. Hallucination F. Secondary gain (DNB NEET 2014-15)

mebooksfree.com mebooksfree.com mebooksfree.com mebooksfree.com mebooksfree.com mebooksfree.com

mebooksfree.com mebooksfree.com mebooksfree.com mebooksfree.com mebooksfree.com mebooksfree.com mebooksfree.com mebooksfree.com mebooksfree.com mebooksfree.com mebooksfree.com mebooksfree.com

84 Review of Psychiatry

A. Impaired attention Amnestic Syndrome B. Anxiety 16. Anterograde amnesia is seen in:(AIIMS Nov 2010) C. Hyperactivity A. Head injury D. Clouding of consciousness mebooksfree.com mebooksfree.com mebooksfree.com mebooksfree.comB. Stroke mebooksfree.com mebooksfree.com 10. Delirium is defined as: (DNB NEET 2014-15) C. Spinal cord injury (traumatic paraplegia) A. Acute onset of disturbed consciousness D. Alzheimer’s disease B. Chronic onset of disturbed consciousness 17. Cause of organic amnestic syndrome include(s): C. Progressive generalized impairment of intellec­ (PGI May 2013) tual functions and memory without impairment A. Multiple sclerosis B. Hypoglycemia of consciousness. C. Hyperglycemia D. Hypoxia D. Disorientation without clouding of conscious­ E. Hypercapnia ness 18. Not diagnostic/defining criteria for amnestic dis- 11. Features of delirium: (PGI Nov 2010, June 2008) order: (PGI Nov 2009) A. Deficit of attention (attention deficit) A. Visual hallucination B. Autonomic instability (dysfunction) mebooksfree.com mebooksfree.comC. Altered sleep wake mebooksfree.compattern mebooksfree.comB. Transient delusion mebooksfree.com mebooksfree.com D. Visual hallucination and clouding of conscious­ C. Impaired concentration/attention ness D. Good recall of recent events E. Delirium cannot be diagnosed clinically E. Ability to form new memories 12. Delirium and schizophrenia differ from each other 19. All are true except: (PGI Feb 2008) by: (DNB 2003, WB 2001, KA 2004) A. Procedural learning is from past experiences A. Change in mood B. Implicit learning is procedural skill acquirement B. Clouding of consciousness C. Amnestic syndromes lose semantic memory C. Tangential thinking D. Implicit memory is declarative D. All of the above E. Anterograde amnesia affects long-term memory more in amnestic syndrome 13. Slow waves in EEG activity are seen in:(PGI 1998) mebooksfree.com mebooksfree.comA. Depression mebooksfree.comB. Delirium Dementiamebooksfree.com mebooksfree.com mebooksfree.com C. Schizophrenia D. Mania 20. Delirium and dementia can be differentiated by? 14. A patient with pneumonia for 5 days is admitted (DNB June 2010) to the hospital in altered sensorium. He suddenly A. Loss of memory B. Apraxia ceases to recognize the doctor and staff. He thinks C. Delusion D. Altered sensorium that he is in jail and complains of scorpion attack- 21. Most common cause of dementia is: ing him. His probable diagnosis is: (AI 2001) (DNB NEET 2014-15) A. Acute dementia B. Acute delirium A. Alzheimer’s disease B. Vascular dementia C. Acute schizophrenia D. Acute paranoia C. Wilson’s disease D. Pick’s disease 15. A 60-year man had undergone cardiac bypass 22. True about dementia is all except: (AI 1994) surgery 2 days back. Now he started forgetting A. Often irreversible mebooksfree.com mebooksfree.comthings and was not ablemebooksfree.com to recall names and phone mebooksfree.com mebooksfree.com mebooksfree.com B. Hallucinations are not common numbers of his relatives. What is the probable C. Clouding of consciousness is common diagnosis? (AI 2010) D. Nootropics have limited role A. Depression B. Post-traumatic psychosis 23. Catastrophic reaction is a feature of: (MH 2011) C. Cognitive dysfunction A. Dementia B. Delirium D. Alzheimer’s disease C. Schizophrenia D. Anxiety

mebooksfree.com mebooksfree.com mebooksfree.com mebooksfree.com mebooksfree.com mebooksfree.com

mebooksfree.com mebooksfree.com mebooksfree.com mebooksfree.com mebooksfree.com mebooksfree.com mebooksfree.com mebooksfree.com mebooksfree.com mebooksfree.com mebooksfree.com mebooksfree.com

Organic Mental Disorders 85

24. All are causes of subcortical dementia except: A. Down’s syndrome (AIIMS May 2009) B. Head trauma A. Alzheimer’s disease C. Smoking B. Parkinson’s disease D. Low education group mebooksfree.com mebooksfree.comC. Supranuclear palsymebooksfree.com mebooksfree.com mebooksfree.com mebooksfree.com 33. Dementia of Alzheimer’s type is not associated D. HIV associated dementia with one of the following: (AIIMS Nov 2005) 25. Dementia is/are present in all except: A. Depressive symptoms A. Alzheimer’s disease B. Pick’s disease B. Delusions C. Lewy body D. Binswanger’s disease C. Apraxia and aphasia E. Gansers syndrome D. Cerebral infarcts

26. Reversible causes of dementia: (PGI June 2004) 34. All the following are features of Alzheimer’s dis- A. Hypothyroidism ease except: (DNB 1994, WB 2002) B. Alzheimer’s disease A. Cerebellar atrophy C. Vitamin B12 deficiency B. Common in 5th and 6th decade D. Vitamin A deficiency C. Atrophied gyri widened sulci mebooksfree.com mebooksfree.com 27. Treatable causes of dementiamebooksfree.com are: (PGI 2001) mebooksfree.comD. Progressive dementia mebooksfree.com mebooksfree.com A. Alzheimer’s disease 35. In Alzheimer’s disease (AD) which of the following B. Hypothyroidism is not seen: (AIIMS Nov 2011) C. Multi-infarct dementia A. Aphasia B. Acalculia D. subdural hematoma (SDH) C. Agnosia D. Apraxia E. Hydrocephalus 36. False regarding Alzheimer’s disease (AD) is: 28. Vascular dementia is characterized by: (PGI 2003) A. Number of senile neural plaques correlates A. Disorientation B. Memory deficit (increases) with age C. Emotional lability D. Visual hallucination B. Presence of tau protein suggest neurodegenera­ E. Personality deterioration tion 29. A 65-year-old male is brought to the outpatient C. Number of neurofibrillary tangles is associated mebooksfree.com mebooksfree.comclinic with one yearmebooksfree.com illness characterized by mebooksfree.comwith the severity of dementia mebooksfree.com mebooksfree.com marked forgetfulness, visual hallucinations, sus- D. Extracellular inclusions (lesions) can occur in piciousness, personality decline, poor self care the absence of intracellular inclusions to make and progressive deterioration in his condition. His pathological diagnosis of AD Mini Mental Status Examination (MMSE) score is 37. Area of brain resistant to neurofibrillary tangles 21. His most likely diagnosis is: (AIIMS Nov 2002) in Alzheimer’s disease: (AI 2012) A. Dementia B. Schizophrenia A. Visual association area C. Mania D. Depression B. Entorhinal cortex 30. Which of the following neurotransmitters are C. Lateral geniculate body decreased in Alzheimer’s disease? D. Cuneal gyrus area VI/temporal lobe (DNB NEET 2014-15) 38. Regarding Alzheimer’s disease which is/are not A. Acetylcholine B. Norepinephrine true: (PGI Dec 2008, June 2009) (AIIMS Nov 2011) C. Corticotropin D. All of the above mebooksfree.com mebooksfree.com mebooksfree.com mebooksfree.comA. Initial loss of long-term memory mebooksfree.com mebooksfree.com 31. Protein involved in Alzheimer’s disease: B. Delayed loss of short-term memory (NIMHANS 2001, DNB 2002) C. Step ladder pattern A. APOE4 gene B. Presenilin-1 D. Cognitive impairment C. Amyloid protein D. All of the above E. Judgment impaired 32. Following are predispositions to Alzheimer’s dis- 39. All are true regarding Alzheimer’s disease except: ease except: (DNB 1996, AI 1999) A. Gradually progressive (PGI Feb 2008)

mebooksfree.com mebooksfree.com mebooksfree.com mebooksfree.com mebooksfree.com mebooksfree.com

mebooksfree.com mebooksfree.com mebooksfree.com mebooksfree.com mebooksfree.com mebooksfree.com mebooksfree.com mebooksfree.com mebooksfree.com mebooksfree.com mebooksfree.com mebooksfree.com

86 Review of Psychiatry

B. Abrupt onset and acute exacerbations C. Dementia C. Episodic memory can be affected D. Anxiety D. Frontotemporal disorder 47. The psychiatric disorder most commonly associ- E. Ubiquitin Lewy bodies mebooksfree.com mebooksfree.com mebooksfree.com mebooksfree.comated with myxedema: mebooksfree.com mebooksfree.com 40. Frontotemporal dementias include all except: A. Depression B. Mania (DNB 2003, UP 2007) C. Phobia D. Psychosis A. Pick’s disease 48. Myxedema madness includes:(DNB NEET 2014-15) B. Nonfluent aphasia A. Auditory hallucinations and paranoia C. Semantic dementia B. Visual hallucinations and depression D. Alzheimer’s disease C. Auditory hallucinations and depression 41. A 70-year-old man presents with h/o prosopag- D. Paranoia and depression nosia, loss of memory, 3rd person hallucinations since 1 month. On examination deep tendon ANSWERS reflexes are increased, mini-mental examination score is 20/30. What is most likely diagnosis? 1. C. If a patient presents with prominent visual hal­ mebooksfree.com mebooksfree.com mebooksfree.com(AIIMS 2001) mebooksfree.comlucinations, organic mental disordersmebooksfree.com (organic mebooksfree.com A. Dissociated dementia brain lesions) should always be looked for. B. Schizophrenia 2. D. Perseveration of speech is suggestive of organic C. Alzheimer’s disease mental disorders. Few books are giving the D. Psychotic disorder answer as delusion which is completely wrong. 3. B. Mini mental status examination is used to evalu­ 42. Not a feature of Alzheimer’s disease: ate cognitive functions in illnesses like dementia (PGI May 2013) and delirium. A. Hirano bodies 4. C, D. B. Amyloid angiopathy As organic mental disorders commonly have C. Granulovacuolar degeneration of neurons disturbances of cognition, they are also known D. Senile plaque E. Cerebellar atrophy as cognitive disorders. mebooksfree.com mebooksfree.com mebooksfree.com 5. mebooksfree.comB. Presence of disturbances of consciousnessmebooksfree.com and mebooksfree.com 43. Rivastigmine and Donepezil are drugs used pre- disorientation is suggestive of organic mental dominantly in the management of: (AI 2006) disorders. A. Depression B. Dissociation 6. B. The complex delusions are frequently seen in C. Delusion D. Dementia psychotic disorder. In organic mental disor­ 44. True regarding FTD are all except: ders, the delusions are usually transient and (AIIMS 2011, NEET 2013) fragmented. Presence of complex delusions in A. Semantic dementia organic mental disorder is very rare. The lack of B. Nonfluent aphasia insight is a feature of both whereas confusion and C. Apathetic, disinhibited personality impairment of consciousness is seen in organic D. Rapid onset static course mental disorders. 7. A, E. 45. All are true regarding frontotemporal dementia: Third person hallucinations are quite suggestive mebooksfree.com mebooksfree.com mebooksfree.com(AlIMS Nov 2012) mebooksfree.com mebooksfree.com mebooksfree.com of schizophrenia. Also systematized delusions A. Stereotypic behavior B. Insight present (elaborate delusions) are much more likely in C. Age less than 65 years D. Affective symptoms schizophrenia. Please remember that schizo­ 46. The following are the psychiatric sequelae after phrenia is not a disorder of personalty and hence stroke inelderly: (PGI 2003) there is no “split personality” in schizophrenia. A. Depression Visual hallucinations and altered sensorium B. Post-traumatic stress disorder are more suggestive of organic mental disorders

mebooksfree.com mebooksfree.com mebooksfree.com mebooksfree.com mebooksfree.com mebooksfree.com

mebooksfree.com mebooksfree.com mebooksfree.com mebooksfree.com mebooksfree.com mebooksfree.com mebooksfree.com mebooksfree.com mebooksfree.com mebooksfree.com mebooksfree.com mebooksfree.com

Organic Mental Disorders 87

although visual hallucinations can also be seen whereas implicit memory (nondeclarative in schizophrenia. memory) doesn’t involve awareness.­ For exam­ 8. B. In older age (>60 years) dementia is the most ple, if you have to chose the correct option for a common psychiatric disorder followed by particular MCQ, you first try to remember the mebooksfree.com mebooksfree.comdepression. mebooksfree.com mebooksfree.comcorrect answer, i.e you try to bringmebooksfree.com the memory mebooksfree.com 9. D. Please remember that the hallmark symptom of associated with MCQ into awareness , hence its delirium is clouding of consciousness, which is an example of explicit memory. However, when associated with impairment­ of global cognitive you drive a car, you don’t really try to remember functions, most importantly attention. everything every time. Changing clutches, press­ 10. A. ing breaks and accelerator happens automati­ 11. A, B, C, D. cally and you don’t have to remember anything, 12. B. Delirium presents with clouding of conscious­ its an example of implicit memory. ness whereas in schizophrenia consciousness is Explicit memory is further divided into epi- intact. The mood changes and tangential think­ sodic memory for events (e.g. the memory of ing cannot be used for differentiation. your first day in medical college) and semantic 13. B. for facts (e.g. memory for the most mebooksfree.com mebooksfree.com 14. B. History of a medicalmebooksfree.com disorder (pneumonia ) fol­ mebooksfree.commemory mebooksfree.com mebooksfree.com lowed by disturbances in consciousness (altered common , least common type of questions). sensorium), disorientation (failure to recognize Procedural memory (for procedures like driv­ doctor and staff and thinking that he is in jail) and ing) is a type of implicit memory. Now, looking hallucinations (scorpions attacking) is suggestive at options. Option A is true, procedural learning of delirium. depends on past experience. Initially we have to 15. C. The history of cardiac surgery 2 days prior fol­ remember every detail about how to use clutch, lowed by ­behavioral changes is suggestive of break and accelerator however with repeated delirium. The question here is stressing on experience it becomes implicit. Option B is also “disturbances of memory” which can be seen correct as procedure learning is a type of impli­ in delirium, however are usually restricted to cit memory. Option C is wrong, in amnestic short- term memory loss. The other important syndrome, episodic memory is lost more and features such as clouding of consciousness and not the semantic memory. Option D is wrong as mebooksfree.com mebooksfree.comattention impairmentmebooksfree.com has not been provided. mebooksfree.comimplicit memory is nondeclarative.mebooksfree.com Option E is mebooksfree.com Nonetheless, the most likely diagnosis appears to also wrong, in amnestic syndrome short-term be delirium. As delirium has prominent cognitive and recent memory are more affected and not dysfunction, that is the correct answer. Alzheimer the long-term memory. disease does not have such sudden onset. 20. D. Please remember that the hallmark of delirium 16. B. Anterograde amnesia is seen in stroke. is disturbance of consciousness (altered senso­ 17. A, B,D. rium) whereas in dementia, there is no distur­ See text. bance of consciousness. 18. A, B, C, D, E. 21. A. None of the options are included in diagnostic criterion for amnestic disorder. Amnestic syn­ 22. C. There is no disturbance of consciousness in drome is characterized by inability to form new dementia. It is often irreversible. The halluci­ mebooksfree.com mebooksfree.commemories (anterogrademebooksfree.com amnesia) and the inabi­ mebooksfree.comnations can be present but are mebooksfree.comnot common. mebooksfree.com lity to recall previously remembered knowledge Nootropics (or cognitive enhancers) have very (retrograde amnesia). Short-term and recent limited role in the management of dementia. memory are usually impaired with preservation 23. A. See text. of remote and immediate memory. 24. A. Alzheimer’s disease is a cortical dementia. 19. C, D, E. 25. E. Ganser’s syndrome is a type of dissociative disor­ Explicit memory (declarative memory) is the der. The other options are examples of dementia. memory which is associated with awareness, 26. A, C.

mebooksfree.com mebooksfree.com mebooksfree.com mebooksfree.com mebooksfree.com mebooksfree.com

mebooksfree.com mebooksfree.com mebooksfree.com mebooksfree.com mebooksfree.com mebooksfree.com mebooksfree.com mebooksfree.com mebooksfree.com mebooksfree.com mebooksfree.com mebooksfree.com

88 Review of Psychiatry

27. B, D, E. 38. A, B, C. Perhaps the use of word “treatable” is inappro­ Short-term memory is lost first, long-term mem­ priate here since all the types of dementia can be ory gets lost only in the later stages of illness. Step “treated”. Theexaminer ­ most likely wants to ask ladder pattern is typical of vascular dementia. mebooksfree.com mebooksfree.comthe types which canmebooksfree.com be “reversed” or “cured”. 39. mebooksfree.comB, D, E. mebooksfree.com mebooksfree.com 28. A, B, C, D. Alzheimer’s has an insidious onset and gradual Vascular dementia presents with memory loss, progression. Later in the course of disease epi­ mood changes (depression, irritability, emo­ sodic memory does get disturbed. Alzheimer’s tional lability), delusions and hallucinations, disease primarily involves parietal and temporal confusion and disorientation. lobe. 29. A. Old age with history suggestive of a progressive 40. D. impairment in memory, presence of behavioral 41. C. The presence of loss of memory, prosopagnosia and psychological symptoms (hallucinations, suspiciousness), poor self care and personality (difficulty in identifying face) in a 70-year-old decline and a MMSE score <24, are all ­suggestive man is quite suggestive of Alzheimer’s disease. of dementia. Third person auditory hallucinations are usu­ mebooksfree.com mebooksfree.com 30. A. mebooksfree.com mebooksfree.comally seen in schizophrenia, howevermebooksfree.com they can be mebooksfree.com 31. D. present in Alzheimer’s disease too. Further on 32. C. Smoking is considered to be one of the protec­ examination, deep tendon reflexes are increased, tive factors in Alzheimer’s disease however this which again can be seen in late stages of Alzhei­ finding has beeninconsistent ­ across the studies. mer’s disease. Finally MMSE score below 24 seals 33. D. Cerebral infarcts are a feature of vascular the diagnosis. dementia and not dementia of Alzheimer’s type 42. E. See text. (Alzheimer’s disease). 43. D. 34. A. In Alzheimer’s , the disease process usually spares 44. D. The frontotemporal dementias have a progressive cerebellum. Especially neurofibrillary tangles are course and not static course. never seen in cerebellum. 45. B. Insight is usually lost. 35. B. The best answer here is B. In reality, all four 46. A, C, D. options given here are seen in Alzheimer’s how­ mebooksfree.com mebooksfree.com mebooksfree.com mebooksfree.com The psychiatric sequelae of strokemebooksfree.com includes mebooksfree.com ever, the DSM criterion for Alzheimer’s disease dementia, ­depression, mania, apathy, psychosis, does not include acalculia as a symptom, while emotional instability. other three, aphasia, apraxia and agnosia have been included. 47. A. The most common psychiatric disorder associ­ 36. D. Please remember that the neuropathological ated with hypothyroidism is cognitive slowing diagnosis of Alzheimer’s disease requires followed by depression. extensive presence of both senile plaques (extra­ 48. A. Myxedematous madness has been described in cellular deposits) and neurofibrillary tangles a small number of patients with hypothyroidism. (intracellular inclusions). The characteristic symptoms include auditory 37. C. hallucinations and paranoia (persecutory ideas). mebooksfree.com mebooksfree.com mebooksfree.com mebooksfree.com mebooksfree.com mebooksfree.com

mebooksfree.com mebooksfree.com mebooksfree.com mebooksfree.com mebooksfree.com mebooksfree.com

mebooksfree.com mebooksfree.com mebooksfree.com mebooksfree.com mebooksfree.com mebooksfree.com mebooksfree.com mebooksfree.com mebooksfree.com mebooksfree.com mebooksfree.com mebooksfree.com

mebooksfree.com mebooksfree.com mebooksfree.com mebooksfree.com mebooksfree.com mebooksfree.com

Chapter 7 Personality Disorders

Personality is defined as the dynamic organization within In other words, the individual with a personality disorder the individual that determines his/her unique adjustment doesn’t find anything wrong with himself and hence is mebooksfree.com mebooksfree.comto his/her environment. The mebooksfree.compersonality can be described often unwillingmebooksfree.com to take any treatment. DSM-5mebooksfree.com has classi- mebooksfree.com under five broad dimensions. These five dimensions, also fied the personality disorders into three clusters. called personality traitsQ can be remembered with the pneumonic, OCEAN. Cluster A Personality Disorders 1. Openness to experience: It reflects the curiosity,nov ­ The following personality disorders are included in clus- elty seekingQ and desire to have new experiences. ter A: Individuals with high openness to experience may A. Paranoid personality disorder: The characteristic indulge in activities such as skydiving, bungee jump- feature is excessive suspiciousness and distrust of ing, gambling, etc. others. These patients may beexcessively sensitiveQ 2. Conscientiousness: It reflects the tendency to be and may be quick to react angrily. They giveexcessive organized, disciplined and dutiful. importance to themselves and believe in conspiracy mebooksfree.com mebooksfree.com 3. Extraversion: It reflects mebooksfree.comthe sociability, talkativeness theories.mebooksfree.com Psychotherapy is the treatmentmebooksfree.com of choice. mebooksfree.com and preference for group activities over solitary activi- Medications like benzodiazepines and antipsychotics­ ties. may be used for agitation and paranoia (excessive sus- 4. Agreeableness: It reflects compassion and coopera- piciousness). tion for others and a trusting and helpful nature. B. Schizoid personality disorder: These patients are 5. Neuroticism: It reflects the tendency to experience detachedQ from social relationships and prefer soli­ unpleasant emotions easily. It also refers to the degree tary activities.They areemotionally coldQ and are of emotional stability. indifferent to praise or criticism. They appear self If the personality of an individual deviates from social absorbed and lost in day dreams and may be preoc- norms and is a cause of unhappiness and impairment, cupied with fantasies. Since they are uncomfortable the individual is diagnosed with a personality disorder. with human interaction, they have little interest in Personality disorder is defined as presence of abnor- sexual activities. The management revolves around mebooksfree.com mebooksfree.commal behavior and subjectivemebooksfree.com experiences which causes psychotherapy.mebooksfree.com The medications whichmebooksfree.com are occasion- mebooksfree.com significant impairment. The prevalence of personality ally used include antipsychotics, antidepressants and disorder is around 10–20% in the general population. benzodiazepines. The onset is inadolescence or early adulthoodQ, the C. Schizotypal personality disorder: These patients symptoms remain stable throughout the adult life and have disturbances of thinking and communication. maturingQ occurs by around 40 years. Maturing means They frequently exhibit odd beliefs or magical think­ the resolution of abnormal patterns of behavior. The per- ingQ (e.g. superstitiousness, belief in telepathy or “sixth sonality disorder are “ego syntonic”Q (agreeable to self). sense”). Their inner world may be like that of a child,

mebooksfree.com mebooksfree.com mebooksfree.com mebooksfree.com mebooksfree.com mebooksfree.com

mebooksfree.com mebooksfree.com mebooksfree.com mebooksfree.com mebooksfree.com mebooksfree.com mebooksfree.com mebooksfree.com mebooksfree.com mebooksfree.com mebooksfree.com mebooksfree.com

90 Review of Psychiatry

filled with fears and H frequently get involved in unlawful behaviors such fantasies. They may In ICD-10, schizotypal disorder is as theft, lying, truancy and conning. They have a lack not considered as a personality dis- have strange ways order, instead it is classified as a of remorse or guilt for their actions. Substance use of communication psychotic disorder along with schizo- disorders are frequently present in these patients. mebooksfree.com mebooksfree.commaking it diffi- phrenia.mebooksfree.com Treatmentmebooksfree.com usually is psychotherapy. Medicationsmebooksfree.com like mebooksfree.com cult to understand. carbamazepine, beta blockers are occasionally used. They may also report illusions and other perceptual D. Borderline personality disorder: These patients are disturbances. They usually don’t have any close rela- almost always in a state of crisis. They have significant tionships and appear “odd and eccentric” to others. mood swings. They may start feeling angry, anxious When in severe stress, they may decompensate and or frustrated without any reason. Their interpersonal have psychotic symptoms, but these are usually brief. relationships are intense and tumultuous. They swing The management revolves around psychotherapy. from being excessively dependent to being hostile to The medications which are occasionally used include persons close to them. Hence, they have a history of antipsychotics, antidepressants and benzodiazepines. unstable relationshipsQ. Another characteristic fea- The “cluster A” personality disorders (especially schi- ture is the repetitive self destructive actsQ such as zotypal personality disorder) are considered to be on a slashing of wrists, or overdosage of medications. The mebooksfree.com mebooksfree.com“schizophrenia continuum” mebooksfree.comwhich means that they lie patientsmebooksfree.com indulge in these behaviors to elicitmebooksfree.com help from mebooksfree.com somewhere in between the “normal” and “schizophrenia”. others, to express the anger or just to numb them- selves to the overwhelming painful feelings they have. Cluster B Personality Disorders These patients are alsoimpulsive Q in areas such as spending, sex and substance use. Finally, these patient The following personality disorders are included in clus- excessively use the defense mechanism of splitting ter B: (wherein they consider each person to be either “all A. Histrionic personality disorder: These patients are good” or “all bad”). Management involves psycho- excitable and overtly emotional and behave in a therapy. “Dialectical behavior therapy” is a therapy dramatic and extroverted way. They want to be the which has been designed for treatment of borderline center of attention and exaggerate everything, mak- personality disorder. Medications used include anti­ ing it sound more important than it really is. They psychotics, antidepressant and mood stabilizers like tend to behave in a sexually seductive manner and carbamazepine. In ICD-10, the borderline personality mebooksfree.com mebooksfree.comuse physical appearance mebooksfree.comto draw attention towards mebooksfree.com mebooksfree.com mebooksfree.com disorder has been described as a subtype of a broader self. Management usually involves psychotherapy. diagnosis of “emotionally unstable personality dis­ Medications like antidepressants are occasionally order”. useful. B. Narcissistic personality disorder: These patients have a heightened sense of self importanceQ. They Cluster C Personality Disorders believe that they are special and very talented. The following personality disorders are included in clus- They are preoccupied with fantasies of unlimited ter C: success and power. They want to be admired by A. Avoidant personality disorder: These patients are others. If condemned, they may become very angry excessively sensitive to rejection. They are afraid that or they may show complete indifference to criticism. they would be criticized or rejected in social situa- They have a fragile self esteem and are susceptible to tions. Hence, they tend to remain socially withdrawn. mebooksfree.com mebooksfree.comdevelopment of depression,mebooksfree.com when faced with rejec- Thesemebooksfree.com persons are usually unwilling tomebooksfree.com enter into a mebooksfree.com tion. Management usually involves psychotherapy. relationship unless they are given a strong guarantee Medications like antidepressants are occasionally of uncritical acceptance. The ICD-10, uses the diagno- useful. sis of anxious personality disorder for such patients. C. Antisocial personality disorder (dissocial persona­ Management mostly involves psychotherapy. Beta lity disorder): These patients don’t have regard for blockers and selective serotonin reuptake inhibitors rights of others and frequently violate them. They (SSRIs) are also useful.

mebooksfree.com mebooksfree.com mebooksfree.com mebooksfree.com mebooksfree.com mebooksfree.com

mebooksfree.com mebooksfree.com mebooksfree.com mebooksfree.com mebooksfree.com mebooksfree.com mebooksfree.com mebooksfree.com mebooksfree.com mebooksfree.com mebooksfree.com mebooksfree.com

Personality Disorders 91

B. Dependent personality disorder: These patients are In comparison individuals with Type B personality dependent on others for everyday decisions. All the are easy going and relaxed, they are not excessively com- major decisions in their lives are taken by someone petitive and may focus more on enjoyment and less on else. They ask for excessive amount of advice and reas- winning or losing. Recent studies have suggested a new mebooksfree.com mebooksfree.comsurance from others. Theymebooksfree.com also have difficulty express- personalitymebooksfree.com type, Type D personalityQ whichmebooksfree.com is charac- mebooksfree.com ing disagreement with others because of fear of loss terized by negative affectivity (a tendency to experience of support. They get very uncomfortable and helpless negative emotions) and social inhibition (tendency to when alone and fear that they wont be able to take inhibit expression of emotions). Individuals with Type D care of themselves. Management usually involves psy- personality are predisposed to development of coronary Q chotherapy. Benzodiazepines and SSRIs can be used heart disease . for symptomatic relief. C. Obsessive compulsive personality disorder: These IMPULSE CONTROL DISORDERS patients are preoccupied with rules and regula- These disorders are characterized by irresistible impulses tions. They give excessive importance to details and or temptations to perform a particular act which is harm- show perfectionism that interferes with task com- ful to self or others. Impulse is described by patients as a pletion (since they want everything to be perfect, it feeling of increasing tension and arousal that leads to per- mebooksfree.com mebooksfree.comoften results in significantmebooksfree.com delays). They areinfle­ formancemebooksfree.com of a certain behavior. The performancemebooksfree.com of the mebooksfree.com xible and insist that others agree to their demands. behavior gives a sense of relief and also gratification. After They are excessively devoted to work and may not some time, however the person feels guilty or remorseful. The following are described as impulse control disorders. have any time for leisure activities. They are for- All of them are preceded by the irresistible impulses: mal and serious and often lack a sense of humor. 1. Pyromania: Recurrent and purposeful setting of fires. The ICD-10 , used the diagnosis of “anankastic per­ 2. KleptomaniaQ: Recurrent stealing of objects which sonality disorder” for these patients. Management are not needed for personal use or are of no monetary usually involves psychotherapy. value. 3. Intermittent explosive disorder: It is characterized by Type A and B Personality episodes of aggression resulting in serious assault or Another way of classifying per­sonality is what is known destruction of properties. 4. Pathological gambling: Recurrent episodes of gam- mebooksfree.com mebooksfree.comas Type A and Type B personality.mebooksfree.com Type A personality mebooksfree.com mebooksfree.com mebooksfree.com bling which causes economic troubles and serious is characterized by competitiveness, time urgency, relationship problems. hostility and anger. The people with Type A personality 5. Trichotillomania: Recurrent episodes of hair pulling. are ambitious, impatient and hard working workaholics. 6. Others: These include, Oniomania or compulsive buy- Many studies have suggested that Type A personality ing: Recur­rent episodes of buying or shopping despite (especially the hostility and anger traits) is a risk factor the buying behavior causing significant monetary and for coronary heart diseaseQ. socio occupational distress.

QUESTIONS AND ANSWERS mebooksfree.com mebooksfree.comQUESTIONS mebooksfree.com 2. mebooksfree.comTrue about personality disorder: (PGImebooksfree.com June 2007) mebooksfree.com A. Onset in early childhood and adolescence 1. Which of the following is not a personality trait? B. Matures around adulthood (AIIMS Nov 2009) C. Not associated with social norms A. Sensation seeking D. Direct result of disease or damage B. Neuroticism 3. Characteristic disorder that appears in late child C. Open to experience hood and continues in adulthood: D. Problem solving (DNB NEET 2014-15)

mebooksfree.com mebooksfree.com mebooksfree.com mebooksfree.com mebooksfree.com mebooksfree.com

mebooksfree.com mebooksfree.com mebooksfree.com mebooksfree.com mebooksfree.com mebooksfree.com mebooksfree.com mebooksfree.com mebooksfree.com mebooksfree.com mebooksfree.com mebooksfree.com

92 Review of Psychiatry

A. Somatoform disorder 11. Antisocial personality is associated with: B. Personality disorder (PGI 1999) C. Anxiety disorder A. Drug abuse D. Mood disorder B. Paranoid schizophrenia mebooksfree.com mebooksfree.com mebooksfree.com mebooksfree.com mebooksfree.com mebooksfree.com 4. True about personality disorder: (PGI 2003) C. Obsessive compulsive disorder A. Typically onset at early childhood and adoles- D. None cence 12. Features of borderline personality are: B. Mature around at 30-40 years (PGI May 2012) C. Ego dystonic A. Impulsivity D. Dramatic, emotional and erratic behavior in B. Recurrent suicidal behavior paranoid PD C. Anger and anxiety E. Pervasive and maladaptive behavior D. Extreme suspiciousness 5. True about personality disorder: (PGI June 2008) E. Pattern of unstable and intense interpersonal A. Onset in early childhood and adolescence relationships B. Matures around adulthood 13. Cardinal feature of antisocial personality disorder C. Suspiciousness is seen in paranoid personality mebooksfree.com mebooksfree.com mebooksfree.com mebooksfree.comis: (DNB NEETmebooksfree.com 2014-15) mebooksfree.com disorder D. Excessive preoccupation with fantasy is seen in A. Violation of rules of society schizoid personality disorder B. Attention-seeking behavior C. Unstable interpersonal relationships 6. Oddities of speech, mannerism, odd clothing with D. Grandiose behavior magical thinking is seen in which type of persona­ lity disorder: (DNB 2003, JIPMER-2K) 14. A 16-year-old girl was brought to psychiatric emer­ A. Schizoid B. Paranoid gency after she slashed her wrist in an attempt to C. Schizotypal D. Borderline commit suicide. On enquiry her father revealed that she had made several such attempts of wrist 7. Which personality disorder of DSM-IV is not slashing in past, mostly in response to trivial fights classified as PD and is placed with schizophrenia in her house. Further she has marked fluctuations in ICD 10? (WB 1998, DNB 1997) in her mood with a pervasive pattern of unstable mebooksfree.com mebooksfree.comA. Schizoid mebooksfree.comB. Paranoid mebooksfree.com mebooksfree.com mebooksfree.com interpersonal relationships. The most probable C. Narcissistic D. Schizotypal diagnosis is: (AIIMS Nov 2002) 8. Characteristic feature of schizoid personality A. Borderline personality disorder disorder is: (AIIMS 1999) B. Major depression A. Conversion reaction C. Histrionic personality disorder B. Not concerned with disease D. Adjustment disorder C. Check details of all things D. Emotional coldness 15. Patients who are grandiose and require admira­ tion from others, have which type of personality? 9. Which personality disorder can be consi­ (DNB NEET 2014-15) dered a part of autistic spectrum disorders? A. Narcissistic B. Histrionic (DNB NEET 2014-15) C. Borderline D. Antisocial mebooksfree.com mebooksfree.comA. Schizoid mebooksfree.comB. Schizotypal mebooksfree.com mebooksfree.com mebooksfree.com C. Borderline D. All of the above 16. A young lady was admitted with h/o taking over­ dose of diazepam after broken affair. She has 10. Markedly inappropriate sensitivity, self impor­ tance and suspiciousness are clinical features of: history of slitting her wrist previously. Most likely (DNB NEET 2014-15, DNB 2001, TN 1999, AMU 2002) diagnosis is: (AIIMS 2000) A. Antisocial PD B. Histrionic PD A. Narcissistic PD B. Dependent PD C. Schizoid PD D. Paranoid PD C. Borderline PD D. Histrionic PD

mebooksfree.com mebooksfree.com mebooksfree.com mebooksfree.com mebooksfree.com mebooksfree.com

mebooksfree.com mebooksfree.com mebooksfree.com mebooksfree.com mebooksfree.com mebooksfree.com mebooksfree.com mebooksfree.com mebooksfree.com mebooksfree.com mebooksfree.com mebooksfree.com

Personality Disorders 93

17. A person has the habit of inflicting repeated inju­ A. Coronary artery disease ries to self, what is the type of personality? B. Depression (PGI June 2004) C. Schizophrenia A. Borderline B. Schizoid D. Mania mebooksfree.com mebooksfree.comC. Histrionic mebooksfree.comD. Narcissistic mebooksfree.com mebooksfree.com mebooksfree.com E. Depressive Impulse Control Disorder 18. Pervasive pattern of instability of interpersonal 25. Kleptomania is: (PGI May 2011, 2007) relationships, self image and affect, with marked A. Delusional disorder impulsivity that begins at early adulthood and B. Obsession present in varieties of context is characteristics C. Impulse disorder of: (Bihar 2006) D. Compulsion seclusion A. Bipolar disorder E. Hallucination B. Schizoaffective disorder C. Borderline personality disorder 26. One of the following is not a compulsive and habit D. Schizotypal personality disorder forming disorder: (KA 1995) 19. A lady has changed multiple boyfriends in last 6 A. Kleptomania mebooksfree.com mebooksfree.commonths, she keeps breakingmebooksfree.com her relationships, and mebooksfree.comB. Pyromania mebooksfree.com mebooksfree.com she also has attempted suicide many times. Most C. Nymphomania likely diagnosis is: (MP 2006) D. Pathological gambling A. Borderline personality disorder B. Post-traumatic stress ANSWERS C. Acute depression D. Acute panic attack 1. D. Sensation seeking is a part of “openness to experi- ence”. Problem solving is not a personality trait. 20. A person with shy, anxious avoidant personality 2. A,B,C. comes under which cluster? (AIIMS May 2015) Personality disorders have onset in early child- A. Cluster A B. Cluster B hood and adolescence and maturing occur in C. Cluster C D. Cluster D adulthood by 30-40 years of age. People with mebooksfree.com mebooksfree.com 21. Obsessive personalitymebooksfree.com disorder is also called: mebooksfree.compersonality disorders tend to havemebooksfree.com conflicts with mebooksfree.com (DNB NEET 2014-15) the societal norms (e.g. patients with antisocial A. Anankastic personality disorder personality disorders tend to break societal rules B. Dissocial personality disorder and regulations). C. Eccentric personality disorder 3. B. D. Histrionic personality disorder 4. A, B, E. Personality disorders are “ego syntonic” and not 22. True about treatment of personality disorder: “ego dystonic”. Option D is description of histrio­ (PGI May 2010) A. Antipsychotics are used nic personality disorder. B. SSRI are used 5. A, B, C, D. C. Behavior therapy is used See text D. No need for treatment 6. C. Odd behavior including odd speech, manner- mebooksfree.com mebooksfree.com mebooksfree.com mebooksfree.comisms and magical thinking is seenmebooksfree.com in schizotypal mebooksfree.com 23. False regarding Type A personality: personality disorder. (AIIMS Nov 2007) 7. D. In ICD-10, schizotypal PD is placed with schizo- A. Hostile B. Time pressure phrenia spectrum and not in personality disor- C. Competitiveness D. Mood fluctuations der. 24. Individual with Type D personality are recently 8. D. See text. found to be at risk of developing: 9. A. The characteristic feature of autistic spectrum (AIIMS Nov 2011) disorder (ASD) is impairment in social interaction

mebooksfree.com mebooksfree.com mebooksfree.com mebooksfree.com mebooksfree.com mebooksfree.com

mebooksfree.com mebooksfree.com mebooksfree.com mebooksfree.com mebooksfree.com mebooksfree.com mebooksfree.com mebooksfree.com mebooksfree.com mebooksfree.com mebooksfree.com mebooksfree.com

94 Review of Psychiatry

and communication. These features are also seen 16. C. The repetitive episodes of self harming behavior in schizoid personality disorder. There can be after stressors is suggestive of borderline perso­ significant difficulty differentiating between nality disorder. schizoid PD and milder forms of ASD. It must 17. A. mebooksfree.com mebooksfree.combe remembered thatmebooksfree.com patients with ASD have 18. mebooksfree.comC. mebooksfree.com mebooksfree.com more severe social impairment and also have 19. A. stereotypical behaviors and interests. 20. C. 10. D. See text. 21. A. 11. A. Antisocial PD is frequently associated with sub- 22. A, B, C. The mainstay of treatment in personality disorders stance use disorders. is psychotherapy. Medications used include 12. A, B, C, E. SSRIs, anti­psychotics and mood stabilizers. See text. 23. D. 13. A. People with antisocial PD characteristically 24. A. disregards rights of others, don’t follow norms 25. C. Kleptomania is an impulse control disorder in of society and indulge in antisocial behaviors. which the patient has recurrent irresistible desire 14. A. This patient has history suggestive of self harming to steal objects, which he/she doesn’t need for mebooksfree.com mebooksfree.combehavior with moodmebooksfree.com fluctuations and pervasive mebooksfree.compersonal use or for monetary value.mebooksfree.com mebooksfree.com unstable pattern of interpersonal relationships, 26. C. Nymphomania is the condition of excessive all of which are features of borderline PD. sexual desire in females. It is not an impulse 15. A. control disorder.

mebooksfree.com mebooksfree.com mebooksfree.com mebooksfree.com mebooksfree.com mebooksfree.com

mebooksfree.com mebooksfree.com mebooksfree.com mebooksfree.com mebooksfree.com mebooksfree.com

mebooksfree.com mebooksfree.com mebooksfree.com mebooksfree.com mebooksfree.com mebooksfree.com

mebooksfree.com mebooksfree.com mebooksfree.com mebooksfree.com mebooksfree.com mebooksfree.com mebooksfree.com mebooksfree.com mebooksfree.com mebooksfree.com mebooksfree.com mebooksfree.com

mebooksfree.com mebooksfree.com mebooksfree.com mebooksfree.com mebooksfree.com mebooksfree.com

Chapter 8 Eating Disorders

ANOREXIA NERVOSA Subtypes mebooksfree.com mebooksfree.comAnorexia nervosa is most commonlymebooksfree.com seen in adolescent Anorexiamebooksfree.com nervosa has the following two subtypes.mebooksfree.com mebooksfree.com females. Initially, it was reported to be more common 1. Restricting type: This type is seen in around 50% of in upper class, however recent data doesn’t support that patients and is characterized by highly restricted food fact. It must be noted that anorexia nervosa is a misnomer intake. since the appetite of these patients is usually normalQ 2. Binge eating/purging subtype: It is seen in 25–50% and hence there is no symptom of anorexia in anorexia of patients. In this type, patient alternates attempts at nervosa. rigorous dieting with intermittent binging and purg- ing episodes. The binging involves intake of a large It is characterized by the following signs and symp- amount of food in a short duration with an associated toms: feeling of lack of self control during binge episode. The 1. Disturbance of body image (patient perceives that she purging is a compensatory mechanism wherein patient is fat despite being quite thin in reality). tries to compensate for excess calories by self induced mebooksfree.com mebooksfree.com 2. Excessive fear of fatness mebooksfree.comand excessive emphasis on vomiting,mebooksfree.com laxative use, diuretic use or emeticmebooksfree.com use. The mebooksfree.com thinness. repeated vomiting episodes may cause dental caries, 3. Restriction of energy intake resulting in a significantly parotitis, and hypokalemic alkalosis. less weightQ than normal. 4. Medical symptoms secondary to starvation such as Treatment amenorrheaQ, lanugo (appearance of neonatal hairs), H hypothermia, dependent edema and bradycardia. The treatment may DSM-5 Update: In DSM-4, ameno­ include hospitaliza- rrhea was a necessary symptom The adolescent patients often havepoor sexual deve­ tion to restore patients for diagnosis of anorexia nervosa, lopmentQ whereas the adult patients usually report lowQ however in DSM-5 this criterion has nutritional status and been removed and anorexia nervosa interest in sexual activities. Patients often exhibit pecu­ manage complications can be diagnosed in the absence of liar behaviorQ about food such as hiding food in the amenorrhea now. like dehydration and house, trying to dispose food in napkins, cutting food into electrolyte imbalances. The treatment focusses on a com- mebooksfree.com mebooksfree.comvery small pieces and rearrangingmebooksfree.com the food repeatedly binationmebooksfree.com of behavioral management (praisemebooksfree.com for healthy mebooksfree.com around the plate. These patients are preoccupied with eating habits, restriction of self induce vomiting), indi- the thoughts about food and may spend a large amount vidual psycho­therapy and family education. Medications of time collecting recipes or cooking food for others. such as cyproheptadine, tricyclic antidepressants (TCAs) Patients are usually secretive and deny any symptoms and selective serotonin reuptake inhibitors (SSRIs) have and refuse for treatment. been tried with varied success.

mebooksfree.com mebooksfree.com mebooksfree.com mebooksfree.com mebooksfree.com mebooksfree.com

mebooksfree.com mebooksfree.com mebooksfree.com mebooksfree.com mebooksfree.com mebooksfree.com mebooksfree.com mebooksfree.com mebooksfree.com mebooksfree.com mebooksfree.com mebooksfree.com

96 Review of Psychiatry BULIMIA NERVOSA 4. Weight is usually normalQ, and is an important dif- ferentiating factor between bulimia nervosa and ano- Bulimia nervosa is characterized by episodes of binge rexia nervosa. eating combined with inappropriate ways of prevent­ The patients with bulimia nervosa usually tend to have mebooksfree.com mebooksfree.coming weight gain. Bulimia nervosamebooksfree.com is more common than featuresmebooksfree.com secondary to purging such as enamelmebooksfree.com erosionQ mebooksfree.com anorexia nervosa, is usually seen in females, and the age and dental cariesQ, salivary gland inflammations,callus of onset is mostly late adolescence. The following are the on knucklesQ (as knuckles get injured against teeth during clinical features: episodes of self induced vomiting). The patient may develop 1. Episodes of binge eating in which large amount of hypokalemia and hypochloremic alkalosis and rarely gas- food is usually consumed in a small duration with an tric and esophageal tear during forceful vomiting. associated feeling of lack of self control during binge Patients have normal sexual functioningQ and are episode. usually not secretive about their symptoms as p atients 2. Compensatory behavior after binge eating to prevent with anorexia nervosa. weight gain. These measures usually include purg- ing behaviours like self induced vomiting, laxatives Treatment or diuretics abuse, use of eme­tics and in few patients It is usually outpatient and involves psychotherapeutic mebooksfree.com mebooksfree.comexcessive exercising (hyperglycemia)mebooksfree.com and dieting. techniquesmebooksfree.com like cognitive behavioural therapymebooksfree.com (first line) mebooksfree.com 3. Like patients of anorexia nervosa, the patients with and dynamic psychotherapy. The medications mostly bulimia nervosa too have a morbid fear of gaining used are antidepressants like selective serotonin reuptake weight and give excessive emphasis to thinness. inhibitors.

QUESTIONS AND ANSWERS

QUESTIONS 4. With regard to anorexia nervosa all of the follow­ ing are true except: 1. Which of the following is not a common feature of (DNB NEET 14-15, DNB 03, Kerala 2K) mebooksfree.com mebooksfree.comanorexia nervosa? mebooksfree.com(DNB 2007, AI 2006) mebooksfree.com mebooksfree.com mebooksfree.com A. Phobic avoidance of normal weight A. Binge eating B. Over perception of body image B. Amenorrhea C. Self induced vomiting C. Self perception of being fat D. Menorrhagia D. Under weight E. Excessive exercise 2. Anorexia nervosa can be differentiated from bulimia by: (AIIMS NOV 2008) 5. A young lady presents with h/o repeated episodes A. Intense fear of weight gain of over eating ( binge) followed by purging using B. Disturbance of body image laxatives, she is probably suffering from: C. Adolescent age (AI 2002, UP 2004, AIIMS 10,07, DNB 2009) D. Peculiar patterns of food handling A. Bulimia nervosa B. Schizophrenia mebooksfree.com mebooksfree.com 3. Which of the followingmebooksfree.com is not true about bulimia mebooksfree.com mebooksfree.com mebooksfree.com C. Anorexia nervosa nervosa? (UPSC 2009) D. Benign eating disorder A. Recurrent bouts of binge eating B. Lack of self control over eating during binge 6. Which of the following is not true about bulimia C. Self induced vomiting or dieting after binge nervosa? (UPSC-1 08) D. Weight gain A. Invariable weight loss with endocrine disorder

mebooksfree.com mebooksfree.com mebooksfree.com mebooksfree.com mebooksfree.com mebooksfree.com

mebooksfree.com mebooksfree.com mebooksfree.com mebooksfree.com mebooksfree.com mebooksfree.com mebooksfree.com mebooksfree.com mebooksfree.com mebooksfree.com mebooksfree.com mebooksfree.com

Eating Disorders 97

B. Occurrence of both binge eating and inappropri- ANSWERS ate compensatory behaviors at least twice weekly 1. A. All the four options are features of anorexia nervosa. on an average for 3 months C. Recurrent episodes of binge eating However, if one has to chose, the best answer mebooksfree.com mebooksfree.comD. Recurrent self inducedmebooksfree.com vomiting mebooksfree.comwould be binge eating. Though mebooksfree.combinge eating is mebooksfree.com seen in almost 50% of patients with anorexia 7. False regarding anorexia nervosa: nervosa, however its not a core symptom of (DNB 2008, AI 2006) anorexia nervosa. A. Evident psychosis 2. D. Unlike patients with bulimia, patients with ano- B. Vigor exceeding physical well-being C. Weight loss rexia remain preoccupied with food and show D. Decreased appetite peculiar behavior like hiding food in the house, trying to dispose food in napkins, cutting food 8. False regarding anorexia nervosa: into very small pieces and rearranging the food (DNB NEET 2014-15) repeatedly around the plate. A. Psychiatric symptoms such as depression may be associated 3. D. The patients with bulimia nervosa usually have mebooksfree.com mebooksfree.comB. Excessive exercisingmebooksfree.com can be a feature mebooksfree.comnormal weight. mebooksfree.com mebooksfree.com C. Weight loss is a feature 4. D. Amenorrhea and not menorrhagia is the men- D. Decreased appetite is a feature strual disturbance seen in anorexia. 5. A. 9. Following are true about bulimia nervosa except: (DNB NEET 2014-15) 6. A. Weight loss and endocrine abnormality are seen A. Uncontrolled eating episodes in anorexia not bulimia nervosa. B. Overweight individuals 7. A. There are no psychotic symptoms in anorexia C. Depressive symptoms are present nervosa. D. Patients are sexually active 8. D. The appetite of patients with anorexia is normal 10. Not true about bulimia nervosa is: and as such there is no anorexia in anorexia (DNB NEET 2014-15) nervosa. A. Onset is in late adolescence 9. B. mebooksfree.com mebooksfree.comB. Dental caries/toothmebooksfree.com decay is a finding 10. mebooksfree.comC. Presence of amenorrhea is a differentiatingmebooksfree.com fea- mebooksfree.com C. Amenorrhea is a common finding ture between anorexia and bulimia. It is seen only D. Normal weight is usually seen in patients with anorexia.

mebooksfree.com mebooksfree.com mebooksfree.com mebooksfree.com mebooksfree.com mebooksfree.com

mebooksfree.com mebooksfree.com mebooksfree.com mebooksfree.com mebooksfree.com mebooksfree.com

mebooksfree.com mebooksfree.com mebooksfree.com mebooksfree.com mebooksfree.com mebooksfree.com mebooksfree.com mebooksfree.com mebooksfree.com mebooksfree.com mebooksfree.com mebooksfree.com

mebooksfree.com mebooksfree.com mebooksfree.com mebooksfree.com mebooksfree.com mebooksfree.com

Chapter 9 Sleep Disorders

ELECTROENCEPHALOGRAM • Stage 2, NREM: It is the stage with maximum durationQ. It is characterized by two typical find­ mebooksfree.com mebooksfree.comIt is the recording of electricalmebooksfree.com activity of the brain. It is mebooksfree.comings on electroencephalogram: mebooksfree.com mebooksfree.com recorded by placing electrodes on the scalp and recording a. Sleep spindlesQ: These are bursts of regular the potential difference between various electrodes. A waves (frequency of 13–15 Hz, 50 microvolt) and normal EEG has following types of rhythm. b. K-complexesQ: These are high voltage spikes which are seen intermittently. Stages of Sleep • Stage 3, NREM: The sleep deepens and there is appearance of delta waves. Sleep can be divided into two stages: • Stage 4, NREM: This is deep sleep and is characte­ A. Nonrapid eye movement sleep (NREM) or slow wave rized by predominance of delta waves on EEG. sleep and During the NREM sleep, there is pulsatile release of B. Rapid eye movement (REM) sleep or paradoxical gonadotropins and growth hormones. Further, the blood sleep. mebooksfree.com mebooksfree.com mebooksfree.compressure,mebooksfree.com heart rate and respiratory rate alsomebooksfree.com decreases. mebooksfree.com A. Nonrapid eye movement sleep: It is further divided B. Rapid eye movement sleep: It follows the NREM into following four stages: sleep. It is characterized­ by the following: • Stage 1, NREM: It is the first stage and the sleep • The EEG shows increased activity similar to awake is light (person can be easily aroused). The EEG state (beta activity) along with return of alpha shows, loss of alpha waves (which predominate activity when person has eyes closed but is still awake) and • Presence of rapid eye movements predominance of theta waves. • There is generalized loss of muscle tone.

Table 1: EEG rhythms. EEG rhythm Frequency (Hz) Amplitude (microvolt) Salient points Region mebooksfree.com mebooksfree.comAlpha (a) 8–12 mebooksfree.com50–100 Seen when individualmebooksfree.com is awake, at rest, Present maximallymebooksfree.com in occipital mebooksfree.com eyes closed and mind wandering and parieto-occipital area Beta (b) 14–30 5–10 Normal awake pattern, when attention Predominantly in frontal area is focussed beta waves appear Theta (q) 4–7 10 Transition from wakefulness to sleep, Parietal region and temporal early sleep region (hippocampus) Delta (d) 1–4 20–200 Deep sleep

mebooksfree.com mebooksfree.com mebooksfree.com mebooksfree.com mebooksfree.com mebooksfree.com

mebooksfree.com mebooksfree.com mebooksfree.com mebooksfree.com mebooksfree.com mebooksfree.com mebooksfree.com mebooksfree.com mebooksfree.com mebooksfree.com mebooksfree.com mebooksfree.com

Sleep Disorders 99

• Increased rateQ of metabolism in brain contractions, however the bed partner frequently • Penile erectionQ, autonomic hyperactivity (increase gets disturbed. The patient may report non restora­ in pulse rate, respiratory rate and blood pressure) tive sleep and day time sleepiness. The treatment • DreamsQ, which can be recalled are seen during usually involves benzodiazepines. mebooksfree.com mebooksfree.comREM sleep. mebooksfree.com • mebooksfree.comRestless leg syndrome (Ekbom syndrome)mebooksfree.com: It is char­ mebooksfree.com Ponto geniculo occipital spikesQ (large phasic poten­ acterized by uncomfortable sensation in legs (such tials that originate from cholinergic neurons in pons and as insect crawling on the skin) which get relieved by pass rapidly to lateral geniculate body and then to occipi­ moving the leg or walking around. This can cause tal cortex) are a characteristic feature. difficulty in initiation of sleep as patient keeps on REM sleep is called paradoxical sleepQ because moving the leg. The only approved drug for treat­ though the EEG is quite similar to awake state, its quite ment is ropiniroleQ (a dopamine agonist). difficult to awaken the patient. B. Hypersomnia: Primary hypersomnia is diagnosed In a 8 hour sleep, maximum time (around 6-6.5 hours) when no cause can be found for excessive sleepi­ is spent in NREM sleep and the rest (around 1.5 hours) in ness which can present with either prolonged sleep REM sleep. Most of the stage 4, NREM occurs in the first episodes or excessive day time sleep episodes. mebooksfree.com mebooksfree.comone-third of the night whereasmebooksfree.com most of REM sleep occurs Fewmebooksfree.com other disorders which can presentmebooksfree.com with hyper- mebooksfree.com in the last one-third of the night. The REM sleep occurs somnia include: regularly after every 90-100 minutes with a total of around • Narcolepsy: This disorder is characterized by the 4-5 REM sleeps in the entire night. following symptoms: a. Sleep attacks: The patient has irresistible urge SLEEP DISORDERS for sleep which can occur at any time during The various sleep disorders can be divided into two cate­ the day. gories: b. CataplexyQ: It is sudden loss of muscle tone, 1. Dyssomnias due to which patient can even have a fall. 2. Parasomnias c. Hypnagogic hallucinationsQ: These are the hallucinations, which occur while going to Dyssomnias sleep. Patient may also have hypnopompic mebooksfree.com mebooksfree.com mebooksfree.com mebooksfree.comhallucinationsQ (hallucinations mebooksfree.comwhile getting mebooksfree.com These disorders are characterized by abnormality in the up from sleep). duration or quality of sleep. They include: d. Sleep paralysis: It usually occurs when the A. Insomnia: Primary Insomnia is diagnosed when no cause can be found for decreased sleep and may patient gets up in the morning. Though he has present with difficulty in initiation of sleep, difficulty woken up, he is not able to move his body. in maintenance of sleep (frequent awakening during The hallmark of narcolepsy is reduced latency of Q night or early morning awakening) or nonrestora­ REM sleep . Normally, it takes around 90 minutes to tive sleep (not feeling refreshed in the morning due reach REM sleep (after crossing all the stages of NREM to poor quality of sleep). The management usually sleep) however in patients with narcolepsy, patient involves use of benzodiazepines, zolpidem and other reaches REM sleep much earlier. hypnotics. The management includes a regimen of forced naps mebooksfree.com mebooksfree.com Few other disorders whichmebooksfree.com can present with insomnia at regularmebooksfree.com time. The medications used aremodafinil mebooksfree.com and mebooksfree.com include: other stimulants like amphetamines. • Periodic limb movement disorder: It is charac­ • Kleine-Levin syndrome: This is a rare disorder terized by sudden contraction of muscle groups which is characterized by episodes of hypersom- (usually leg) while sleeping. This results in partial niaQ, hyperphagia and hyper sexualityQ (increased or complete awakening, repeatedly in the night. sexual activity). In between the episodes patient is The patient is usually not aware of these sudden essentially asymptomatic.

mebooksfree.com mebooksfree.com mebooksfree.com mebooksfree.com mebooksfree.com mebooksfree.com

mebooksfree.com mebooksfree.com mebooksfree.com mebooksfree.com mebooksfree.com mebooksfree.com mebooksfree.com mebooksfree.com mebooksfree.com mebooksfree.com mebooksfree.com mebooksfree.com

100 Review of Psychiatry Parasomnias of choice is bed alarmsQ, which start ringing, as soon as child passes urine. The medications These disorders are characterized by dysfunctional which can be used include tricyclic antidepres­ events associated with the sleep. These include: sants such as imipramineQ, although their use A. Stage 4, NREM sleep disorders: These disorders occur mebooksfree.com mebooksfree.com mebooksfree.com mebooksfree.comis associated with severe side effects.mebooksfree.com Intranasal mebooksfree.com during stage 4, NREM (also stage 3, NREM). Since desmopressinQ is a better alternative. most of the stage 4, NREM is present in first third • Bruxism (teeth grindingQ): The patient grinds his of the sleep, these disorders are also seen in the teeth making loud sounds and there may be dam­ same period. Also, the patient is not able to recall age to the enamel of teeth. the events in the morning. These disorders are usu­ • Sleep talking (somniloquy): Patient talks during ally seen in children and include: stage 3 and 4, NREM and is unable to recall the • Night terror or sleep terror (pavor nocturnusQ): same in the morning. The patient suddenly gets up screaming and has In most cases these disorders do not require any symptoms of intense anxiety such as tachycardia treatment and the parents must be reassured. In some and sweating. The patient is not able to recall any cases, benzodiazepinesQ are prescribed. As benzodia­ dream or reasons for feeling scared. zepines decrease the duration of stage 4, NREM, they Q mebooksfree.com mebooksfree.com• Sleep walking (somnambulismmebooksfree.com): The patients also decreasemebooksfree.com these episodes. mebooksfree.com mebooksfree.com may carry out a range of activities for which he B. Other sleep disorders: doesn’t have any memory later on. It may include • Nightmare: It occurs during REM sleep, wherein leaving the bed and walking about and also activi­ patient has a bad dream and gets up scared and ties like dressing, moving around or even driving. has behavioral signs of anxiety such as tachycar­ • Sleep related enuresis: The enuresis which is dia and hypertension. In contrast to night terror,­ defined as voiding of urine at inappropriate in nightmare, the patient is able to recall the places, is nocturnal in around 80% of cases. The dream. Agents that reduce duration of sleep, such most common cause of bed wetting are psy­ as tricyclic antidepressants can be used for treat­ chosocial such as sibling rivalry. The treatment ment. mebooksfree.com mebooksfree.com mebooksfree.com mebooksfree.com mebooksfree.com mebooksfree.com QUESTIONS AND ANSWERS

QUESTIONS C. Brain shows increased metabolism D. EEG shows decreased activity 1. Maximum duration of time spent is in which of the following NREM stage? (NEET/DNB) 4. Slow wave in hippocampal area is: (MP 00) A. I B. II A. Delta B. Theta C. III D. IV C. Beta D. Alpha 5. Alpha-rhythm is seen in: (PGI 1997) 2. A middle aged man complains of lack of sleep dur- A. Sleep with eyes closed with mind wandering ing the night time. The duration of the time he is B. Mental activity truly asleep or awake can be ascertained by which mebooksfree.com mebooksfree.com mebooksfree.com mebooksfree.comC. Awake with eyes open mebooksfree.com mebooksfree.com of the following? (AIIMS Nov 2012) D. REM sleep A. Barograph B. Kymograph C. Actigraphy D. Plethysmography 6. Pontogeniculo occipital spike is characteristic of which of the following sleep stage? 3. Not a feature of paradoxical sleep is: (PGI 1999) (DNB NEET 2014-15) A. Decreased muscle tone A. Stage 1 NREM B. Stage 2 NREM B. Rapid eye movements C. Stage 3 NREM D. REM

mebooksfree.com mebooksfree.com mebooksfree.com mebooksfree.com mebooksfree.com mebooksfree.com

mebooksfree.com mebooksfree.com mebooksfree.com mebooksfree.com mebooksfree.com mebooksfree.com mebooksfree.com mebooksfree.com mebooksfree.com mebooksfree.com mebooksfree.com mebooksfree.com

Sleep Disorders 101

7. The EEG recorded shown below is normally 14. Drug of choice for night terrors: (PGI 1998) recordable during which stage of sleep: (AI 2003) A. Meprobamate B. Tricyclic antidepressant C. Clonazepam mebooksfree.com mebooksfree.com mebooksfree.com mebooksfree.comD. Diazepam mebooksfree.com mebooksfree.com 15. Feature of narcolepsy include (s) all except: (PGI May 2013) A. Disorder of REM sleep regulation B. Disorder of NREM sleep regulation C. Hypnagogic hallucination D. Hypnopompic hallucinations E. Cataplexy

A. Stage I B. Stage II 16. Not true about narcolepsy: (PGI Dec 2006) C. Stage III D. Stage IV A. Sudden sleep B. Long duration (>3 hrs) of sleep mebooksfree.com mebooksfree.com 8. What are the EEG wavesmebooksfree.com recorded for parieto mebooksfree.comC. Cataplexy mebooksfree.com mebooksfree.com occipital region with subject awake and eyes D. Presents in IInd decade closed? (Kerala 1997) 17. Modafinil is approved by FDA for treatment of all A. Alpha waves B. Beta waves except: (DNB 2006, AI 2009) C. Delta waves D. Theta waves A. Obstructive sleep apnea syndrome (OSAS) 9. Which one of the following phenomenon is closely B. Shift work syndrome (SWS) associated with slow wave sleep? C. Narcolepsy (AIIMS Nov 2004) D. Lethargy in depression A. Dreaming B. Sleep walking 18. Following is true about ropinirole: C. Atonia D. Irregular heart rate (DNB NEET 2014-15) A. Selective D2/3 receptor agonist 10. Not true about nocturnal penile tumescence is: mebooksfree.com mebooksfree.com mebooksfree.com mebooksfree.comB. It is used in restless leg syndromemebooksfree.com mebooksfree.com A. Totals about 100 min/night (AIIMS 1995) C. Both A and B B. Normal phenomenon D. None of the above C. Occurs in NREM sleep 19. Regarding, Kleine-Levin syndrome which of the D. Can be used to distinguish between psychologi­ following is not true: (DNB NEET 2014-15) cal or organic impotence A. Hypersomnia 11. Which of the following conditions are seen during B. Hyposexulity NREM sleep? (DNB NEET 2014-15) C. Spontaneous resolution A. Teeth grinding B. Night mares D. Also called sleeping beauty syndrome C. Narcolepsy D. Sleep paralysis ANSWERS 12. Pavor nocturnus is: (APPG 1997) mebooksfree.com mebooksfree.comA. Sleep terror mebooksfree.comB. Sleep apnea 1. mebooksfree.comB. mebooksfree.com mebooksfree.com 2. C. Actigraphy is the procedure which is used for C. Sleep bruxism D. Somnambulism studying the sleep patterns. It usually involves 13. Antidepressant drug used in nocturnal enuresis wearing a small sensor on the wrist, which is: (AI 2011) ­detects the movements. However, the gold A. Imipramine B. Fluoxetine standard technique for studying sleep disorders C. Trazodone D. Sertraline is polysomnography.

mebooksfree.com mebooksfree.com mebooksfree.com mebooksfree.com mebooksfree.com mebooksfree.com

mebooksfree.com mebooksfree.com mebooksfree.com mebooksfree.com mebooksfree.com mebooksfree.com mebooksfree.com mebooksfree.com mebooksfree.com mebooksfree.com mebooksfree.com mebooksfree.com

102 Review of Psychiatry

3. D. In paradoxical sleep or REM sleep, the EEG shows 12. A. increased activity, similar to awake state. 13. A. Remember its not the drug of choice. Desmopres­ 4. B. sin is the drug of choice and bed alarms are the 5. B, D. treatment of choice. mebooksfree.com mebooksfree.com Alpha rhythm is seenmebooksfree.com when a person is awake with 14. mebooksfree.comC,D. mebooksfree.com mebooksfree.com eyes closed and his mind is wandering (having Benzodiazepines can be used in night terrors mental activity) and not when a person is sleep­ though usually no treatment is required. ing with eyes closed. Also, alpha rhythm is seen 15. B. in REM sleep. 16. B. The onset of narcolepsy is mostly in adolescence 6. D. See text. or young adulthood. There are sudden sleep 7. B. The sleep spindles and K complexes are seen in attacks which last for 10–20 minutes (and not stage II, NREM. more than 3 hours) and cataplexy is a feature. 8. A. 17. D. Modafinil is FDA approved for narcolepsy, shift 9. B. Somnambulism is usually seen in NREM III and work sleep disorder and as an adjunct in obstruc­ IV (slow wave sleep). 10. C. Nocturnal penile erections are a feature of REM tive sleep apnea. mebooksfree.com mebooksfree.comsleep. mebooksfree.com 18. mebooksfree.comC. Ropinirole is a dopamine agonist (D2,mebooksfree.com D3 recep­ mebooksfree.com 11. A. Teeth grinding or bruxism is seen in NREM III tors) and is approved for restless leg syndrome. and IV. 19. B. There is hypersexuality and not hyposexuality.

mebooksfree.com mebooksfree.com mebooksfree.com mebooksfree.com mebooksfree.com mebooksfree.com

mebooksfree.com mebooksfree.com mebooksfree.com mebooksfree.com mebooksfree.com mebooksfree.com

mebooksfree.com mebooksfree.com mebooksfree.com mebooksfree.com mebooksfree.com mebooksfree.com

mebooksfree.com mebooksfree.com mebooksfree.com mebooksfree.com mebooksfree.com mebooksfree.com mebooksfree.com mebooksfree.com mebooksfree.com mebooksfree.com mebooksfree.com mebooksfree.com

mebooksfree.com mebooksfree.com mebooksfree.com mebooksfree.com mebooksfree.com mebooksfree.com

Chapter 10 Sexual Disorders

Gender Identity Disorders there is no desire to permanently change the sexQ. There isno sexual arousalQ associated with cross mebooksfree.com mebooksfree.comGender is the sense of beingmebooksfree.com a male or a female. Mostly dressing.mebooksfree.com (Remember, in fetishistic mebooksfree.comtransvestism, mebooksfree.com the gender corresponds to the anatomical sex, (i.e. a man which is a type of paraphilia, the cross dressing is with male body organs, also psychologically considers associated with sexual arousal). himself as a male), however there might be a mismatch resulting in gender identity disorder. The following are Treatment: In patients who insist for sex change, sex types of gender identity disorder: reassignment surgery can be done. In a person born A. Gender identity disorder of childhood: It usually mani­ anatomically male, removal of penis, scrotum and tes­ fests in preschool years. The child shows preoccupa­ tes and construction of labia and vagina is done. In a tion with the dress and activities of the opposite sex person born anatomically female, bilateral mastectomy, (e.g. the male child insists on wearing skirts and frocks hysterectomy, removal of ovaries and con­ H and may play exclusively with dolls and reject the cars DSM-5 Update: In DSM-5, the diag- and other toys which are usually preferred by boys). struction of a neophal­ nosis of “gender dysphoria" is used in The child expresses the desire to be of the opposite sex lus (penis) is done. The place of DSM-4 diagnosis of "gender mebooksfree.com mebooksfree.com mebooksfree.com mebooksfree.comidentity disorder". mebooksfree.com mebooksfree.com and rejects behaviors, attire and attributes of his ana­ hormonal treatment tomical sex. Usually, there is no feeling of rejection of usually accompanies. the anatomical structures however in a small minority it may be present (e.g. the male child may repeatedly Disorders of Sexual Orientation assert that the penis and testicles are disgusting and It must be remembered that homosexuality is not a psy­ will disappear in due course of time). chiatric disorder (homosexuality is considered as a normal B. Transsexualism: In adolescents and adults, the symp­ variant, if it is ego syntonic, i.e. the individual accepts his toms are quite similar to gender identity disorder of sexual orientation) however ego dystonic homosexuality Q childhood. The patients manifest a desire to live (where in the individual doesn’t accepts his sexual ori­ and be treated as the other sex, usually accompanied entation and wants to change it) has been classified as Q by a discomfort with one’s anatomi­cal sex and a a disorder. mebooksfree.com mebooksfree.comdesire to changeQ it with themebooksfree.com help of a surgery or some mebooksfree.com mebooksfree.com mebooksfree.com other form of treatment. The patient frequently uses Disorders of Sexual Response the phrases like “I am a man trapped in body of woman”. The homosexual orientation is frequently Phases of Sexual Response Cycle present. Normally sexual response has been divided into four C. Dual-role transvestism: The patient wears the clothes phases. of opposite sex, to enjoy the temporary feelingQ of A. Desire: It is characterized by a desire to have sex (hypo­ belonging to the other sex. Unlike transsexualism, active sexual desire disorder is a disorder of this phase).

mebooksfree.com mebooksfree.com mebooksfree.com mebooksfree.com mebooksfree.com mebooksfree.com

mebooksfree.com mebooksfree.com mebooksfree.com mebooksfree.com mebooksfree.com mebooksfree.com mebooksfree.com mebooksfree.com mebooksfree.com mebooksfree.com mebooksfree.com mebooksfree.com

104 Review of Psychiatry

B. Excitement (arousal): This phase is characterized Treatment: The medications with best evidence by penile erection and vaginal lubrication. Other include PDE-5 inhibitorsQ (phosphodiesterase-5 changes such as nipple erection, enlargement of inhibitors like sildenafil, tadalafil and vardenafil, size of testes and elevation of testes, engorgement which facilitate blood flow into penis and enhance mebooksfree.com mebooksfree.comand thickening of labiamebooksfree.com minor and clitoris, and mebooksfree.comerection. The other medications whichmebooksfree.com can be used mebooksfree.com physiological changes like increased heart rate, include oral phentolamine (decreases sympathetic blood pressure and respiratory rate are also seen. tone and relaxes smooth muscles of corpora caver­ There is an associated subjective sense of pleasure nosa) and injectable and transurethral alprostadil. (erectile dysfunction is a disorder of this phase). Alprostadil contains naturally occurring prosta­ C. Orgasm: There is a peaking of sexual pleasure, glandin E and hence has vasodilator action. It can followed by release of sexual tension and ejaculation be injected into corpora cavernosa or administered of semen. In females, orgasm is characterized by intraurethrally. involuntary contraction of lower third of vagina Apart from medications, psychotherapy also plays and contractions from fundus downward to cervix. an important role. The most successful isdual- Q (premature ejaculation and anorgasmia are disorders sex therapy (or simply sex therapy) which was of this phase). developed by Masters and Johnson. This therapy mebooksfree.com mebooksfree.com mebooksfree.com mebooksfree.comtreats the “couple”Q and not the individualmebooksfree.comQ. The mebooksfree.com D. Resolution: The body goes back to the resting state. couple is taught ways to improve their communi­ There are disorders specific to each phase of sexual cation. The couple is also taught exercises which cycle as described below: increases the sensory awareness. These exercises A. Sexual desire disorders: It has been further subdivided are called, sensate focus exercises. Initially, the into two categories: hypoactive sexual desire disorder, couple is asked to touch, rub, kiss on each oth­ characterized by lack of desire for sexual activity and ers body parts, excluding breasts and genitals (this sexual aversion disorder, characterized by active aver­ stage is called nongenital sensate focus). In next sion and avoidance of sexual activity.­ The only FDA stage, the same activities are done on breasts and approved drug for treatment of hypoactive sexual genitals (called genital sensate focus). The whole desire disorder in females is flibanserin, which got purpose is to make the couple aware that pleasure approval in August 2015. Due to risk of severe hypo­ can be given and received by methods other than tension, flibanserin mebooksfree.com mebooksfree.com H mebooksfree.com mebooksfree.comsexual intercourse. The sex therapy mebooksfree.comis effective not mebooksfree.com should not be taken DSM-5 Update: In DSM-5, the diag- only for erectile dysfunction but other sexual dis­ concomitantly with nosis of sexual aversion disorder has orders like premature ejaculation. been removed. alcohol. Other techniques such as behavioral therapy, B. Disorders of excitement (arousal) phase: hypnotherapy and psychoanalysis have also been • Male erectile disorder (erectile dysfunction): It is used. characterized by recurrent or persistent inability • Female sexual arousal disorder: It is characterized to attain or to maintain the erection required for by inability to achieve adequate vaginal lubrication satisfactory sexual intercourse. Erectile dysfunction required for sexual intercourse. The management is usually caused by psycho­logical factors such as involves use of lubricants during the intercourse. anxiety and poor marital relation. C. Disorders of orgasm phase: The presence ofearly morning erections and • Premature ejaculation: It is characterized by a pattern mebooksfree.com mebooksfree.comerections during REM sleepmebooksfree.com (nocturnal erectionsQ) mebooksfree.comof persistent or recurrent ejaculation mebooksfree.comwith minimal mebooksfree.com are suggestive of psychogenic erectile dysfunction. sexual stimulation before or immediately after the Investigation such as penile plethysmography and vaginal penetra­ Q H nocturnal penile intumescence (NPT) are used tion. In DSM-5, the criterion for prema- to record nocturnal erections. The cause of ture ejaculation has been defined more clearly, and states that prema- The physical causes include vascular and neuro­ premature ejacu­ ture ejaculation is a pattern of ejacula- logical disorders like arteriolosclerosis and auto­ lation is usually tion within approximately one minute nomic neuropathy. psychogenic. following vaginal penetration.

mebooksfree.com mebooksfree.com mebooksfree.com mebooksfree.com mebooksfree.com mebooksfree.com

mebooksfree.com mebooksfree.com mebooksfree.com mebooksfree.com mebooksfree.com mebooksfree.com mebooksfree.com mebooksfree.com mebooksfree.com mebooksfree.com mebooksfree.com mebooksfree.com

Sexual Disorders 105

Treatment: Specific techniques have been described • Male orgasmic disorder (retarded ejaculation): It for the management of premature ejaculation. is characte­rized by recurrent delay or absence of These include: orgasm in males. It is less common than premature a. Squeeze techniqueQ: When the man gets the feel­ ejaculation and is treated with psychotherapy. mebooksfree.com mebooksfree.coming of impending ejaculation,mebooksfree.com the female partner D. Othermebooksfree.com disorders: mebooksfree.com mebooksfree.com (or the man himself) squeezes the coronal ridge • Dyspareunia: It is recurrent or persistent genital of glans, which results in inhibition of ejaculation. pain in either men or women, before, during or b. Stop-start technique (Semans technique): Here, after sexual intercourse. when the man gets the feeling of impending • Vaginismus: It is involuntary muscle constriction of ejaculation, the sex is stopped for some time and outer third of vagina which makes penile insertion once excitement has decreased, it is restarted. difficult. Vaginismus and dyspareunia frequently Apart from these techniques, sex therapy (as coexist. described earlier) is also an effective method of • NymphomaniaQ: treating premature ejaculation. H SSRIs (selective serotonin reuptake inhibitors) It is the term used DSM-5 Update: Genito-pelvic pain/ to describe exces­ penetration disorder is new in DSM-5 are also frequently used as they can delay the and represents a merging of the ejaculation. sive sexual desire DSM-4 categories of vaginismus and mebooksfree.com mebooksfree.com mebooksfree.com mebooksfree.comin females. dyspareunia, which weremebooksfree.com highly com­ mebooksfree.com • Female orgasmic disorder (anorgasmia): It is charac­ orbid and difficult to distinguish. terized by recurrent delay or absence of orgasm in • SatyriasisQ: It is females. It is a common sexual disorder in females the term used to describe excessive sexual desire and the treatment involves psychotherapy. in males.

QUESTIONS AND ANSWERS

QUESTIONS C. Sildenafil induced erection D. Squeeze technique mebooksfree.com mebooksfree.com 1. Most accurate treatmentmebooksfree.com of erectile dysfunction mebooksfree.com mebooksfree.com mebooksfree.com is: (PGI 2002) 5. Squeeze technique is used for: (AI 2000) A. Sildenafil A. Impotence B. Master and Johnson technique B. Premature ejaculation C. b-blockers C. Infertility D. Papaverine D. Priapism 2. Excessive sexual desire in males in known as: 6. A 30-year-old male presents to OPD with erectile (AIIMS May 2008) dysfunction. Basic screening evaluation is unre- A. Nymphomania B. Satyriasis markable. The next step in evaluation/manage- C. Tribadism D. Sadism ment should be: (AI 2008) 3. A homosexual person feels that he is imposed by A. Oral sildenafil titrate trial a female body and has persistent discomfort with B. Cavernosometry mebooksfree.com mebooksfree.comhis sex. Most likely diagnosismebooksfree.com is: (PGI 2003) mebooksfree.comC. Doppler study mebooksfree.com mebooksfree.com A. Gender identity disorder B. Transvestism D. Neurological testing C. Voyeurism D. Paraphilias 7. A 20-year-old girl Neelu enjoys wearing male 4. How to differentiate between psychological and clothes. Wearing male clothes gives her feeling of organic erectile dysfunction? (NEET/DNB) more confidence and after these episodes she is A. Nocturnal penile tumescence an absolutely normal girl. The likely diagnosis is: B. PIPE test (AIIMS 1997)

mebooksfree.com mebooksfree.com mebooksfree.com mebooksfree.com mebooksfree.com mebooksfree.com

mebooksfree.com mebooksfree.com mebooksfree.com mebooksfree.com mebooksfree.com mebooksfree.com mebooksfree.com mebooksfree.com mebooksfree.com mebooksfree.com mebooksfree.com mebooksfree.com

106 Review of Psychiatry

A. Transsexualism disorder. Most of the patients with gender iden­ B. Fetishism tity disorder have homosexual orientation. C. Dual role transvestism 4. A. Presence of early morning erections and erections D. Fetishistic transvestism during REM sleep (nocturnal erections) are mebooksfree.com mebooksfree.com mebooksfree.com mebooksfree.com mebooksfree.com mebooksfree.com 8. True about dual sex therapy is: (DNB June 2011) suggestive of psychogenic erectile dysfunction. A. Patient alone is not treated As during sleep, there is no anxiety, hence a B. Uses sildenafil patient with psychogenic erectile dysfunction C. It treats sexual perversion is able to have erections. Whereas, a patient D. It is used for people with dual gender identities with organic erectile dysfunction (due to vas­ cular or neurological causes ) won’t have erec­ ANSWERS tions even during sleep. Investigation such as penile plethysmography and nocturnal penile 1. A, B. intumescence (NPT) can be used to record The pharmacological treatment with best nocturnal erections. evidence in erectile dysfunction is phospho­ 5. B. Squeeze technique and stop-start techniques are diesterase-5 inhibitors like sildenafil. The used for treatment of premature ejaculation. mebooksfree.com mebooksfree.compsychotherapeuticmebooksfree.com technique which is most 6. mebooksfree.comA. In a young patient with negativemebooksfree.com screening, mebooksfree.com commonly used is Master’s and Johnson’s tech­ the most likely cause of erectile dysfunction is nique. psychogenic erectile dysfunction. He should be 2. B. Satyriasis is the condition of excessive sexual given a trial of oral sildenafil. desire in males while the same in females is 7. C. Here the person only enjoys wearing clothes of known as nymphomania. opposite sex and there is no discomfort with her 3. A. As mentioned in the question the person is own sex and there is no desire to be of other sex. uncomfortable with his sex and feels that he is Hence, it is a case of dual role transvestism. imposed by a female body (i.e. he is of another 8. A. In dual sex therapy, the couple is treated and not an sex), both are characteristics of gender identity individual. mebooksfree.com mebooksfree.com mebooksfree.com mebooksfree.com mebooksfree.com mebooksfree.com

mebooksfree.com mebooksfree.com mebooksfree.com mebooksfree.com mebooksfree.com mebooksfree.com

mebooksfree.com mebooksfree.com mebooksfree.com mebooksfree.com mebooksfree.com mebooksfree.com

mebooksfree.com mebooksfree.com mebooksfree.com mebooksfree.com mebooksfree.com mebooksfree.com mebooksfree.com mebooksfree.com mebooksfree.com mebooksfree.com mebooksfree.com mebooksfree.com

mebooksfree.com mebooksfree.com mebooksfree.com mebooksfree.com mebooksfree.com mebooksfree.com

Chapter 11 Child Psychiatry

ATTENTION DEFICIT HYPERACTIVITY Depending on the H DISORDER (ADHD)/HYPERKINETIC predominant symp- DSM-4 required the presence of mebooksfree.com mebooksfree.com mebooksfree.comtoms, threemebooksfree.com subtypes symptoms before agemebooksfree.com of 7 years, mebooksfree.com DISORDER however according to DSM-5 the have been defined. onset of symptoms should be before The DSM-5 uses the diagnosis of ADHD whereas in ICD- A. Combined presen- 12 yrs of age to make a diagnosis of ADHD. 10, the corresponding diagnosis is hyperkinetic disor­ tation der. It is a common neuropsychiatric disorder of child- B. Predominantly inattentive presentation hood, which is more prevalent in boys in comparison to C. Predominantly hyperactive/impulsive presentation. girls. The predominant symptoms and signs in the ADHD are as follows: Course A. Inattention: The child hasdifficulty in giving close Q attention to details, makes frequent mistakes in Around 50 percent of patients achieve remission before school work and other activities. The child is distracti­ puberty and early adulthood. Others achieve only partial bleQ and frequently shifts from one acti­vity to another remission and are at risk of developi2ng substance use Q mebooksfree.com mebooksfree.comas he loses interest in onemebooksfree.com task quickly. disordersmebooksfree.com (particularly­ alcoholism), antisocialmebooksfree.com persona­ mebooksfree.com B. Hyperactivity and Impulsivity: The child ishyper ­ lity disorder and mood disorders. active and appears restless. Teacher frequently com- plains that child keeps on roaming in the class and is Treatment excessively talkative and disturbs other studentsQ. ADHD is a serious medical illnessQ and must be promptly The child is also impulsiveQ and often blurts out treated. The pharmacological treatment is the mainstay of answer before question has been completed. He also the treatment. The following medications are used: has difficulty in waiting for his turn and often inter- rupts others or intrudes in others conversation. A. Stimulant medications: CNS stimulants are the Along with the core symptoms of inattention and first line drugs in the treatment of ADHD.Methyl­ hyperactivity/impulsivity, children with ADHD frequently phenidateQ is the drug of choice. Dexmethylphenidate mebooksfree.com mebooksfree.comshow destructive and aggressivemebooksfree.com behavior and are irri­ (containingmebooksfree.com only the d-enantiomer) hasmebooksfree.com also been mebooksfree.com table. used recently. Other stimulant medications used Neurological examination may reveal soft neurologi­ are dextroamphetamine, lisdexamphetamine and cal signsQ (the neurological soft signs are fine abnormali- “dextroamphetamine and amphetamine salt” com- ties found during detailed neurological examination such binations. Modafinil has also been used with varied as difficulty in copying age appropriate figures, difficulty success. in performing rapid alternating movements, difficulty in B. Nonstimulant medications: The nonstimulant medi- right left discrimination, etc). cations are used if stimulants are not effective or

mebooksfree.com mebooksfree.com mebooksfree.com mebooksfree.com mebooksfree.com mebooksfree.com

mebooksfree.com mebooksfree.com mebooksfree.com mebooksfree.com mebooksfree.com mebooksfree.com mebooksfree.com mebooksfree.com mebooksfree.com mebooksfree.com mebooksfree.com mebooksfree.com

108 Review of Psychiatry

contraindicated. They includeatomoxetine (norepi- have development of abnormal language such as nephrine reuptake inhibitor), clonidine, guanfacine, difficulty in making sentences properly (articula- venlafaxine and bupropion. tion difficulties) and pronoun reversals (using “me” Apart from medications, psychosocial interventions instead of “you”). mebooksfree.com mebooksfree.comsuch as social skill training, psychoeducationmebooksfree.com for parents, • mebooksfree.comRestricted, repetitive and stereotyped mebooksfree.combehavior: The mebooksfree.com behavioral therapy and cognitive behavioral therapy are activities and play of these children tend to be Q also effective in the management. repetitive and boring. They may show stereotyped behaviors like hand wringingQ, spinning and bang­ PERVASIVE DEVELOPMENTAL ing. These children are quite resistant to changes and may become extremely upset if their routines DISORDERS (AUTISM SPECTRUM are disturbed (e.g. bathing routine is changed or DISORDER) furnitures are rearranged in the room). These These are group of neurodevelopmental orneurobeha­ restrictive behaviors usually result from a lack of vioral disordersQ, which are characterized by disturbance imagination and creativity. of social interaction, abnormalities of communication Apart from these three characteristic features, the and restricted behaviors. The following are the subtypes patients with autism also have abnormal responses mebooksfree.com mebooksfree.comof pervasive developmental disorders:mebooksfree.com to mebooksfree.comstimuli. They may have a higher thresholdmebooksfree.com for pain mebooksfree.com A. Autism (Childhood autism, autistic disorder): It is and may show intense interest in some sounds (like a neurodevelopmental disorder (neurobehavioral dis- that of a ticking watch) and may totally ignore other order) with a strong genetic basis. The onset is before sounds. They may also have self destructive behaviors the age of 3 years. Chromosome 7,2,4,15 and 19 have like head banging, biting , scratching, etc. been found to contribute to the disorder. Fragile X Precocious skills or islets of precocity: Some indi- syndromeQ, tuberous sclerosis, congenital rubella and viduals with autism may have skills in certain areas, phenylketonuria are associated with autism and are which are much higher than their normal peers. For found with high frequency in children with autistic example, hyperlexia (early ability to ready very well), disorders. Around 70%Q of children with autism have extremely good rote memory or calculating ability, comorbid mental retardation. The prevalence of peri- etc. natal insults like birth asphyxia has also been found Treatment: Educational interventions such as a mebooksfree.com mebooksfree.comto be higher in children withmebooksfree.com autism. Following are the structuredmebooksfree.com classroom teaching along mebooksfree.comwith use of mebooksfree.com symptoms: behavioral therapy is the recommended treatment. • Impairment in social interaction: The patients with The role of medications is limited. Atypical antipsy- autism have impaired reciprocal social skills. As chotics such as risperidone and aripiprazole have infants they have poor eye contactQ, lack social been used to reduce aggressive and self injurious smileQ and anticipatory posture (the posture which behavior. the kid assumes when he wants to be picked up). B. Rett’s disorder (Rett’s syndrome): Earlier it was They may have poor attachment to their parents believed that Rett’s disorder occurs exclusively in and other important persons and may not acknow­ femalesQ, however of late males with similar presen­ ledge their presence (e.g. they won’t come running tation have been described. It is characterized by nor- to meet when the father returns to home after mal development till the age of 5 months. Between office). However, if the routine of these children is 5-48 months , the child starts to lose acquired hand mebooksfree.com mebooksfree.comdisturbed (e.g. if someonemebooksfree.com rearranges the furniture skillsmebooksfree.com (such as fine motor skills) and theremebooksfree.com is loss of mebooksfree.com in their room), they may show excessive reactionQ. acquired speech. Also, there is deceleration of head When they grow up, they may have difficulty in mak- circumference producing microcephalyQ. The child ing friends and getting into a romantic relationship. gradually develops stereotyped hand movements • Impairment of communication and language: These such as hand wringing, licking or biting of fingers. children usually have significant delay in lan­ The language function remains impaired and there guage milestonesQ, whereas the motor milestones is also loss of social interaction. The child also deve­ are usually normally achieved. The patients also lops poorly coordinated gait or trunk movements.

mebooksfree.com mebooksfree.com mebooksfree.com mebooksfree.com mebooksfree.com mebooksfree.com

mebooksfree.com mebooksfree.com mebooksfree.com mebooksfree.com mebooksfree.com mebooksfree.com mebooksfree.com mebooksfree.com mebooksfree.com mebooksfree.com mebooksfree.com mebooksfree.com

Child Psychiatry 109

Along with these symptoms around 75% of children requests and rules, frequent loss of temper and often have seizures. The disorder is usually progressive and deliberately annoying adults. However unlike conduct treatment is symptomatic. disorder, there are no serious violations like theft, fire C. Childhood disintegrative disorder (Heller’s syn- setting, destruction, etc. mebooksfree.com mebooksfree.comdome): It is characterisedmebooksfree.com by normal development Themebooksfree.com management for both involves familymebooksfree.com interven- mebooksfree.com till the age of 2 H tion and behavioural therapy. In some cases, low dose years. Between 2-10 DSM-5 Update: In DSM-5, the term antipsychotics have been found to be effective. years there is loss pervasive developmental disorder has of acquired motor been replaced by autism spectrum disorder. Also, all the subtypes, such LEARNING DISORDERS (SPECIFIC skills, social skills, as autism, Rett’s disorder, Aspergers language skill and syndrome have been removed. Now, DEVELOPMENTAL DISORDERS OF autism spectrum disorder is consi­ bowel or bladder dered as a continuum with difference SCHOLASTIC SKILLS) control. The child in severity, rather than presence of subtypes. These developmental disorders are characterized by develops the three significant­ impairment in one or more of the scholas­ core symptoms of impaired communication, impaired tic skillsQ which are out of proportion to the intellectual social interaction and repetitive, stereotyped behavior. functioning of the child. For example, a child may pre- The course is usually mebooksfree.com mebooksfree.com H mebooksfree.comsent withmebooksfree.com significant difficulty in readingmebooksfree.com while having mebooksfree.com progressive though DSM-5 Update: In DSM-5, in normal writing and arithmetic skills and a normal IQQ. some patients may autism spectrum disorders, language dysfunction has been removed as a Depending on the symptoms, the subtypes have been show improvement. core criterion. Only impaired social The treatment is interaction and repetitive, restrictive described. symptomatic. behaviors form the core criterion now. A. Specific reading disorder (Dyslexia): The child’s D. Asperger’s syndrome: It is characterized by impair- reading performance is significantly impaired and he ment of social interaction and restricted, repetitive may make errors while reading, may have slow read- and stereotyped behavior. However no language delay ing speed or may have difficulty in comprehension. or disturbance is seen. The treatment is usually sup- B. Disorder of written expression (specific spelling dis- portive. order): The child may make frequent spelling mis- takes, errors in grammar and punctuations and may mebooksfree.com mebooksfree.comCONDUCT DISORDERmebooksfree.com AND havemebooksfree.com poor hand writing. mebooksfree.com mebooksfree.com OPPOSITIONAL DEFIANT DISORDER C. Specific disorder of arithmetic skills: The area of impairment is arithmetics. A. Conduct disorder: It is characterized by repetitive D. Mixed disorders of scholastic skills: There is impair- and persistent pattern of disregard for rights of ment in reading, writing and arithmetics combined. othersQ and aggressiveQ and dissocial behaviorQ, Apart from the above mentioned symptoms, the child such as excessive levels of fighting or bullying, cruelty may have associated problems such as inattention, hyper- to animals or other people, severe destruction of activity and emotional disturbances. pro­perty, fire setting, stealing , truancy from school, repeated lyingQ, frequent running from school and MENTAL RETARDATION home, defiance of authority figures and a pattern of disobedience. Conduct disorder is frequently Mental retardation is a condition characterized by incom- mebooksfree.com mebooksfree.comassociated with unsatisfactorymebooksfree.com family relationships plete developmentmebooksfree.com of intellectual functionsmebooksfree.com and adaptive mebooksfree.com and failure at school. These children may later on skills (skills which help an individual live a successful develop antisocial personality disorder (dissocial life). The intelligence is usually measured by calculating personality disorder). the Intelligence Quotient (IQ). B. Oppositional defiant disorder: It is less severe than IQ = Mental age/Chronological age × 100Q conduct disorder and is characterized by persistently In this formula, the maximum denominator is 15, even negativistic and defiant behavior such as frequent if assessment of an older individual is being performed. arguing with adults, refusal to comply with adults Mental retardation is diagnosed if the IQ is less than 70.

mebooksfree.com mebooksfree.com mebooksfree.com mebooksfree.com mebooksfree.com mebooksfree.com

mebooksfree.com mebooksfree.com mebooksfree.com mebooksfree.com mebooksfree.com mebooksfree.com mebooksfree.com mebooksfree.com mebooksfree.com mebooksfree.com mebooksfree.com mebooksfree.com

110 Review of Psychiatry

Category IQ An earlier classification of retardation used the words Normal 90-109 “idiots, imbecile, and moron”. Borderline 70-89 Term IQ range Mild mental retardation 50-69 mebooksfree.com mebooksfree.com mebooksfree.com Moronmebooksfree.com mebooksfree.com51-70 mebooksfree.com Moderate mental retardation 35-49 Imbecile 26-50 Severe mental retardation 20-34 Idiot 0-25 Profound mental retardation < 20

The level of functioning varies in different severity of men- The most common H tal retardation. The following table summarizes the same. chromosomal cause of DSM-5 Update: In DSM-5, the mental retardation is diagnosis of mental retardation has Mental been replaced with “intellectual dis- age as Educational Down syndromeQ fol- ability”. Category Class adults achievement Life Work lowed by fragile-X syn­ Mild MR Educable 9-12 yrs Upto 6th Independent Unskilled Q class living or semi- drome . skilled work Moderate Trainable 6-8 yrs Upto 2nd Needs some Unskilled Behavioral problems in mental retardation: The patients mebooksfree.com mebooksfree.comMR classmebooksfree.comeducation or semi- mebooksfree.com mebooksfree.com mebooksfree.com skilled with mental retardation may have maladaptive behavior work such as aggression, self injurious behaviors, hyperacti­ Severe Dependent 3-6 yrs No formal Needs Simple vity, etc. These behaviors can usually be modified using MR education attention task-under supervision behavioral therapy techniques like contingency manage­ Profound Needs life < 3 yrs No formal Needs None mentQ, in which the desired behaviors are rewarded and MR support education continuous supervision undesired behaviors are punished.

QUESTIONS AND ANSWERS mebooksfree.com mebooksfree.com mebooksfree.com mebooksfree.com mebooksfree.com mebooksfree.com QUESTIONS 3. A 10-years-old child presents with hyperactivity and inatten­tion. Parents are extremely worried, 1. Which of the following is not seen in a hyperkinetic what would you say to the parents? child? (DNB 1993, AI 1991) (AIIMS Nov 2008) A. Aggressive outbursts A. It is a normal behavior B. Decreased attention span B. Child has a behavioral problem and should C. Left to right disorientation receive behavior therapy D. Soft neurological signs C. Child has a serious problem and should receive medical therapy 2. A 9-year-old child disturbs other people, is des­ D. There should be a change in environment tructive, interferes when two people are talking, mebooksfree.com mebooksfree.comdoes not follow instructionsmebooksfree.com and cannot wait for 4. mebooksfree.comADHD in childhood can lead to whatmebooksfree.com in future: mebooksfree.com his turn while playing a game. He is likely to be (PGI 2000) suffering from: (AIIMS Nov 2005) A. Schizophrenia B. Alcoholism A. Emotional disorders C. Intellectual changes D. Antisocial behavior B. Behavioral problems 5. Following drugs are used in the treatment of C. No disorder ADHD: (DNB NEET 2014-15) D. Attention deficit hyperactivity disorder A. Amphetamine

mebooksfree.com mebooksfree.com mebooksfree.com mebooksfree.com mebooksfree.com mebooksfree.com

mebooksfree.com mebooksfree.com mebooksfree.com mebooksfree.com mebooksfree.com mebooksfree.com mebooksfree.com mebooksfree.com mebooksfree.com mebooksfree.com mebooksfree.com mebooksfree.com

Child Psychiatry 111

B. Modafinil C. Impaired folate level C. Methylphenidate D. A socioeconomic hazard D. All of the above E. Result of wrong parenting mebooksfree.com mebooksfree.com 6. Drug(s) used in treatmentmebooksfree.com of attention-deficit 12. mebooksfree.comAutism is: mebooksfree.com(PGI 2000) mebooksfree.com hyperactivity disorder: (PGI Dec 2008) A. Neurodevelopmental disorder A. Atomoxetine B. Methylphenidate B. Social and language communication problem C. Dexmethylphenidate D. Quetiapine C. Metabolic disease E. Dextroamphetamine D. Mainly due to hypothalamus damage 7. Not an associated comorbid condition in children 13. A 6-year-old child with history of birth asphyxia with hyperkinetic attention deficit disorder is: does not communicate well, has slow mental and (DNB Dec 2010) physical growth, does not mix with people, has A. Elimination disorder B. Anxiety disorder limited interests and gets widely agitated if dis­ C. Sleep disorder D. Language disorder turbed. Diagnosis is: (AIIMS 2001) 8. A neurodevelopmental disorder which is charac­ A. Hyperkinetic child mebooksfree.com mebooksfree.comterized by impaired socialmebooksfree.com interaction, impaired mebooksfree.comB. Autistic disorder mebooksfree.com mebooksfree.com verbal and nonverbal communication, and C. Attention deficit disorder restricted and repetitive behavior is description D. Mixed receptive–expressive language disorder for: (DNB NEET 2014-15) A. Autism 14. A girl with normal milestones spend her time B. Anxiety disorder seeing her own hand and does not interact with C. Antisocial personality disorder others. What is the likely diagnosis: (AIIMS 2008) D. Paranoid schizophrenia A. ADHD B. Autism C. Asperger’s syndrome D. Rett’s disorder 9. 10-years-old child presents with impaired social interaction, impaired communication and stereo­ 15. A 2-year-old girl child is brought to the out patient typed behavior. He has normal IQ and language with features of hand wringing stereotyped move­ skills. What is the most probable diagnosis? ments, impaired language and communication mebooksfree.com mebooksfree.com mebooksfree.com(DNB NEET 2014-15) mebooksfree.comdevelopment, breath holding spells,mebooksfree.com poor social mebooksfree.com A. Asperger’s syndrome skills and deceleration of head growth after six B. Autism months of age. The most likely diagnosis is: C. Rett syndrome (AIIMS Nov 2003) D. Childhood depression A. Asperger syndrome B. Rett’s syndrome 10. A 3-year-old child has normal developmental C. Fragile X-syndrome D. Cotard syndrome milestones except delayed language development 16. Which of the following is not seen in autism? (poor speech development). He has difficulty (AIIMS Nov 2014) in concentration, communication, and making A. 2/3rd patients are mentally retarded friends (i.e. he has no friends) and spends time see­ B. Poor eye contact ing his own hands. The most probable diagnosis is: (AI 2012, AIIMS Nov 2006) C. Language is impaired mebooksfree.com mebooksfree.comA. Autism mebooksfree.com mebooksfree.comD. Abnormal dermatoglyphics mebooksfree.com mebooksfree.com B. ADHD 17. A child with pervasive developmental disorder C. Specific learning disability will have all except: (AIIMS Nov 2015) D. Mental retardation A. Stereotyped behavior 11. Infantile autism is characterized by:(PGI Dec 2004) B. Reduced social interaction A. Impaired vision C. Poor language skills B. Impaired neurobehavioral development D. Impaired cognition

mebooksfree.com mebooksfree.com mebooksfree.com mebooksfree.com mebooksfree.com mebooksfree.com

mebooksfree.com mebooksfree.com mebooksfree.com mebooksfree.com mebooksfree.com mebooksfree.com mebooksfree.com mebooksfree.com mebooksfree.com mebooksfree.com mebooksfree.com mebooksfree.com

112 Review of Psychiatry

18. Which of the following disease is seen only in B. Can study upto 8th standard females? (DNB Dec 2011) C. Can follow simple verbal commands A. Autism B. Asperger’s syndrome D. Can handle money C. Rett’s disease D. Colard disease mebooksfree.com mebooksfree.com mebooksfree.com mebooksfree.comE. Recognize family members mebooksfree.com mebooksfree.com 19. Rett’s syndrome is characterized by all except: 27. All of the following statements about ‘Imbecile’ (DNB NEET 2014-15, AIIMS 2013) are true, except: (AI 2011) A. Regression of acquired skills A. IQ is 50-60 B. Breath holding spells B. Intellectual capacity is equivalent to a child of C. Autistic behavior 3-7 years of age D. Macrocephaly C. Impaired self care 20. IQ is: (DNB NEET 2014-15) D. Condition usually congenital or acquired at an A. Mental age/chronological age × 100 early age B. Chronological age/mental age × 100 28. X-linked disease leading to mental retardation is: C. Mental age + chronological age × 100 (PGI 2000) D. Mental age – chronological age × 100 A. Myotonic dystrophy B. Fragile X-syndrome mebooksfree.com mebooksfree.com 21. According to Wechslermebooksfree.com intelligence scale scoring, mebooksfree.comC. Tuberous sclerosis D. Phenylketonuriamebooksfree.com mebooksfree.com average IQ of a normal child is: (AIIMS 2013) 29. Best therapy suited to teach daily life skill to a A. 50 B. 75 mentally challenged child: (AIIMS May 2011, 2009) C. 90 D. 111 A. CBT (Cognitive behavior therapy) 22. A 16-year-old male is found to have a mental age B. Contingency management of 9 years on IQ testing. He has: (AIIMS May 2005) C. Cognitive reconstruction A. Mild mental retardation D. Self instruction B. Moderate mental retardation 30. A 14-year-old boy is not able to get good grades C. Severe mental retardation in 9th standard exam. But he is very sharp and D. Profound mental retardation intelligent. Best test to diagnose his problem: 23. Which of the following score is not included in (AIIMS 2012) mebooksfree.com mebooksfree.commild mental retardation?mebooksfree.com (PGI May 2012) mebooksfree.comA. Child behavior checklist mebooksfree.com mebooksfree.com A. 85 B. 50 B. Bhatia’s battery C. 45 D. 75 C. Specific learning disability test E. 65 D. Child behavior battery 24. A patient with IQ 30 will be diagnosed with: 31. A child finds difficulty to spell and read, otherwise (DNB NEET 2014-15) his IQ is normal, interacts well with parents and A. Mild mental retardation friends. Vision is normal. Most probable diagnosis B. Moderate mental retardation of the condition is? (DNB June 2011) C. Severe mental retardation A. ADHD B. Dyslexia D. Profound mental retardation C. Autism D. Asperger syndrome 25. True about mental retardation: (PGI Nov 2011) 32. A 14-year-old boy has difficulty in expressing A. More common in females than males himself in writing and makes frequent spelling mebooksfree.com mebooksfree.comB. Severe MR is IQ < 20mebooksfree.com mebooksfree.com mebooksfree.com mebooksfree.com mistakes. He passes his examination with poor C. Antenatal factor can cause mental retardation marks. However his mathematical abi­lity and D. Common cause is down’s syndrome social adjustment are appropriate for his age. E. Life long inability to learn and progress Which of the following is the most likely diagnosis? 26. In a child with IQ 50, which of the following is (AIIMS Nov 2004) true: (PGI 2001) A. Mental retardation A. Can look after himself independently B. Specific learning disability

mebooksfree.com mebooksfree.com mebooksfree.com mebooksfree.com mebooksfree.com mebooksfree.com

mebooksfree.com mebooksfree.com mebooksfree.com mebooksfree.com mebooksfree.com mebooksfree.com mebooksfree.com mebooksfree.com mebooksfree.com mebooksfree.com mebooksfree.com mebooksfree.com

Child Psychiatry 113

C. Lack of interest in studies adversely affect his future life. Also, medications D. Examination anxiety like methylphenidate are the first line treatment. 33. A boy presents with history of abnormal excessive 4. B, D. Kindly note, that few books have also mentioned blinking and grunting. He says he has no control mebooksfree.com mebooksfree.com mebooksfree.com mebooksfree.comintellectual changes as an answer.mebooksfree.com This is not mebooksfree.com over his symptoms which have risen in frequency true. Though children with ADHD tend to have of late. This has started affecting his social life and lower educational achievements, however it is is making him depressed. Which of the following not because of any intellectual impairment but medications should be used in the management? because of poor attention and hyperactivity. (AIIMS May 2015) 5. D. A. Carbamazepine B. Imipramine 6. A, B, C, E. C. Risperidone D. Methylphenidate See text 34. Appetite for nonnutritive substances is called: 7. A. Elimination disorders are not a common comor- (DNB NEET 2014-15) bidity in ADHD. A. Pica B. Anorexia 8. A. C. Bulimia D. Binge 9. B mebooksfree.com mebooksfree.com 35. Scholastic performancemebooksfree.com is impaired in all of the 10. mebooksfree.comA. The child has all the three core featuresmebooksfree.com of autism, mebooksfree.com following except: (AI 2012) impairment in social interaction (difficulty in A. Attention deficit hyperactivity disorder making friends), impaired communication and B. Specific learning disorder repetitive, stereotyped behavior (spends most C. Anxiety time seeing own hands). D. PICA 11. B, D. Autism is a neurodevelopmental or neurobeha­ 36. Conduct disorder in a child manifests with: vioral disorder. It causes socioeconomic pro­ (PGI 2001) blems as a majority of autistic patients remain A. Disregard for right of others dependent on others however use of term B. Doesn’t care for authority “hazard” is a bit insensitive here. C. Backward in studies 12. A,B. mebooksfree.com mebooksfree.comD. Decreased head circumferencemebooksfree.com 13. mebooksfree.comB. The history of poor social interactionmebooksfree.com and mebooksfree.com E. Steals things restricted behaviors along with history of agitation when disturbed supports the diagnosis of autism. ANSWERS Around 70% of children with autism have comorbid mental retardation. The prevalence 1. C. The best answer here would be left to right dis­ of perinatal insults like birth asphyxia has been orientation. Please remember, in ADHD, “left to found to be higher in children with autism. right discrimination” difficulties can be found. 14. B. The history of restricted behaviors and poor social However, the term “left to right disorientation” interaction is suggestive of autism. is used for describing gross inability to distin- 15. B. The deceleration of head growth after 6 months guish left from right and is usually a feature of of age followed by repetitive, stereotyped Gerstmann’s syndrome. behavior (wringing hand movements), impaired mebooksfree.com mebooksfree.com 2. D. This child has symptomsmebooksfree.com of hyperactivity and mebooksfree.comcommunication and poor socialmebooksfree.com interaction is mebooksfree.com impulsivity and the most likely diagnosis would suggestive of Rett’s syndrome. be attention deficit hyperactivity disorder. 16. D. 3. C. The symptoms are suggestive of ADHD. ADHD 17. D. is a serious medical problem and should be 18. C. Rett’s syndrome was earlier believed to occur treated properly. The symptoms of ADHD inter- exclusively in females, however of late males fere with education of child, and if not treated with similar presentation have been described. child’s education may suffer greatly and will 19. D.

mebooksfree.com mebooksfree.com mebooksfree.com mebooksfree.com mebooksfree.com mebooksfree.com

mebooksfree.com mebooksfree.com mebooksfree.com mebooksfree.com mebooksfree.com mebooksfree.com mebooksfree.com mebooksfree.com mebooksfree.com mebooksfree.com mebooksfree.com mebooksfree.com

114 Review of Psychiatry

20. A. money, can have an independent living, study till 21. C. According to Wechsler intelligence scale, the 6th class. following is the classification. 27. A. The IQ of imbeciles is between 26-50. Hence, mebooksfree.com mebooksfree.comIQ range IQmebooksfree.com classification mebooksfree.commost of their features would correspondmebooksfree.com to that mebooksfree.com 130 and above Very superior of moderate mental retardation. 120-129 Superior 28. B. 29. B. See text. 110-119 High average 30. C. The history of poor academic performance 90-109 Average despite good intelligence should raise suspicion 80-89 Low average of learning disorders (specific learning disabi­ 70-79 Borderline lity). Hence, he should take a specific learning 69 and below Extremely low disability test to rule out the same. 22. A The formula for IQ is mental age/chronological 31. B. The history of difficulty in reading and spelling × 100. However, please remember that the maxi- mistakes in presence of normal IQ is suggestive of mum denominator can be 15. In this case 9/15 X learning disorders (specific learning disability). mebooksfree.com mebooksfree.com100 = 60. Hence, it willmebooksfree.com come under the category 32. mebooksfree.comB. Scholastic difficulty in a particularmebooksfree.com skill (written mebooksfree.com of mild mental retardation. expression) is suggestive of specific learning dis- 23. A, C, D. ability. The range for mild mental retardation is IQ from 33. C. The history of motor tics (abnormal excessive 50-69. blinking) and vocal tics (grunting) is suggestive 24. C. of tics disorder (possibly tourette syndrome). 25. C, D. Antipsychotics like haloperidol and risperidone Mental retardation is more common in boys, are the drugs of choice. severe MR is IQ < 35 and patients with mild and moderate MR can learn. 34. A. 26. A, C, D, E. 35. D. IQ of 50 corresponds to mild mental retardation. 36. A, B, C, E. mebooksfree.com mebooksfree.comPeople with mild mentalmebooksfree.com retardation can handle mebooksfree.com See text. mebooksfree.com mebooksfree.com

mebooksfree.com mebooksfree.com mebooksfree.com mebooksfree.com mebooksfree.com mebooksfree.com

mebooksfree.com mebooksfree.com mebooksfree.com mebooksfree.com mebooksfree.com mebooksfree.com

mebooksfree.com mebooksfree.com mebooksfree.com mebooksfree.com mebooksfree.com mebooksfree.com mebooksfree.com mebooksfree.com mebooksfree.com mebooksfree.com mebooksfree.com mebooksfree.com

mebooksfree.com mebooksfree.com mebooksfree.com mebooksfree.com mebooksfree.com mebooksfree.com

Chapter 12 Psychoanalysis

The term “psychoanalysis” was coined by started treating hysterical patients, wherein he FreudQ who is also known as “father of psychoanalysis”Q. would try to retrieve the unconscious memories during mebooksfree.com mebooksfree.comFreud (1856-1939)Q was bornmebooksfree.com in Freiburg, Moravia (now the treatmentmebooksfree.com procedure. Freud developedmebooksfree.com a technique mebooksfree.com in Czech republic) and lived most of his life in ViennaQ. called “free association” in which the patient was asked He died in London in 1939. to say whatever came into their minds without censoring ­ Psychoanalysis is a theory which states that the child­ their thoughts. With the help of this technique, Freud hood experiences and memories and unconscious men­ was able to gain access to unconscious memories, which tal activity (activity of mind which we are not aware of) would come out as patient would start saying all that came plays an important role in determining human behavior into their minds and would not try to stop any thought. and emotions and also in the development of psychiatric ­ Freud also gave a lot of importance to slips of the tongue disorders. The term “psychoanalysis” is used not only to (which he called parapraxisQ). Freud believed that these refer to this theory but also for the treatment method “slips of tongues” were not simple mistakes, and that which is based on this theory. these slips actually conveyed important informationQ mebooksfree.com mebooksfree.com The theory of psychoanalysismebooksfree.com was developed by about mebooksfree.comwhat was going on in the unconsciousmebooksfree.com mind. mebooksfree.com Freud while working with patients of hysteria (the term The psychoanalytic treatment provided by Freud also ­hysteria is no longer used, these patients will get a used the principles of and countertransfe­ diagnosis of “dissociative disorder” according to cur- rence. rent classification). In particular Freud came to know TransferenceQ is the feeling that the patient develops about a patient Anna O, who had developed multiple for the doctor. This feeling is a combination of the feelings unexplained neurological symptoms including paralysis patient had for figures from the past and the real feeling of limbs, after the death of her father. Whenever she was for the clinician. For example, if the doctor reminds the able to recall how a particular symptom originated, that patient of his dominating and insensitive father, the patient symptom would improve. For example, once she was able will develop a negative feeling for the doctor, despite the to recall that on one occasion while she was sitting at her fact that doctor has not done anything to offend him. sick father’s bedside, she had a daydream that a snake CountertransferenceQ is the feeling that the clinician mebooksfree.com mebooksfree.comwas crawling towards her fathermebooksfree.com and while she wanted developsmebooksfree.com for the patient. mebooksfree.com mebooksfree.com to ward off the snake she couldn’t do it as her arm had gone into sleep. As soon as Anna O, was able to recall Topographical Theory of Mind this event, the paralysis of her arm improved. This case In 1900Q, Freud published a book called “The interpre- provided Freud a strong demonstration, that unconscious tation of dreams”Q. In this book, Freud said that dreams memories (memories which an individual has forgotten, were meaningful and by understanding dreams, one can but which are still present in the ) can understand about the unconscious mind of an individual. result in development of symptoms. In this book, Freud proposed a theory of mind, called the mebooksfree.com mebooksfree.com mebooksfree.com mebooksfree.com mebooksfree.com mebooksfree.com

mebooksfree.com mebooksfree.com mebooksfree.com mebooksfree.com mebooksfree.com mebooksfree.com mebooksfree.com mebooksfree.com mebooksfree.com mebooksfree.com mebooksfree.com mebooksfree.com

116 Review of Psychiatry

topographical theory of mind. According to this theory consequences). The primary process thinking is illogical the mind can be divided into three regions: and contradictory. A. The conscious B. The Structural Theory of Mind mebooksfree.com mebooksfree.com C. The unconscious mebooksfree.comLater inmebooksfree.com his life, Freud replaced the topographicalmebooksfree.com theory mebooksfree.com A. The conscious: It is the part of mind which is acces­ of mind with a newer theory, called the structural theory sible to us. We are aware of the contents of conscious of mind. According to this theory, there are three com­ mind. Everything you know about yourself is a part of ponents of mind: id, ego and superego. conscious mind. A. Id: It is the most primitive part of mind with which B. The preconscious: The content of preconscious an infant is born. Id consists of the instinctual drives. mind are not normally available to us, but they can It is that part of mind which wants to have pleasure be recalled or brought into awareness by focusing and that too immediately. Id doesn’t care about the attention. For example, you may not be aware of the external word or any consequences. Id hence works appearance of your 5th class teacher, however if you on “pleasure principal”. Id uses the primary process try to focus and remember hard, you might be able thinking. Id is completely in the unconscious domain mebooksfree.com mebooksfree.comto recall her appearance. mebooksfree.comThe preconscious separates of mebooksfree.commind. mebooksfree.com mebooksfree.com the conscious and unconscious mind. The precon­ B. Ego: Ego is that part of mind which deals with the scious mind has a barrier, called repression, which external world. The part of your mind which is reading normally doesn’t allow the contents of unconscious this book is “ego”. Apart from dealing with the external mind to reach the conscious mind. If any uncon­ world, another important function of ego is to deal scious memory has to reach the conscious awareness, with the “id” and “superego” and maintain a balance it must find a way to overcome the force of “repres­ between the two and the external word. Since, the ego sion”. Freud reported that during sleep, the repression maintains a balance and helps in dealing with the rea­ force becomes lax, and many unconscious memories lities of the outside world, it is said to work on “reality and desires are able to reach the conscious in the principal”. Ego is said to be the “executive organ” of form of dreams. That’s why Freud believed that the the mind. Ego has both conscious and unconscious interpretation of dreams can reveal the contents of components. The “defense mechanisms” reside in the mebooksfree.com mebooksfree.comunconscious memories andmebooksfree.com desires. Further, when a unconsciousmebooksfree.com component of ego. mebooksfree.com mebooksfree.com person indulges in “free association”, few unconscious C. Superego: It is that part of our mind, which wants to contents are able to cross the barrier of repression and follow the moral principles and do the right thing. are able to come out in the form of “slips of tongue”. The voice of conscience, which scolds you, when you C. The unconscious: The unconscious mind is not acces­ are not studying, comes from superego. Superego is sible to an individual. The unconscious mind contains, mostly unconscious, but also has a conscious com- the instinctual drives (i.e. the drives and desires one is ponent. born with) such as sexual instinct and aggressive instinct. To understand how these components work, an exam­ Further, distressing childhood memories and distressing ple can be illustrated. While you are studying, your id desires are also buried inside the unconscious. These wants you to throw away the books and instead go out contents are not available to the conscious mind due to and have fun and indulge in some pleasurable activity. the barrier of “repression” Freud believed that by not On the other hand, your superego wants you to study very mebooksfree.com mebooksfree.comallowing these memories tomebooksfree.com reach conscious, repression hard withoutmebooksfree.com taking many breaks and stay mebooksfree.comaway from all mebooksfree.com causes development of psychiatric symptoms and disor­ distractions. Finally, your ego does a balancing act and ders. you decide that you will study for two hours and after The unconscious mind is characterized by “primary that you will take a break and will watch a movie. This is process thinking”Q. This is primitive way of thinking in how, ego always keeps a balance. which the mind wants immediate ‘wish fulfillment” and As mentioned in this example, conflicts keep on going instinctual discharge (wants all desires and instincts in the mind (between id, ego and superego) and these to be fulfilled immediately without considering the unconscious conflictsQ in the mind are believed to cause mebooksfree.com mebooksfree.com mebooksfree.com mebooksfree.com mebooksfree.com mebooksfree.com

mebooksfree.com mebooksfree.com mebooksfree.com mebooksfree.com mebooksfree.com mebooksfree.com mebooksfree.com mebooksfree.com mebooksfree.com mebooksfree.com mebooksfree.com mebooksfree.com

Psychoanalysis 117

psychiatric disorders according to the psychodynamic (or dropped the glass. Here, the child was able to express psychoanalytic) theories. his anger indirectly by dropping the glass. E. Regression: Attempt to return to an earlier phase of Defense Mechanisms development (i.e. childhood) to avoid the tensions mebooksfree.com mebooksfree.com mebooksfree.com andmebooksfree.com conflicts of present phase of developmentmebooksfree.com (i.e mebooksfree.com An important function adulthood). For example, extremely stressed because of ego is to prevent a H All the defense mechanisms oper- of an upcoming entrance examination, a medical build up of excessive ate at an unconscious level (except, suppression which is a conscious and students goes to a park and starts playing cricket and unbearable anxi­ voluntary defense mechanism). along with the children. Regression is involved in ety. Many unaccep­ development of neurosisQ. table urges, if they reach the conscious awareness, can F. Projective identification: In this defense mechanism, produce excessive anxiety. Defense mechanisms are the intolerable aspects of self are projected on to another tools used by the “ego” to prevent the development of person, that person is induced to play the projected excessive anxiety. The defense mechanisms have been part and the two persons than act in unison. For divided into four groups: narcissistic, imma­ture, neurotic ­ example, a wife who has lots of aggression can and mature defense mechanisms. Following are the project her aggression on to the husband, and make mebooksfree.com mebooksfree.comimportant defense mechanisms:mebooksfree.com himmebooksfree.com behave in an aggressive mannermebooksfree.com and finally mebooksfree.com a system develops where the husband indulges in Narcissistic Defenses aggression and wife is the recipient of aggression. A. Denial: It is refusal to acknowledge the reality. The Please remember all of this happens unconsciously person continues to behave as if nothing has hap­ without entering into awareness of either the wife pened. For example, a mother refused to accept that or the husband. Projective identification is seen in her seven year old son died in an accident and insists patients with borderline personality disorder. that he will be back for dinner. B. Projection: Projecting “own” unacceptable feeling Neurotic Defenses about others, on to others. For example, a husband G. Displacement: Shifting emotions about one object/ with an unacceptable wish of indulging in infidelity, individual onto another object/individual. For exam­ starts accusing his wife of indulging in infidelity. ple, after being scolded by his consultant, a senior Here, the husband has “projected” his own wish on resident comes to the ward and started shouting at mebooksfree.com mebooksfree.comto the wife. This defense mebooksfree.commechanism is responsible for themebooksfree.com intern. Here, actually the senior residentmebooksfree.com is angry mebooksfree.com development of delusions and hallucinations. at the consultant but he is displacing his anger on the intern. Displacement is involved in the development Immature Defenses of phobiasQ. C. Acting out: Acting on unconscious desires without H. Intellectualization: Excessive use of intellectual becoming aware of them. For example, a person process to avoid the painful emotions. For example, suddenly steals an item from a shop without any prior a doctor who was diagnosed with pancreatic cancer planning. In this case, this person had an unconscious has a long discussion about the pathophysiology of desire of indulging in stealing. His mind however did the cancers with his treating physician. Here, the not allow this feeling to enter his conscious, as that doctor is trying to avoid the painful emotion of being would result in this person feeling bad about himself. diagnosed with the cancer by discussing excessively Hence, this person resorts to straight away acting on about the pathophysiology of cancers. mebooksfree.com mebooksfree.comthe unconscious desire withoutmebooksfree.com even becoming aware I. Isolationmebooksfree.com of affect: Removing the feelingsmebooksfree.com associated mebooksfree.com of the same. This defense mechanism is involved in with a stressful life event. For example, without development of impulse control disorders. showing any emotions, a woman tells her family D. Passive-aggressive behavior: Indirectly expressing the members that she has been diagnosed with advanced anger towards others. For example, a young boy was stage cholangiocarcinoma. forced to bring a glass of water for the father, while J. Repression: It is one of the most important defense bringing the water, the child accidentally tripped and mechanism, often referred to as the “primary” defense

mebooksfree.com mebooksfree.com mebooksfree.com mebooksfree.com mebooksfree.com mebooksfree.com

mebooksfree.com mebooksfree.com mebooksfree.com mebooksfree.com mebooksfree.com mebooksfree.com mebooksfree.com mebooksfree.com mebooksfree.com mebooksfree.com mebooksfree.com mebooksfree.com

118 Review of Psychiatry

mechanism. It is unconsciously forgetting something, students joked and laughed at themselves after getting which can not be retrieved later. For example, a young humiliated by the examiner during the viva. girl who was sexually abused by her father, “forgets” S. Sublimation: Expression of unacceptable feelings in this incidence of sexual abuse. Now, even if she wants a socially acceptable manner. For example, a middle mebooksfree.com mebooksfree.comto recall it, she can’t do itmebooksfree.com in normal circumstances. agedmebooksfree.com man with unacceptable sexual desiremebooksfree.com becomes mebooksfree.com K. Rationalization: Offering rational explanations to a painter and starts making nude paintings. Here, the justify own unacceptable behavior. For example, an sexual desires are getting an outlet and its socially alcoholic blamed his family environment for his habit acceptable since painting nudes is considered an art. of excessive drinking. It is a commonly used defense T. Suppression: It is the only voluntary or conscious mechanism in substance use disorders. defense mechanism. It involves a voluntary decision L. Dissociation: Splitting of a single (e.g. memory, to not think about an event for some time and hence identity) or group of mental functions from the avoid the accompanying emotions. For example, a remaining mental functions. It is seen in disorders like medical student who is extremely stressed out because dissociative identity disorder, where for example, of an upcoming entrance exam decides to take a one the identity of an individual gets split from rest of the day break during which he doesn’t think at all about the exam. mebooksfree.com mebooksfree.commental functions. mebooksfree.com mebooksfree.com mebooksfree.com mebooksfree.com M. Reaction formation: Transformation of feelings into Defense mechanisms in psychiatric disorder: All the exact opposite. For example, a man who is actually defense mechanisms are used at times by all of us. infatuated by an office colleague tells his friend that However when used excessively, they can result in deve­ he really hates her. Here, the actual feeling is that lopment of psychiatric disorders. Following is a list of few of infatuation but that is being transformed into the defense mechanisms and associated disorders: feeling of “hatred”. A. Obsessive compulsive disorder: Reaction formationQ, N. Undoing: An act which is done to nullify a previous displacementQ, undoing and inhibitionQ act. For example, a husband brings gifts for wife next B. Phobia: Displacement and inhibitionQ day after having a fight with her the previous day. The C. Dissociative disorder: DissociationQ defense mechanism of undoing is used in obsessive D. Neurosis: RegressionQ. compulsive disorderQ. mebooksfree.com mebooksfree.com O. Aim inhibition: Placing a mebooksfree.comlimitation upon instinctual Psychosexualmebooksfree.com Stages of Developmentmebooksfree.com mebooksfree.com demands, accepting partial or modified fulfillment Sigmund FreudQ proposed that the sexuality develops of desires. For example, a student who wanted to in multiple stages. Freud used the term “sexuality” in a became a doctor but who was not able to clear the pre broader concept that included others forms of pleasure­ medical tests takes admission in a veterinary course also and not only genital sexua­lity. He proposed five and becomes a veterinary doctor. stages of development. Freud further proposed that the Mature Defenses development may get arrested at a particular stage (called P. Altruism: Satisfying internal needs by helping others. “fixation”) and may result in development of psychiatric For example, while driving in a drunk state, a man disorders: met an accident and lost his son who was travelling A. Oral stage (0-1.5 years): This is the first stage of deve­ alongside him. Later, he started a campaign against lopment where in the pleasure is derived from the oral drunk driving and started educating people about ills cavity. The child derives pleasure in cutting, biting, mebooksfree.com mebooksfree.comof drunk driving. mebooksfree.com chewing,mebooksfree.com etc. mebooksfree.com mebooksfree.com Q. Anticipation: Planning in advance to deal with an B. Anal stage (1.5-3 years): The site of pleasure is anal uncomfortable event. For example, a student plans all region. The child gets a sense of achievement by get­ his arguments compre­hensively before going to home ting toilet trained. If the after a bad exam result. gets arrested at this stage (called fixation at anal R. Humor: Using comedy to deal with unpleasant stage), it can result in development of obsessive com- feeling and situations. For example, two medical pulsive disorderQ.

mebooksfree.com mebooksfree.com mebooksfree.com mebooksfree.com mebooksfree.com mebooksfree.com

mebooksfree.com mebooksfree.com mebooksfree.com mebooksfree.com mebooksfree.com mebooksfree.com mebooksfree.com mebooksfree.com mebooksfree.com mebooksfree.com mebooksfree.com mebooksfree.com

Psychoanalysis 119

C. Phallic stage (3-5 years): The site of pleasure is the doesn’t have a penis and desires to get one (known as genital area. According to Freud, penis becomes the “penis envy”). The female child believes that she was organ of principal interest to children of both sexes. castrated and that’s why does not have a penis and The male child develops what is known asoedipus holds her mother responsible for it, developing­ anger mebooksfree.com mebooksfree.comcomplexQ in which he startsmebooksfree.com developing­ sexual feeling againstmebooksfree.com the mother. The stage gets resolvedmebooksfree.com when mebooksfree.com towards the mother and wants to replace the father. the female child starts identifying with the mother. However , the male child also becomes fearful, that if Failure to resolve the oedipus and electra complex father finds it out, his father might castrate him (and can result in development of neurotic illnesses (like hence the child deve­lops castration anxietyQ). The hysteria). Hence, the neurotic illness develops due oedipus complex in male child gets resolved once the to fixation at phallic stageQ. child shifts his affection away from mother to some other D. Latent stage (5-12 years): During this stage, there is female and starts identifying (starts imitating father and relative quiescence or inactivity of sexual drive and trying to become like him) with the father. child focuses on learning and gaining skills. In females, the oedipus stage unfolds differently E. Genital stage (12 years onward till young adulthood): (at times the term used for female child is “electra This stage is characterized by maturation of genital complex”). The girl child develops sexual desire for the functioning and gra­dual achievement of a mature mebooksfree.com mebooksfree.comfather. At the same time, shemebooksfree.com becomes aware that she sexualmebooksfree.com and adult identity. mebooksfree.com mebooksfree.com

QUESTIONS AND ANSWERS

QUESTIONS 6. Counter transference is: (AIIMS Nov 2011) A. Type of defense mechanism 1. The term ‘id’ and “superego” were coined by: B. Psychic connection between patient and disease (DNB 2003, DNB 1994, WB 2001) with transfer of psychic energy from body parts A. Freud B. Skinner to brain mebooksfree.com mebooksfree.comC. mebooksfree.comD. Bleuler mebooksfree.comC. Implies doctor’s feelings towards mebooksfree.compatient mebooksfree.com D. Patient’s feelings towards doctor during psycho­ 2. That part of mind which works on reality principle therapy is: (DNB 2004, Karnataka 2001) A. ID B. Ego 7. According to Sigmund Freud, primary process think- C. Super ego D. Ego-ideal ing is: (JIPMER 2011) 3. The term ‘free association’ which is a fundamental A. Illogical and bizarre technique of psychoanalysis was coined by: B. Rational (DNB 2006, JIPMER 2001) C. Absent during sleep A. Freud B. Adler D. Logical and unconscious C. Erikson D. Jung 8. Psychodynamic theory of mental illness is based 4. Theory of “Psychosexual development” was given on: (AIIMS Nov 2007) mebooksfree.com mebooksfree.comby: mebooksfree.com(DNB Dec 2010) mebooksfree.comA. Unconscious internal conflict mebooksfree.com mebooksfree.com A. B. Sigmund Freud B. Maladjusted reinforcement C. Jean Piaget D. Skinner C. Organic neurological problem D. Focuses on teaching patients to restrain absurd 5. Interpretation of dreams by Freud was published thoughts in: (UP 2001, KA 2002, DNB 1999) A. 1990 B. 1900 9. Wrong statement about psychoanalysis is: C. 1956 D. 1919 (DNB 2007, J&K 2008; TN 2006)

mebooksfree.com mebooksfree.com mebooksfree.com mebooksfree.com mebooksfree.com mebooksfree.com

mebooksfree.com mebooksfree.com mebooksfree.com mebooksfree.com mebooksfree.com mebooksfree.com mebooksfree.com mebooksfree.com mebooksfree.com mebooksfree.com mebooksfree.com mebooksfree.com

120 Review of Psychiatry

A. Parapraxis has meaning 18. Which of the following excludes painful stimuli from B. Transference is patient’s feeling for therapist awareness? (AIIMS 1998) C. Counter transference is clinician’s feelings for A. Repression B. Reaction formation patient C. Projection D. Rationalization mebooksfree.com mebooksfree.comD. Unguided communicationmebooksfree.com has no meaning mebooksfree.com mebooksfree.com mebooksfree.com 19. Avoiding awareness of pain of reality by negative 10. Oedipus complex (given by Sigmund Freud) is sensory data is seen in which of the following seen in: (PGI 1998, Delhi 1998, DNB 2004) defense mechanisms? (MH 2011) A. Boys of 1-3 years of age A. Distortion B. Denial B. Girls of 1-3 years of age C. Displacement D. Dissociation C. Boys of 3-5 years of age 20. Postponing paying attention to a “conscious D. Girls of 3-5 years of age impulse” or “conflict” is a mature defense mecha- 11. In psychoanalytic terms, obsessive compulsive nism known as: disorder is fixed at: A. Sublimation B. Suppression (DNB 1998, Delhi 1998, TN 2002, Mah. 2003) C. Humor D. Anticipation A. Oedipal stage B. Genital stage 21. A reluctant child forced to bring sugar from a shop mebooksfree.com mebooksfree.comC. Oral stage mebooksfree.comD. Anal stage mebooksfree.comspills half of it on the way. This is anmebooksfree.com example of: mebooksfree.com 12. Fixation of hysteria is: (JIPMER 1997, Delhi 2002, DNB 2004) A. Hysteria B. Passive aggression (DNB 1999, WB 2002, J&K 2004, PGI 2005) C. Disobedience D. Active aggression A. Genital B. Anal C. Oral D. Phallic 22. A chronic alcoholic blames the family environ- ment as a cause of his alcoholism. This is pheno- 13. Following name(s) is/are associated with psycho- menon of: (AIIMS 2000) dynamic theory: (PGI Nov 2009) A. Projection B. Denial A. B. Sigmund Freud C. Rationalization D. Sublimation C. Emil Kraepelin D. Eugen Bleuler E. Kurt Schneider 23. Ego’s defense mechanism “Undoing” is typically seen in: (PGI 2001, AIIMS 1993, 1995) mebooksfree.com mebooksfree.comDefense Mechanisms mebooksfree.com mebooksfree.comA. Depression mebooksfree.com mebooksfree.com B. Schizophrenia 14. Not a defense mechanism: (PGI 1998) C. Obsessive compulsive neurosis A. Derailment B. Repression D. Hysteria C. Distortion D. Undoing 24. Most important cause of neurotic reaction is the 15. Which of the following is a mature defense mecha- excessive use of: nism? (DNB 2002, JIPMER 1991, UP 2007) (DNB 2005, PGI 1998, Nimhans 2001, Mah. 2004) A. Projection B. Reaction formation A. Projection B. Regression C. Anticipation D. Denial C. Suppression D. Sublimation 16. Which of the following is not a neurotic defense 25. Displacement reaction is characteristically seen mechanism? (DNB NEET 2014-15) in: (DNB 1998, MP 1998) mebooksfree.com mebooksfree.comA. Isolation mebooksfree.comB. Regression mebooksfree.comA. Mania B. Phobiamebooksfree.com mebooksfree.com C. Reaction formation D. Undoing C. Conversion disorder D. Depression 17. Which of the following is a neurotic defense 26. Defense mechanism in phobia is: mechanism? (DNB NEET 2014-15) (DNB NEET 2014-15) A. Repression B. Anticipation A. Inhibition B. Dissociation C. Projection D. Undoing C. Distorsion D. Conversion

mebooksfree.com mebooksfree.com mebooksfree.com mebooksfree.com mebooksfree.com mebooksfree.com

mebooksfree.com mebooksfree.com mebooksfree.com mebooksfree.com mebooksfree.com mebooksfree.com mebooksfree.com mebooksfree.com mebooksfree.com mebooksfree.com mebooksfree.com mebooksfree.com

Psychoanalysis 121

27. Defense mechanisms involved in OCD are: 11. D. See text (PGI 2012, PGI 2007) 12. D. See text A. Repression B. Undoing 13. A, B. C. Rationalization D. Sublimation Apart from Sigmund Freud, other big names mebooksfree.com mebooksfree.comE. Reaction formationmebooksfree.com mebooksfree.comassociated with psychoanalysis includemebooksfree.com Carl Jung mebooksfree.com and . Initially Jung and Adler worked ANSWERS along with Freud, however later they separated 1. A. and gave their own theories. 2. B. 14. A. Derailment is a formal thought disorder and not 3. A. a defense mechanism. 4. B. 15. C. 5. B. 16. B. Regression is an immature defense mechanism. 6. C. Rest all are neurotic defense mechanism. 7. A. The primary process thinking is a characteristic 17. A,D. of uncons­cious mind. It is illogical and aims for However, if you have to chose, go for repression. mebooksfree.com mebooksfree.comimmediate wish fulfilment.mebooksfree.com mebooksfree.comIt is one of the most important neuroticmebooksfree.com defense mebooksfree.com 8. A. Psychodynamic (or psychoanalytic) theory mechanism. stresses that unconscious memories and con­ 18. A. Repression is the defense mechanism which flicts are responsible for development of psychi­ removes painful memories or unacceptable desi­ atric disorders. The “conflict” may be between res away from the consciousness or awareness. different parts of mind such as id and ego or ego 19. B. Denial is the defense mechanism which helps and superego. a person to avoid (or refuse to accept) the rea­ 9. D. According to , “parapraxis” lity. Don’t get confused by the phrase “negative or “slips of tongue” are believed to reveal uncon­ sensory data”. scious content and hence are believed to have 20. B. Postponing or delaying action on a conscious meaning. The description of transference and impulse (a conscious wish) and its accompany­ given in this question is also ing emotions is known as suppression. mebooksfree.com mebooksfree.comcorrect. The last statementmebooksfree.com is wrong. In psycho­ mebooksfree.com mebooksfree.com mebooksfree.com 21. B. See text. analysis, unguided communication is believed 22. C. See text. to have meaning. Unguided communication here refers to the technique of “free association” 23. C. Undoing is typically seen in obsessive compulsive in which patient speaks all that comes into his disorder. mind, without any censoring. The “free associa­ 24. B. Excessive use of regression causes neurotic ill­ tion” helps in understanding the unconscious nesses. contents of mind and hence is meaningful. 25. B. Displacement and Inhibition are the defense 10. C. Sigmund Freud described oedipus complex for mechanisms involved in phobia. both sexes, however that term is mostly associ­ 26. A. ated with male sex now a days. 27. B, E. mebooksfree.com mebooksfree.com mebooksfree.com mebooksfree.com mebooksfree.com mebooksfree.com

mebooksfree.com mebooksfree.com mebooksfree.com mebooksfree.com mebooksfree.com mebooksfree.com

mebooksfree.com mebooksfree.com mebooksfree.com mebooksfree.com mebooksfree.com mebooksfree.com mebooksfree.com mebooksfree.com mebooksfree.com mebooksfree.com mebooksfree.com mebooksfree.com

mebooksfree.com mebooksfree.com mebooksfree.com mebooksfree.com mebooksfree.com mebooksfree.com

Chapter 13 Miscellaneous

ELECTROCONVULSIVE THERAPY (ECT) B. Unilateral ECTS: In an attempt to decrease the side effects of ECTs, the unilateral electrode placements The convulsive therapies have long been used for mebooksfree.com mebooksfree.com mebooksfree.com havemebooksfree.com been introduced. Theright unilateralmebooksfree.com ECT has mebooksfree.com treatment­ of psychiatric disorders. Initially, intramuscular been found to have better side effect profile in compari­ injections of camphor were used to produce convulsions son to the bilateral ECTs and is being increasingly used. in patients with psychosis, with good therapeutic results. Later, electricity was used as an agent to induce convul­ Mechanism of Action sions and it was called “electroconvulsive therapy.” The nductioni of a bilateral generalized seizure is consi­ Types dered necessary for the beneficial effect of ECTs. Earlier it was considered that the response to ECTs was an “all A. Direct ECT: In this technique, anesthetic agents and or none” phenomenon, however of late it has been found muscle relaxants are not used. The generalized con­ that at least in right unilateral ECTs, a dose response rela­ Q vulsions produced can result in fractures or teeth tion is present. The mechanism of action of ECTs is still mebooksfree.com mebooksfree.comdislocations. Due to highermebooksfree.com incidence of side effects not completelymebooksfree.com understood. Various hypothesismebooksfree.com include mebooksfree.com this technique is rarely used now. changes in the neurotransmitters (especially down regu­ B. Modified ECT (Indirect ECT): Here, anesthetic lation of postsynaptic b-adrenergic receptors),­ changes agents and muscle relaxants are administered before in growth factors and molecular mechanisms (latest giving ECT. As muscles are relaxed, the risk of bone research suggests increase in brain derived neurotrophic fractures and other injures from the motor activity factor, BDNFQ as an important mechanism) and neuro­ during the seizures gets minimized. genesis in areas like hippocampus.

Electrode Placement Indications Various configurations have been developed for electrode A. Depression (Major depressive disorder): The ECT was placement. These include: initially invented for the treatment of schizophre­ mebooksfree.com mebooksfree.com A. Bilateral ECT: This is usedmebooksfree.com most commonly and it niamebooksfree.com and other psychotic illnesses, howevermebooksfree.com currently mebooksfree.com involves placement of electrodes on both sides of it is mostly used for treatment of depressionQ. ECT the skull. In bilateral ECTs, various configurations of is effective for depression in both major depressive electrode placement have been devised. Thebifronto­ disorder as well as bipolar disorder. The clearest temporal electrode placement is deployed most indication for ECT is depression with suicide riskQ. commonly. Other commonly used configuration uses The indications of ECT in depression include the bifrontal electrode placement. following:

mebooksfree.com mebooksfree.com mebooksfree.com mebooksfree.com mebooksfree.com mebooksfree.com

mebooksfree.com mebooksfree.com mebooksfree.com mebooksfree.com mebooksfree.com mebooksfree.com mebooksfree.com mebooksfree.com mebooksfree.com mebooksfree.com mebooksfree.com mebooksfree.com

Miscellaneous 123

• Depression with suicide risk (ECT is treatment of A. Raised intracranial tensionQ (space occupying choice in acutely suicidal patientsQ due to imme­ lesion in CNSQ) diate onset of action) B. Recent myocardial infarction • Depression with stuporQ C. Severe hypertension mebooksfree.com mebooksfree.com• Depression with psychoticmebooksfree.com symptoms (psychotic D. Cerebrovascularmebooksfree.com disease mebooksfree.com mebooksfree.com depression or delusional depression) E. Severe pulmonary disease • In case of failed medication trials or intolerance to F. Retinal detachment. medications. B. Manic episode: Electroconvulsive therapy can be used COGNITIVE DEVELOPMENT STAGES in the treatment of acute mania, however since effec­ tive pharmacotherapy is available for mania, ECT is The thinking process undergoes a series of changes as the Q not the first line treatment. The ECT is used in only child grows up into an adult. Jean Piaget , described four those patients who are either intolerant/unrespon­ stages of development of thinking processes, also known sive to pharmacotherapy or when mania is so severe as cognitive develop­mental stages. These are described that there is a risk of homicide/suicide or danger of below: physical­ violence and immediate control of symptoms A. Sensorimotor stage (Birth to 2 years): This is the first mebooksfree.com mebooksfree.comis required. mebooksfree.com stage.mebooksfree.com During this stage, child learns throughmebooksfree.com sensory mebooksfree.com C. Schizophrenia: Electroconvulsive therapy is the first observations and gradually gains control of his motor functions. Initially, the child thinks that if he cannot line treatment in catatonic schizophreniaQ. It is also effective in other types of schizophrenia however­ see an object, it means that the object has ceased since the advent of antipsycho­tics, is used only if to exist. For example, if a rattle with which child is patient is unresponsive/intolerant to medications. playing, ­is taken away from the child and is covered, so that the child can no longer see it, the child will Electroconvulsive therapy is not effective in chronic think that the rattle no longer exists and will not try schizophreniaQ. D. Other indications where ECT is occasionally used to look for it. This type of thinking is also described as Q and Q include intractable seizuresQ, neuroleptic malig­ “out of sight, out of mind” “here and now” type of thinking. In the end of the sensorimotor stage nant syndromeQ, delirium, on-off phenomenon of Parkinson’s disease, etc. the child develops “object permanence”, which is the mebooksfree.com mebooksfree.com mebooksfree.com developmentmebooksfree.com of the concept that objectmebooksfree.com continue to mebooksfree.com Adverse Effects exist even if they are not visible currently. In the above example, once the child develops object permanence, A. Memory disturbances: It is the most common side he will try to search for the rattle by removing the effect of ECT. Both retrograde and anterograde amne­ covering cloth as he now knows that the rattle con­ Q sia is seen, however retrograde­ amnesia is much tinues to exist though he is not able to see it. Another more common. It is however mild and recovery occurs important development at around 18 months, is a usually within 1-6 months after treatment. process known as “symbolization”. It means that B. Other side effects include delirium, headache, muscle the infants now start developing mental symbols and aches, fractures (very rare with modified ECT), nausea using words for objects. For example, they make a and vomiting. mental symbol to represent a ball and use a word for mebooksfree.com mebooksfree.com mebooksfree.com it. Themebooksfree.com development of “object permanence”mebooksfree.com indicates mebooksfree.com Contraindications the transition to the next stage of development i.e. There are no absolute contraindicationsQ of ECT. Earlier stage of preoperational thought. raised intracranial tension was considered as an absolute B. Stage of preoperational thought (2–7 years): In this contraindication, however it is now regarded as a relative stage, use of symbols and language becomes more contraindication. Pregnancy is not a contraindication for extensive. The thinking process is characterised by ECT. The following are the relative contraindications of “intuitive thought”,Q which refers to thinking with­ ECT: out use of reasoning and an inability to use logica­lity.

mebooksfree.com mebooksfree.com mebooksfree.com mebooksfree.com mebooksfree.com mebooksfree.com

mebooksfree.com mebooksfree.com mebooksfree.com mebooksfree.com mebooksfree.com mebooksfree.com mebooksfree.com mebooksfree.com mebooksfree.com mebooksfree.com mebooksfree.com mebooksfree.com

124 Review of Psychiatry The children are also “egocentric” in this stage which LEARNING THEORY means that they are only concerned about their own Learning is acquiring of new behavioral patterns. The two needs and cannot think from others perspective. types of learning are: C. Stage of concrete operations (7–11 years): In this stage, A. Classical conditioning mebooksfree.com mebooksfree.comthe egocentric thought ismebooksfree.com replaced by “operational mebooksfree.com mebooksfree.com mebooksfree.com B. Operant conditioning thought” and hence the children start to see things A. Classical conditioning: Classical conditioning (also from others perspective also. The thinking is concrete called res­pondent conditioning) results from the (concrete thinking is the literal thinking. For example, repeated pairing of a neutral stimulus with one that when asked, the meaning of proverb “people who live naturally produces a response. The concepts of clas­ in glasshouses should not throw stones” the child will sical conditioning emerged from the experiments say that “if my house is of glass, I should not throw of Russian physiologist, Ivan Pavlov. The Pavlovian stones as it will break my house”. The child is not able to experiment included the following: understand the deeper meaning. The logical thinking Under normal circumstances, a dog would salivate starts to develop and children are able to understand to the smell of food. The ringing of bell would not and follow rules and regulations. Two important produce any salivation response. In the experiment, mebooksfree.com mebooksfree.comdevelopments in this stagemebooksfree.com are attainment of “con­ a bellmebooksfree.com was rung everytime before themebooksfree.com presentation mebooksfree.com servation” and “reversibility”. Conservation is the of food. The dog ultimately paired the bell with the ability to understand that despite changes in shape, food. Eventually the ringing of bell alone started to the object remains the same. For example, water may produce salivation, even if no food was presented to be transferred from a cup to a glass, and may appear the dog. The following are the elements of classical different in shape, however the amount will remain conditioning:­ the same. Reversibility is the capacity to understand • Unconditioned stimulus: It is a stimulus that natu­ that one thing can turn into another and back again, rally without any learning, produces a response. For e.g. water and ice. example, smell of food, which produces a response D. Stage of formal operations (11 to end of adolescence): of salivation. This stage is characterized by development ofabstract • Unconditioned response: It is the natural response thinkingQ, which is ability to understand the deeper to an unconditioned stimulus. For example, sali­ mebooksfree.com mebooksfree.com mebooksfree.com mebooksfree.comvation is the unconditioned responsemebooksfree.com to smell of mebooksfree.com meaning and deduce the larger meanings. For exam­ food. ple, when asked to explain the meaning of phrase • Conditioned stimulus: It is a stimulus which when “pen is mightier than sword”, a child with concrete paired with unconditioned stimulus, starts produc­ thinking will say that the pen is heavier and stronger ing a response. For example, ringing of bell usually than the sword, whereas a child who has achieved doesn’t produce any response. However, when it is abstract thinking will say that “power of knowledge repeatedly paired with food (unconditioned stimu­ is stronger than power of brute force”. The thinking lus), it also starts to produce a response. becomes logical,­ the child understands the concept­ • Conditioned response: The response which results of permutation and combination and probability. from pairing of conditioned stimulus to the uncon­ There is development of “hypothetico deductive ditioned stimulus. For example, the salivation thinking”. Hypothetico deductive thinking is ability to which results secondary to ringing of bell is a con­ mebooksfree.com mebooksfree.commake hypothesis and use mebooksfree.comdeductive reasoning (ability mebooksfree.comditioned response. mebooksfree.com mebooksfree.com to deduce, e.g. a child while playing a video game • Extinction: If the conditioned stimulus (ringing of observes that whenever he breaks a banana, apple or bell) is presented repeatedly without the uncondi­ cherry, he loses point, and hence is able to deduce tioned stimulus (smell of food), the response (sali­ that in this game to win he should avoid breaking the vation) will decrease and eventually disappear. This fruits). is called extinction.

mebooksfree.com mebooksfree.com mebooksfree.com mebooksfree.com mebooksfree.com mebooksfree.com

mebooksfree.com mebooksfree.com mebooksfree.com mebooksfree.com mebooksfree.com mebooksfree.com mebooksfree.com mebooksfree.com mebooksfree.com mebooksfree.com mebooksfree.com mebooksfree.com

Miscellaneous 125 • Stimulus generalizationQ: Here, a conditioned Behavior Therapy response gets transferred from one stimulus to other. For example, apart from the bell, ringing of According to learning theory, the maladaptive behaviors a tuning fork also starts resulting in salivation. are learned by either classical conditioning or operant mebooksfree.com mebooksfree.com B. Operant conditioning (Instrumentalmebooksfree.com conditioning): conditioningmebooksfree.com and hence can be unlearnt. Amebooksfree.com large number ­ mebooksfree.com The principles of operant conditioning were given of psychiatric disorders can be treated, if the psychiatric by BF Skinner. According to this theory, a behavior symptoms are considered as learned maladaptive beha­ is determined by its consequencesQ for the indi­ viors. Behavior therapy is a psychological treatment in vidual. Hence, according to this theory any behavior which the maladaptive behaviors of patients are changed can be learned or unlearned and its frequency can to improve the quality of life. Behavior therapy is a generic be changed by modifying the consequences of that term and is used to describe a variety of specific tech­ behavior. If a behavior is followed by pleasant con­ niques which intend to remove maladaptive behaviors. sequence (called reward), that behavior will get rein­ The techniques of behavior therapy include. forced i.e. its frequency will increase. For example, A. Systematic desensitisation: This technique was deve­ if a child is given a chocolate on studying for a par­ loped accor­ding to the principle of “reciprocal ticular amount of time, the frequency of studying will inhibition”Q. According to this principle if an anxiety mebooksfree.com mebooksfree.comincrease. Similarly if the consequencemebooksfree.com is negative, the provokingmebooksfree.com stimulus is provided while a mebooksfree.comperson is in a mebooksfree.com frequency of beha­vior will decrease. For example, if a relaxed state, the anxiety gets inhibited. For example, child is slapped on using a bad word, the frequency if a person who is phobic to spiders is first made to of using bad words will decrease. relax and then is exposed to a spider, he may develop Types: The frequency of a behavior is increased by posi­ much lesser anxiety. In systematic desensitisation, the tive or negative reinforcement and decreased by punish­ patient is first taught relaxation techniques (usually ment or extinction. progressive muscle relaxation) and then a hierarchy is made of anxiety provoking stimuli. For example, if a Table 1: Types of operant conditioning. person is afraid of heights, the list may have “standing Type Effect Example at the roof of a ten storey building” at the top, “stand­ Positive Behavior is A child increases his study ing on the balcony at second floor” in the middle reinforcementQ increased by hours as every study a positive session is rewarded with a and “standing on third stair” at the bottom of list. The mebooksfree.com mebooksfree.comconsequence mebooksfree.comchocolate patientmebooksfree.com is then exposed (or asked to imaginemebooksfree.com that expo­ mebooksfree.com (reward) sure) to a series of anxiety provoking stimuli, start­ Negative Behavior is A child increases cleaning ing with the least anxiety provoking stimulus while Q reinforcement increased of his room to avoid he is also using relaxation techniques. As the patient to avoid a scolding by the mother masters­ the technique of relaxation in the presence negative consequence of an anxiety provoking stimuli, he moves up to the PunishmentQ Behavior is A child stops using foul next stimulus. decreased by language after getting Systematic desensitization is used in the treatment a negative slapped for the same of phobiasQ, obsessive compulsive disordersQ and consequence certain sexual dis­orders. Extinction Behavior is An intern who used to B. Therapeutic graded exposure or in vivo exposure (or decreased work very hard in the exposure and response prevention): It is similar to mebooksfree.com mebooksfree.comdue to lack of mebooksfree.comward, becomes inefficient mebooksfree.com mebooksfree.com mebooksfree.com reinforcement as he was never praised systematic desensitisation except that no relaxation by his seniors. techniques are used and that real life situations are used. For example, if a patient is afraid of dogs, the PSYCHOTHERAPY exposure will start with looking at a picture of dog, Psychotherapy is treatment of psychiatric disorders by then looking at a video of dog, followed by looking at a using psychological methods. The following are impor­ dog from a distance and finally holding a dog in arms. tant kinds of psychotherapy: The patient learns to get habituated to anxiety (i.e. he

mebooksfree.com mebooksfree.com mebooksfree.com mebooksfree.com mebooksfree.com mebooksfree.com

mebooksfree.com mebooksfree.com mebooksfree.com mebooksfree.com mebooksfree.com mebooksfree.com mebooksfree.com mebooksfree.com mebooksfree.com mebooksfree.com mebooksfree.com mebooksfree.com

126 Review of Psychiatry

learns that anxiety gradually decreases by itself). It is disorders which are caused by dysfunction in autonomic used in phobiasQ and obsessive-compulsive disorder. control such as asthma, tension headaches, arrhythmias, C. Flooding (Implosion): Here, the patient is made to etc. The technique uses a feedback instrument, the choice confront the feared situation directly, without any of which depends on the patients problem. This instru­ mebooksfree.com mebooksfree.comhierarchy, as in systematicmebooksfree.com desensitisation or graded ment givesmebooksfree.com patient a feedback about the currentmebooksfree.com status of mebooksfree.com exposure. No relaxation exercises are used either. The a specific autonomic function. For example, an electro­ patient is exposed to the feared situation, experiences myogram (EMG) may be used to give patient feedback fear and anxiety which gradually subsides, and the about muscle tension in a particular muscle group. When patient is not allowed to escape. the muscle tension is high, the EMG will emit a higher D. Modeling (Participant modeling): Here, therapist tone and when muscle tension is low (i.e. when muscle himself­ makes the contact with phobic stimulus and is relaxed), the EMG will emit a lower tone. Using feed­ demonstrates this to the patient. Patient learns by back, patient learns to control his muscle tone and hence imitation and observation. For example, a therapist is able to control symptoms caused by increased muscle himself­ took a dog in his arms while a patient who tone (e.g. bruxism). had phobia of dogs observed him. This technique is used in phobias as well as obsessive compulsive dis­ Cognitive Therapy mebooksfree.com mebooksfree.comorders. mebooksfree.comThe mebooksfree.comcognitive theory assumes that themebooksfree.com cognitions mebooksfree.com E. Assertiveness training: Here a person is taught to be (thoughts) are at the core of psychiatric symptoms. On assertive while asking for his rights and while refusing the basis of early experiences, an individual may develop unjust demands of others. wrong patterns of thinking, known as cognitive distor­ F. Social skills training: Usually used in patients with tions (or maladaptive assumptions). For example, a child schizophrenia, it involves imparting skills required who was praised when he came first and was scolded for dealing with others and living a social life. when he got second rank, may develop a cognitive dis­ G. Aversive conditioning (Aversion therapy): It is the tortion that “To be successful it is necessary to get first clinical use of principles of classical conditioning. It rank, otherwise I would be considered as a failure”. These is used for treatment of unwanted behaviors (such as cognitive distortions (or maladaptive assumptions) give paraphiliasQ). Here, the patient is asked to imagine rise to “negative automatic thoughts”, which are thoughts mebooksfree.com mebooksfree.comthat he is indulging into mebooksfree.coman unwanted behavior and with amebooksfree.com negative connotation and appear mebooksfree.comautomatically. mebooksfree.com immediately a painful stimulus (such as an electric For example, in the above example, when the child with shock) is given. An association gets created between the above mentioned cognitive distortion has a below the unwanted behavior and painful stimuliQ and the expectation performance in the exam, he may start hav­ unwanted behavior ceases. It is now rarely used due ing “negative automatic thoughts” like “ I am a failure”, “I to ethical considerations. performed badly in exams, I will perform badly in every other exam” “I will never get a post graduation seat”, etc. Uses: The various technique of behavior therapy are used The cognitive therapy aims to correct these “negative primarily in treatment of anxiety disorders (like phobia, automatic thoughts” and “cognitive distortions”. When panic disorders). Behavior therapy can also be used in along with these, behavioral techniques are also used, the depression, dissociative disorders, eating disorders, sexual therapy method is known as “cognitive behavioral the­ disorders, personality disorders, substance used disor­ rapy”. Cognitive therapy and cognitive behavioral therapy mebooksfree.com mebooksfree.comders and schizophrenia. mebooksfree.comare usedmebooksfree.com in the treatment of depression, panicmebooksfree.com disorder, mebooksfree.com obsessive compulsive disorder, personality disorder and Biofeedback somatoform disorder. It is a treatment technique that uses the principles of ope­ Cognitive Distortions: Following is the list of common rant conditioning. The biofeedback is based on the idea cognitive distortionsQ (maladaptive assumptions): that autonomic nervous system (which is usually invol­ A. All or nothing thinking: Seeing things in black and untary) can be brought under voluntary control with the white. For example, if I failed to get a particular job, help of operant conditioning. It is used for treatment of it means that I would never ever get any job.

mebooksfree.com mebooksfree.com mebooksfree.com mebooksfree.com mebooksfree.com mebooksfree.com

mebooksfree.com mebooksfree.com mebooksfree.com mebooksfree.com mebooksfree.com mebooksfree.com mebooksfree.com mebooksfree.com mebooksfree.com mebooksfree.com mebooksfree.com mebooksfree.com

Miscellaneous 127

B. Approval seeking: Belief that you should always be K. Personalization: Blaming yourself for event, which you liked and loved by others, otherwise life would be are not responsible for. For example, a wife blames terrible. herself for her husbands extramarital affair. C. Disqualifying positive: It is a tendency of refusal to L. Should statements: Having a lots of rules about how mebooksfree.com mebooksfree.comacknowledge the positivemebooksfree.com events in life and insisting shouldmebooksfree.com you and others behave. For example,mebooksfree.com I should mebooksfree.com that they “don’t count”. For example, a housewife was exercise daily, I shouldn’t be lazy. praised by her husband, however she thought that “he is praising me just to make me feel better, in reality I Substance Use Disorder: Psychosocial don’t deserve to be praised”. Treatment D. Emotional reasoning: Belief that your emotions reflect The patients with substance use disorders (and other the reality. For example, if I am having a bad feeling problematic behaviors) go through a series of changes about a person, it means that the person in reality is a before quitting the substance use. Various models of bad human being even if I have no evidences for the these changes have been described, the most acceptable same. model is known as transtheoretical model of change. E. Fallacy of fairness: Tendency to judge a random nega­ According to this model, the following are the stages of mebooksfree.com mebooksfree.comtive event as an issue ofmebooksfree.com justice. For example, you change:mebooksfree.com mebooksfree.com mebooksfree.com missed the flight due to heavy traffic and you believe A. Precontemplation: In this stage, the substance user “life is always unfair to me”. doesn’t see any problem in his behavior and doesn’t F. Jumping to conclusions: Making an interpretation with think about quitting. minimal evidence. For example, a friend did not reply B. Contemplation: In this stage, the substance user starts to your message and you made a conclusion that the realising that he has a problem and that he is taking friend hates you. substance excessively. He considers about the pros G. Labelling mislabelling: Giving labels to self or others. and consQ of stopping substance use. However, he is For example, if your roommate didn’t clean room yet to make any decision. once, you label him as a “lazy slob”. C. Preparation: In this stage, the substance user decides H. Magnification (catastrophizing) and minimization: to quit the substance and starts making a plan to quit. Focussing on worst possible outcome is maximization D. Action: In this stage, the substance user actually stops mebooksfree.com mebooksfree.comand in its extreme form, mebooksfree.comit is called catastrophizing. takingmebooksfree.com the substance and makes changesmebooksfree.com in his beha­ mebooksfree.com For example, if you lose a hundred rupee note and viors (e.g. he stops meeting with the friends who use you say that its one of the biggest losses I ever had, its drugs in an attempt to keep away himself from drugs), maximization. If you say that now there is nothing left starts taking treatment. in my life, its catastrophization. Minimization is trying E. Maintenance: In this stage, the patient continues to to minimise the importance of events. For example, stay away from substances (drugs) and continues with an alcoholic when criticised about his heavy drinking the treatment and other behaviors to prevent relapse. says that “I don’t really drink much, just a peg here A patient may remain in maintenance stage or may relapse if he starts taking substance again. Usually, a and there”. patient has few relapses before attaining complete absti­ I. Mental filtering/selective perception: Picking a single nence (freedom) from substance. negative detail while ignoring the rest. For example, Various psychological treatment methods have been in a party, everybody gave you a complement for devised to help patient quit substance use and move your looks, however a single person said that “have mebooksfree.com mebooksfree.com mebooksfree.comfrom stagesmebooksfree.com of precontemplation to maintenance.mebooksfree.com One mebooksfree.com you gained weight” and you give all the importance of the most commonly used technique which focuses on to that one person’s remark and ignore all the praise. increasing the motivation of the patient to quit substance J. Overgeneralization: Considering a single negative is known as motivation enhancement therapy or moti­ event and making a general rule out of it. For exam­ vational interviewing. ple, you made a mistake at work and then you start Once the patient has reached maintenance stage, thinking “I always mess up everything”. Labelling is an relapse prevention techniques are used to prevent any extreme form of overgeneralisation. relapses (return to previous pattern of substance intake). mebooksfree.com mebooksfree.com mebooksfree.com mebooksfree.com mebooksfree.com mebooksfree.com

mebooksfree.com mebooksfree.com mebooksfree.com mebooksfree.com mebooksfree.com mebooksfree.com mebooksfree.com mebooksfree.com mebooksfree.com mebooksfree.com mebooksfree.com mebooksfree.com

128 Review of Psychiatry PSYCHOSURGERY c. Bhatia’s battery of performance tests of intelli­ gence. The surgical techniques for treatment of psychiatric • Personality assessment: The personality assessment disorder­ are rarely used and are reserved for only the can be done using two types of test: mebooksfree.com mebooksfree.comchronic and severe cases whichmebooksfree.com have not responded to all mebooksfree.coma. Objective test: These are standardizedmebooksfree.com tests which mebooksfree.com other methods of treatment. The psychosurgeries involve give numerical scores and can be analyzed using creating a lesion in the limbic system or its connecting standard result tables. For example, Minnesota fibres (limbic system is considered to be responsible for Multiphasic Personality Inventory (MMPI). normal and abnormal emotional reactions). The lesions b. Projective tests: In these tests, patients are pro­ are now a days produced with precision using stereotactic vided with ambiguous stimuli (unclear stimuli) methods. The following are the commonly used psycho­ and it is believed that the patient’s response surgeries. to such unclear stimulus reflects his internal A. Stereotactic subcaudate tractotomy: It produces a thought processes and emotional factors. The subcaudate lesion and is used in chronic, severe and patient “projects” his internal situation on to intractable cases of depression, obsessive compulsive the test question and finally an expert analyses disorder and schizoaffective disorder. the patients answers and deduces the aspects of mebooksfree.com mebooksfree.com B. Stereotactic limbic leucotomymebooksfree.com: Small lesion is made mebooksfree.compatients personality. The projectivemebooksfree.com tests include: mebooksfree.com in subcaudate and also a lesion is made in cingulate – Rorschach testQ: The patient is shown ten bundle. It is used in treatment of chronic, severe and cards which have inkblots and is asked intractable obsessive compulsive disorder and schizo­ what he sees in the card. phrenia. – Thematic apperception test (TAT)Q: Here C. Amygdalotomy: A lesion is made in amygdala in patients are shown certain pictures and patients with severe, uncontrolled aggression. asked to make stories about them – Sentence completion testQ: Here patients are NEUROPSYCHOLOGICAL TESTS given incomplete sentences and are asked to complete them. For example, a sentence Neuropsychology is a brach of psychology which exami­ may be like “I wish I ……….” nes the relationship between the behavior and brain – Word association technique: Here the mebooksfree.com mebooksfree.comfunctioning. It tries to locatemebooksfree.com the areas of disturbances mebooksfree.comexaminer says a word and patientmebooksfree.com has to mebooksfree.com in brain, on the basis of beha­vioral symptoms (includ­ respond with the first word that comes in ing cognitive, sensory, motor and emotional symptoms). to his mind. Neuropsychological tests are used extensively for various – Draw a person test (DAPT): Here patient purposes. Few of them have been discussed below: is asked to draw a person and then specific A. Neuropsychological assessment of intelligence and questions are asked about what he drew. personality: B. Neuropsychological assessment for brain disorders • Intelligence testing: The simplest way of measuring or organic mental disorders: Several tests have been intelligence is in terms of Intelligence Quotient, IQ devised which extensively measure a wide range of IQ = MA/CA × 100, MA is the mental age and CA cognitive functions like memory, motor functions, is the chrono­logical age, In this formula, the maxi­ sensory functions, problem solving, reading, writing, mum chronological age can be 15. arithmetic, etc. Few such important tests include: mebooksfree.com mebooksfree.com Now, much better andmebooksfree.com precise tests have been • mebooksfree.comLuria Nebraska Neuropsychological batterymebooksfree.com mebooksfree.com devised that measure the intelligence, few com­ • Halstead Reitan battery of neuropsychological monly used tests include: testsQ a. Wechsler adult intelligence scaleQ • Bender Gestalt TestQ (Bender visual motor gestalt b. Malin’s intelligence scale for Indian children test): This test is used mostly as a screening tool for (MISIC) organic brain disorders.

mebooksfree.com mebooksfree.com mebooksfree.com mebooksfree.com mebooksfree.com mebooksfree.com

mebooksfree.com mebooksfree.com mebooksfree.com mebooksfree.com mebooksfree.com mebooksfree.com mebooksfree.com mebooksfree.com mebooksfree.com mebooksfree.com mebooksfree.com mebooksfree.com

Miscellaneous 129

QUESTIONS AND ANSWERS

QUESTIONS C. Neuroleptic malignant syndrome mebooksfree.com mebooksfree.com mebooksfree.com mebooksfree.comD. Acute anxiety mebooksfree.com mebooksfree.com ECT 8. Absolute contraindication to ECT is:(AIIMS 1995) 1. Indications for ECT is/are: (PGI May 2010) A. Glaucoma B. Brain tumor A. Psychotic depression C. Aortic aneurism D. MI B. Catatonic schizophrenia 9. ECT is absolutely contraindicated in: C. Cyclothymia A. Pregnancy (AI 1992, DNB 1995) D. Dysthymia B. Very ill patient E. Post traumatic stress disorder C. Raised intracranial tension 2. Best marker for electroconvulsive therapy: D. Severe heart disease A. CSF 5 HIAA (AIIMS Nov 2008) 10. Most common complication of ECT is: mebooksfree.com mebooksfree.comB. CSF serotonin mebooksfree.com mebooksfree.comA. Anterograde amnesia mebooksfree.com(AIIMS 1996) mebooksfree.com C. Brain derived growth factor B. Retrograde amnesia D. CSF dopamine C. Psychosis 3. ECT is currently indicated as a line of treatment D. Depression in the following conditions except: (UPSC 2008) 11. Memory disturbance of ECT recovers in: A. Catatonic schizophrenia A. Few days to few weeks (AIIMS 1996) B. Severe depression with psychosis B. Few weeks to few months C. Manic-depressive psychosis C. Few months to few year D. Obsessive compulsive disorder D. Permanent 4. ECT is indicated in: (AIIMS 1998) 12. Most common complication of modified ECT: A. Neurotic depression (AIIMS 1991, AI 2, DNB 1997) mebooksfree.com mebooksfree.comB. Auditory hallucinationmebooksfree.com mebooksfree.comA. Intracerebral bleed mebooksfree.com mebooksfree.com C. Chronic Schizophrenia B. Fracture spine D. Delusional depression C. Body ache 5. ECT is not useful in treatment of: D. Amnesia A. Chronic schizophrenia (AI 1993, DNB 1994) 13. True about ECT is: (PGI May 2012, AIIMS 2011) B. Catatonic schizophrenia A. It is not a treatment for dysthymic disorder C. Endogenous depression B. Used to treat complex partial seizures D. Acute psychosis C. Used for those major depressive patients not 6. ECT in depressive phase of MDP is useful because responding to medication it: (PGI 1999) D. Memory impairment is a side effect A. Produces recurrence E. Effective in OCD mebooksfree.com mebooksfree.comB. Reduces recurrencemebooksfree.com mebooksfree.com mebooksfree.com mebooksfree.com C. Shortens duration Names D. Increases drug effects 14. Who introduced cocaine in psychiatry: 7. All of the following are indications for ECT except: (Kerala 1998, DNB 1992) A. Intractable seizures (DNB NEET 2014-15) A. Freud B. Jung B. Depressive stupor C. Miller D. Stanley

mebooksfree.com mebooksfree.com mebooksfree.com mebooksfree.com mebooksfree.com mebooksfree.com

mebooksfree.com mebooksfree.com mebooksfree.com mebooksfree.com mebooksfree.com mebooksfree.com mebooksfree.com mebooksfree.com mebooksfree.com mebooksfree.com mebooksfree.com mebooksfree.com

130 Review of Psychiatry

15. Moral treatment of mentally ill-patient was first 22. ‘Reinforcement’ is used in: (AIIMS 1994, 1999) stressed by: A. Psychoanalysis (AIIMS 1995, CMC 1998, DNB 2001, TN 2004) B. Hypnosis A. Pinel B. Morel C. Abreaction mebooksfree.com mebooksfree.comC. Kraepelin mebooksfree.comD. Sigmund Freud mebooksfree.comD. Conditioned learning mebooksfree.com mebooksfree.com 16. The eight stage classification of human life is pro­ 23. Behavior therapy to change maladaptive behavior posed by: (DNB 2K, WB 2004, UP 2005) using response as reinforcer uses the principles A. Sigmund Freud B. Pavel of: (AI 2003) C. Strauss D. Erikson A. Classical conditioning B. Modeling 17. Which of the following scientist propagated ‘thera­ C. Social learning peutic community concept: D. Operant conditioning (Karnataka 2K, DNB 2003) A. JB Watson B. Maxwell Jones 24. Many of our bad habits of day to day life can be C. Freud D. Adler removed by: (AIIMS Nov 2004) A. Positive conditioning mebooksfree.com mebooksfree.comCognitive Developmentmebooksfree.com Stages mebooksfree.comB. Negative conditioning mebooksfree.com mebooksfree.com C. Bio feed back 18. Which of the following is a stage of intuitive D. Generalization thought appearance in Jean-Piaget scheme: (PGI 1999) 25. Operant conditioning in which pain stimulus are A. Sensorimotor given to a child for decreasing a certain undesired B. Concrete behavior can be classified as: (AI 2010, 1997) C. Preoperational stage A. Positive reinforcement D. Formal operations stage B. Negative reinforcement C. Punishment 19. Ability to form a concept and generalize is known D. Negotiation as: (JIPMER 2011) A. Concrete thinking 26. A child is not eating vegetables. His mother starts mebooksfree.com mebooksfree.comB. Abstract thinking mebooksfree.com mebooksfree.comgiving a chocolate each time he finishesmebooksfree.com vegetables mebooksfree.com C. Intellectual thinking in the diet. The condition is: (AIIMS Nov 2012) D. Delusional thinking A. Operant conditioning B. Classical conditioning 20. In Piaget’s theory of cognitive development ‘out C. Social training of sight, out of mind’ and ‘here and now’ is seen D. Negative reinforcement in the stage of: (AIIMS 2013) A. Sensorimotor stage 27. Patient of contamination phobia was asked by B. Preoperational stage therapist to follow behind him and touch every­ C. Concrete operational stage thing he touches. During process therapist kept D. Formal operational stage talking quietly and calmly to the patients. The patient was asked to repeat the procedure twice mebooksfree.com mebooksfree.comLearning Theory and mebooksfree.comPsychotherapy mebooksfree.comdaily. The procedure is: (AIIMSmebooksfree.com May 2010) mebooksfree.com 21. Pavlov’s experiment is an example of: (AI 2006) A. Flooding B. Modeling A. Operant conditioning C. Positive reinforcement D. Aversion therapy B. Classical conditioning 28. Therapeutic exposure is a form of: (MH 2011) C. Learned helplessness A. Behavior therapy B. Psychoanalysis D. Modeling C. Cognitive therapy D. Supportive therapy

mebooksfree.com mebooksfree.com mebooksfree.com mebooksfree.com mebooksfree.com mebooksfree.com

mebooksfree.com mebooksfree.com mebooksfree.com mebooksfree.com mebooksfree.com mebooksfree.com mebooksfree.com mebooksfree.com mebooksfree.com mebooksfree.com mebooksfree.com mebooksfree.com

Miscellaneous 131

29. Reciprocal inhibition is done by: (SGPGI 2000) about quitting but is reluctant to do so because he A. Systematic desensitisation is worried that quitting will make him irritable. B. Flooding Which of the following option best describes the C. Exposure and response prevention stage of behavior change: (AI 2011) mebooksfree.com mebooksfree.comD. Psychoanalysis mebooksfree.com mebooksfree.comA. Precontemplation and preparationmebooksfree.com mebooksfree.com B. Contemplation and cost factor 30. Along a pleasant stimulus,a noxious stimuli is C. Contemplation and sickness susceptibility given in treatment of alcohol dependence and D. Belief paraphilias. This is an example for which kind of behavior therapy: (MH 2008) 38. A smoker is worried about the side effects of smok­ A. Negative reinforcement B. Aversive therapy ing. But he does not stop smoking thinking that C. Punishment D. Fooding he smokes less as compared to others and takes a good diet. This thinking is called as: 31. Behavior therapy is useful in: (PGI June 2008) (AIIMS May 2015) A. Psychosis B. OCD A. Self-exemption B. Cognitive error C. Personality disorder D. Panic attack C. Self-protection D. Distortion mebooksfree.com mebooksfree.comE. Anxiety disorders mebooksfree.com mebooksfree.com mebooksfree.com mebooksfree.com 32. A patient can be taught to control his involuntary Neuropsychological Tests physiological­ responses by which of the following 39. A Study comparing the behavioral and develop­ therapies: (MH 2009) mental changes in a normal brain with a damaged A. Breathing exercise brain is: (AIIMS 2013) B. Stress modification A. Neuropsychology C. Biofeedback B. Neurodevelopmental psychology D. Rational emotive therapy C. Child psychology 33. Tics, hair pulling, nail biting can be treated by: D. Criminal psychology (DNB December 2011) 40. Rorschach inkblot test is: (BIHAR 2003) A. Mind fullness A. Projective B. Subjective B. Social habit training mebooksfree.com mebooksfree.comC. Habit reversal trainingmebooksfree.com mebooksfree.comC. Both D. Nonemebooksfree.com of the above mebooksfree.com D. No intervention required 41. Best test for diagnosis of organic mental disorder: 34. Which of the following is not a cognitive error/ A. Sentence completion test (AI 2000) dysfunction? (AI 2010) B. Bender gestalt test A. Catastrophic thinking B. Arbitrary inference C. Rorschach test C. Overgeneralization D. Thought block D. Thematic appreciation test 35. Typically changes in problem behavior shows how 42. Rorschach test measures: (PGI 1999) many stages: (DNB NEET 2014-15) A. Intelligence B. Creativity A. 2 B. 3 C. Personality D. Neuroticism C. 4 D. 5 43. Signs of organic brain damage are evident on: mebooksfree.com mebooksfree.com 36. All of the following aremebooksfree.com parts of cognitive behavior mebooksfree.comA. Bender-Gestalt test mebooksfree.com(AI 2004) mebooksfree.com change technique except: (AI 2010) B. Rorschach test A. Precontemplation B. Consolidation C. Sentence completion test C. Action D. Contemplation D. Thematic apperception test 37. A chronic smoker taking 20 cigarettes per day has 44. Halstead Reitan battery involves all except: developed chronic cough. His family suggested A. Finger oscillation quitting cigarettes. He is ready to quit and thinks B. Constructional praxis

mebooksfree.com mebooksfree.com mebooksfree.com mebooksfree.com mebooksfree.com mebooksfree.com

mebooksfree.com mebooksfree.com mebooksfree.com mebooksfree.com mebooksfree.com mebooksfree.com mebooksfree.com mebooksfree.com mebooksfree.com mebooksfree.com mebooksfree.com mebooksfree.com

132 Review of Psychiatry

C. Rhytm A. Severe depression D. Tactual performance B. Conversion disorder C. Personality disorder Miscellaneous D. Somatization disorder mebooksfree.com mebooksfree.com mebooksfree.com mebooksfree.com mebooksfree.com mebooksfree.com 45. A person laughs to a joke, and then suddenly loses 53. Serial 7 subtraction is used to test: tone of all his muscles. Most probable diagnosis (DNB NEET 2014-15) of this condition is: (DNB Dec 2009) A. Working memory B. Long-term memory A. Cataplexy B. Catalepsy C. Mathematical ability D. Recall power C. Cathexis D. Cachexia 54. Erotomania is seen in: (DNB NEET 2014-15) 46. Hypomimia is: (DNB NEET 2014-15) A. Schizophrenia B. Mania A. Decreased ability to copy C. Neurosis D. OCD B. Decreased execution 55. Highest level of insight is: (DNB NEET 2014-15) C. Deficit of expression by gesture A. Intellectual B. Emotional D. Deficit of fluent speech C. Psychological D. Affective (Kerala 1994) mebooksfree.com mebooksfree.com 47. Deja vu is seen in: mebooksfree.com 56. mebooksfree.comWhich category of ICD is associatedmebooksfree.com with mood mebooksfree.com A. Temporal lobe epilepsy disorders: (DNB NEET 2014-15) B. Normal person A. F 10-19 B. F 20-29 C. Psychosis C. F 30-39 D. F 40-49 D. All of the above 57. Which category of ICD is associated with schizo­ 48. Unfamiliarity of familiar things is seen in: phrenia? (DNB NEET 2014-15) (Kerala 1999, JIPMER 2002) (Karnataka 1994) A. F 10-19 B. F 20-29 A. Deja vu B. Jamais vu C. F 30-39 D. F 40-49 C. Deja entendu D. Deja pence 58. DSM-IV classification of psychiatric disorder as 49. Patient wanting to scratch for itching in his proposed by American Psychiatric Association amputated limb is an example of: classifies and helps in diagnosing patients on mul­ A. Illusion (DNB NEET 2014-15) mebooksfree.com mebooksfree.com mebooksfree.com mebooksfree.comtiple axes. Of these, axis V representsmebooksfree.com the degree mebooksfree.com B. Pseudohallucination of: (MH 2009) C. Phantom limb hallucination A. Present state of symptoms D. Autoscopic hallucination B. Comorbid medical condition 50. All of the following are true about pseudohalluci­ C. Global assessment of function nations except: (DNB NEET 2014-15) D. Comorbid psychological problem A. It arises in inner subjective self 59. When information memorized afterwards is inter­ B. Patient describes that the sensations are being fered by the information learnt earlier, it is called: perceived by “mind’s eye” A. Retroactive inhibition (AIIMS May 2004) C. They are under voluntary control B. Proactive inhibition D. Distressing flashbacks of PTSD is an example C. Simple inhibition mebooksfree.com mebooksfree.com 51. Catatonia is most commonlymebooksfree.com seen with: mebooksfree.comD. Inhibition mebooksfree.com mebooksfree.com A. Schizophrenia (DNB NEET 2014-15) 60. Methods of learning in psychiatry are all except: B. Depression (AIIMS Nov 2007) C. Anxiety disorder A. Modelling B. Catharsis D. Obsessive compulsive disorder C. Exposure D. Response prevention 52. Catatonic features are seen in schizophrenia, they 61. According to Disabilities Act,1995, the seventh dis­ are also seen in: (PGI Jun 2008) ability is usually referred to as? (AIIMS Nov 2008)

mebooksfree.com mebooksfree.com mebooksfree.com mebooksfree.com mebooksfree.com mebooksfree.com

mebooksfree.com mebooksfree.com mebooksfree.com mebooksfree.com mebooksfree.com mebooksfree.com mebooksfree.com mebooksfree.com mebooksfree.com mebooksfree.com mebooksfree.com mebooksfree.com

Miscellaneous 133

A. Neurological abnormality 10. B. Amnesia is the most common side effect of ECT. B. Mental illness Both retrograde and anterograde amnesia are C. Substance abuse seen, however retrograde amnesia is much more D. Disability due to road traffic accident common. mebooksfree.com mebooksfree.com mebooksfree.com 11. mebooksfree.comB. Amnesia caused by ECT is mildmebooksfree.com and recovery mebooksfree.com 62. Patients suffering from which of the following ­occurs usually within 1-6 months after treatment. disease as per ICD/DSM criteria are eligible for 12. D. disability benefit as per National Trust Act? 13. A,C,D. (AI 2009) A. Schizophrenia B. Bipolar disorder Names C. Dementia D. Mental retardation 14. A. Sigmund Freud studied about the effects of 63. Consultation—liaison (C-L) psychiatry involves ­cocaine. It is also believed that he was addicted diagnosing:­ (MAHE 2006, SGPGI 2004) to cocaine for a long period. A. Psychiatric illness in medically ill 15. A. Moral treatment of mentally ill patients using B. Medical illness in psychiatric patients humane methods was first stressed by Pinel. mebooksfree.com mebooksfree.comC. Suicidal tendency inmebooksfree.com psychiatric patients 16. mebooksfree.comD. Erik Erikson divided the human mebooksfree.comlife into eight mebooksfree.com D. Suicidal tendency in medically ill stages, known as Erikson’s psychosocial stages. 17. B. is a group based ANSWERS approach for treatment of substance use disorders and other psychiatric disorders. It is a ECT residential approach where in patients live in a 1. A,B. house for long-term and have defined roles dur­ See text. ing the stay. The term “therapeutic community” 2. C. Latest research suggests that increase in brain was given by Thomas Main and the concept was derived neurotrophic factor, BDNF mediates the developed by Maxwell Jones. response to ECT and is the best marker for the same. Cognitive Development Stages mebooksfree.com mebooksfree.com 3. D. ECT is rarely used inmebooksfree.com the treatment of OCD. 18. mebooksfree.comC. Intuitive thinking is seen in stagemebooksfree.com of preopera­ mebooksfree.com 4. D. Delusional depression or psychotic depression tional thought. is an indication for ECT. 19. B. Abstract thinking is the ability to make concepts 5. A. Electroconvulsive therapy is not effective in (i.e. ability to grasp essential of whole) and to chronic schizophrenia. generalise. 6. C. ECT shortens the duration of depressive episode. 20. A. See text. It doesn’t prevent the recurrence unless given as a maintenance treatment. Learning Theory and Psychotherapy 7. D. ECT is occasionally used in intractable seizures, 21. B. neuroleptic malignant syndrome, delirium, on- 22. D. off phenomenon of Parkinson’s disease. Acute 23. D. Use of rewards as a reinforcer (in positive rein­ mebooksfree.com mebooksfree.comanxiety is not an indication.mebooksfree.com mebooksfree.comforcement) is a technique of operantmebooksfree.com condition­ mebooksfree.com 8. B. There are no absolute contraindications for ECT. ing. Earlier, raised intracranial tension and space 24. B. Negative conditioning is used to decrease the occupying lesions were considered as absolute frequency of a particular behavior. contraindications, hence the best answer here is 25. C. Punishment is decrease in frequency of a beha­ brain tumor. vior due to unpleasant consequences. 9. C. Again, the best answer is raised intracranial ten­ 26. A. This is an example of positive reinforcement, a sion. type of operant conditioning.

mebooksfree.com mebooksfree.com mebooksfree.com mebooksfree.com mebooksfree.com mebooksfree.com

mebooksfree.com mebooksfree.com mebooksfree.com mebooksfree.com mebooksfree.com mebooksfree.com mebooksfree.com mebooksfree.com mebooksfree.com mebooksfree.com mebooksfree.com mebooksfree.com

134 Review of Psychiatry

27. B. This is an example of participant modeling in 47. D. Deja vu refers to the feeling that an event which which patient learns by observation and imita­ is being currently experienced has also happened tion of therapist. in the past. It can be seen in normal persons 28. A. and also in certain disorders like temporal lobe mebooksfree.com mebooksfree.com 29. A. The principle of reciprocalmebooksfree.com inhibition is used in mebooksfree.comepilepsy. mebooksfree.com mebooksfree.com the technique of systematic desensitisation. 48. B. Jamais vu refers to the feeling of unfamiliarity for 30. B. Aversive therapy. familiar things. 31. A,B,C,D,E. 49. C. Phantom limb is the experiencing of sensations in an Behavioral therapy is primarily used in treatment amputated limb. of anxiety disorders (including panic disorder), 50. C. Pseudohallucinations are not under voluntary obsessive compulsive disorder. It is also useful control. in personality disorders. Though, in psychotic 51. B. Catatonia is most commonly seen in mania fol­ disorders like schizophrenia, behavioral therapy lowed by depression and than schizophrenia. is not the first line treatment, however it can be 52. A. used. 53. A. Serial 7 subtraction test, in which the patient is 32. C. asked to serially subtract 7 from 100 is a test for mebooksfree.com mebooksfree.com 33. C Habit reversal trainingmebooksfree.com is a kind of behavioral therapy mebooksfree.comworking memory. mebooksfree.com mebooksfree.com which is used in the management of tics, tri­ 54. A. Erotomania or delusion of love is most commonly chotillomania, nail biting. skin picking and seen in schizophrenia and delusional disorder. other similar disorders. The technique involves 55. B. Emotional insight is the highest level of insight. In getting aware of the urge that precedes tics and emotional insight, the patient is aware of the ill­ other impulsive behaviors and developing an ness and also changes his behavior accordingly. alternative response. Intellectual insight is next to emotional insight 34. D. Thought block is not a cognitive error. in the hierarchy of insight. In intellectual insight, 35. D According to the transtheoretical model, there the patient is aware that he has illness, however are 5 stages of change in substance use and other he doesn’t change his behavior in any manner problem behaviors. based on this knowledge. 36. B. Consolidation is not a stage of change. 56. C. The fifth chapter of ICD-10 classifies psychiatric 37. C. In this question, patient is considering quitting disorders. The chapter has been further sub mebooksfree.com mebooksfree.comand thinking aboutmebooksfree.com the pros and cons of it. This mebooksfree.comdivided into blocks as described below:mebooksfree.com mebooksfree.com is characteristic of stage of contemplation. F00-F09: Organic, including symptomatic, mental 38. A. Self-exemption refers to the beliefs that give disorders smokers false reassurances and allow them to F10-F19: Mental and behavioral disorders due to avoid thinking deeply about the importance of psychoactive substance use quitting. F20-F29: Schizophrenia, schizotypal and delu­ Neuropsychological Tests sional disorders F30-F39: Mood (affective) disorders 39. A. See text. F40-F48: Neurotic, stress-related and somato­ 40. A. Rorschach inkblot test is a projective test. form disorders 41. B. See text. F50-F59: Behavioral syndromes associated with 42. C. Personality. physio-logical disturbances and physical factors mebooksfree.com mebooksfree.com 43. A. mebooksfree.com mebooksfree.com F60-F69: Disorders of adult personalitymebooksfree.com and mebooksfree.com 44. B. Constructional praxis is not a part of halstead reitan behavior­ battery. F70-F79: Mental retardation F80-F89: Disorders of psychological development Miscellaneous F90-F98: Behavioral and emotional disorders 45. A. with onset usually occurring in childhood and 46. C. Hypomimia refers to decrease in facial expres­ adolescence sions, usually seen in parkinsonism. F99-F99: Unspecified mental disorder.

mebooksfree.com mebooksfree.com mebooksfree.com mebooksfree.com mebooksfree.com mebooksfree.com

mebooksfree.com mebooksfree.com mebooksfree.com mebooksfree.com mebooksfree.com mebooksfree.com mebooksfree.com mebooksfree.com mebooksfree.com mebooksfree.com mebooksfree.com mebooksfree.com

Miscellaneous 135

57. B. 59. B. The tendency of previously learned information 58. C. In DSM-IV, a multiaxial system was used to hinder subsequent learning is known as pro­ while making the diagnosis. The diagnosis was active inhibition. described in the following five axes: 60. B. Catharsis is not a method of learning. The term mebooksfree.com mebooksfree.com Axis I: Clinical syndromes/Disordersmebooksfree.com (psychiatric mebooksfree.com“catharsis” is used to denote themebooksfree.com process of mebooksfree.com disorder) ­release of pent-up emotions (emotional outlet). Axis II: Personality disorders/Mental retardation 61. B. According to persons with disability Act, 1995; the Axis III: Medical conditions sixth disability is mental retardation and seventh Axis IV: Psychosocial and environmental stress­ disability is mental illnesses. ors 62. D. The National Trust Act is applicable for autism, Axis V: Global assessment of functioning cerebral palsy, mental retardation and multiple In DSM-5, the multiaxial system has been disabilities. ­removed. The former axis I, II and III have been 63. A. Consultation liaison psychiatry is the speciality combined and for the last two, separate notations of psychiatry which deals with the psychiatric are being used. illnesses in medically ill patients. mebooksfree.com mebooksfree.com mebooksfree.com mebooksfree.com mebooksfree.com mebooksfree.com

mebooksfree.com mebooksfree.com mebooksfree.com mebooksfree.com mebooksfree.com mebooksfree.com

mebooksfree.com mebooksfree.com mebooksfree.com mebooksfree.com mebooksfree.com mebooksfree.com

mebooksfree.com mebooksfree.com mebooksfree.com mebooksfree.com mebooksfree.com mebooksfree.com

mebooksfree.com mebooksfree.com mebooksfree.com mebooksfree.com mebooksfree.com mebooksfree.com